You are on page 1of 242

HI TON HC H NI

S GIO DC V O TO K LK

NGUYN VN MU, PHAN HNG (Ch bin)

CC CHUYN TON HC
BI DNG HC SINH GII
KHU VC DUYN HI NAM
TRUNG B V TY NGUYN

K YU HI THO KHOA HC
Bun Ma Thut, ngy 14-15 thng 03 nm 2015

1. TRANG BA:
Nh xut bn: THIT K LI MU SC, CHNH SA PH HP VI BN IN
2. MC SNG BA:
CC CHUYN TON HC BI DNG HC SINH GII
TRUNG HC PH THNG

HI TON HC H NI

S GIO DC V O TO K LK

****************

*******************

CH BIN:

NGUYN VN MU, PHAN HNG

CC CHUYN TON HC
BI DNG HC SINH GII
KHU VC DUYN HI NAM
TRUNG B V TY NGUYN

K YU

HI THO KHOA HC

BUN MA THUT 14-15/03/2015

Cc bo co khoa hc
CC CHUYN TON HC BI DNG HC SINH GII
KHU VC DUYN HI NAM TRUNG B V TY NGUYN
ti Thnh ph Bun Ma Thut vo cc ngy 14-15/03/ 2015
Chiu ngy 13.03.2015
16h00-17h30 Hp BTC v Ban Chng trnh
18h00-20h00 n ti
Ngy 14.03.2015, bui sng
08h00-08h30 Vn ngh cho mng
08h30-09h00 Khai mc
Pht biu khai mc: Gim c s GD v T c Lk - Phan Hng
Pht biu tham lun ca cc i biu i din cc s GD:
Pht biu: GS.TSKH Nguyn Vn Mu
Pht biu tng kt: Gim c s GD v T c Lk - Phan Hng
11h00-12h30 n tra
Ngy 14.03.2015, bui chiu
13h30-14h00 Khai mc
Pht biu khai mc: Hiu trng THPT Chuyn Nguyn Du - L Vn Thi
Pht biu ca cc i biu:
Pht biu dn: GS.TSKH Nguyn Vn Mu
14h00-15h30 Cc bo co khoa hc phin hp ton th
iu khin: PGS.TS Trn Huy H, PGS.TS Nguyn Thy Thanh
1. m Vn Nh, a thc bt kh quy v mt vi vn dng lin quan
2. Trnh o Chin, Cc dng ton chn hc sinh gii cp quc gia nhng nm gn y
3. Trn Nam Dng, Ton hc v thut ton
4. H Thng, Mt s ng dng ca bt bin trong dy s v s hc
5. Nguyn B ang, Mt gc nhn thng qua cc bi hnh hc cc k thi Olympic Ton quc t
6. Phm ng Long, p dng mnh trong cc bi ton logic
7. Trng Ngc c, Mt s dy s sinh bi cc hm lng gic
8. L Sng, Hunh B Lc, Mt s ng dng ca php bin i Abel
15h30-15h45 Ngh gii lao
15h45-17h30 Cc bo co khoa hc phin chuyn
iu khin: TS. Trn Nam Dng, TS Trnh o Chin
1.
2.
3.
4.
5.

L Quc Bo, Mt hng tip cn gii bi ton dy s


Nguyn Ti Chung, Gii hn ca cc dy s sinh bi tng
L Th nh oan, Mt s vn lin quan n hm phn nguyn
Nguyn Vn Giang, Mt s dng ton v bt phng trnh hm
Cao Trn T Hi, Mt s phng php gii ton t hp
1

6. Nguyn Vn Quang, Mt s ng dng ca tm v t


7. on Vn Khim, Mt s dng ton v bt ng thc trong t din tng t nh tam gic
8. Hong t H, Mt s p dng khai trin Taylor trong bt ng thc v tnh gii hn
9. Nguyn Th Minh Nguyt, Phng php gii bi ton chia ht
10. H Duy Ngha, Mt s ng dng ca thng d bc hai
11. L Th Kim Uyn, a thc Fibonacci v ng dng
12. Mai c Thanh, Mt s dng ton v dy s trong t hp
13. Lng Ngc Tin, Mt s bi ton gii bng phng php m theo hai cch
14. Li Th nh Vn, Mt s dng ton suy lun logic
15. Nguyn Mnh Quyn, Mt s ng dng ca nguyn l Dirichlet trong ton hc
16. Nguyn Th Phng Tho, p dng nguyn l xung thang gii mt s dng ton
17h30-18h00 Tng kt hi tho
iu khin: GS.TSKH. Nguyn Vn Mu
18h00-20h00 n ti
20h00-22h30 Giao lu vn ngh

Ngy 15.03.2015
08h00-9h30: Chng trnh hi ngh bn trn v
Trao i cc kinh nghim bi dng hc sinh gii
iu khin: GS.TSKH. Nguyn Vn Mu, G Phan Hng
1. T Ton - Tin, Trng THPT Mc nh Chi, Gia Lai, Mt s phng php xc nh s hng
tng qut ca dy s
2. L Trang Tr, Mt s bi tp v dy s tun hon
3. Ng Minh Ngc Richard 10CT, Nguyn Vn Quang, Mt s ng dng ca nguyn l Dirichlet
trong t hp v bt ng thc
4. Hong Huy Thng - Bi B Anh 10CT, Nguyn Vn Quang, Phng php phn chng trong
hnh hc
5. Phan Ngc Ton, ng dng cp phn t trong bi ton s hc

9h30-18h00: Chng trnh tham quan thc a.

Mc lc
Chng trnh: Cc bo co khoa hc

Li ni u

1. L Vn Thi
Bo co dn Hi tho khoa hc

2. Trn Nam Dng,


Ton hc v thut ton

3. m Vn Nh,
a thc bt kh quy v mt vi vn dng lin quan

14

4. Trnh o Chin,
Cc dng ton chn hc sinh gii cp quc gia nhng nm gn y

26

5. Nguyn B ang,
Mt gc nhn thng qua cc bi hnh hc cc k thi Olympic Ton quc t

30

6. L Quc Bo,
Mt hng tip cn gii bi ton dy s

38

7. Nguyn Ti Chung,
Gii hn ca cc dy s sinh bi tng

46

8. Trng Ngc c,
Mt s dy s sinh bi cc hm lng gic

58

9. Phm ng Long,
p dng mnh trong cc bi ton logic

65

10. L Th nh oan,
Mt s vn lin quan n hm phn nguyn

72

11. Nguyn Vn Giang,


Mt s dng ton v bt phng trnh hm

83

12. Hong t H,
Mt s p dng khai trin Taylor trong bt ng thc v tnh gii hn

92

13. Nguyn Th Minh Nguyt,


Phng php gii bi ton chia ht

99

14. Cao Trn T Hi,


Mt s phng php gii ton t hp

105

15. H Duy Ngha,


Mt s ng dng ca thng d bc hai

114

16. H Thng,
Mt s ng dng ca bt bin trong dy s v s hc
17. L Th Kim Uyn,
3

123

a thc Fibonacci v ng dng

135

18. Nguyn Vn Quang,


Mt s ng dng ca tm v t

144

19. L Sng, Hunh B Lc,


Mt s ng dng ca php bin i Abel

148

20. L Trang Tr,


Mt s bi tp v dy s tun hon

153

21. on Vn Khim,
Mt s dng ton v bt ng thc trong t din tng t nh trong tam gic

160

22. Mai c Thanh,


Mt s dng ton v dy s trong t hp

171

23. Lng Ngc Tin,


Mt s bi ton gii bng phng php m theo hai cch

174

24. Li Th nh Vn,
Mt s dng ton suy lun logic

185

25. Nguyn Mnh Quyn,


Mt s ng dng ca nguyn l Dirichlet trong ton hc

193

26. Nguyn Th Phng Tho,


p dng nguyn l xung thang gii mt s dng ton

207

27. T Ton - Tin, Trng THPT Mc nh Chi, Gia Lai,


Mt s phng php xc nh s hng tng qut ca dy s

215

28. Ng Minh Ngc Richard 10CT, Nguyn Vn Quang,


Mt s ng dng nguyn l Dirichlet trong t hp v bt ng thc

225

29. Hong Huy Thng - Bi B Anh 10CT, Nguyn Vn Quang,


Phng php phn chng trong hnh hc

236

Lin kt bi dng hc sinh gii khu vc


Duyn hi Nam Trung b v Ty Nguyn
- ng lc thc y pht trin hc sinh gii
cc tnh trong khu vc
Ha chung trong khng kh k nim 40 nm gii phng Bun Ma Thut m mn cho cuc
Tng tin cng v ni dy ma Xun nm 1975, nh cao l Chin dch H Ch Minh lch s,
gii phng hon ton min Nam, thng nht T quc; cng vi vic trin khai Ngh quyt s
29-NQ/TW Hi ngh ln th 8, Ban Chp hnh Trung ng kha XI vi ni dung i mi cn
bn, ton din gio dc v o to, p ng yu cu cng nghip ha hin i ha trong iu
kin kinh t th trng nh hng x hi ch ngha v hi nhp quc t; nhm pht trin hc
sinh gii ca cc tnh trong khu vc, S Gio dc v o to k Lk cng vi cc S Gio dc
v o to cc tnh khu vc duyn hi Nam Trung b v Ty Nguyn phi hp vi Hi Ton hc
H Ni ng t chc Hi ngh lin kt bi dng hc sinh gii cc tnh khu vc duyn hi Nam
Trung b v Ty Nguyn ln th V.
Qua 4 k t chc Hi ngh, ln lt ti Ph Yn, Khnh Ha, Bnh nh v Gia Lai, n nay
cng tc bi dng hc sinh gii Trung hc ph thng c cc tnh quan tm v thng xuyn
chia s kinh nghim. S lng hc sinh gii hng nm ca cc tnh trong khu vc c pht trin
v c s lng ln cht lng, nhiu tnh c hc sinh ot gii cao trong k thi hc sinh gii
Trung hc ph thng Quc gia; mt s tnh c hc sinh ot gii trong k thi Olympic Quc
t. Thnh tch ny th hin c tinh thn v ch phn u ca hc sinh v gio vin ca
cc tnh trong khu vc.
Trn c s thc trng cng tc pht hin, tuyn chn, bi dng v kt qu thi hc sinh gii
Trung hc ph thng trong nhng nm qua ca cc tnh khu vc duyn hi Nam Trung b v
Ty Nguyn, Hi ngh ln ny nh gi mt cch khch quan, thc cht v cng tc lin kt bi
dng hc sinh gia cc tnh, t xut nhng gii php hu hiu nhm pht trin hc sinh
gii cho nhng nm tip theo.
Hi ngh vinh d c n tip nhiu nh khoa hc, nh gio lo thnh, cc chuyn gia u
ngnh thuc b mn Ton hc v Ng vn; cc chuyn gia gio dc, lnh o v cn b ph trch
chuyn mn cc S Gio dc v o to; cc thy gio, c gio b mn cc tnh khu vc duyn
hi Nam Trung b v Ty Nguyn ang trc tip bi dng hc sinh gii. Ban t chc Hi ngh
hy vng vic trao i kinh nghim v tuyn chn, bi dng hc sinh gii ca i biu tham d
Hi ngh chc chn s l bi hc qu gi cho cc tnh khu vc duyn hi Nam Trung b v Ty
Nguyn trong cng tc pht trin hc sinh gii.
Cng tc pht hin, tuyn chn v bi dng hc sinh gii l mt nhim v c bit quan
trng trong vic thc hin nhim v chung v Gio dc v o to ca cc tnh. Nhng nm qua,
so vi mt s tnh c truyn thng khu vc ng bng Bc b v Bc Trung b v cc thnh
ph trc thuc Trung ng th kt qu v hc sinh gii Trung hc ph thng ca cc tnh khu
vc duyn hi Nam Trung b v Ty Nguyn vn cn khim tn, cn phi phn u nhiu hn
na. y l iu trn tr ca cn b qun l cng nh nhng gio vin trc tip tham gia bi
dng hc sinh gii. V vy, vic t chc Hi ngh lin kt bi dng hc sinh gii cc tnh khu
vc duyn hi Nam Trung b v Ty Nguyn s l ng lc cc tnh hoch nh chin lc
pht trin hc sinh gii.
L n v t chc Hi ngh, S Gio dc v o to k Lk chn thnh cm n cc nh
khoa hc, nh gio lo thnh, chuyn gia u ngnh thuc b mn Ton hc v Ng vn; chuyn
gia gio dc, lnh o v cn b ph trch chuyn mn cc S Gio dc v o to; thy gio,
c gio tham gia v vit bi gp phn cho vic t chc thnh cng Hi ngh.

Bo co dn Hi tho khoa hc
Cc chuyn bi dng hc sinh gii
Khu vc Duyn hi Nam Trung b v Ty Nguyn
L Vn Thi
Hiu trng trng THPTC Nguyn Du, tnh k Lk
Knh tha qu v i biu, qu thy c gio !
Hm nay trong khng kh vui ti ca thnh ph Bun Ma Thut va t chc l k nim 110
nm ngy thnh lp v 40 nm ngy gii phng Bun Ma Thut. S Gio dc v o to k
Lk kt hp vi Hi Ton hc H Ni t chc hi tho cc chuyn bi dng hc sinh gii mn
Ton THPT khu vc Duyn hi Nam Trung b v Ty nguyn. Chng ti rt vui mng c n
tip lnh o cc S GD& T trong khu vc, cc chuyn gia b mn Ton, cc thy c l chuyn
vin ch o b mn Ton ca cc S , Nh trng v gio vin trc tip ging dy bi dng hc
sinh gii mn Ton n tham d hi tho.
Chng ta bit nhm to s lin kt bi dng trong khu vc t nm hc 2010 2011 S
Gio dc o to Ph Yn ng cai t chc Hi tho Khoa hc Cc chuyn Ton hc bi
dng hc sinh gii khu vc Duyn hi Nam Trung b v Ty nguyn ln th nht; sau l
n cc S Gio dc v o to Khnh Ha, Bnh nh, Gia Lai. Hm nay, ngy 14/3/2015 ti
trng THPTC Nguyn Du tnh k Lk din ra Hi tho Khoa hc ln th 5 do S Gio dc
v o to k Lk c tip tc ng cai t chc.
Trong nhng nm qua, c s quan tm ca lnh o B v cc S Gio dc v o to,
ca Vin nghin cu Ton cao cp, Hi Ton hc H Ni v ca ton x hi, vn bi dng
hc sinh gii mn Ton c ch trng v to nhiu iu kin thun li, cc gio vin v hc sinh
trong khu vc c nhiu n lc, c gng v quyt tm cao hn nn kt qu c nhiu khi sc
hn, c nhng hc sinh trong khu vc t gii quc t, t gii nht, nhiu gii nh mn Ton
v tt c cc S u c hc sinh gii quc gia mn Ton. Tuy nhin s lng v cht lng gii
cn thp so vi cc khu vc khc, s gii khng n nh, s lin kt trong bi dng c nhng
mang tnh cht ring l, khng n nh.
Mc tiu ca hi tho nhm xc nh v tm quan trng ca vic ging dy v bi dng hc
sinh gii mn Ton, tho lun mt s chuyn Ton hc do cc gio vin, chuyn gia trnh by,
tm ra cc gii php nng cao cht lng bi dng hc sinh gii trong khu vc. Ni dung ca
hi tho tp trung vo cc vn sau:
- Nghe bo co mt s chuyn Ton hc .
- Tho lun hon chnh cc bo co.
- Trao i cc kinh nghim bi dng hc sinh gii.
- nh hng cho ni dung Hi tho v Lin kt bi dng trong nm hc 2015 2016.
Hi tho Khoa hc nm nay c s quan tm rt ln ca Lnh o B v cc S Gio
dc v o to trong khu vc, Hi Ton hc H Ni, mt s chuyn gia u ngnh mn Ton, cc
trng THPT chuyn trong khu vc, cc trng THPT ca tnh k Lk v mt s tnh trong
khu vc, cc gio vin ang bi dng hc sinh gii mn Ton. Ban t chc hi tho nhn
c 28 chuyn ca cc gio vin thuc cc S Gio dc v o to Bnh nh, Khnh Ha,
Lm ng, Ninh Thun, k Nng, k Lk. c bit c 2 chuyn ca mt s hc sinh lp
10 chuyn Ton trng THPT Chuyn Nguyn Du, k Lk. Cc chuyn thuc cc lnh vc
khc nhau ca Ton hc nh: T hp; S hc; Hnh hc; Dy s; Bt ng thc; Phng trnh
hm; Cc phng php chng minh Ton hc;... Nhiu chuyn th hin s c gng ca tc
gi trong nghin cu, tng hp, c nhiu ng dng v c cp trong cc k thi chn hc sinh
gii cc cp.
Chng ti rt mong t cc bo co c th v nhng kin trao i thng nht trong hi tho,
6

cng nh nhng kin ng gp ca cc chuyn gia, cc lnh o s gip chng ta t c cc


mc tiu ca hi tho nm nay.
Cui cng chng ti xin chn thnh cm n Hi Ton hc H Ni; Lnh o cc S Gio dc
v o to trong khu vc; qu v i biu; qu thy c gio tham d hi ngh v mong c qu
v b qua nhng thiu st ca chng ti trong qu trnh t chc hi tho. Knh chc qu v t
c nhiu thnh cng, nhiu nim vui v sc khe. Knh chc hi tho thnh cng tt p.

Ton hc v thut ton


Nguyn Tin Dng, i hc Toulouse, Php
Trn Nam Dng, i hc KHTN, HQG Tp. HCM

Li gii thiu

Nguyn Tin Dng l gio s H Toulouse, Php. Thi cn hc sinh, Nguyn Tin Dng
tng tham gia k thi IMO 1985 v t huy chng vng khi mi 15 tui. GS Nguyn Tin Dng
c nhiu bi vit v ton hc v ging dy ton hc ng trn trang web c nhn zung.zetamu.net.
Trn Nam Dng l ging vin trng H KHTN Tp HCM. Thi cn hc sinh, Trn Nam
Dng tng tham gia k thi IMO 1983 v t huy chng bc khi mi 17 tui. TS Trn Nam
Dng c nhiu bi vit v cc chuyn Olympic Ton v l ngi khi xng v t chc cc hot
ng cho phong tro chuyn ton cc tnh pha Nam.
Nguyn Tin Dng v Trn Nam Dng u hc i hc ti trng HTH Matxcova mang tn
L-m-n-xp v l ngi c GS Nguyn Vn Mu dn dt qua nhiu thi k.
Bi vit ny l bi vit chung ca hai Dng dnh tng cho GS Nguyn Vn Mu v Hi tho.
Trong cuc sng v trong cng vic, chng ta hay gp phi nhng tnh hung l, nhng tnh
hung m chng ta mi gp ln u, cha bit trc cch gii quyt. Nhng tnh hung gy
kh khn cho chng ta, i hi chng ta phi suy ngh v nhng vn l. Nhng ng thi
cng chnh l c hi cho chng ta vn ln v khm ph c nhng ci mi, cn nhng cng
vic lp i lp li c my mc dn dn lm thay cho con ngi.
gii quyt mt vn m chng ta gp phi, chng ta cn tm ra mt cch gii, hay c th
gi l mt thut ton i n li gii. Trong cc mn hc nh trng, th mn ton chnh l
mn tt nht dy v hc v cch tip cn thut ton i vi cc vn . Nu nh qua vic hc
ton m hc sinh hiu c bn cht cc thut ton quan trng nht p dng trong cc tnh
hung khc nhau trong cuc sng v cng vic, th c th coi y l s thnh cng ca bn thn
mn ton trong gio dc. Rt tic rng, iu ny cn rt lu mi tr thnh hin thc. Vo thi
im hin ti, trn th gii, kha cnh thut ton vn cn t c quan tm ti trong vic dy v
hc ton.
Mt v d in hnh cho khng nh trn chnh l k thi ton olympic IMO 2014 va qua.
bi ca IMO 2014 khng h rm ri phc tp, c bit nu so vi thi HSG ca Vit Nam. V
ch cn kin thc ton PTCS ch cng khng cn n kin thc PTTH c th hiu bi v
li gii ca ton b thi. Th nhng, im thi nm nay tng i thp so vi cc nm khc,
nn ch cn c 29/42 im (tc l ch cn lm c 4/6 bi v vit c mt hai g cho 2
bi cn li) l c huy chng vng. Mt s bi nm nay kh, khng phi l v chng phc tp,
m l v chng l, khin cho cc th sinh ng ngng khng bit lm th no. Chng ti s ly bi
s 5 v bi s 6 ca IMO 2014 lm v d, chnh l hai bi m on Vit Nam c t im.
Nhng trc khi bc vo cc bi ton ny, ta s xem xt qua mt s v d m u, n gin
v d hiu hn.

Cc v d

Bi ton 1. C mt s hn , mi hn nng khng qu 0,5 kg. Tng trng lng ca chng


khng qu 2,5 kg. Chng minh rng c th chia cc hn ny thnh 4 nhm, mi nhm c tng

trng lng khng ln hn 1 kg. Kt lun bi ton cn ng khng nu ta thay 4 nhm bng 3
nhm?
iu kh chu trong bi ton ny l ta khng bit c bao nhiu hn , v trng lng ca cc
hn . Cu hnh l kh lng lo vi cc iu kin dng bt ng thc. Ta nhn thy rng,
iu kin hn nng khng qu 0,5 kg s suy ra tng trng lng hai hn bt k khng qu
1 kg.
Nh vy, nu s hn 8 th kt lun bi ton l hin nhin (ta c th chia thnh 4 nhm,
mi nhm c 1 hoc 2 hn ). Th nu c 9 s th sao? Lc ny nu chia lm 4 nhm, s c mt
nhm c t nht 3 hn , v tng trng lng 3 hn ny c th khng cn 1 kg. Nhng nu
ta chn c 3 hn c tng trng lng 1 kg th trng hp ny gii quyt xong, v lc
6 hn cn li c th chia thnh 3 nhm, mi nhm 2, c tng trng lng hin nhin 1 kg.
Lm sao tm c 3 hn c tng trng lng 1 kg? Mt cch t nhin, ta chn 3 hn
c trng lng nh nht v chng minh tng trng lng ba hn ny khng ln hn 1 kg.
iu ny l kh hin nhin v tng trng lng 3 hn nh nht bao gi cng khng ln hn
1/3 tng ca c 9 hn . Ta gii quyt xong trng hp c 9 hn .
Nu c 10 s th sao?
Lc ny c th c n 2 nhm c 3 hn . V ta khng chc l c th chn sao cho c hai
nhm 3 vin ny u c tng trng lng 1 kg. Lm th no a v trng hp 9 hn ?
n y, c mt tng mang tnh t ph ny sinh: V c 10 hn , ta chia thnh 5 cp th
t c 1 cp c tng trng lng 0, 5 kg. Ta ct hai hn ny lm mt v a bi ton v bi
ton 9 hn . V tng ny c th p dng c mt li gii hon chnh nh sau:
a) Nu c hai hn no c tng trng lt 0, 5 kg th ta ct hai hn li. Sau qu
trnh thay nh th, ta c mt s hn , mi hn c trng lng 0, 5, tng trng lng
cc hn 2, 5 kg v tng hai hn bt k > 0,5. Ta chng minh vi cc hn nh vy ta
lun chia c thnh 4 nhm tha mn yu cu bi, v nh th cc hn ban u cng chia
c.
Vi tnh cht tng trng lng hai hn bt k > 0,5 ta thy s cc hn khng qu 9.
Nu s cc hn l 8 tr xung th cch chia l hin nhin: ta chia thnh 4 nhm, mi nhm 1
hoc 2 hn . Vy ta ch cn xt TH c 9 hn .
Gi s trng lng 9 hn l
a1 a2 a3 ... a9 .
Ta thy
a1 + a2 + a3 (a1 + .. + a9 )/3 < 1.
Nh vy ta c th chia 9 hn ny thnh 4 nhm c tng trng lng tng ng l:
a1 + a2 + a3 , a4 + a5 , a6 + a7 , a8 + a9
tha mn iu kin.
b) Kt lun khng ng: Ta c th chn 7 hn c trng lng u bng 5/14 kg. Nu chia
thnh 3 nhm, s c 1 nhm cha t nht 3 hn , tng trng lng ca chng s bng 15/14 > 1
(kg).
T duy thut ton trong bi ton ny rt r rng: t nhiu hn , ta "thu gn" li thnh mt
b mi vn tha mn cc tnh cht bi nhng c thm tnh cht b sung (tng trng lng
hai hn bt k > 0, 5) v t tm ra li gii. Chnh tng thu gn ny cng c p dng
hiu qu trong li gii bi ton 5 ca IMO 2014 m ta s cp n phn sau.
Bi ton 2. Tng ca 50 s nguyn dng bng 99. Chng minh rng vi mi s nguyn dng
S nh hn 100 tm c mt s s trong cc s cho c tng bng S.
9

C gng gii trc tip s dn n vic xt kh nhiu trng hp. Mi lin h gia cc con s
50 v 99 gi chng ta n mt cch tip cn rt quen thuc trong cc bi ton s dng thut
ton: t bi ton tng qut ri gii n, s dng t duy quy np.
Ta chng minh bng quy np mnh mnh : T n s nguyn dng c tng bng 2n 1 c
th chn ra mt s hoc mt vi s c tng l mt gi tr bt k t 1 n 2n 1.
Gi s mnh ng n n. Xt n + 1 s nguyn dng c tng bng 2n + 1. Trong cc
s , s c mt s s bng 1 (c t nht 1 s). Ta loi nhng s ny ra. Trong cc s cn li, gi
k l s nh nht. Nu k = 2 th ta loi 1 s 2 i, cn li n s c tng bng 2n 1. Theo GTQN
th cc tng c gi tr t 1 n 2n 1 l "chn c". c tng bng 2n, ta ch cn loi 1 s 1.
Vy l TH ny n. Nu k = 3 th do trung bnh ca tt c cc s < 2 nn phi c t nht 2 s 1
na. Ta loi i 1 s 1, 1 s 3 th cn li n 1 s c tng bng 2n 3. Nh vy cc tng c gi tr
t 1 n 2n 3 u chn c. c 2n 2 th ta thm 1 vo, c 2n 1 th thm 3 vo b 2
s 1 i, c 2n v 2n + 1 li ln lt thm cc s 1 vo.
Ta c th p dng l lun ny cho k bt k. Vi k bt k, trung bnh ca tt c cc s < 2 nn
phi c t nht k 1 s 1 na. Ta b i s k v k 2 s 1 th cn li n (k 1) s c tng bng
2n 1 (k + k 2) = 2(n k + 1) 1. Theo gi thit quy np, cc tng t 1 n 2(n k + 1) 1
u chn c. Ta ln lt cho thm cc s 1 vo th s c tip cc tng 2(n k + 1) n
2(n k + 1) + k 3. Tip theo, ta ly k 1 s 1 trong tng 2(n k + 1) + k 3 ra v thm k
vo th c tng 2(n k + 1) + k 2, ri ln lt cho cc s 1 thm vo c cc tng cho
n 2n 1.
Bi ton 3. Hai bn An v Bnh chi tr chi bc ko. Ban u trn bn c 25 vin ko. Bt
u t An, hai bn lun phin nhau bc ko, mi ln c php bc 1, 2 hoc 3 vin. n khi
ht ko trn bn ai bc c tng cng mt s chn vin ko s thng. Hi ai l ngi c chin
thut thng nu c hai cng chi ng?
gii cc bi ton kiu ny, chng ta c mt cch tip cn chung l lm t di ln, tc l
xt bi ton vi s ko nh hn. Do tnh cht ca bi ton, bi ton s c ngha nu s vin ko
ban u l l.
V d nu trn bn c 1 vin ko, 3 vin ko, 5 vin ko th sao.
R rng nu trn bn c 1 vin ko th An s phi bc v s thua. Nu l 3 th An s bc 2
vin v thng. Cn nu c 5 vin ko th sao?
An c 3 phng n bc: Nu An bc 2 hoc 3 vin th Bnh s bc 2 vin v thng. Nhng
nu An bc 1 vin th cho d Bnh bc th no, An cng thng:
- Bnh bc 1, An bc 3
- Bnh bc 2, An bc 1
- Bnh bc 3, An bc 1
Xt n trng hp c 7 vin ko. An cng c 3 cch bc, tng ng 1 vin, 2 vin, 3 vin.
- Nu An bc 1 vin th Bnh bc 1 vin v Bnh s thng, v nu tip theo An bc 1 hoc 2
vin th Bnh bc 3, An bc 3 vin th Bnh bc 1.
- Nu An bc 2 vin th Bnh bc 1 vin v Bnh s thng (1, 3), (2, 1), (3, 1).
- Nhng nu An bc 3 vin th An thng, v nu Bnh bc 1 th An bc 3, Bnh bc 2 th An
bc 1, Bnh bc 3 th An bc 1.
C nh vy, ta c th xy dng ln. Tuy nhin, c th thy r mt lin h quy nh sau.
Ta quy c nh s cc trng thi bng 2 tham s (X, Y ) vi X l tnh chn l ca s vin ko
mnh c v Y l s vin ko cn li khi n lt mnh bc. V d, vi An lc bt u cuc chi
vi bi ton ban u c 5 vin ko th trng thi l (C, 5). T trng thi ny, bng cch bc, An
s t Bnh vo 1 trong 3 trng thi (C, 4), (C, 3), (C, 2). Cc trng thi (C, 3) v (C, 2) l trng
thi thng (Bnh ch vic bc 2 vin), cn (C, 4) l trng thi thua (v Bnh bc 1 An s bc 3,
Bnh bc 2, 3 An s bc 1). Do An c quyn chn nn An s a v (C, 4). Nh vy, mt trng
10

thi s l trng thi thng nu c 1 cch a v trng thi thua (cho i phng), cn 1 trng
thi s l thua nu mi cch i u a v trng thi thng cho i phng.
Cui cng, ch l do tng s vin ko ban u l l, nn t trng thi (C, 5) s chuyn v trng
thi (C, 4), (C, 3), (C, 2), nhng t trng thng (C, 6) s chuyn v trng thi (L, 5), (L, 4), (L, 3).
T trng thi (L, 5) s chuyn v cc trng thi (L, 4), (L, 3), (L, 2).
T y, ta d dng xy dng c bng cc gi tr ca cc trng thi (1 tng ng A thng,
0 tng ng Bnh thng)

C
L

1
0
1

2
1
1

3
1
1

4
0
1

5
1
0

6
1
1

7
1
1

8
1
0

9
0
1

10
1
1

11
1
1

12
0
1

13
1
0

14
1
1

15
1
1

16
1
0

17
0
1

18
1
1

19
1
1

20
0
1

Tnh tip ta tm c (C, 25) = 0, tc l An thua, Bnh thng.


By gi, sau khi lm quen vi cch tip cn thut ton, ta s xem xt n phn tch hng
gii cho hai bi ton IMO 2014.
Bi ton 4 (IMO 2014). Bi s 5 ca IMO 2014 nh sau:
C mt ng ng xu c tng mnh gi nh hn hoc bng 99,5, sao cho mnh gi ca ng
xu no cng c dng 1/n vi n l s t nhin (cc ng xu khc nhau c th c mnh gi khc
nhau). Chng minh rng ta c th chia c ng ng xu thnh khng qu 100 ti sao cho tng
mnh gi ca mi ti khng vt qu 1.
Lm sao tip cn mt bi ton l hoc nh trn, vi gi s l bn khng trng t, cha
tng lm mt bi rt tng t nh th? Mt s phng php tip cn chung gm 4 bc sau (lp
di lp li nu cn thit): quan st, suy lun, tnh ton, kim tra. Trong cc bc quan st v l
lun c vic quan st cc tng quan trong bi ton, lm th cc trng hp ring, da vo cc
trng hp ring a ra cc gi thuyt lin quan ri xt cc gi thuyt , bin i n gin
ha bi ton, a v cc dng bi ton quen thuc hn hay d gii hn. Nhiu khi, tng qut ha
cng chnh l mt cch n gin ha. Nu l vn ln, phc tp, th phi chia tr, tc l
lm sao chia n ra thnh cc bc nh hn, gii quyt tng bc d dng hn l gii quyt
ton b vn .
C mt anh bn ca ti, l chuyn gia my tnh, c ni vi ti mt cu t cch y hn 20
nm m n by gi ti vn rt tm c: khi c giao mt nhim v lp trnh, lp trnh vin
no m ngi vo h hi vit chng trnh ngay th l lp trnh vin ti. Lp trnh vin tt trc
khi vit chng trnh th phi quan st, suy ngh . Trong ton hay trong nhiu lnh vc khc
cng vy, trc khi b ci cn quan st v suy lun hnh dung vn .
Trong trng hp bi ton s 5 trn, th c th quan st ngay l s 99,5 gn vi s 100.
T ta c gi thuyt l nu thay 99,5 v 100 bi N 1/2 v N trong N l s t nhin
bt k th vn ng. Nu nh bi ton ch ng khi N = 100 ch khng ng vi N khc, khi
th bi ton chc l kh phc tp, nhng y l thi hc sinh c th lm trong vng mt
vi ting, khng qu phc tp c, nn gi thuyt l bi ton ng vi N bt k l gi thuyt
tng i kh d. Vic thay 100 bng N ty ny cho php chng ta gi s l khng c ng xu
no c mnh gi 1. Tht vy, nu c m ng c mnh gi 1 th cho cc ng vo m ti, cn li
N - m ti v s tin N m 1/2.
Mt bc l lun thng dng quan trng l: ta c th gii quyt vn mt cch ngy th
nht, xem n tc ch no, kh ch no. y c ngha l, ta c th b dn tin vo 100
ti, cho n bao gi b c ht hoc b tc khng b c na th thi. Nhng khi no th c
th b tc? l khi cn 1 ng tin m c b vo ti no cng khin cho mnh gi ca ti
(N 1/2)
1
1
= 1
. iu c
vt qu 1. V c N ti m tin N , nn c ti c tin
2
N
2N
11

ngha l, nu c ng tin cn li khng b c vo ti no, th mnh gi ca n phi > 1/2N .


Cch gii quyt ngy th ny cho ta quan st sau, lm n gin vn : ta c th gi s tt
c cc ng tin c mnh gi > 1/2N , ch khng cn quan tm n cc ng tin c mnh gi
1/2N na (cc ng c mnh gi 1/2N lun c th b vo, sau khi gii quyt cc ng
c mnh gi > 1/2N ).
Mt bc l lun thng dng quan trng khc l: lm sao n gin ha bi ton, a v nhng
trng hp n gin nht c th (gi l phng php thu gn trong ton).
y, c th hiu l c t xu th n gin, cng c nhiu xu th phc tp hn. Nh vy, ta c
th tm cch lm gim s ng xu. C 1 cch hin nhin l: nu c th i 2 hay nhiu ng xu
trong ng ly 1 ng xu khc (c mnh gi bng tng mnh gi ca cc ng kia), th ta n
gin ha c ng ng xu: nu gii c sau khi i, th sau khi gii xong ta c th i li
c li gii cho ng ng xu ban u. C i xu n gin ha nh vy, ta a c bi
ton v trng hp m khng cn i xu c na.
n y, bi ton n gin ha i ng k, v ta ch cn cn xt trng hp sau: tng mnh
1
gi N , khng c ng xu no c mnh gi bng 1, khng c mt nhm cc ng xu no c
2
1
th i thnh 1 ng xu khc, v khng c ng xu no c mnh gi <
. Tt c cc trng
2N
hp cn li u suy gin v c trng hp ny. By gi li quan st tip xem trng hp cui
cng ny ra sao.
1
v
Ta s quan st thy l vi mi s t nhin k th khng c qu 1 ng xu c mnh gi
2k
1
khng c qu (2k 2) ng xu c mnh gi
(v sao th?). T c cch xp xu vo ti
2k 1
1
1
nh sau: ti k cha cc ng xu mnh gi
v
, vi k = 1, . . . , N . (Bn c hy t
(2k 1)
2k
kim tra rng cch ny thc hin c).
Ti vit li gii cho bi s 5 pha trn kh di dng gii thch qu trnh suy ngh dn n li
gii m bn thn ti s dng khi th lm bi , cn tt nhin khi vit thnh li gii trn tra
th c th vit ngn hn nhiu. Nhn t quan im thut ton, bi s 5 ny tht ra kh n gin,
v ch sau vi bc lc gin ha d thy l a v c trng hp chia c ngay thnh cc
ti. Bn no hc v lp trnh my tnh chc s ngh ngay c cch vit chng trnh chia xu da
trn cc bc pha trn. Rt tic l on VN ch c 2 bn gi c bi ny.
Bi ton 5 (IMO 2014). Bi s 6 ca IMO 2014 th cn l hn bi s 5 na, v khng c bn
VN no gii c, tuy c mt bn c 3/7 im. Cc on khc cng rng gn ht bi ny.
bi c th pht biu nh sau:
Gi s c n ng thng trn mt phng, sao cho khng c 2 ng no song song v khng
c 3 ng no ng qui. CMR c th t t nht ng bng mu xanh, sao cho khng c min
b chn no trn mt phng c bin ton l mu xanh.

Con s n trong bi c l l con s gy hoang mang, bi thot nhn chng bit n t u


n(n 1)
(n 1)(n 2)
, cn s cc min b chn th l
, tc l
ra. S cc im ct nhau th l
2
2
u so c vi n2 , ch chng c ci g trn mt phng c s lng dng . Nhng hoang mang
nh vy c th nh hng xu n tm l, khin vic tm li gii tr nn kh khn hn. B qua
cc hoang mang, ta c th lm mt thut ton ngy th gii quyt vn t mu, xem n
s gp kh khn b tc u. Thut ton ngy th l: u tin ta t mt ng bt k mu
xanh. Sau c cn t c thm ng mu xanh (sao cho khng c min b chn no trn
mt phng c bin ton l mu xanh) th t tip, cn n lc khng t c thm na th dng
li. Gi s ng t c lc dng li l k. Nu thut ton n gin ny m tt, th tc l ta c
k 2 n. Nh vy, nu ta chng minh dc k 2 n th bi ton c gii quyt.
12

Mt phng php chng minh thng dng l dng phn chng. Tc l ta s chng minh rng,
nu t c k ng, v n > k 2 , th cn t thm c ng na. chng minh iu , ch
cn chng minh rng s ng cm khng vt qu k 2 k = k(k 1). ng cm tc l ng m
nu t n th s c min b chn vi bin ton mu xanh. By gi li vn dng nhn xt sau: s
(k 1)
, nn nu ta lm sao thit lp quan
im nt xanh (= giao im ca cc ng xanh) l
2
h c gia cc ng cm v cc nt xanh, kiu ng cm no cng cho bi nt xanh, v mi
nt xanh khng cho qu 2 ng xanh th l xong. tng l nh vy. i vo c th hn, th
quan h s l: mi ng cm ng vi t nht 2 on cm (1 on cm l mt on mu xanh
i t 1 nt xanh n ng cm), cn t mi nt xanh ch i ra c nhiu nht l 4 on cm
thi. V li gii n y gn nh l kt thc (bn c c th t vit li li gii hon chnh, s
khng qu 1 trang giy).
y l chuyn thi ton quc t. Quay li chuyn dy ton v hc ton bnh thng.
Nm nay, ti c dy ton cho mt lp i hc nm th nht chuyn ngnh tin hc, thay cho
mt ng nghip ngh sinh con. Mt s bn sinh vin phn nn vi ti l chng trnh ton chn
qu, chng thy lin quan g n tin hc. M ng th tht, ti c chng trnh cng thy chn.
Nhng ti ch l lnh nh thu ch khng son chng trnh nn khng thay i c n.
Ci m ti c gng lm b tr li l gii thch thm cho cc sinh vin nhng khi nim
m h hc chnh l cc thut ton c ngha ra sao. V d nh, cng thc ni suy Lagrange chnh
l thut ton v ng cong n gin nht, p mt nht i qua mt s im cho trc, v
khi ngi ta thit k t, tu bay th thay v v ton b cc im ngi ta ch cn v t im thi
ri ni suy ra cc im cn li theo kiu cng thc . Hay nh chui Taylor lm g? N cng
chnh l thut ton cho php chng ta tnh xp x mt cch rt chnh xc cc gi tr ca cc hm
phc tp, v.v. V d nh, ta c th tnh nhm bng tay cc s nh e, sin(1), v.v. vi chnh xc
rt cao m khng cn my tnh (my tnh n cng dng cc thut ton nh ta tnh nhm thi),
v.v.
Khi hc, sinh sinh vin (c bit l sinh vin tin hc) c gii thch ngha thut ton ca
cc khi nim ton hc, th h cng phn khi ln nhiu. i vi sinh vin sinh vt, ha hc, v.v.
th c th cn dy hi khc i, vi nhiu minh ha ng dng ton hc trong ngnh ca h hn,
v d nh ti sao li dng hm ly tha khi kho st dn s, v.v.
Cui cng th ton hc vn l mt tng hp cc cng c m hnh ha v cc thut ton
gii quyt cc vn ny sinh trong mi lnh vc, ch khng phi l mt m khi nim t trn
tri ri xung v xa vi thc t.

13

a thc bt kh quy v mt vi
vn dng lin quan
m Vn Nh, Trng HSP H Ni

1
1.1

a thc
Mt vi tiu chun v bt kh quy

Gi s hai a thc f (x), g(x) R[x]. a thc f (x) c gi l chia ht cho a thc g(x) nu
c a thc h(x) R[x] f (x) = g(x)h(x). a thc p(x) R[x], deg p(x) > 0, c gi l mt
a thc bt kh quy trn R nu n khng th biu din thnh tch hai a thc p(x) = p1 (x).p2 (x)
vi p1 (x), p2 (x) R[x] v deg p1 (x), deg p2 (x) > 0.
p
B 1. Cho a thc f (x) = a0 xn + a1 xn1 + + an Z[x], a0 6= 0. Nu s hu t
vi
q
(p, q) = 1 l nghim ca phng trnh f (x) = 0 th
(1) p l mt c ca an v q l mt c ca a0 .
(2) p mq l mt c ca f (m) cho mi s nguyn m.
p
Chng minh. (1) Gi s s hu t vi (p, q) = 1 l nghim ca f (x) = 0. Khi
q
a0 pn + a1 pn1 q + + an q n = 0.
V (p, q) = 1 nn p l mt c ca an v q l mt c ca a0 .
(2) Khai trin f (x) theo cc lu tha ca x m ta c
f (x) = a0 (x m)n + b1 (x m)n1 + + bn1 (x m) + f (m) Z[x].
p
v quy ng a0 (p mq)n + b1 (p mq)n1 q + + bn1 (p mq)q n1 + f (m)q n = 0. V
q
(p, q) = 1 nn p mq l mt c ca f (m) cho mi s nguyn m.
Cho x =

H qu 1. Nghim hu t ca a thc f (x) = xn + a1 xn1 + + an Z[x] phi l s nguyn.


Chng minh. Suy ra t b trn.
V C l trng ng i s nn a thc bt kh quy mt n trn C ch l nhng a thc bc
1. Chnh v l do ny m ta ch xt a thc bt kh quy trn Q v trn R. Mt s tiu chun sau
y c th kim tra khi no mt a thc vi cc h s nguyn l bt kh quy.
nh l 1 (Tiu chun Eisenstein). Cho f (x) = an xn + an1 xn1 + + a0 , an 6= 0, l a thc vi
cc h s nguyn v p l s nguyn t sao cho an khng chia ht cho p v cc ai , i < n, chia ht
cho p nhng a0 khng chia ht cho p2 . Khi f (x) l a thc bt kh qui trn Z.
Chng minh. Gi s f = gh = (

r
P
i=0

bi xi )(

s
P

cj xj ) vi g, h Z[x] v r = deg g, s = deg h >

j=0

0, r + s = n. V b0 c0 = a0 chia ht cho p nn ti thiu mt s b0 hoc c0 phi chia ht cho p, chng


hn b0 chia ht cho p. V a0 khng chia ht cho p2 nn c0 khng chia ht cho p. Nu tt c cc bi
u chia ht cho p th an cng phi chia ht cho p : mu thun vi gi thit. Vy phi c mt bi
khng chia ht cho p. Gi i l ch s nh nht bi khng chia ht cho p. Khi 0 < i 6 r. V
ai = bi c0 + bi1 c1 + + b0 ci chia ht cho p vi tt c cc s hng bi1 c1 , . . . , b0 ci u chia ht cho
p nn bi c0 cng chia ht cho p : mu thun. iu ny chng t f l a thc bt kh qui trn Z.
14

x2
xn
V d 1. Vi bt k s nguyn dng n a thc f (x) = 1 + x +
++
l bt kh quy trn
2!
n!
Q.
x2
Bi gii. Ta phi chng minh n!f (x) = n! + n!x +
+ + xn l bt kh qui trn Z. Ta chn
2!
s nguyn t p vi p 6 n < 2p v n chia ht cho p, nhng n! khng chia ht cho p2 . Theo tiu
chun Eisenstein, a thc n!f l bt kh qui trn Z.
V d 2. Vi bt k s nguyn t p a thc f (x) = 1 + x + + xp1 l bt kh quy trn Z.


p
Bi gii. Theo Tiu chun Eisenstein, a thc f (x + 1) = xp1 + p1 xp2 + + p1
l bt
kh qui trn Z. Do f l bt kh qui trn Z.
nh l 2. Cho f (x) = b0 xn + b1 xn1 + + bn l a thc vi cc h s nguyn v p l s nguyn
t sao cho b0 khng chia ht cho p nhng bk+1 , . . . , bn chia ht cho p, bn khng chia ht cho p2 .
Khi f (x) c nhn t bt kh qui bc > n k.
Chng minh. Phn tch f (x) thnh tch cc nhn t bt kh quy. Gi s g(x) = c0 xm +c1 xm1 +
+ cm Z[x] l mt nhn t bt kh quy vi cm chia ht cho p. Biu din f (x) = g(x)h(x) vi
h(x) = d0 xh + d1 xh1 + + dh Z[x]. Khi dh khng chia ht cho p. Gi ci l h s u tin
ca g(x) khng chia ht cho p trong khi bm , . . . , bi+1 chia ht cho p. Ta c cm dh = bn chia ht cho
p, nhng khng chia ht cho p2 . V bh+i = ci dh + bi+1 dh1 + khng chia ht cho p nn h + i 6 k
hay n m + i 6 k. Do m > n + i k > n k.
Ngoi ra ta cn c mt vi tiu chun kim tra tnh bt kh quy ca a thc vi cc h s nguyn
khc na qua bt ng thc.
nh l 3 (Tiu chun Osada). Cho f (x) = xn + a1 xn1 + + an1 x p l a thc vi cc h
s nguyn v p l s nguyn t. Nu p > 1 + |a1 | + + |an1 | th f (x) l bt kh qui.
Chng minh. Gi s f (x) l kh qui. Khi f (x) = g(x)h(x), g, h l nhng a thc bc
dng vi cc h s nguyn. V p l s nguyn t nn mt trong cc s hng t do ca g hay h
phi bng 1, chng hn h s t do ca g bng 1. Vy gi tr tuyt i ca tch cc nghim
ca g phi bng 1. Khi g(x) = 0 phi c mt nghim vi || 6 1. V cng l nghim ca
f (x) = 0 nn
p = |n + a1 n1 + + an1 | 6 1 + |a1 | + + |an1 |.
iu mu thun ny chng t f (x) l bt kh qui.
V d 3. Vi s t nhin n > 2, a thc q(x) = xn 18xn1 + 3x2 2011 lun lun l bt kh
quy.
Bi gii. V 2011 l s nguyn t v 2011 > 1 + 18 + 3 nn q(x) l bt kh qui theo nh l 3.
V d 4. a thc p(x) = x9 + x8 + + x2 + x + 11 lun lun l bt kh quy.
Bi gii. V 11 l s nguyn t v 11 > 1 + 1 + + 1 = 10 nn p(x) l bt kh qui theo nh
l 3.
n+1
].
nh l 4 (Tiu chun Polya). Cho f (x) l a thc bc n vi cc h s nguyn. t m = [
2
m!
Gi s cho n s nguyn khc nhau d1 , . . . , dn c |f (di )| < m v cc s di u khng l nghim
2
ca f (x). Khi f (x) l bt kh qui.

15

Chng minh. Gi s f (x) l mt a thc kh qui. Khi ta c biu din f (x) = g(x)h(x) vi
g, h l nhng a thc bc dng, cc h s nguyn. Hin nhin deg g(x), deg h(x) < n. Khng hn
ch ta c th gi thit deg h(x) 6 deg g(x) = s. Ta c m 6 s < n. Ta thy ngay g(di ) 6= 0 v g(di )
chia ht f (di ). Do
m!
|g(di )| 6 |f (di )| < m .
2
s!
s!
m!
Ta c i |g(di )| > s . V s > m nn s > m , (chng minh). Vy
2
2
2
m!
s!
> |g(di )| > s .
m
2
2
Mu thun ny ch ra f (x) l mt a thc bt kh qui.

V d 5 (VMO 1984). Xc nh a thc bt kh quy f (x) Z[x] nhn 2 + 3 3 lm mt nghim.

Bi gii. t x = 2 + 3 3. Khi 3 = (x 2)3 = x3 + 6x (3x2 2) 2. Nh vy


2
f (x) = (x3 + 6x 3)2 2(3x
+
2)2 = x6 6x4 6x3 + 12x2 36x + 1 l a thc bc 6 vi cc

h s nguyn nhn x1 = 2 + 3 3 lm mt nghim. Vi p = 2, d dng kim tra f (x + 1) l bt


kh quy theo nh l 1. Do
a thc
vi cc h s nguyn x6 6x4 6x3 + 12x2 36x + 1 l

3
bt kh quy, bc 6, nhn x1 = 2 + 3 lm mt nghim.
V d 6. Tm tt c cc cp (n, r) vi s t nhin n > 1 v s thc r a thc p(x) = (x+1)n r
chia ht cho 2x2 + 2x + 1.
1 + i
. Vy p(x) = (x+1)n r
Bi gii. a thc 2x2 +2x+1 l bt kh quy vi nghim phc =
2
1+i n
1
n
n
chia ht cho 2x2 +2x+1 khi v ch khi (+1)n = r. Do vy r = (
) = (cos
+i sin
).
n
2
4
4
2
n
= 0 hay n = 4k vi k N+ .
r l s thc cn v sin
4
V d 7. Vi cc s nguyn phn bit a1 , a2 , . . . , an , k hiu a thc f (x) = (xa1 )(xa2 ) . . . (x
an ). Nu n > 7 v ax2 + bx + 1 Z[x], a 6= 0, l bt kh quy th af (x)2 + bf (x) + 1 cng l bt
kh quy.
Bi gii. Trc tin ta chng minh nhn xt sau: Nu a thc g(x) Z[x] nhn gi tr 1, (hoc
-1), ti nhiu hn ba gi tr nguyn phn bit ca bin x th n khng th nhn gi tr -1, (hoc
1), khi x nguyn.
Tht vy, gi s g(x) Z[x] nhn gi tr 1 ti nhiu hn ba gi tr nguyn phn bit ca bin x.
Khi g(x) 1 c t nht 4 nghim nguyn khc nhau, chng hn a1 , a2 , a3 , a4 . Biu din
g(x) 1 = (x a1 )(x a2 )(x a3 )(x a4 )h(x), h(x) Z[x].
Vi a nguyn v a 6= ak , k = 1, 2, 3, 4, th (a a1 )(a a2 )(a a3 )(a a4 ) l tch 4 s nguyn
phn bit. Cc tha s ny c th l 1 v p. Nu c s nguyn a g(a) = 1 th 2 =
(a a1 )(a a2 )(a a3 )(a a4 )h(a). 2 = (1).1.2 cng lm ch c ba nhn t phn bit. Vy,
khng c s nguyn a g(a) = 1.
S dng kt qu ny vo chng minh bi ton t ra. Gi s c s phn tch af (x)2 + bf (x) + 1 =
g(x)h(x) vi g(x), h(x) Z[x] v deg g(x), deg h(x) > 1. V g(ai )h(ai ) = 1 vi mi i = 1, . . . , n nn
g(ai ) = 1 vi i = 1, . . . , n. Do n > 7 v nhn xt trn nn hoc g(ai ) = 1 hoc g(ai ) = 1 vi
mi i = 1, 2, . . . , n. Tng t i vi h(x). Chng hn g(x) = 1 + f (x). Khi h(x) = 1 + f (x).
Ta c = a. V af (x)2 + bf (x) + 1 = [1 + f (x)][1 + f (x)] nn ax2 + bx + 1 = [1 + x][1 + x] :
mu thun v ax2 + bx + 1 l bt kh quy. Vy af (x)2 + bf (x) + 1 l bt kh quy.
16

V d 8. a thc p(x) = x2010 + 318x1952 + 2011 khng th phn tch c thnh tch hai a thc
vi bc > 1 v cc h s nguyn.
Bi gii. a thc p(x) = x2010 + 318x1952 + 2011 l bt kh quy theo Tiu chun Osada, nh
l 3. Vy khng th phn tch a thc p(x) = x2010 + 318x1952 + 2011 ra thnh tch hai a thc
vi bc > 1 v cc h s nguyn.

V d 9. Vi s nguyn dng n v gc c x2n 2xn cos + 1 = (x2 2x cos + 1) (x2


n
2 +
(2n

2)
+

2x cos
+ 1) (x2 2x cos
+ 1) khi cos 6= 1.
n
n
Bi gii. Xt phng trnh xn = cos i sin . Hai phng trnh c 2n nghim x. Vy x2n

2 +
(2n 2) +
2xn cos + 1 = (x2 2x cos + 1)(x2 2x cos
+ 1) (x2 2x cos
+ 1).
n
n
n

V d 10. Chng minh rng khng tn ti hai a thc f (x), g(x) R[x] tha mn : 3 x3 + 2x + 2 =
f (x)
.
g(x)

f (x)
Bi gii. Gi s tn ti f (x), g(x) R[x] tha mn : 3 x3 + 2x + 2 =
. Ta c th chn
g(x)


f (x), g(x) = 1. V g(x)3 x3 + 2x + 2 = f (x)3 . V x3 + 2x + 2 l bt kh quy nn f (x) chia

2
ht cho x3 + 2x + 2 hay f (x) = x3 + 2x + 2 h(x). Vy g(x)3 = x3 + 2x + 2 h(x)3 v nh th

g(x) : x3 + 2x + 2 : mu thun vi vic chn f (x), g(x) = 1.
Mnh 1 (Tiu chun Perron). Gi s f (x) = xn + an1 xn1 + + a1 x + a0 Z[x] vi bc
n > 2, a0 6= 0. Nu |an1 | > 1 + |an2 | + + |a1 | + |a0 | th f (x) l a thc bt kh quy trn Z.
Chng minh. Trc tin ta ch ra f (x) c ng mt nghim vi mun || > 1. Gi s
f (x) = 0 c nghim . Khi an1 n1 = n +an2 n2 + +a1 x+a0 . Nu || = 1 th |an1 | 6
1 + |an2 | + + |a1 | + |a0 | : mu thun gi thit. Do || =
6 1. Gi s f (x) = 0 c cc nghim
phc 1 , 2 , . . . , n . Ta c |a0 | = |1 ||2 | . . . |n |. V |a0 | =
6 0 v nguyn nn |1 ||2 | . . . |n | > 1.
V |i | 6= 1 nn c i , chng hn: 1 vi |1 | > 1. t p(x) = xn1 + bn2 xn2 + + b1 x + b0
tha mn f (x) = (x 1 )p(x). Khi ta c h
a0 = 1 b0 , a1 = b0 1 b1 , . . . , an2 = bn3 1 bn2 , an1 = bn2 1
v c |bn2 1 | = |an1 | > 1 + |bn3 1 bn2 | + + |1 b0 |. Do vy
|bn2 | + |1 | > 1 + |1 ||bn2 | |bn3 | + + |1 ||b1 | |b0 | + |1 ||b0 |
v suy ra |1 | 1 > (|1 | 1)(|bn2 | + + |b0 |). V |1 | 1 > 0 nn ta nhn c |bn2 | + |bn3 | +
+ |b0 | < 1. Vi s phc , || > 1, c
|p()| = |n1 + bn2 n2 + + b1 + b0 |
> |n1 | |bn2 ||n2 | |b1 ||| |b0 |
> |n1 | |bn2 | |b1 | |b0 | > 0.
Nh vy, mi nghim ca p(x) u phi tha mn || < 1. Tm li, ta ch ra f (x) c ng
mt nghim vi mun || > 1.
Gi s f (x) l kh quy vi f (x) = g(x)h(x), trong g(x), h(x) Z[x] v deg g, deg h > 1. V
f (x) ch c mt nghim vi mun ln hn 1 nn mt trong hai a thc, chng hn h(x) ch c cc
nghim vi mun nh hn 1. Gi s 1 , . . . , r l tt c cc nghim ca h(x) = xr +d1 xr1 + +dr
vi dr nguyn, khc 0. Ta c 1 6 |dr | = |1 | . . . |r | < 1 : mu thun. Nh vy, iu gi s l sai
v suy ra f (x) l a thc bt kh quy.
17

V d 11. a thc f (x) = x62 + 2013x61 + 2x60 + 3x59 + + 61x + 62 bt kh quy trong Z[x].
2n
V d 12. Vi s nguyn n > a thc f (x) = x2 + x
+ 1 l bt kh quy trn Z.
Bi gii. Vi n = 0, a thc f (x) = x2 + x + 1 l bt kh quy. Gi thit n > 1. Vi a thc
g(x) = am xm + am1 xm1 + + a0 Z[x] ta k hiu g(x) = am xm + am1 xm1 + + a0 Z2 [x].
D dng kim tra g(x)h(x) = g(x) h(x) vi g(x), h(x) Z[x]. Bin i
n

(x2 + x + 1)2

n1

n2

= ((x2 + x)2 + 1)2


= ((x2 + x)2 + 1)2
n
= = (x2 + x)2 + 1.

Do vy f (x) = (x2 +x+1)2 . Gi s f (x) l kh quy v f (x) = g(x)h(x). Khi g(x) h(x) = f (x) =
n
n
(x2 +x+1)2 . V a thc x2 +x+1 l bt kh quy nn g(x) = (x2 +x+1)s v h(x) = (x2 +x+1)2 s .
n
Ta c vit trong Z[x] : g(x) = (x2 + x + 1)s + 2u(x) v h(x) = (x2 + x + 1)2 s + 2v(x), trong
2n
u(x), v(x) Z[x]. Vi nghim phc ca x2 + x + 1, t x2 + x
+ 1 = f (x) = g(x)h(x) =
2
s
2
2n s
[(x + x + 1) + 2u(x)][(x + x + 1)
+ 2v(x)] ta suy ra 2 = f () = g()h() = 4u()v(). V
1
2
2 = 1 nn u()v() c dng a + b vi a, b Z. V ta c a + b = = : v l. iu gi
4
2
s l sai hay a thc f (x) l bt kh quy.

1.2

a thc bt kh quy v s nguyn t

V d 13. Cho s nguyn t p. Gi s x1 , x2 , x3 l ba nghim ca phng trnh x3 ax2 +bxap =


0. Chng minh biu thc sau y lun lun l mt s nguyn t
(x21 + p)(x22 + p)(x23 + p)
.
(b p)2
Bi gii. Xc nh phng trnh nhn y1 = x21 + p, y2 = x22 + p, y3 = x23 + p lm ba nghim. Kh
x t h phng trnh
(
x3 ax2 + bx ap = 0
x2 + p y = 0.
(
x3 ax2 + bx ap = 0
Ta c ngay h
v ax2 px + yx + bx ap = 0 hay phng trnh
x3 + px yx = 0
ay
a2 y 2
a(y p) + yx + (b p)x ap = 0. t T = b p. Khi x =
. Vy
+py = 0
y+T
(y + T )2
hay y 3 + (2T p a2 )y 2 (T 2 + 2pT )y pT 2 = 0. Phng trnh ny c ba nghim l y1 , y2 , y3 .
y1 y2 y3
pT 2
(x21 + p)(x22 + p)(x23 + p)
=
= 2 = p.
Do
(b p)2
(b p)2
T
nh l 5. Khng tn ti a thc f (x) = a0 xm + a1 xm1 + + am vi cc h s ai nguyn v
a0 6= 0, m > 1, tha mn f (n) l s nguyn t vi mi n = 1, 2, . . . .
Chng minh. Vi a thc f (x) nh trn, hin nhin |f (n0 )| > 1 vi mt s nguyn dng
ln n0 . Chn s nguyn t p chia ht f (n0 ) v khai trin
f (n0 + kp) = f (n0 ) + kpg(n0 , p, k).
S g(n0 , p, k) l s nguyn. Nh vy c ba s hng trong h thc ny u chia ht cho p vi mi
k = 1, 2, . . . . Vi k ln ta c |f (n0 + kp)| > p v f (n0 + kp) chia ht cho p. Ta suy ra iu phi
chng minh.
18

V d 14. Cho a thc f (x) Z[x] vi bc n = deg f (x) > 0. Chng minh rng, nu c nhiu v
hn s t nhin m f (m) u l nhng s nguyn t th f (x) l a thc bt kh quy trn Q.
Bi gii. Gi s a thc f (x) kh quy. Khi ta c th phn tch f (x) = g(x)h(x) vi
g(x), h(x) Z[x], n > deg g(x), deg h(x) > 0. Theo gi thit c nhiu v hn s t nhin m
g(m)h(m) = f (m) l s nguyn t. Nh vy, c nhiu v hn s t nhin m tha mn g(m) hoc
h(m) nhn gi tr r {1, 1}, chng hn: C nhiu v hn s t nhin m g(m) = a. T y
suy ra, phng trnh g(x) a = 0 c nhiu v hn nghim thuc tp N, (mu thun). Nh vy,
f (x) l a thc bt kh quy trn Q.
V d 15. Vi s nguyn n > 3, Xc nh s nguyn dng k nh nht tha mn iu kin: Cho
bt k n im Ai (xi , yi ) trn mt mt phng, khng c ba im no cng nm trn mt ng
thng, v cho bt k n s thc di vi i = 1, 2, . . . , n, tn ti mt a thc f (x, y) bc khng vt
qu k sao cho f (xi , yi ) = di cho mi i = 1, 2, . . . , n.
Bi gii. Kt lun 1: Cho n im bt k Ai (xi , yi ), i = 1, 2, . . . , n, thuc mt mt phng
h n i sao
tha
cho khng c ba im no thng hng. Ta lun xc nh mt a thc f (x, y) bc s 6
2
mn f (xn , yn ) = 1 v f (xi , yi ) = 0 vi i = 1, 2, . . . , n 1.
hni
Ch rng, c s =
ng thng `i , i = 1, 2, . . . , s, tha mn An khng thuc mt ng `i
2
no, nhng mi im A1 , A2 , . . . , An1 thuc t nht mt ng. C th: Khi n l mt s t nhin
l th chn cc ng thng A1 A2 , A3 A4 , . . . , An4 An3 , An2 An1 ; Cn khi n l s t nhin chn
th ta chn cc ng thng A1 A2 , A3 A4 , . . . , An3 An2 , An2 An1 . Gi s phng trnh ng
thng th i l ai x + bi y + ci = 0 vi i = 1, 2, . . . , s. a thc di y tha mn cc yu cu:
f (x, y) =

(a1 x + b1 y + c1 ) . . . (as x + bs y + cs )
.
(a1 xn + b1 yn + c1 ) . . . (as xn + bs yn + cs )

hni
.
2
Vi mi i = 1, 2, . . . , n, ta xc nh a thc pi (x, y) tha mn pi (xi , yi ) = 1 v pi (xj , yj ) = 0 khi
j 6= i. Xt a thc sau:
Kt lun 2: k =

f (x, y) = d1 p1 (x, y) + d2 p2 (x, y) + + dn pn (x, y).


Hin nhin f (xi , yi ) = di pi (xi , yi ) = di vi mi i = 1, 2, . . . , n. Ta ch ra, nu k <

hni

th c
2
n im lm kt lun u bi khng ng. Tht vy, xt h n im phn bit Ai (i, i2 ) thuc
Parabl P : y = x2 v d1 = d2 = = dn1 = 0, dn = 1. Hin nhin, khng c ba im
no trong s cc im A1 , A2 , . . . , An thng hng bi v khng c ng thng no ct P ti
nhiu hn hai im. Gi s c a thc g(x, y) bc khng ln hn k tha mn g(xi , yi ) = di
vi mi i = 1, 2, . . . , n. t G(x) = g(x, x2 ). a thc G(x) c bc khng ln hn 2k tha mn
G(1) = G(2) = = G(n 1) = 0, G(n) = 1. ah thc
G(X) c n 1 nghim phn bit. Do vy
ni
deg G(x) > n 1. Do vy 2k > n 1 v c k >
.
2
V d 16. Gi s hm s f (x) nhn gi tr nguyn khi x nguyn tha mn iu kin: Cho s
nguyn t p bt k c a thc qp (x) vi h s nguyn v bc khng vt qu 2015 f (n) qp (n)
chia ht cho p vi mi s nguyn n. Chng minh rng, tn ti a thc g(x) vi h s thc tha
mn g(n) = f (n) vi s nguyn n ty .
Bi gii. Ta xc nh a thc f0 (x) bc khng vt qu 2015 tha mn f0 (i) = f (i) vi mi
i = 1, 2, . . . , 2016. S dng cng thc ni suy Lagrange, ta xt a thc bc khng qu 2015 sau
19

y:
(x 2)(x 3) . . . (x 2016)
+
(1 2)(1 3) . . . (1 2016)
(x 1) . . . (x i + 1)(x i 1) . . . (x 2016)
+
+ f (i)
(i 1) . . . (i (i 1))(i (i + 1)) . . . (i 2016)
(x 1)(x 2) . . . (x 2015)
+ f (2016)
(2016 1)(2016 2) . . . (2016 2015)

f0 (x) = f (1)

tha mn f0 (i) = f (i) vi i = 1, 2, . . . , 2016. t c = (2016!)2 . a thc cf0 (x) Z[x]. Gi s p > c
l s nguyn t. a thc cqp (x) cf0 (x) c bc khng ln hn 2015, c 2016 nghim phn bit
1, 2, . . . , 2016, theo modulo p. Bi vy, hai a thc ny trng nhau theo modulo p. Ta bit rng,
s c(f (x) qp (x)) + (cqp (x) cf0 (x) = cf (x) cf0 (x), vi x nguyn, chia ht cho s nguyn t p
bt k, ln. Nh vy, f (x) = f0 (x).
B 2. Gi s a thc f (x) = an xn + an1 xn1 + + a1 x + a0 thuc Z[x] tha mn an >
1, an1 > 0 v |ai | 6 vi i = 0, 1, . . . , n 2, l mt hng s dng btk. Khi mi
1 + 1 + 4
.
nghim phc ca f (x) u tha mn hoc phn thc R() 6 0 hoc || <
2
Chng minh. Nu |z| > 1 v R(z) > 0 th ta c cc nh gi sau:


f (z)

> |an + an1 | 1 + + 1
zn
z
|z|2
|z|n



an1
2
> R an +
z
|z| |z|

|z|2 |z|
> 1 2
=
.
|z| |z|
|z|2 |z|

1 + 1 + 4
2
.
Nu f () = 0 th || || < 0 v ta suy ra || <
2
nh l 6. Gi s s nguyn t p c th biu din c thnh dng p = an dn + an1 dn1 + +
a1 d + a0 vi s nguyn d > 2 v cc s ai thuc tp {0, 1, 2, . . . , d 1}, an 6= 0. Khi a thc
f (x) = an xn + an1 xn1 + + a1 x + a0 l bt kh quy trn Q.
Chng minh. Theo B Gauss, ta ch cn xt tnh bt kh quy ca f (x) trn Z. Gi
s ta c phn tch f (x) = g(x)h(x) vi g(x), h(x) Z[x] v 0 < deg g(x), deg h(x) < n. V
p = f (d) = g(d)h(d) nn g(d) = 1 hoc h(d) = 1. Ta ch cn xt g(d) = 1. Trn C,
r
Q
ta c biu din g(x) = c (x i ). V mi i cng l nghim ca f (x) v c l s nguyn
i=1

1 + 1 + 4
dng nn hoc R(i ) 6 0 hoc |i | <
theo B 6, trong = d 1. V
2
p
1 + 1 + 4(d 1)
d > 3 nn |i | <
6 d 1 v suy ra |d i | > 1 vi i = 1, . . . , r. Do vy,
2
r
Q
|g(d)| == c |d i | > 1, mu thun v |g(d)| = 1.
i=1

V d 17. a thc 2x6 +2x5 +2x3 +x2 +x+1 l bt kh quy v 2011 = 2.36 +2.35 +2.33 +32 +3+1.

20

1.3

a thc bt kh quy theo modulo p

Vi s nguyn t p ta k hiu trng Zp cc lp thng d modulo p qua Fp . Mt a thc thuc


Z[x] l bt kh quy vn c th tr thnh kh quy trn Fp vi mi s nguyn t p. V d: a thc
x2 + 1 l bt kh quy trn Z, nhung li l kh quy trn Z17 bi v x2 + 1 = x2 16.
Mnh 2. Vi cc s nguyn a v b, a thc f (x) = x4 + ax2 + b2 l kh quy trn Fp cho mi
s nguyn t p.
Chng minh. Vi p = 2, cc a thc x4 , x4 + x2 = x2 (x2 + 1), x4 + 1 = (x + 1)4 v x4 + x2 + 1 =
(x2 + x + 1)2 trn F u l kh quy. By gi ta xt cc s nguyn t P l. Ta c th chn s
nguyn s tha mn a 2s( mod p). Khi c th biu din
x4 + ax2 + b2 x4 + 2sx2 + b2 (x2 + s)2 (s2 b2 )( mod p)
x4 + ax2 + b2 x4 + 2sx2 + b2 (x2 + b)2 (2b 2s)x2 ( mod p)
x4 + ax2 + b2 x4 + 2sx2 + b2 (x2 b)2 (2b 2s)x2 ( mod p).
Vn cn li l ch ra mt trong ba s s2 b2 , 2b 2s, 2b 2s s ng d vi mt s r2 theo
modulo p. Trong l thuyt thng d bc hai ta bit rng, trong mt h thng d bc hai thu gn
p1
p 1 2
modulo p c
thng d bc hai tng ng cng lp vi cc thng d 12 , 22 , . . . ,
v
2
2
p1
khi y l mt thng d bc hai modulo p th y 2 1( mod p); Cn khi z l mt bt thng d
p1
bc hai modulo p th z 2 1( mod p). T y suy ra, nu z1 , z2 l hai bt thng d bc hai
modulo p th z1 z2 mt thng d bc hai modulo p. iu ny dn n ngay kt qu: Nu 2b 2s,
v 2b 2s l hai bt thng d bc hai modulo p th 4(s2 b2 ) = (2b 2s)(2b 2s) mt thng
d bc hai modulo p hay s2 b2 k 2 ( mod p) v suy ra x4 + ax2 + b2 l kh quy trn Fp .
V d 18. x4 + 1 l bt kh quy trn Z, nhng kh quy trn Fp .
Bi gii. a thc x4 + 1 l bt kh quy trn Z theo tiu chun Eisenstein. a thc x4 + 1 l
kh quy trn Fp theo Mnh 2.
Tip theo, ta xt a thc bt kh quy trn Fp . Gi s F l mt khng gian n chiu trn Fp
vi mt c s e1 , e2 , . . . , en . Mi phn t thuc F biu din c mt cch duy nht di dng:
a1 e1 + a2 e2 + + an en , ai Fp .
Do vy F c q = pn phn t. V nhm nhn F c cp q 1 nn mi phn t F u tha
n
n
mn p 1 = 1 hay p = . Nh vy, mi phn t F, k c = 0, u l nghim ca phng
trnh
n
xp x = 0.
nh l 7. a thc (x) Fp [x] l bt kh quy bc d. Ta c
d

(1) (x)|xp x trong Fp [x].


n

(2) Vi n > 0, (x)|xp x khi v ch khi d|n.


Chng minh. (1) V a thc (x) Fp [x] l bt kh quy bc d nn dimFp Fp [x]/((x)) = d.
Vy trng Fp [x]/((x)) c pd phn t v suy ra nghim ca (x) = 0 cng tha mn phng
d
d
trnh xp x = 0. Theo nh l ?? ta nhn c kt qu (x)|xp x trong Fp [x].

21

(2) V trng F c c s p nn o hm
0
n
xp x = 1 6= 0.
n

Nh vy, phng trnh xp x = 0 khng c nghim bi. Gi thit d|n. t n = hd. Gi l


d
d
2d
hd
n
nghim ca (x) = 0. Khi p = theo (1). Vy = p = p = = p = p . T y
n
n
suy ra p = 0. Theo nh l ??, a thc xp x chia ht cho a thc (x). Ngc li, gi thit
n
(x)|xp x. Biu din n = hd + r vi 0 6 r 6 d 1. Gi s r > 0. Gi l nghim ca (x) = 0.
n
hd+r
r
d
, v suy ra = p . Nh vy, dimFp Fp [x]/((x)) 6 r < d : mu
Khi = p , = p = p
thun. Ta c r = 0 v d|n.
H qu 2. Vi s nguyn dng n, trong Fp [x] c s phn tch
Y Y
n
xp x =
(x)
d|n deg (x)=d

vi cc a thc bt kh quy (x) v h t cao nht ca mi (x) u bng 1. K hiu NP


p (k) l s
n
pn
nhn t bt kh quy (x) bc k vi h t cao nht bng 1 ca a thc x x. Ta c p =
dNp (d).
d|n

1.4

Mt vi vn dng

V d 19. Gi s l mt nghim ca a thc thuc R[x] l f (x) = a0 xn + a1 xn1 + + an vi


ar
a0 6= 0 v n > 1. Khi ta c || 6 1 + max{| ||r = 1, 2, . . . , n}.
a0
Bi gii. t = max{|

a1
an
ar
||r = 1, . . . , n} v vit f (x) thnh f (x) = a0 xn [1 +
++
].
a0
a0 x
a0 x n

Khi || > 1, ta c
a1
an
+ +
|
a0
a0 n

1 
1
+ + n
> |a0 xn | 1
|x|
|x|
1
1 ni
h
h
i
||
= |a0 n | 1
> |a0 n | 1
|x| 1
|| 1
h || 1 i
= |a0 n |
.
|| 1

|f ()| = |a0 n ||1 +

h || 1 i
> 0. Do vy, || khng th lm nghim ca f (x)
Nu || > 1 + th |f ()| > |a0 n |
|| 1
ar
v suy ra rng nu l nghim ca f (x) th || 6 1 + max{| ||r = 1, 2, . . . , n}.
a0
V d 20. Gi s f (x) = a0 xn + a1 xn1 + + an Z[x] vi a0 =
6 0 v n > 1. t =
ar
max{| ||r = 1, 2, . . . , n}. Nu tn ti s t nhin q > + 2 f (q) l mt s nguyn t th f (x)
a0
l mt a thc bt kh quy trn Q.
Bi gii. Gi s f (x) l mt a thc kh quy trn Z. Khi ta c biu din f (x) = g(x)h(x) vi
g(x), h(x) Z[x] v deg g(x), deg h(x) > 0. V g(q)h(q) = f (q) l mt s nguyn t nn g(q) = 1,

22

s
Q

chng hn. Biu din g(x) = r

(x j )vi s nguyn dng r, s v cc j l cc nghim phc

j=1

ca g(x). nh gi 1 = |g(q)| = r

s
Q

|q j | qua bt ng thc

j=1

1=r

s
Y

s
Y

|q |j || >

j=1

|q ( + 1)| > 1.

j=1

T mu thun ny suy ra f (x) bt kh quy trn Q.


V d 21. Cho a thc f (x) Z[x] vi n = deg f (x) > 0. Nu c s nguyn t p f (x) Zp [x]
tha mn deg f (x) = n v f (x) l a thc bt kh quy trn Zp th a thc f (x) l bt kh quy trn
Q. T ch ra: Khng th c hai a thc p(x) v q(x) thuc Z[x] vi bc ln hn 0 tha mn
iu kin
s
31x4 25x3 + 41x2 14x + 11
= p(x).
q(x)
Bi gii. Theo B Gauss, ta ch cn xt tnh bt kh quy ca f (x) trn Z. Gi s ta c phn
tch f (x) = g(x)h(x) vi g(x), h(x) Z[x] v 0 < deg g(x), deg h(x) < n. V f (x) = g(x).h(x)
trong Zp [x] nn n = deg f (x) = deg g(x) + deg h(x) > deg(g(x)) + deg(h(x)) = deg f (x) = n. Do
vy, du = xy ra v ta nhn c deg(g(x)) = deg g(x) > 0 v deg(h(x)) = deg h(x) > 0. iu
ny chng t deg(f (x)) l kh quy: mu thun. Do vy, f (x) bt kh quy trn Z.
Xt a thc f (x) = 31x4 25x3 +41x2 14x+11. Trn Z5 ta c f (x) = x4 +x2 +x+1. V a thc ny
khng c nghim trn Z5 nn n khng c nhn t bc nht. Gi s f (x) = (x2 +ax+b)(x2 +cx+d)
vi a, b, c, d Z5 . Khi

a + c = 0, bd = 1
+b + ac + d = 1

ad + bc = 1.
V vai tr ca b v d nh nhau nn trong tp {0, 1, 2, 3, 4} v iu kin bd = 1 nn ta ch cn
xt (b, d) = (1, 1), (2, 3), (4, 4). D dng ch ra khng cp no tha mn h. Nh vy f (x) =
x4 + x2 + x + 1 l a thc bt kh quy trn Z5 . T suy ra f (x) l a thc bt kh quy trn
Q. Nu c hai a thc p(x) v q(x) thuc Z[x] vi bc ln hn 0 tha mn iu kin
s
31x4 25x3 + 41x2 14x + 11
= p(x).
q(x)
Khi f (x) = p2 (x)q(x), v l.
n

V d 22. Vi s nguyn n > 0, a thc f (x) = (x2 + x)2 + 1 l bt kh quy trn Z.


Bi gii. Vi n = 0, a thc x2 + x + 1 l bt kh quy trn Z. Gi thit n > 1. Vit g(x) trong
Z2 [x] cho a thc g(x). Biu din
n

n1

(x2 + x + 1)2 = ((x2 + x)2 + 1)2


n

n2

= ((x2 + x)2 + 1)2

= .

Nh vy (x2 + x + 1)2 = (x2 + x)2 + 1 = f (x). Gi s ta c phn tch f (x) = g(x)h(x) vi


g(x), h(x) Z[x] v 0 < deg g(x), deg h(x) < n. Vy f (x) = g(x).h(x) trong Z2 [x]. V x2 + x + 1
n
l bt kh quy trn Z2 nn g(x) = (x2 + x + 1)s v h(x) = (x2 + x + 1)2 s . Biu din trong Z[x],
n
g(x) = (x2 + x + 1)s = 2v(x) v h(x) = (x2 + x + 1)2 s + 2t(x), trong v(x), t(x) Z[x]. Vi
nghim phc u ca x2 + x + 1, t
n

(x2 + x)2 + 1 = f (x) = g(x)h(x)


23

ta c (x2 + x)2 + 1 = [(x2 + x + 1)s + 2v(x)][(x2 + x + 1)2

n s

+ 2t(x)] suy ra 2 = f (u) = g(u)h(u) =


1
2
4v(u)t(u). V u2 = u 1 nn v(u)t(u) = a + bu vi a, b Z. Nh vy, a + bu = = : v l.
4
2
iu gi s l sai hay f (x) l a thc bt kh quy.
6
Q

V d 23. Tnh tch

k=1

tan

k
.
13

tan 13z

. S dng phng trnh


= 0. Khi ta c phng trnh
13
tan z

tan12 z + 286 tan2 z + 13 = 0. Phng trinh ny c nghim tan z = tan . V tan kz =


13
6
Q
k
tan(13 k)z vi k = 1, . . . , 6, nn
tan
= 13.
13
k=1

Bi gii.

t z =

V d 24. Cho s nguyn t p. Gi s x1 , x2 , x3 l ba nghim ca phng trnh x3 ax2 +bxap =


(x2 + p)(x22 + p)(x23 + p)
0. Chng minh biu thc sau y lun lun l mt s nguyn t: 1
.
(b p)2
2
y2 = x22 + p, y3 = x23 + p lm ba nghim. Kh
Bi gii.
( Xc nh phng trnh nhn y1 = x1 + p,(
x3 ax2 + bx ap = 0
x3 ax2 + bx ap = 0
Ta
c
ngay
h
v ax2 px +
x t h
x2 + p y = 0.
x3 + px yx = 0
yx + bx ap = 0 hay phng trnh a(y p) + yx + (b p)x ap = 0. t T = b p. Khi
ay
a2 y 2
x=
. Vy
+ p y = 0 hay y 3 + (2T p a2 )y 2 (T 2 + 2pT )y pT 2 = 0. Phng
y+T
(y + T )2
(x2 + p)(x22 + p)(x23 + p)
y1 y2 y3
pT 2
trnh ny c ba nghim l y1 , y2 , y3 . Do 1
=
=
= p.
(b p)2
(b p)2
T2

V d 25 (Vit Nam MO 2015). Dy cc a thc {fn } c xc nh bi f0 = 2, f1 = 3x v


fn+2 = 3xfn+1 + (1 x 2x2 )fn , n > 0. Xc nh tt c cc s nguyn dng n fn chia ht cho
a thc x3 x2 + x.
Bi gii. Phng trnh c trng t2 3xt (1 x 2x2 ) = 0 c hai nghim t1 = x + 1
t2 = 2x 1. Bng quy np theo n ta ch ra fn (x) = (2x 1)n + (x + 1)n . fn (x) chia ht cho x
cn v fn (0) = 0 hay (1)n + 1 = 0. Vy n phi l s l. Gi u l nghim phc ca x2 x + 1.
V u2 + u = 2u 1 nn fn (x) chia ht cho x2 x + 1 khi v ch khi (2u 1)n + (u + 1)n = 0 hay

(u2 + u)n + (u + 1)n = 0. V u + 1 6= 0 nn un + 1 = 0. V u = cos + i sin nn un + 1 = 0 tng


3
3
n
n
ng cos
+ i sin
= 1. Gii ra n = 6m + 3. Tm li f6m+3 (x) chia ht cho x3 x2 + x vi
3
3
m N.
V d 26. Dy cc a thc {fn } c xc nh bi f0 = 3, f1 = 2x + 2, f2 = 2x2 + 2x + 2 v
fn+3 = 2(x + 1)fn+2 (x2 + 3x + 1)fn+1 (x2 + x)fn , n > 0.
Xc nh s t nhin m fm chia ht cho (x2 + x + 1)2 .
Bi gii. Phng trnh c trng t3 2(x + 1)t2 + (x2 + 3x + 1)t (x2 + x) = 0 c ba nghim
t1 = x + 1, t2 = x v t3 = 1. Bng quy np theo n ta ch ra fn (x) = (x + 1)n + xn + 1. a thc
2
2
x2 + x + 1 l bt kh quy trn Q vi nghim phc = cos
+ i sin , 3 = 1. V 2 + + 1 = 0
3
3
nn 1 + = 2 . Vy (1 + )6 = 1. Biu din m = 6k (
+ r vi r {0, 1, 2, 3, 4, 5}. a thc
(1 + )m + m + 1 = 0
m
m
2
(x + 1) + x + 1 chia ht cho (x + x + 1) khi v ch khi
tng
m(1 + )m1 + mm1 = 0
24

(
r = 2, r = 4
ng
Kim tra r = 4 tha mn h. Vy, a thc (x + 1)m + xm + 1
(1 + )r1 + r1 = 0.
chia ht cho (x2 + x + 1)2 khi v ch khi m = 6k + 4.

Ti liu
[1] D. Faddev et I. Sominski, Recueil DExercices DAlgbre Suprieure, Editions Mir-Moscou
1977.
[2] D. V. Nhi, A new Inequality and Identity (M, N ), Journal of Science and Arts, No. 1 (22)
2013, 5-16.
[3] D. V. Nhi and L. B. Thang, Some identities of a triangle, Journal of Science of HNUE, Vol.
57, No. 7 2012, 35-43.
[4] T. T. Nam, . V. Nh v L. B. Thng, Mt s ng dng ca i s hin i vo gii ton s
cp, Nh Xut Bn HSP TP H Ch Minh 2013.
[5] D. V. Nhi, P. M. Phuong and T. T. Tinh, Some new identities on the conic sections, Journal
of Science and Arts, Year 14, No. 3(28), 2014, 199-210.
[6] D. V. Nhi and L. B. Thng, Triangle identities via Elimination theory, Mathematical Reflections - Issue 1, 2015.
[7] V. Prasolov, Polynomials, Springer-Verlag Berlin Heidelberg 2004.
[8] J. Rivaud, Exercices DAlgbre 1, Paris Librairie Vuibert 1964.
[9] D. Q. Vit v . V. Nh, Gio trnh i s S cp, Nh Xut Bn HSP H Ni 2007.
[10] D. Q. Vit v . V. Nh, C s L thuyt s v a thc, Nh Xut Bn HSP H Ni 2008.

25

Cc dng ton chn hc sinh gii


cp quc gia nhng nm gn y
Trnh o Chin, Trng CSP Gia Lai
y l nhng nm 2010 - 2015. Cc nm 2010 v 2011, mi k thi ch t chc mt bui thi.
Cc nm 2012 - 2015, t chc 2 bui thi cho mi k thi.
Cc bi ton v Dy s, Hnh hc phng v Ton ri rc lin tc xut hin trong cc k thi (t
l 100 o/o). Trong 6 nm gn y, cc bi ton S hc, Bt ng thc v a thc gp mt 3 ln
(t l 50 o/o). H phng trnh v Phng trnh hm ri rc c mt, 2 ln cho mi dng (t l 30
o/o).
c bit, nm 2012, bi ton Dy s xut hin 2 ln; cc nm 2013, 2014, bi ton Hnh hc
phng xut hin 2 ln; cc nm 2012, 2014, bi Ton ri rc xut hin 2 ln; nm 2015, bi ton
S hc xut hin 2 ln.
S liu thng k nu trn cho thy nhng chuyn cn c u tin bi dng cho hc sinh
v mt k hoch cn kp nhm xy dng chuyn gia cc tnh khu vc Duyn hi Min Trung Ty nguyn cho tng dng ton hp.
Di y l mt s dng ton c th trong cc thi nu trn.

Cc bi ton Dy s

Nh cp trn, bi ton Dy s lin tc xut hin trong 6 nm gn y. a s gi thit


cc bi ton u cho mt (hoc hai) dy s xc nh bi mt cng thc truy hi v yu cu chng
minh dy s (hoc hai dy s) c gii hn hu hn, ri tm gii hn . Chng hn, cc dy
s xc nh nh sau:
n+2
(xn1 + 2), n > 2. . . (Bi 1 - VMO 2012)
3n
an + 2
a1 = 1 v an+1 = 3 an , n > 1. . . (Bi 2 - VMO 2013)
( 2

xn+1 yn+1 xn = 0
x1 = 1, y1 = 3 v
, n > 1. . . (Bi 1 - VMO 2014)
x2n+1 + yn = 2
1
n2 p 2
u1 = 3 v un+1 = un + 2
un + 3, n > 1, a > 0. . . (Bi 1 - VMO 2015)
2
4n + a
x1 = 3 v xn =

C cm gic rng, c s ngi ra to ra chng l nhng phng trnh (hoc h phng


trnh) sai phn tuyn tnh (hoc phi tuyn) vi h s bin thin. Cn tuyn tnh ha chng v p
dng nhng nh l, tnh cht ca dy s mi gii quyt c bi ton.
S hi t ca dy s trong cc thi ny thng c chng minh t tnh n iu v b
chn ca n. Tnh n iu thng c chng minh bng quy np hoc suy ra t vic xt mt
hm s tng ng, ri xt du o hm ca n. Gii hn cn tm thng l mt nghim no
ca mt phng trnh tng ng c thit lp. Tuy nhin, vic xt tnh n iu v b chn ca
mt dy s trong cc thi thng rt kh v quan h truy hi l kh phc tp. i khi n dn
n mt bt ng thc cn c chng minh m kh ca chng minh ny ngang nga vi
kh ca bi ton.

26

Cc bi ton Hnh hc phng

Mt trong nhng cch gii cho mt s bi ton hnh hc phng nhng nm gn y l phng
php p dng khi nim phng tch, trc ng phng v tm ng phng.
Khi nim tm ng phng hnh thnh t nh l sau y:
nh l 1. Cho 3 ng trn (C1), (C2) v (C3). Khi 3 trc ng phng ca cc cp ng
trn trng nhau hoc song song hoc cng i qua mt im (gi l tm ng phng ca ba
ng trn).
Li gii ca cc bi ton Hnh hc phng nhng nm gn y u da trn mt s nh l c
bn ca hnh hc m ch nhng hc sinh lp Chuyn mi c bi dng, nh nh l Brocard,
nh l Miquel... Chng hn Bi 3 - VMO 2012, Bi 4 - VMO 2014, Bi 4 - VMO 2015.
nh l 2. (nh l Brocard). Cho t gic ABCD c cc cp cnh i khng song song ni tip
ng trn tm O. Gi M, N, P ln lt l giao im ca AC v BD, AD v BC, AC v BD. Khi
O l trc tm tam gic NMP.
nh l 3. (nh l Miquel). Cho tam gic ABC v cc im D, E, F ln lt thuc cc ng
thng BC, CA, AB. Khi cc ng trn ngoi tip cc tam gic AEF, BFD, CDE c mt im
chung M (gi l im Miquel).
H qu 1. Cc im D, E, F thng hng khi v ch khi im Miquel M thuc ng trn (ABC).
H qu 2. Cho t gic ABCD c AB giao CD ti E, AD giao BC ti F . Khi cc ng trn
(EAD), (EBC), (FAB), (FCD) c mt im chung M (im Miquel ca t gic ABCD).
im Miquel M trong H qu 2 c nhiu tnh cht th v khi t gic ABCD ni tip. Mt
trong nhng tnh cht quan trng l:
H qu 3. Cho t gic ABCD ni tip ng trn ( O). Gi s AB giao CD ti E, AD giao BC
ti F, AC giao BD ti G . Khi cc ng trn (EAD), (EBC), (FAB), (FCD) c mt im
chung M v O, G, M thng hng.
Cc kt qu nu trn thng n trong nhng li gii m vic vn dng chng phi ht sc tinh
t. iu ng lu y l cc gc s dng trong chng minh thng l cc gc nh hng.
iu ny i hi hc sinh cn nm vng cc kin thc c bn v vn ny.

Cc bi Ton ri rc

y l cc bi ton "t trn tri ri xung" m th sinh cc tnh l i ngi nht. i khi ch
cn ch cn hiu c bi ton y ngi ta ni g l kh lm ri, ch cha bn n li gii ca
n. Chng hn:

27

Bi ton 1. (VMO 2015). Cho m hc sinh n v n hc sinh nam (m, n > 2) tham gia mt Lin
hoan Song ca. Ti Lin hoan song ca, mi bui biu din mt chng trnh vn ngh. Mi chng
trnh vn ngh bao gm mt s bi song ca nam-n m trong mi i nam-n ch ht vi nhau
khng qu mt bi v mi hc sinh u c ht t nht mt bi. Hai chng trnh c coi l
khc nhau nu c mt cp nam-n ht vi nhau chng trnh ny nhng khng ht vi nhau
chng trnh kia. Lin hoan Song ca ch kt thc khi tt c cc chng trnh khc nhau c th c
u c biu din, mi chng trnh c biu din ng mt ln.
a) Mt chng trnh c gi l l thuc vo hc sinh X nu nh hy tt c cc bi song ca
m X tham gia th c t nht mt hc sinh khc khng c ht bi no trong chng trnh .
Chng minh rng trong tt c cc chng trnh l thuc vo X th s chng trnh c s l bi
ht bng s chng trnh c s chn bi ht.
b) Chng minh rng Ban t chc Lin hoan c th sp xp cc bui biu din sao cho s cc
bi ht ti hai bui biu din lin tip bt k khng cng tnh chn l.
Ch trong ngn y thi gian lm bi, vic hiu cho thu ni dung ton ni trn cng
th sinh "chong vng"!
Nhng dng ton kiu nh th c th l s s cp ha mt kt qu no ca ton hin i,
c th l ca L thuyt th hu hn. gii c nhng bi ton ny, hc sinh cn phi c
bi dng mt cch bi bn t nhng chuyn gia c chuyn mn su, am hiu v lnh vc ny.
M nhng chuyn gia y, trong c nc, ch m khng qu mi u ngn tay.
Gn y, nhiu bi Ton ri rc c th c gii bng phng php m hnh ha di dng
bng vung (bng nh phn) m bi ton trn l mt v d. Chng hn, nu ta nh s cc hc
sinh n theo th t t 1 n m v cc hc sinh nam t 1 n n. ng vi mi chng trnh vn
ngh, vic ghp cp ca cc i song ca c th c biu din bi mt bng vung gm m hng
v n ct. Mi vung ca bng ny c nh s 1 hoc 0, trong nm hng i ct j c
in s 1 nu hc sinh n th i v hc sinh nam th j c ht vi nhau v c in s 0 nu hc
sinh n th i v hc sinh nam th j khng ht vi nhau. Bng cch "m ha" nh th, li gii
bi ton ny c th xem trong [1].
Vic thay th li gii ca bi ton cho bi nhng thao tc trn bng vung nu trn l
mt bc "m ha" quan trng i vi nhiu bi Ton ri rc tng t. Bn c c th tham
kho thm v vn ny trong [2].

Cc bi ton S hc

Bi ton S hc trong thi nhng nm gn y thng lin quan n phng trnh nghim
nguyn v mt trong nhng cch gii ca n l p dng mt kt qu no ca S hc. M
phng trnh nghim nguyn y i khi li n sau nhng phng trnh Diophant kinh in nh
Phng trnh Pell, Phng trnh dng Markov, ... Chng hn bi ton sau y:
Bi ton 2. (VMO 2014). Vi a, n l cc s nguyn dng, xt phng trnh a2 x + 6ay + 36z = n,
trong x, y, z l cc s t nhin.
a) Tm tt c cc gi tr ca a vi mi n > 250, phng trnh cho lun c nghim t
nhin (x, y, z).
b) Bit rng a > 1 v nguyn t cng nhau vi 6. Tm gi tr ln nht ca n theo a phng
trnh cho khng c nghim (x, y, z).

28

Gi tr 250 trong cu a) d lm cho hc sinh lng tng, v chc chn n s l gi tr c


th ca mt biu thc no m khng d t ngh ra. By gi, nu bin i phng trnh
a2 x + 6ay + 36z = n v dng a(ax + 6y) + 36z = n th v tri ca phng trnh s c dng
ax + by = n. Lin h gia cc nhn xt trn, ta nhn ra rng bi ton c th c thit lp t
kt qu sau:
nh l 4. (nh l Sylvester). Cho a, b nguyn t cng nhau. Khi s nguyn ln nht khng
biu din c di dng ax+by vi x, y t nhin l N0 = abab. Ni cch khc, N0 = abab+1
l s nguyn dng nh nht sao cho phng trnh ax+by = n c nghim t nhin vi mi n > N0 .
Vi b = 36, bi ton s c s lin h vi nh l trn theo tham s a v li gii ca n c th
tham kho trong [1].
Phng trnh Markov c in l phng trnh dng x21 + x22 + ... + x2n = kx1 x2 ...xn , trong n
v k l cc tham s nguyn dng. Mt tnh cht c bn ca phng trnh ny l, nu n c mt
nghim th n s c rt nhiu nghim c to ra theo mt quy lut no . Mt s trng hp
ring ca phng trnh ny l c s hnh thnh rt nhiu bi ton trong cc thi nhng nm
gn y. Chng hn:
Bi ton 3. (VMO 2012). Cho a, b l hai s t nhin l tha mn a l c ca b2 + 2 v b l c
ca a2 + 2. Chng minh rng a v b l cc s hng ca dy s t nhin vn xc nh bi
v1 = v2 = 1, vn+2 = 4vn+1 vn , n > 1.
T gi thit ca bi ton, c th suy lun rng khi (a, b) = 1. Do ab l c ca a2 +b2 +2.
Nh vy a v b tha mn mt phng trnh dng Markov x2 + y 2 + 2 = 4xy.
Tm li, vic bi dng mt cch bi bn v cc phng trnh Diophant kinh in s gip cho
hc sinh phn no c hng gii quyt mt s bi ton S hc thng gp trong cc k thi chn
hc sinh gii ...
Ton s cp vn mnh mng, d ch l bin nh. M sc ngi th c hn!

Pleiku, Tt t Mi.

Ti liu tham kho


[1] Trn Nam Dng (ch bin), Nguyn Tt Thu, V Quc B Cn, L Phc L - Li gii v
bnh lun thi VMO 2015 - Thnh ph H Ch Minh, 16/01/2015 - Ngun: Internet.
[2] Trnh o Chin - i tm "ci gc" ca mt s bi Ton ri rc - K yu hi tho khoa
hc Cc chuyn bi dng hc sinh gii mn Ton Trung hc ph thng - S Gio dc v
o to Gia Lai, 19-20/04/2014.

29

Mt gc nhn thng qua cc bi hnh


hc cc k thi Olympic Ton quc t
Nguyn B ang, Hi THHN

Bi ton 1.
Cho tam gic ABC, O l im trong tam gic tha mn AB + BO = AC + CO, v M l im
bt k trn BC qua M k ng thng song song vi BO, CO ct AC, AB ln lt ti P v Q.
Chng minh chu vi t gic APMQ khng ph thuc vo v tr ca im M.
Li gii. Ko di AC ly i E sao cho CE = CO. Ko di AB ly im F sao cho BF = BO

\
[
Theo gi thit AB + BO = AC + CO AF = AE phn gic ca cc gc F
BO, F[
AE, ECO
vung gc vi EF, OE, OF ct nhau ti mt im I
I l tm ng trn ngoi tip OEF.
Gi hnh chiu ca I trn AC, AB, BO, CO K, J, G, H IK = IJ = IG = IH.
Ko di BO ct AE ti Y v CO ct AB ti X
AB + BY + Y A = 2AJ AC + CX + XA = 2AK, AJ = AK
AC + CX + XA BX + BY = CX + CY
CM
BM
=x
= 1 x.
t
BC
BC
MQ//CX
QM
BM
XQ
CM

=
= x,
=
=1x
XC
BC
XB
CB
QM = xXC v XQ = (1 x)XB
MP//BY
YP
BM
MP
CM

=
= x,
=
= 1 x P Y = xCY, M P = (1 x)BY
YC
BC
BY
CB
XQ + QM + M P + P Y = (1 x)XB + xXC + (1 x)BY + xCY
XQ + QM + M P + P Y = XB + BY + x(XC + CY XB BY ) = XB + BY
Cng hai v AX + AY
30

AQ + QM + M P + P Y = XB + BY + AX + AY = AB + BY + AY
AB + BY + AY khng i chu vi t gic APMQ khng ph thuc vo v tr ca im M.

Bi ton 2.
b < 900 ), ng trn ng knh AC ct AB, AD ti M v N,
Cho hnh bnh hnh ABCD (A
ng cho BD ct tip tuyn ti C vi ng trn ti E. Chng minh rng M, N, E thng hng.
Li gii.

\
\
Theo gi thit AC l ng knh AM
C = AN
C = 900
AM, AN ct tip tuyn ti C ca ng trn ti Q v P CQ2 = QM.QA
ACQ: AC 2 = AM.AQ
(1) CP 2 = P N.P A, CA2 = AN.AP
PN
CP 2
=
(2), chia (1) ch0 (2)

2
CA
AN
CQ2
QM P N

=
(3).
2
CP
AM AN
T gic ABCD l hnh bnh hnh AB//CD v BC//AD
CQ
AD
QC
QB
CQ2
AD QB
=
v
=

=
(4)

2
CP
DP
CP
BA
CP
DP BA
BA EQ DP
AQP c B, D, E thng hng theo nh l Menelaus
= 1 (5)
BQ EP DA
M A EQ N P
T (3), (4) v (5)
= 1 M, N, E thng hng.
M Q EP N A

Bi ton 3 (APMO 2012).


b ct cnh AC ti
[ = 300 . ng phn gic trong v ngoi gc B
Cho tam gic ABC, BAC
b ct cnh AB ti C1 , C2 . ng trn ngoi tip tam
B1 , B2 , ng phn gic trong v ngoi gc C
gic BB1 B2 ct ng trn ngoi tip tam gic CC1 C2 ti im P trong tam gic ABC, gi O
l trung im B1 B2 . Chng minh rng OP vung gc vi BP.
b
\ = OBB
\1 CBB
\1 =BB
\
\
Li gii. OBC
1 O B1 BA = A OBA v OCB ng dng .
OB
OA
OA
OP

=
OA.OC = OB 2 = OP 2
=
OC
OB
OP
OC
[
[
OPC ng dng vi OAP OP C = P AC.
\
\
\
\
[
[
\
\
P
BC P[
BA = (P\
BB1 + B
1 BC) (ABB1 P BB1 ) = 2P BB1 = = P OB1 = P CA OP C =
31

P[
CA P[
AC
\
P[
AC + P
BC = P[
BA + P[
CA
[
\
hon ton tng t ta c: P AB + P
CB = P[
BA + P[
CA
[
\
[
\
[
Cng cc v P AC + P BC + P AB + P CB = P BA + P[
CA + P[
BA + P[
CA.
0
0
b = 300
[
[
[
[
[
[
180 (P BA + P CA) = 2(P BA + P CA) P BA + P CA = 60 , gi thit A
\
\
P
BC + P
CB = 900 P BP C.

Bi ton 4.
Cho t gic ABCD ni tip ng trn (O; R). Chng minh ng thc:
AC
AB.AD + CB.CD
=
BD
BA.BC + DA.DC
Li gii. Ta lun c

dt(ABD) + dt(CBD)
=1
dt(DAB) + dt(BAC)
abc
ta c:
4R
AD.AB.BD CD.CB.BD
+
dt(ABD) + dt(CBD)
BD(AD.AB + CD.CB)
4R
4R
=
=
BA.BC.AC DA.DC.AC
dt(BAC) + dt(DAB)
AC(BA.BC + DA.DC)
+
4R
4R

s dng cng thc S =

32

BD(AD.AB + CD.CB)
AC
AB.AD + CB.CD
=1
=
.
AC(BA.BC + DA.DC)
BD
BA.BC + DA.DC

Bi ton 5.
Cho t gic ABCD ni tip trong mt ng trn. Tnh di ng cho AC v BD theo
a, b, c, d. (AB = a, BC = b, CD = c, DA = d) Li gii. Theo nh l Ptoleme AC.BD =
AB.CD + BC.DA = ac + bd (1)
Theo kt qu bi ton trn
AC
AB.AD + CB.CD
ad + bc
=
=
(2)
BD
BA.BC + DA.DC
ab + dc
r
(ad + bc)(ac + bd)
(ad + bc)(ac + bd)
Nhn (1) vi (2) AC =
AC =
ab + dc
ab + dc
r
(ab + dc)(ac + bd)
(ab + dc)(ac + bd)
BD =
Chia (1) cho (2) BD2 =
ad + bc
ad + bc
2

Bi ton 6 (IMO 53-2012).


Cho tam gic ABC, gi J l tm ng trn bng tip gc A v I, D, E l cc tip im
tng ng vi cc cnh BC, CA, AB, ng thng BJ ct ID ti M v ng thng CJ ct EI
ti N, ng thng JC ct IE ti N. ng thng AM, AN ct cnh BC t P v Q. Chng minh
IP = IQ . (Hy Lp)
b J l tm ng trn bng tip gc A
b EBJ
d
[ = 1800 B,
[ = JBI
Cch 1. Li gii. EBI

d = 900 1 B
b , EIBJ
[ = IBJ
EBJ
2
b tng t CID
b
[ = 1 B,
[ = 1C
BEI
2
2
b + C)
b
\
[I = 900 M
\
[ = 900 1 (B
M JEIv ct EI ti trung im EM
J = JM
IB BIE
2
1b
b
\
[ = 1A
\
EM
J= A
EM
J = EAJ
2
2
33

AMEJ l t gic ni tip, AEEJ M AM J MA//IE PEIA l hnh thang cn


IP = AE.
b t gic AEJN ni tip AEEJ AN JN
[J = EAJ
[ = 1A
tng t EN
2
t gic AIDQ l hnh thang cn IQ = AD, mt khc AE = AD IP = IQ.
Cch 2.
JEAE, JDAD A, E, J, D nm trn ng trn ng knh AJ.

d = 900 1 B.
b
[ = JBI
EBJ
2
d = 900 1 C
b BIM
d = 1C
b
\ = CID
[ = 900 ICJ
Tng t ICJ
2
2
d BIM
b 1C
b = 1A
b AM M J
\I = JBI
\ = 900 1 B
BMI: BM
2
2
2
b P B = BA
MJ l phn gic ngoi gc B
P I = P B + BI = BA + BE = AE
b M thuc ng trn (AEJ), tng t N cng thuc thuc ng trn
\
[ = 1A
JM
D = JAD
2
(AEJ), tng t nh trn QI = QC + CI = CA + CD = AD
Mt khc AD = AE IP = IQ.

Bi ton 7 ( d tuyn 2006 Hy Lp).


Cho tam gic ABC, ng trn bng tip gc A tip xc vi cc cnh BC, CA, AB ln lt
ti I, D, E. Gi s ng thng EI vung gc vi AC ti K, gi H l hnh chiu ca D trn JK.
[ = AHE
\ = 900 . Li gii.
Chng minh rng CHI
JBEF , EKAC KFJD l hnh ch nht .
F J = KDJE 2 = JD2 = JF.JB = DK.JB
DK
JD

=
KDJ ng dng DJB
JD
JB
DKJ = JDB JKDB
B, H, D thng hng BIJ = BEJ = CEJ = 900
E, I, H nm trn ng trn ng knh JB
KCI = IJB = KHI CHIK ni tip IHC = 900
t gic AKHE ni tip HEA = HJB = HKD AHE = CHI = 900 .
34

Bi ton 8 (IMO-53).
b = 900 ), AHBC, M AH, P BM sao cho CP = CA,
Cho tam gic vung ABC (A
Q CM sao cho BQ = BA, CP ct BQ ti E. Chng minh EP = EQ. (Cng ha Czech)
Li gii. BP v CQ ct ng trn ngoi tip tam gic ABC ti G v D.ng thng thng BD

b = 900 BC l ng knh ng trn ngoi tip ABC


v CG ct nhau ti I, A
\ = BGC
\ = 900 M l trc tm IBC I nm trn ng thng AH.
BDC
BQ
BC \
BQ2 = BA2 = BH.BC
=
, QBH chung BQH ng dng vi BCQ
BH
BQ
\ = BCQ,
\ mt khc BCD
\ = BIH
[ BIH
[ = BQH
\ B, Q, I, H nm trn mt ng trn,
BQH
IHBC BI l ng knh ng trn ngoi tip t gic BQIH
[ = 900 v QDBI QI 2 = ID.IB
BQI
[I = 900 v P I 2 = IG.IC
Tng t nh trn CP
[ chung hai tam gic ng dng
Xt hai tam gic IDC v IGB l hai tam gic vung c DIG
ID
IC
=
ID.IB = IG.IC QI 2 = P I 2 QI = P I

IG
IB
Xt hai tam gic: EQI v EPI l hai tam gic vung c QI = P I v cnh huyn EI chunghai
tam gic bng nhau EP = EQ.

35

Bi ton 9 (APMO 2013).


Cho tam gic nhn ABC ni tip ng trn tm O, cc ng cao AD, BE, CF. Chng minh
rng cc ng thng OA, OF, OB, OD, OC, OE chia tam gic ABC thnh ba cp tam gic c
din tch bng nhau.
1\
\
=
Li gii. Gi M, N l trung im cnh BC v CA, OM BC, ON CA M
OC = BOC
2

[
BAC
Theo gi thit BECA MOC v EAB ng dng (g.g)
OM
OC
OA

=
=
(OA = OC)
AE
AB
AB
Tng t ADBC NAO v DAB ng dng
OA
OM
ON
ON
=

=
OM.BD = ON.AE SOBD = SOAE .

BD
AB
AE
BD
Tng t SOCD = SOAF v SOCE = SOBF .

Bi ton 10 (APMO 2013).


Cho t gic ABCD ni tip mt ng trn (C). M nm trn ng thng ko di ca ng
cho DB, sao cho MA, MC l tip tuyn ca ng trn (C). Tip tuyn ti B vi ng trn
(C) ct MC ti N v CD ti P, ND ct ng trn (C) ti E. Chng minh rng A, E, P thng
\
\ MCB v MDC ng dng
hng. Li gii. MC l tip tuyn vi (C) N
CB = BDC
CB
MC
=
MD
DC
MA
AB
MA l tip tuyn vi (C), tng t
=
, (do M A = M C)
MD
DA
CB
AB

=
DA.CB = AB.DC
DC
DA
p dng nh l Ptolemy vi t gic ABCD AB.CD + BC.DA = AC.BD
1
AC
2BC
BC.DA = AC.BD
=
; (1)
2
DA
DB
NB, NC l tip tuyn vi ng trn (C) NBE, NDB ng dng.
NB
BE N C
CE
NCE, NDC ng dng
=
,
=
ND
DB N D
DC
BE
CE
kt hp N B = N C
=
BE.DC = CE.DB
DB
DC
p dng nh l Ptolemy vi t gic BECD
36

1
BC
2CE
=
; (2)
BE.DC = CE.DB = BC.DE
2
BD
DE
PB
CB
PC
=
=
P C.P D = P B 2
PB l tip tuyn vi (C)
P
B
P
D
BD

2 
2
PC
P C.P D
PB
CB
mt khc
=
=
=
kt hp vi (2)
PD
P D2
PD
BD

2 
2
CB
2CE
PC
=
=
; (3)

PD
BD
DE
EC
QC
=
Gi s AE ct CD ti Q QEC v QDA ng dng
QA
DA
QD
DE
QDE v QAC ng dng
=
;
QA
AC
QC QD
EC DE

:
=
:
,v kt hp (1), (2)
QA QA
DA AC

2
QC
EC.AC
EC 4EC
2CE

=
=
=
; (4)
QD
DE.DA
DE DE
DE
PC
QC
T (3) v (4)
=
P Q A, E, P thng hng.
PB
QB

37

Mt hng tip cn gii bi ton


dy s
L Quc Bo, Trng THPT Trn Hng o, Cam Ranh
Ton hc khng ch l nhng vn th, m cn l nhng vn th hay, nhng ng th
k diu, bt dit; n din t c th gii, c v tr bao la ch trong nhng th n gin
+
R x2

e dx = chnh l mt trong nhng v


nht! Nhng cng thc ei + 1 = 0 hay

p diu k ca ton hc.


Lipman Bers (1914-1993)

Dy s l khi nim quan trng trong chng trnh Ton ph thng, c bit l i vi chng
trnh chuyn Ton. Cc bi ton v gii hn dy s thng xut hin nhiu trong cc k thi hc
sinh gii quc gia, thi Ton quc t,. . . v c xem nh nhng dng ton kh v kh a dng,
khai thc cc tnh cht s hc, i s, lng gic ca chng.
Trong chuyn ny, ti s khai thc tnh cht gii tch ca chng v gii thiu mt cch tip
cn mi trong vic gii quyt bi ton dy s bng cch s dng gii hn trn v gii hn di,
chng c bit hiu qu bi gi thit i hi ch l dy s b chn nn phm vi p dng rt rng.
Phng php ny tc gi c gii thiu t ngi thy ca mnh khoa Ton i hc Qui
Nhn v qua qu trnh nghin cu ca bn thn thu c mt s kt qu. Hy vng qua chuyn
ny cc bn hc sinh c th c thm cng c mi khi gii quyt bi ton dy s trong cc k thi
hc sinh gii v quan trng hn l thi mt lun gi mi cho cc em tip cn vi nhng kin thc
ca Gii tch c in vi nhng kin thc rt c th, rt s cp cc em yu mn hc mang v
p k diu ny !
Chuyn c chia lm 3 phn vi nhiu v d c th v bi tp.
Phn 1: Trnh by nh ngha v cc tnh cht gii hn trn v gii hn di ca dy s.
Phn 2: Cc dng bi tp c th s dng o hm v gii hn trn v gii hn di.
Phn 3: Cc bi tp p dng.
Chuyn ny mc d c bin son mt cch nghim tc nhng s khng trnh khi mt s
thiu st. Tc gi mong mun nhn c s gp t pha ng nghip v cc bn hc sinh. Xin
chn thnh cm n !

Mt s kin thc chun b

Sau y l nh ngha v nh l v gii hn trn v gii hn di ca dy s. Chi tit v


chng minh cc bn c th tham kho trong [3].
nh ngha 1. Nu dy con (ank ) hi t th gii hn ca n c gi l gii hn ring ca dy
(an ).
nh l 1. (Bolzano-Weierstrass ) T dy b chn bt k (an ) u c th trch ra c mt dy
con (ank ) hi t.
nh ngha 2.
1. Gii hn ring ln nht ca dy (an ) c gi l gii hn trn ca n v k
hiu l lim an .
38

2. Gii hn ring b nht ca dy (an ) c gi l gii hn trn ca n v k hiu l lim an .


nh l 2. Mi dy s thc u c gii hn trn v gii hn di.
nh l 3. Dy s thc (an ) hi t khi v ch khi n b chn v lim an = lim an .
Mt s kt qu quan trng v gii hn trn v gii hn di m ti s vn dng vo gii quyt
bi ton dy s. Chi tit v chng minh ti s khng nu ra y, bn c quan tm c th tham
kho trong [4] v [5].
nh l 4. [4] Cho hai dy s thc (an ) v (bn ). Khi
lim an + lim bn lim(an + bn ) lim an + lim bn lim(an + bn ) lim an + lim bn .
nh l 5. [5] Cho f : R R l lin tc, n iu tng v (an ) l dy b chn.Khi

lim f (an ) = f (lim an ) , lim f (an ) = f lim an .
nh l 6. [5] Cho f : R R l lin tc, n iu gim v (an ) l dy b chn.Khi

lim f (an ) = f lim an , lim f (an ) = f (lim an ) .

ng dng

Bi ton 1. (Olympic 30-4 nm 1999) Cho dy s (un ) vi:

u 1 N
1
un+1 = ln(1 + u2n ) 1999, n 1
2

Chng minh rng dy (un ) hi t.


1
x
Li gii. Ta c f (x) = ln(1 + x2 ) 1999 l hm s kh vi trn R v f 0 (x) =

2
1 + x2
h 1 1i
1
, (x R.) Mc khc, t: g(x) = x + 1999 ln(1 + x2 ) = x f (x) th g cng kh vi trn
2 2
2
x
R v g 0 (x) = 1
>
0,
x

R.
Hn
na
g(0)g(1999)
< 0. Do tn ti L (1999, 0)
1 + x2
sao cho g(L) = 0 f (L) = L. p dng nh l Lagrange, ta c c R sao cho: |un+1 L| =
 n1
1
1
0
|u1 L|, n N.
|f (un ) f (L)| = |f (c)||un L| |un L.| T ta c: |un+1 L|
2
2
Vy nn lim un = L.
1

Bi ton 2. (Olympic 30-4) Cho 1 < a < e e v dy s (xn ) xc nh bi:


(
x1 = a
.
xn+1 = axn , n 1
Chng minh rng dy (xn ) hi t.
ln x
Li gii. Bng qui np ta thu c (xn ) l dy tng. Xt f (x) =
ln a, x > 1. D thy
x
ln x0
f (1)f (e) 0, do tn ti x0 > 1 sao cho f (x0 ) = 0.
= ln a x0 = ax0 Bng qui np
x0
ta chng minh c xn < x0 , n N. Vy (xn ) hi t.
39

Bi ton 3. (VMO nm 2001)

Cho dy s (xn ) tha mn iu kin

(
x0 = a
xn+1 = xn + b sin xn , n 0

a. Chng minh rng nu b = 1 th dy (xn ) c gii hn hu hn v tm gii hn .


b. Chng minh rng vi mi s thc b > 2 th dy (xn ) khng hi t.
Li gii. K thut tng t cc bi trn, xt hm f (x) = x + sin x.
Bi ton 4. (Olympic SV - 2011) Cho , R tha mn


1
1+
n

+n


<e<

1
1+
n

+n
, n R.

Tm min | |.
Li gii. T gi thit ta c
<

Xt hm s f (x) =

V vy dy

1

 n < , n R.
1
ln 1 +
n

1

 x, x 1. D thy hm f gim.
1
ln 1 +
x

1

 n
l dy gim. Do
1
ln 1 +
n

min | | =

Suy ra min | | =

1
1


 1 inf



1
1
ln 1 +
ln 1 +
1
n

1
1 lim

ln2

1

ln 1 +

 n

1
n

1
1
1 .
ln2
2

1
3
.
ln2 2

Nhn xt 1. Vic s dng o hm t ra hiu qu trong vic xt mt hm s lin quan. Tuy


nhin cch ny s gp kh khn khi gp dy s truy hi dng xn+1 = f (xn , xn1 ). Cc bi tp
di y ti s vn dng itgii hn trn v gii hn di gii quyt cc bi ton dy s. Mt
s bi tp di y s minh ha cho phng php ny !
1

Chi tit li gii cc bn c th xem [1].

40

Bi ton 5. Cho dy s (an ) tha mn iu kin

a1 = 1, a2 = 1
2
1

an+1 = (1 + an + a3n1 ), n 2
3

Chng minh rng dy (an ) c gii hn hu hn v tm gii hn .


1
Li gii. Bng qui np ta thu c an 1, n 1. V vy (an ) b chn, do ta t
3
(
lim an = A
.
lim an = a
Khi
1
a = lim an+1 = lim (1 + an + a3n1 )
3
1
(1 + lim an + lim a3n1 )
3
1
= (1 + a + a3 ).
3
a3 2a + 1 0

51
a 1.
2

(1)

51
a = lim an
Bng qui np ta d dng chng minh c inf kn ak an

2
51
51
Cui cng a =
. Kt hp vi (2.1) ta thu c lim an =
.
2
2
Bi ton 6. Cho dy s (an ) tha mn iu kin
(
a1 = 1

an+1 = 2 + an , n 1

51
.
2

Chng minh rng dy (an ) c gii hn hu hn v tm gii hn .


Li gii. Bng qui np ta thu c 0 < an < 2, n 1. V vy (an ) b chn, do ta t
(
lim an = A
.
lim an = a
Khi

A = lim an+1 = lim( 2 + an )

V hm f (x) =
nn

2 + x l hm tng trn khong (0; +) nn lim( 2 + an ) = 2 + lim an . Vy


q

A = 2 + lim an = 2 + A.

A = 2. Tng t ta cng c a = 2. Khi lim an = lim an . Vy lim an = 2.


41

Bi ton 7. Cho dy s (an ) b chn v tha mn iu kin


2
1
an+2 an+1 + an , n 1
3
3
Chng minh rng dy (an ) hi t.
Li gii. t
2
bn = an+1 + an .
3
V (an ) b chn nn (bn ) b chn. Do ta t
(
lim an = A
.
lim an = a
Ta c
2
A = lim bn = lim(an+1 + an )
3
2
lim an+1 + lim an .
3
Ngoi ra
2
a = lim bn = lim(an+1 + an )
3
2
lim an+1 + lim an .
3
Hn na d kim tra (bn ) l dy hi t, suy ra
2
2
A+ aa+ A
3
3
A a A = a.
Bi ton 8. (VMO-1988)

Cho dy s (xn ) b chn v tha mn iu kin


1
(xn+1 + xn ) xn+2 , n 1
2

Dy (xn ) c tn ti gii hn hay khng ?


Li gii. t
1
yn = xn+1 + xn
2
V k thut tng t bi tp trn.
Bi ton 9. Cho dy s (an ) tha mn iu kin

a1 = 2
1
an+1 = 2 +
, n 1
1

3+
an
2

Cc bn c th tham kho cch gii khc ca bi ton ny trong [1].

42

Chng minh rng dy (an ) hi t.


Li gii. Bng qui np ta thu c 2 an 3, n 1. V vy (an ) b chn, do ta t
(
lim an = A
.
lim an = a

A = lim an+1 = lim 2 +

1
3+
an

1
1
v hm f (x) = 3 +
= lim 2 +
1
x
3+
lim an
1
.
=2+
1
3+
A

A=

3+


tng trn(2; +)

15

. Chng minh tng t ta cng thu c a = A.

3 + 15
.
Vy (an ) hi t v lim an =
3

Bi tp
1. Chng minh dy (an ) cho bi

n+ 1
1
2
an = 1 +
n
l dy gim.
2. Cho dy s (yn ) b chn v tha mn iu kin
yk+1 = (1 x)yk +

Ax
1x

yk x
vi A > 0, 0 < x < 1 v y0 > 0.
Chng minh rng dy (yn ) c gii hn hu hn v tm gii hn .
3. Cho (an ) c gii hn bng a. Chng minh rng
lim

a1 + a2 + + an
= a.
n

4. Cho dy s (xn ) tha mn iu kin


xn+1 = 1 +
Chng minh rng lim xn =

2.

5. Cho dy (an ) v (bn ) tha mn


43

1
, x0 = 1.
1 + xn

(a) (bn ) l dy tng v lim bn = +;


an an1
= l.
(b) lim
bn bn1
Khi
lim
6. Cho dy s (an ) tha mn iu kin

a1 = 1,
an+1 =

an
= l.
bn

1 + a1 + a2 + + an

, n 1

Chng minh rng dy (an ) c gii hn hu hn v tm gii hn .


7. Cho dy s (xn ) tha mn iu kin


 
1
2
n
xn = 1 + 2
1 + 2 ... 1 + 2 .
n
n
n
Tm lim(ln xn ).
8. Cho dy s (xn ) tha mn iu kin

x1 = a 6= 2,
p
3 2x2n + 2 2

p
, n 1
xn+1 =
2xn + 2x2n + 2

Chng minh rng dy (xn ) c gii hn hu hn v tm gii hn (nu c) ty theo a.


9. Cho dy s (xn ) b chn v tha mn iu kin
(
xn + xn+1 2xn+2 , n 1
x1 = 2007

Chng minh rng dy (xn ) c gii hn hu hn v tm gii hn .


10. Cho dy s (an ) b chn v tha mn iu kin
an+2

1
2008
an+1 +
an , n 1
2009
2009

Chng minh rng dy (an ) hi t.

44

Ti liu tham kho


[1] Le Hai Chau, Le Hai Khoi, Selected Problems of the Vietnamese Mathematical Olympiad
(1962-2009), Word Scientific, volume 5.
[2] on Qunh (Ch bin), Trn Nam Dng, Nguyn V Lng, ng Hng Thng, Ti liu
chuyn Ton i s v Gii tch 11, nh xut bn gio dc Vit Nam, 2012.
[3] Nguyn Vn Mu, Nguyn Thy Thanh, Chuyn bi dng hc sinh gii Ton Trung hc
ph thng Gii hn dy s v hm s, nh xut bn gio dc, 2008.
[4] W.J.Kaczor, M.T.Nowak, on Qunh(dch), Bi tp gii tch I: S thc-dy s v chui s,
nh xut bn i hc s phm, 2003.
[5] W.J.Kaczor, M.T.Nowak, Problems in Mathematical Analysis II: Continuity and Differentiation, AMS, volume 21.
[6] Nguyn Duy Tin, Bi ging gii tch (Tp 1), nh xut bn i hc quc gia H ni, 2007.
[7] Teodora-Liliana T. Rdulescu, Vicentiu D.Rdulescu, Titu Andreescu, Problems in real anal
Analysis: Advanced calculus on the real axis, Springer, Made in the USA Lexington, KY 05
March 2011.

45

Gii hn ca cc dy s sinh bi tng


Nguyn Ti Chung
THPT Chuyn Hng Vng, Gia Lai

Gii hn ca cc dy s sinh bi tng

1.1

Mt s lu .

+
nh ngha 1. Cho dy s (xn )+
n=1 . Xt dy s (Sn )n=1 nh sau:

Sn =

n
X

xi , tc l Sn = x1 + x2 + + xn .

i=1

Khi dy s (Sn )+
n=1 c gi l dy tng.
Nhn xt 1. i vi nhng dy s (xn ) c dng xn+1 = f (x1 + x2 + + xn ), th bng cch t
Sn = x1 + x2 + + xn , ta a v dy s {Sn }+
n=1 nh sau:
S1 = x1 , Sn+1 = Sn + f (Sn ).
Ch 1. Vn xt s hi t hay phn k ca mt dy tng c cp nhiu trong chng
trnh ton cao cp, tuy nhin trong bi ny ta ch xt vn s cp hn l tm gii hn ca
mt s dy tng thng gp trong cc k thi hc sinh gii trung hc ph thng.

1.2

Phng php tm gii hn ca dy tng.

Ta thng dng cc phng php sau y:


Rt gn hoc tm s hng tng qut ca dy s (Sn ) (y l dng ton rt thng gp trong
cc k thi HSG).
So snh dy s (s dng nguyn l kp).
S dng cc dy con k nhau.
Chuyn v dy tch.
Ch 2. Nu dy (xn ) cho bi h thc truy hi (chng hn bi ton 6 trang 47, bi ton 7
trang 48...) th ta thng lm nh sau:
Chng minh lim xn = + hoc lim xn = bng cch ch ra dy (xn ) tng v khng
n+

n+

b chn trn hoc dy (xn ) gim v khng b chn di.


Rt gn tng Sn , t tm lim Sn .
n+

Ch 3. rt gn tng Sn =

n
P

xi , ta thng bin i

i=1

Sn =

n
X
i=1

xi =

n
X

(i i+1 ) = 1 n+1 .

i=1

46

1.3

Cc bi ton v dy sinh bi tng c th rt gn c.

Bi ton 1. Xt dy (un ) nh sau: un =

2n + 1
, n N . Tm
+ 1)2

n2 (n

lim (u1 + u2 + + un ).

n+

Gii. Ta c
uk =

2k + 1
1
1
= 2
, k = 1, 2, . . .
2
+ 1)
k
(k + 1)2

k 2 (k

Do

 



1
1
1
1
1
u1 + u2 + + un = 1 2 +
+ +

2
22 32
n2 (n + 1)2
1
=1
.
(n + 1)2
Bi vy

lim (u1 + u2 + + un ) = lim

n+

n+

Bi ton 2. Xt dy (xn ) nh sau: xn =


Bi ton 3. Tnh lim

n
P

n+ k=2

n+

n
P
n
,
n
=
1,
2,
.
.
.
Tm
lim
xk .
n+ k=1
(2n 1)2 (2n + 1)2

xk bit: xn = ln


Bi ton 4. Tnh lim 1 +


1
= 1.
1
(n + 1)2

n3 1
, n = 2, 3, . . .
n3 + 1


1
1
1
+
+ +
.
1+2 1+2+3
1 + 2 + + n

Bi ton 5. [ ngh Olympic 30/04/2013] Cho a l mt s thc dng v dy (xn ) c xc


nh bi:
x
1
pn
, n .
x1 = , xn+1 =
2
2
2a
1 + axn + a xn + 4axn
t Sn =

n
P

xk . Tm lim Sn .

k=1

n+

Bi ton 6. Cho s thc a > 1. Xt dy s (un ) nh sau: u1 = a v


un+1 = u2n un + 1, n = 1, 2, . . .


1
1
1
Hy tm lim
+
+ +
.
n+
u1 u2
un
Gii. V a > 1 v un+1 = un + (un 1)2 , n N nn d dng suy ra
1 < a = u1 < u2 < < un < un+1 <
T (1) v (2) ta c:
1
un+1 1
un (un 1)
1
1
1
1
1
1

.
un+1 1
un 1 un
un
un 1 un+1 1
un+1 1 = un (un 1)

47

(1)

(2)

1
1
1
=

, n = 1, 2, . . . ta c
un
un 1 un+1 1


1
1
1
+
+ +
u1 u2
un

 



1
1
1
1
1
1
=

+ +

u1 1 u2 1
u2 1 u3 1
un 1 un+1 1
 


1
1
1
1

, n = 1, 2, . . .
=
u1 1 un+1 1
a 1 un+1 1

Trng hp 1: Dy (un ) b chn trn. Khi dy (un ) tng v b chn trn nn c gii hn hu
hn. t lim un = L. T (2) suy ra L > 1. T (1) cho n + ta c L = L2 L + 1 hay
n+

L = 1. iu ny mu thun vi L > 1. Vy trng hp 1 khng th xy ra.


Trng hp 2: Dy s (un ) khng b chn trn. Khi :
lim un = + lim (un+1 1) = +

n+

Do


lim

n+

1
1
1
+
+ +
u1 u2
un

n+


= lim

n+

1
1

a 1 un+1 1


=

1
.
a1

1
.
n+
a1
Lu . Dy s (un ) c h thc truy hi l un+1 = f (un ), vi f (x) = x2 x + 1. Ta tm im bt
ng ca hm s f bng cch gii phng trnh
Vy lim Sn =

f (x) = x x2 x + 1 = x x2 2x + 1 (x 1)2 x = 1.
Do vy, trong li gii bi ton 6, ta s xt un+1 1 suy ra
1
1
1
=

, n = 1, 2, . . .
un
un 1 un+1 1
T y ta rt gn c tng cn tnh gii hn.
Bi ton 7. [ thi chnh thc OLYMPIC 30/04/2006]Cho dy (xn ) nh sau: x1 = 5 v
1
xn+1 = (x2n xn + 9), n = 1, 2, . . .
5

(1)

n
P

1
.
n+ i=1 xi + 2

Tm lim

1
(xn 3)2 0, n = 1, 2, . . .
5
Suy ra: xn xn1 xn2 x2 x1 = 5 > 3
T (2) v (3) suy ra: 3 < 5 = x1 < x2 < < xn < xn+1 <
Ta c:
Gii. T (1) ta c xn+1 xn =

5 (xn+1 3) = x2n xn 6 = (xn 3) (xn + 2)


1
5

=
(v xn 5).
xn+1 3
(xn 3) (xn + 2)
48

(2)
(3)
(4)

Vy

1
xn+1 3

1
1

, n N , hay
xn 3 xn + 2
1
1
1
=

, n N .
xn + 2
xn 3 xn+1 3

Do
n
X


1
1
1
+
+ +
x1 + 2 x2 + 2
xn + 2
i=1

 



1
1
1
1
1
1

+ +

=
x1 3 x2 3
x2 3 x3 3
xn 3 xn+1 3


1
1
1
1
=

=
, n = 1, 2, . . .
x1 3 xn+1 3
2 xn+1 3
1
=
xi + 2

Trng hp 1: Dy (xn ) b chn trn. Khi dy (xn ) tng v b chn trn nn c gii hn hu
hn. t lim un = L. T
n+

5 = x1 < x2 < x3 < < xn < xn+1 <


suy ra L 5. T (1) cho n tin n + ta c
1
L = (L2 L + 9) L2 6L + 9 = 0 L = 3.
5
iu ny mu thun vi L 5. Vy trng hp 1 khng th xy ra.
Trng hp 2: Dy s (xn ) khng b chn trn. Khi do (4) nn:
lim xn = + lim xn+1 = + lim

n+

n+

Do
lim

n+

n
X
i=1

n+

1
= lim
xi + 2 n+

1
1

2 xn+1 3

1
xn+1 3


= 0.

1
= .
2

1
1

, n N , sau y ta s trnh by
xn 3 xn+1 3
mt phng php khc so vi phng php tm im bt ng ca hm c trng ( c trnh
by lu ngay sau bi ton 6 trang 47), l a thm tham s vo ri la chn. Gi s


1
1
1
1
a(xn+1 xn )
=a

=
xn + 2
xn + b xn+1 + b
xn + 2
(xn + b)(xn+1 + b)
(xn + b)(xn+1 + b) = a(xn + 2)(xn+1 xn )
xn xn+1 + bxn + bxn+1 + b2 = a(xn xn+1 x2n + 2xn+1 2xn )
(2)
(2a b)xn+1 + (a 1)xn xn+1 = ax2n + (2a + b)xn + b2 .

Lu . tm biu thc sai phn 1xn + 2 =

So snh (1) v (2), thy rng cn phi chn

a
1


a 1 = 0,
=
a=1
2a b
5

2
2a
+
b
1
b
9
b = 3.

= ,
=
2a b
5 2a b
5
1
1
Nh vy, ta cn chng minh: 1xn + 2 =

, n N . Qua y, ta thy rng


xn 3
xn+1 3
phng php s dng im bt ng ca hm c trng gip ta tm ra li gii nhanh hn so vi
phng php a thm tham s vo ri la chn.
49

Bi ton 8. [ thi HSG tnh Gia Lai, nm hc 2006-2007] Cho dy {xn }+


n=1 xc nh nh sau:
x1 = 1 v
(1)
xn+1 = x2n + 3xn + 1, n = 1, 2, 3, . . .
n
P
1
, n = 1, 2, . . . Tm lim yn .
t yn =
n+
i=1 xi + 2
Bi ton 9. [ thi HSG tnh Gia Lai-2003]Cho dy s (un ) nh sau:

u1 = 1
u2
un+1 = n + un , n = 1, 2, . . . ()
2003


un
u1 u2
+
+ +
.
Tm lim
n+
u2 u3
un+1
Bi ton 10. [Olympic ton Sinh vin ton quc-2010] Cho dy s {xn } xc nh bi

x1 = 1, xn+1 = xn 1 + x2010
, n 1.
n
 2010

x1
x2010
x2010
2
n
Tm lim
+
+ +
.
n+
x2
x3
xn+1
Bi ton 11. Cho s thc a 1. Xt dy s {xn } xc nh bi x1 = a v xn+1 = xn (1 + x2010
), n
n
1. Tm

x2010
x2010
x2010

n
lim 1 x2 + 2 x3 + +
.
x
n+1
n+
x2 +
x3 +
xn+1 +
x1
x2
xn
Bi ton 12. Xt dy s {xn }+
n=1 nh sau: x1 = 2 v
xn+1 =


1 2
xn + 1 , n = 1, 2, . . .
2

(1)

1
1
1
+
++
. Hy tm phn nguyn ca S2013 v tnh gii hn ca Sn
1 + x1 1 + x2
1 + xn
khi n tng ln v hn.

t Sn =

Bi ton 13. [ ngh OLYMPIC 30/04/2004]T dy s (un ) c xc nh bi:

u1 = 2
u2 + 2003un
un+1 = n
, n N ,
2004
ta thnh lp dy s (Sn ) vi Sn =

n
P

ui
. Tm lim Sn .
n+
i=1 ui+1 1

Bi ton 14. [ ngh thi OLYMPIC 30/04/2004] Cho dy s (Un ) xc nh bi U1 = a (a > 0


cho trc) v
Un2 2004Un+1 + 2003Un = 0 (n = 1, 2, . . . ).
n
P
Ui
Tm a lim
= 2004.
n+ i=1 Ui+1 1

50

Bi ton 15. Cho dy s (xn ) (n = 1, 2, . . .) c xc nh nh sau:


p
x1 = 1; xn+1 = xn (xn + 1)(xn + 2)(xn + 3) + 1 , n = 1, 2, . . .
t yn =

n
P

1
, n = 1, 2, . . . Tm lim yn .
n+
i=1 xi + 2

Bi ton 16. [ ngh Olympic 30/04/2011] Dy s (xn ) c xc nh nh sau: x1 = 1 v


p
xn+1 = xn (xn + 5) (x2n + 5xn + 8) + 16, n = 1, 2, . . .
t yn =

n
P

1
. Tm lim yn .
n+
i=1 xi + 3

Bi ton 17. [ thi OLYMPIC ton Sinh Vin ton quc nm 2001] Cho hm s f (x) xc nh
trn [1; +) v tha mn f (1) = a > 0 v
f (x + 1) = 2001f 2 (x) + f (x), x [1; +).
Hy tnh

lim

n+


f (1) f (2)
f (n)
.
+
+ +
f (2) f (3)
f (n + 1)

Bi ton 18. Cho dy s (xn ) nh sau: x1 = a > 0 v


xn+1 = x2n + xn , n = 1, 2, . . .


1
1
1
Tm lim
+
+ +
.
n+
1 + x1 1 + x2
1 + xn
n
P

1
, bit rng
n+ i=1 xi 1

Bi ton 19. Tm lim

x1 = 3; xn+1 = x2n 3xn + 4, n N .


Bi ton 20. [China Girls Math Olympiad-2003]Cho dy s (an )+
n=1 nh sau: a1 = 2 v
an+1 = a2n an + 1, n N .
Chng minh rng 1

(1)

2003
P 1
1
<
< 1.
20032003
i=1 ai

Bi ton 21. [Olympic Ton Sinh vin Ton quc nm 2007] Cho a, b, c, l cc s thc tha
mn 6= c b. Xt dy s (un ), (vn ) xc nh bi:
n

u1 = a, un+1 =

X
u2n + bun
uk
, vn =
.
c
u
k+1 + b c
k=1

Bit rng lim un = . Tm gii hn ca dy s (vn ).


n+

Bi ton 22. Dy s v hn {un }, n = 1, 2, . . . xc nh nh sau:



u1 = 1
un+1 = 1 + u1 u2 . . . un , n = 1, 2, . . .
t Sn =

n 1
P
. Hy tm lim Sn .
n+
k=1 uk

51

Bi ton 23. Cho dy s (xn )+


n=1 nh sau: x1 = a > 2 v
xn+1 = x2n 2, n = 1, 2, . . .


1
1
1
1
Tm lim
+
+
+ +
.
n+
x1 x1 x2 x1 x2 x3
x 1 x 2 . . . xn
Bi ton 24. Cho dy s (xn )+
n=1 nh sau: x1 = a > 1 v
xn+1 = x2n + 2xn 2, n N .
Tm


lim

n+


1
1
1
+
+ +
.
1 + x1 (1 + x1 )(1 + x2 )
(1 + x1 )(1 + x2 ) . . . (1 + xn )

Bi ton 25. Cho m l s thc ln hn 1 v a l s thc ln hn 4. Xt dy s (xn ) nh sau:


x1 = a v
xm+1 + 3xn + 16
xn+1 = n m
, n = 1, 2, . . .
(1)
xn xn + 11
n
P
1
t yn =
, n = 1, 2, . . . Tm lim yn .
m
n+
i=1 xi + 7
m+1
x
+ 3x + 16
. Khi
Gii. Xt hm s f (x) =
m
x x + 11
xm+1 + 3x + 16
xm+1 4xm + 7x 28

4
=
xm x + 11
xm x + 11
(x 4) (xm + 7)
xm (x 4) + 7 (x 4)
=
.
=
xm x + 11
(xm + 7) (x 4)

f (x) 4 =

Vy nu x > 4 th
1
(xm + 7) (x 4)
1
1
=
=
m
.
m
f (x) 4
(x 4) (x + 7)
x4 x +7

(2)

T (2) ta c
1
xn+1 4

1
1
1
1
1
m
m
=

.
xn 4 xn + 7
xn + 7
xn 4 xn+1 4

Mt khc vi x > 4 th
xm+1 + 3x + 16
> x 3x + 16 > x2 + 11x
xm x + 11

x2 8x + 16 > 0 (x 4) > 0 ng .

f (x) > x

Vy quy np theo n ta chng minh c xn > 4, n = 1, 2, . . . v


a = x1 < x2 < x3 < +.
Do

n
X


n 
X
1
1
1
1
1
yn =
=

.
m
x
+
7
x

4
x

4
x

4
x

4
i
i+1
1
n+1
i
i=1
i=1


1
1
1
Suy ra lim yn = lim

=
.
n+
n+
x1 4 xn+1 4
x1 4
52

(3)

Bi ton 26. [Tp ch Ton hc v Tui tr s 403, thng 1, nm 2011] Xt dy s (xn ) nh sau:
x1 = 5 v
x2010 + 3xn + 16
, n = 1, 2, . . .
xn+1 = n2009
xn xn + 11
n
P
1
t yn =
, n = 1, 2, . . . Tm lim yn .
2009
n+
+7
i=1 xi
Bi ton 27. [ ngh thi Olympic 30/04/2012] Xt dy s (xn ) nh sau: x1 = 3 v
xn+1 =
t yn =

n
P

2011
i=1 xi

+4

x2012
+ 2xn + 4
n
, n = 1, 2, . . .
2011
x n xn + 6

, n = 1, 2, . . . Tm lim yn .
n+

Bi ton 28. [ thi chnh thc Olympic 30/04/2012] Cho dy s (xn ) nh sau: x1 = 4 v
xn+1 =

x4n + 9
, n = 1, 2, . . .
x3n xn + 6

a) Chng minh rng lim xn = +.


n+

b) Vi mi s nguyn dng n, t yn =

n
P
3
k=1 xk

1
. Tnh lim yn .
n+
+3

Bi ton 29. Cho dy s (xn ) c xc nh bi x1 = 3, 01 v


p
xn 3 + x2n + 10xn 3
, n N .
xn+1 =
2

(1)

n
P
1
Vi mi s nguyn dng n, t yn =
. Tm lim yn .
2
n+
i=1 xi+1 9

x 3 + x2 + 10x 3
Gii. Xt hm s f (x) =
. Ta c f (3) = 3 v vi x > 3 ta c
2

x 3 + x2 + 10x 3
x 3 + x2 + 6x + 9
f (x) =
>
= x.
2
2

Do x1 > 3 nn quy np theo n suy ra xn > 3, n = 1, 2, . . . . T


xn+1 xn = f (xn ) xn > xn xn = 0.
Suy ra
3, 01 = x1 < x2 < x3 < < xn < xn+1 <

(2)

Nu dy (xn ) b chn trn th t (2) suy ra dy (xn ) c gii hn hu hn. t lim xn = L. T


n+

(2) suy ra L > 3, 01. T (1) cho n + ta c

L 3 + L2 + 10L 3
L=
L + 3 = L2 + 10L 3
2
L2 + 6L + 9 = L2 + 10L 3 L = 3.

53

n y ta gp mu thun. Vy dy (xn ) khng b chn trn. T (2) suy ra lim xn = +. T


n+

(1) suy ra
2xn+1 xn + 3 =

p
x2n + 10xn 3

(2xn+1 xn + 3)2 = x2n + 10xn 3


4x2n+1 + x2n + 9 4xn+1 xn + 12xn+1 6xn = x2n + 10xn 3
4x2n+1 4xn+1 xn + 12xn+1 = 16xn 12
x2n+1 xn+1 xn + 3xn+1 = 4xn 3
x2n+1 9 = xn+1 xn 3xn+1 + 4xn 12
x2n+1 9 = (xn 3) (xn+1 + 4)
1
xn+1 + 4
xn+1 + 3 + 1
1
1

= 2
=
=
+ 2
2
xn 3
xn+1 9
xn+1 9
xn+1 3 xn+1 9
1
1
1
=

2
xn+1 9
xn 3 xn+1 3

n
n 
X
X
1
1
1
1
1
yn =
=

2
x 9
xi 3 xi+1 3
x1 3 xn+1 3
i=1 i+1
 i=1

1
1
1

= 100.
lim yn = lim
=
n+
n+
x1 3 xn+1 3
x1 3
Lu . Xt v phi ca (1), khi xy dng ton ta c tnh chn h s ca xn v h s ca
x2n sao cho bc bin i rt gn yn ta gin c c x2n ( (1) th hai h s ny bng nhau).
Bi ton 30. Cho a R. Xt dy s (un ) nh sau: u1 = a4 2a2 2 v
p
un + 5 u2n 14un + 21
un+1 =
, n N .
2
n
P
1
Vi mi s nguyn dng n, t yn =
. Hy tnh cc gii hn
2
i=1 ui+1 4ui+1 5
un
lim
.
n+ n

(1)
lim yn ,

n+

Bi ton 31. Cho dy s (xn ) c xc nh bi x1 = 2, 1 v


p
xn 2 + x2n + 8xn 4
xn+1 =
, n N .
2
n
P
1
Vi mi s nguyn dng n, t yn =
. Tm lim yn .
2
n+
i=1 xi+1 4
Bi ton 32. [HSG Quc gia-2009]Cho dy s (xn ) nh sau: x1 =

1
v
2

p 2
xn1 + 4xn1 + xn1
xn =
, n = 2, 3, . . .
2
n 1
P
c gii hn hu hn khi n + v tm gii hn .
Chng minh rng dy (yn ), vi yn =
2
i=1 xi
Bi ton 33. Cho hm s f : R R tha mn cc iu kin f (1) = 2011 v
f (x + 1).f (x) = [f (x)]2 + f (x) 1, x R.
t S1 =

n
P

n
P
1
1
, S2 =
. Tm lim (S1 + S2 ).
n+
i=1 f (i) 1
i=1 f (i) + 1

54

Bi ton 34. [ thi hc sinh gii quc gia, bng A, nm 2004] Xt dy s {xn }+
n=1 nh sau:
x1 = 1 v vi mi n = 1, 2, . . . , th
xn+1 =

(2 + cos 2) xn + cos2
,
(2 2 cos 2) xn + 2 cos 2

trong l mt tham s thc. Tm tt c cc gi tr ca dy s {yn }, vi yn =


n
P
1
, n = 1, 2, . . . c gii hn hu hn khi n +. Hy tm gii hn ca dy s
k=1 2xk + 1
{yn } trong cc trng hp .
Bi ton 35. [ ngh Olympic 30/04/2011] Cho dy s (xn ) xc nh bi x1 = 2011 v
xn+1
t yn =

x2n + (1 n)xn + n2 + n + 1
, n = 1, 2, . . .
=
n+1

n
P

1
. Chng minh rng dy (yn ) c gii hn hu hn v tnh gii hn .
k=1 1 + xk

Bi ton 36. [ ngh thi OLYMPIC 30/04/2006] Cho dy s (an )+


n=1 xc nh bi cng thc:
n

P
1
, n = 1, 2, . . . Chng minh rng lim an = 2.
an =
Cnk
n+

k=0

1.4

Nhng dy sinh bi tng phi s dng cc nh gi

Bi ton 37. Cho dy s {un }+


n=1 c xc nh bi:
un =

n
X
(1)k1
k=1

(n = 1, 2, . . . ).

Chng minh rng dy s ny c gii hn hu hn v tm gii hn .


n (1)k1
P
1 1 1
1
= 1 + + + (1)n1 . . V vy
Gii. Ta c un =
k
2 3 4
n
k=1
u2m =

2m
X
(1)k1
k=1

=1

1 1 1 1 1
1
+ + +
2 3 4 5 6
2m

1 1 1 1
1
1
= 1 + + + + + +
+
 2 3 4 5
 2m 1 2m
1 1 1
1
+ + + +
2
2 4 6
2m
2m
m
2m
m
2m
m
X1
X 1
X1
X
X
1
1 X 1
=
2
=

=
=
.
k
2k
k
k k=m+1 k
m+k
k=1
k=1
k=1
k=1
k=1
Ta chng minh
ln(x + 1) < x < ln(1 x), x (0; 1).

(*)

Xt cc hm s: f (x) = x ln(1 + x), g(x) = x + ln(1 x). Ta cn chng minh: vi 0 < x < 1
th f (x) > 0 v g(x) < 0. Ta c f v g lin tc trn [0; 1) v
1
x
=
0, x [0; 1)
1+x
1+x
1
x
g 0 (x) = 1
=
0, x [0; 1).
1x
1x
f 0 (x) = 1

55

Vy f (x) tng trn [0; 1) v g(x) gim trn [0; 1). Ta c f (0) = 0 v g(0) = 0. Do vi
0 < x < 1 th f (x) > f (0) = 0 v g(x) < g(0) = 0. C ngha l () c chng minh. Do vi
1
(k = 1, 2, . . . , m) th
x=
m+k
ln

m+k+1
1
m+k1
<
< ln
.
m+k
m+k
m+k

Vy
ln (m + k + 1) ln(m + k) <

1
< ln(m + k) ln (m + k 1), k = 1, m.
m+k

Do
1
< ln(m + 1) ln (m),
m+1
1
ln (m + 3) ln(m + 2) <
< ln(m + 2) ln (m + 1),
m+2
1
< ln(m + 3) ln (m + 2),
ln (m + 4) ln(m + 3) <
m+3
.........................................................
1
ln (2m + 1) ln(2m) <
< ln(2m) ln (2m 1).
2m
ln (m + 2) ln(m + 1) <

Cng li v theo v ta c:
ln(2m + 1) ln(m + 1) <

m
X
k=1

1
< ln(2m) ln m
m+k

m
X

2m + 1
1
2m
<
< ln
m+1
m+k
m
k=1

 X
m
1
1
ln 2
< ln 2.
<
m+1
m
+
k
k=1
ln

V lim

m+


ln 2

1
m+1


= ln 2,

lim (ln 2) = ln 2, nn s dng nguyn l kp suy ra

m+

lim

m+

m
X
k=1

1
= ln 2.
m+k

Vy lim u2m = ln 2. Mt khc v u2m+1 = u2m +


m+

1
nn
2m + 1

1
= lim u2m .
m+ 2m + 1
m+

lim u2m+1 = lim u2m + lim

m+

m+

Tm li lim u2m+1 = lim u2m = ln 2, do lim un = ln 2.


m+

m+

n+

Bi ton 38. Cho dy s {un }+


n=1 c xc nh bi:
un =

n
X
k=1

(1)k1

2k + 1
(n = 1, 2, . . . ).
k(k + 1)
56

Chng minh rng dy s ny c gii hn hu hn v tm gii hn .


2k + 1
1
1
Gii. Ta c
= +
, do vi mi n = 1, 2, . . . , ta c:
k(k + 1)
k k+1
un =

n
X

(1)

k1

k=1

1
1
+
k k+1


=

n
X
(1)k1
k=1

n
X
(1)k1
k=1

k+1

Theo bi ton 37 trang 55 ta c


lim

n
X
(1)k1

n+

k=1

= ln 2

Ta c:
n
X
(1)k1
k=1
n
X
k=1

Do :
lim

n+

k+1

1 1 1 1
(1)n1
+ + +
2 3 4 5
n+1

(1)k1
1 1 1 1
(1)n1
= 1 + + +
.
k
2 3 4 5
n

n
X
(1)k1
k=1

k+1

"
= lim

n+

n
X
(1)k1
k=1

Bi vy lim un = ln 2 + 1 ln 2 = 1.
n+

57

#
= 1 ln 2.

Mt s dy s sinh bi
cc hm lng gic
Trng Ngc c, THPT Chuyn L Qu n, Bnh nh
Trong qu trnh ging dy cho hc sinh chuyn Ton v chuyn dy s, ti nhn thy ta
c th xy dng dy s xc nh bi cc hm s lng gic nh s dng cc cng thc lng gic.
Cc dy s c xc nh bi cc hm s lng gic thng cho bi h thc truy hi kh phc tp
v cc s hng ca chng li l nhng s v t cng lm cho hc sinh khi gii chng kh pht hin
ra cc quy lut ca dy s.
Ta xem xt bi ton thi hc sinh gii Ton quc gia nm 2014.
Bi ton 1 (VMO2014). Cho hai dy s dng (xn ) , (yn ) xc nh bi x1 = 1, y1 =
(
xn+1 yn+1 xn = 0
x2n+1 + yn = 2

3 v

vi mi n = 1, 2, ...
Chng minh rng hai dy s trn hi t v tm gii hn ca chng.
Phn tch. y l mt bi ton cho hai dy s tha mn mt h thc truy hi kh phc tp.
Ta tip cn bng cch tnh vi gi tr c bit ban u ca hai dy s.
p
p

1
Ta c x22 + y1 = 2 x2 = 2 3 v x2 y2 x1 = 0 y2 = p
= 2 + 3.
2 3
q
q
p
p

Tng t, ta cng c x3 = 2 2 + 3, y3 = 2 + 2 + 3.
T y ta nhn thy x2n + yn2 = 4(*) ng khi n = 1, 2, 3. Ta c hai cch gii sau.
Li gii 1.
Ta chng minh h thc (*) ng theo quy np.
Gi s (*) ng n n 1, t cng thc xc nh hai dy s v gi thit quy np, ta c
x2
4 yn2
2
2
x2n+1 = 2 yn , yn+1
= 2n =
= 4.
= 2 + yn x2n+1 + yn+1
xn+1
2 yn
Vy theo nguyn l quy np (*) ng vi mi n nguyn dng.

T chng minh trn ta suy ra yn+1 = 2 + yn , n N . (1)


Ta chng minh
dy s (yn ) tng v b chn trn bi 2 nn c gii hn hu hn v lim yn =
2 lim xn = lim 2 yn1 = 0
 x 2  y 2
n
n
Ta vit (*) di dng
+
= 1, khi cho php ta ngh n cng thc lng gic
2
2
sin2 x + cos2 x = 1 x R v ta c li gii 2.
Li gii 2.

Ta nhn thy x1 = 1 = 2 sin , y1 = 3 = 2 cos . Ta chng minh bng quy np rng vi mi


6
6
n nguyn dng, cng thc xc nh hai dy s cho l

xn = 2 sin
. (2)
n , yn = 2cos
3.2
3.2n
Thy vy, vi n = 1 mnh (2) ng. Gi s mnh (2) ng n n 1. p dng cng
thc truy hi, ta c
r
r

.
xn+1 = 2 yn = 2 2cos n = 4sin2 n+1 = 2 sin
3.2
3.2
3.2n+1
58


2 sin
xn
3.2n = 2 cos .
yn+1 =
=

xn+1
3.2n+1
2 sin
n+1
3.2
Theo nguyn l quy np ton hc, ta c (2) ng vi mi n nguyn dng.




T y ta c lim xn = lim 2 sin
= 0, lim yn = lim 2cos n = 2 .
3.2n
3.2
Nhn xt 1. Hai dy s ca bi ton trn c xy dng t hai hm lng gic sin v cosin v
s dng cc cng thc lng gic:
x
x
1
x
sin x = 2 sin cos ; cos2 = (1 + cos x) .
2
2
2
2
Hon ton tng t, ta cng c th xy dng c cc dy s xc nh bi cc hm s lng
gic, nh s dng cc cng thc lng gic.

2
Bi ton 2. Cho hai dy s dng (xn ) , (yn ) xc nh bi x0 =
, y0 = 1 v
2
(
xn yn+1 = x2n+1
xn + 2yn x2n+1 = yn
vi mi n = 0, 1, 2, ...
Vi mi s t nhin n, t zn =

n x
Q
i
. Chng minh rng dy (zn ) c gii hn hu hn v tm
y
i=0 i

gii hn .

Li gii. Ta nhn thy x0 =

= sin
0 , y0 = 1 = tan
2
4.2
4.20

2 2

2
T cng thc xc nh dy s, ta li c x0 + 2y0 x1 = y0 x1 =
= sin
v
2
4.2

2x2
.
y1 = 1 = 2 1 = tan
x0
4.2
Ta chng minh bng quy np rng vi mi n l s t nhin, cng thc xc nh hai dy s

cho l xn = sin
. (*)
n , yn = tan
4.2
4.2n

t n =
, n N.
4.2n
T cng thc truy hi ca dy s, ta c
1
xn
1
sin n
1 cos n
n
n
x2n+1 =
=
=
= sin2
xn+1 = sin
2 2yn
2 2 tan n
2
2
2
V
n
n
2sin2
sin
2x2n+1
2 =
2 = tan n .
yn+1 =
=

n
xn
sin n
2
cos
2
Theo nguyn l quy np ton hc, ta c (*) ng vi mi n l s t nhin.
Vy

x0
2

x0 x1 ...xn

z0 =
=
= cos , ..., zn =
= cos
0 cos
1 ... cos
y0
2
4
y0 y1 ...yn
4.2n
4.2
4.2
p dng cng thc lng gic, ta c

59

n = sin
n cos
n cos
n1 ... cos
4.2
4.2
4.2
4.2
4.20
1

= sin
= n+1
n1 cos
n1 ... cos
0 = ... = n+1 sin
2
2
2
2
4.2
4.2
4.2

zn sin

Suy ra
zn =

1
2n+1 sin

4.2n

v lim zn = lim 4.2 . = .


2
2
sin
4.2n

Bi ton 3. Cho hai dy s dng (xn ) , (yn ) xc nh bi x0 =

2 v

xn yn = 2 4 x2n
2
4 yn+1
xn = 2
2
4 + yn+1
vi mi n = 0, 1, 2, ... Chng minh rng hai dy s trn hi t v tm gii hn ca chng.

Li gii. Ta nhn thy x0 = 2 = 2


= 2 cos
2
4.20
p

.
Vi n = 0 x0 y0 = 2 4 x20 y0 = 2.1 = 2 tan
4.20
T cng thc truy hi ca dy s, ta c

p
2
16

2+ 2

x1 y 1 = 2 4 x1
x2 =

x
=
2
cos
1
2
1 = 2
2
4+

4 y1
2
y1
4.2
2
x0 = 2
2

2
y1 = 2( 2 1) = 2 tan
y1 = 4( 2 1)
4 + y1
4.2
Ta chng minh bng quy np rng vi mi n l s t nhin, cng thc xc nh hai dy s

cho l xn = 2 cos
. (*)
n , yn = 2 tan
4.2
4.2n

t n =
, n N. T cng thc truy hi ca dy s, ta c
4.2n

16
16
n
2

= 4cos2

xn+1 = 4 + y 2 =

n
16

n+1
4 + 4tan2

x2n+1 =
2
2
4 + yn+1
n

sin2
2

x
n

2
2

2
cos

yn+1 = 4
n
2 = 4tan2 n
2

yn+1
=4
=4

2 + xn

2 + 2 cos n
2
cos2
2

xn+1 = 2 cos n
2

yn+1 = 2 tan n
2
.
Theo nguyn l quy np ton hc, ta c (*) ng vi mi n l s t nhin.




T y ta c lim xn = lim 2 cos
=
2,
lim
y
=
lim
2
tan
= 0.
n
4.2n
4.2n

60

Bi ton 4. Cho hai dy s dng (xn ) , (yn ) xc nh bi x0 =


(
2x2n+1 = xn yn+1
x2n+1 yn + 2 (xn yn ) = 0

2, y0 = 2 v

vi mi n = 0, 1, 2, ...
Chng minh rng hai dy s trn hi t v tm gii hn ca chng.
Li gii. Ta nhn thy x0 =

2=2

= 2 sin
0 v y0 = 2 = 2 tan
2
4.2
4.20

T cng thc truy hi ca dy s vi n = 0 x21 y0 + 2(x0 y0 ) x21 = 2

2x0
= 2 2
y0

2x2

v y1 = 1 = 2( 2 1) = 2 tan
.
4.2
x0
4.2
Ta chng minh bng quy np rng vi mi n l s t nhin, cng thc xc nh hai dy s
cho l

xn = 2 sin
. (*)
n , yn = 2 tan
4.2
4.2n

t n =
, n N.
4.2n
T cng thc truy hi ca dy s, ta c
x1 = 2 sin

16
2
4 + yn+1
2 xn
=4
2 + xn

x2n+1 =

2
yn+1

2xn
4 sin n

2 n
2

=
2

x
=
2

=
2

2
cos

=
4sin

n
n+1

yn
2 tan n
2

2 n
4sin
2

2 = 2 tan n

yn+1 = 2 n+1 = 2
xn
2 sin n
2

xn+1 = 2 sin n
2

yn+1 = 2 tan n
2

Theo nguyn l quy np ton hc, ta c (*) ng vi mi n l s t nhin.






T y ta c lim xn = lim 2 sin
= 0, lim yn = lim 2 tan
= 0.
4.2n
4.2n
Bi ton 5. Cho hai dy s dng (xn ) , (yn ) xc nh bi x0 = 1 v

xn = p 2yn
4 + yn2

2
2
2
4xn+1 + xn yn+1
= 2yn+1
xn
.
yn
Chng minh rng dy (zn ) c gii hn hu hn v tm gii hn .

vi mi n = 0, 1, 2, ... Vi mi s t nhin n, t zn =

Li gii. Ta nhn thy x0 = 1 = 2 cos


3
p
4
1

V vi n = 0 x0 4 + y02 = 2y0 y02 = y0 = 2 = 2 cot .


3
3
3
T cng thc truy hi ca dy s, ta c

61

y12

x1 4 + = 2y1

4x21 + y12 = 2y12

x1 = 3 = 2 cos
y1 = 2x1
3.2

y1 = 2 3 = 2 cot
y12 = 12
3.2

Ta chng minh bng quy np rng vi mi n l s t nhin, cng thc xc nh hai dy s

cho l xn = 2 cos
. (*)
n , yn = 2 cot
3.2
3.2n

t n =
, n N.
3.2n
T cng thc truy hi ca dy s, ta c

2
yn+1

= 2yn+1
xn+1 4 +
2
2
2
4xn+1 + 2 cos n yn+1
= 2yn+1

2
2
x2n+1 (4 + yn+1
) = 4yn+1

1
n 2
2
x2n+1 = (1 cos n ) yn+1
= sin2 yn+1
2
2

1 sin2

2
2
y2 = 4
xn+1 = 2 cos n
sin2
4 + yn+1
=4
n+1

n
2

sin2
2n
2

x2n+1 = sin2 n yn+1


2
y
=
2
cot

n+1

2
2
x2 = sin2 n y 2
n+1
n+1
2

Vy xn = 2 cos
, yn = 2 cot
3.2n
3.2n
Theo nguyn l quy np ton hc, ta c (*) ng vi mi n l s t nhin.

xn
= sin
Vy zn =
lim zn = 0.
yn
3.2n

Bi ton 6. Cho hai dy s dng (xn ) , (yn ) , (zn ) xc nh bi x0 = 3, y0 = 1 v

xn = yn zn
xn = xn+1 yn+1

x2n+1 + yn = 2
vi mi n = 0, 1, 2, ...
Chng minh rng ba dy s trn hi t v tm gii hn ca chng.

x0

3 = 2 sin , y0 = 1 = 2 cos v z0 =
= 3 = tan .
3
3
y0
3
T cng thc xc nh dy s, ta li c

(
(

x1 = 1 = 2 sin
x0 = x1 y1
x1 y1 = 3
3.2

y1 = 3 = 2 sin
x21 = 2 y0
x21 = 2 1
3.2
x1
1

v z1 =
= = tan
y1
3.2
3
Ta chng minh bng quy np rng vi mi n l s t nhin, cng thc xc nh ba dy s
cho l

xn = 2 sin
. (*)
n , yn = 2 cos
n , zn = tan
3.2
3.2
3.2n
Li gii. Ta nhn thy x0 =

62


, n N.
3.2n
Vi n = 0, 1th (*) ng. Gi s (*) ng n n 1, ta phi chng minh (*) ng vi n + 1.
Theo cng thc truy hi v gi thit quy np, ta c

(
xn+1 = 2 sin

xn+1 = 2 sin n

2
xn+1 yn+1 = 2 sin n
sin n

2n
y
=
n+1

x2n+1 = 2 2 cos n

y
=
2
cos
n

n+1

sin
2
2
n
xn+1
= tan .
v zn+1 =
yn+1
2
Theo nguyn l quy np ton hc, ta c (*) ng vi mi n l s t nhin.
T y ta c





lim xn = lim 2 sin
=
0,
lim
y
=
lim
2
cos
=
2,
limz
=
lim
tan
= 0.
n
n
3.2n
3.2n
3.2n
Hon ton tng t cch gii cc bi ton trn, ta c th gii c cc bi tp sau.
t n =

Bi tp

Bi tp 1. Cho hai dy s dng (xn ) , (yn ) xc nh bi x0 = 1 v

p
xn 4 + yn2 = 4
2 xn
2
yn+1
=4
2 + xn
vi mi n = 0, 1, 2, ...
Chng minh rng hai dy s trn hi t v tm gii hn ca chng.

2
, y0 = 2 v
Bi tp 2. Cho hai dy s dng (xn ) , (yn ) xc nh bi x0 =
2
(
4x2n+1 + yn = 2
4
16x4n+1 + yn+1
= 16 8x2n
vi mi n = 0, 1, 2, ...
Chng minh rng hai dy s trn hi t v tm gii hn ca chng.

Bi tp 3. Cho hai dy s dng (xn ) , (yn ) xc nh bi x0 = 3, y0 = 2 3 v

4yn
x =
n
yn+1 yn + 4

2xn = yn+1 (4 x2n+1 )


xn
.
yn
Chng minh rng dy (zn ) c gii hn hu hn v tm gii hn .

Bi tp 4. Cho hai dy s dng (xn ) , (yn ) xc nh bi x0 = 3, y0 = 1 v


(
2
x2n+1 = 2xn + yn+1
xn+1 yn+1 = yn
vi mi n = 0, 1, 2, ... Vi mi s t nhin n, t zn =

vi mi n = 0, 1, 2, ...
Chng minh rng hai dy s trn hi t v tm gii hn ca chng.
63

2
2
Bi tp 5. Cho hai dy s dng (xn ) , (yn ) , (zn ) xc nh bi x0 =
, y0 =
v
2
2

yn = xn zn

1 xn
zn+1 =

1 + xn

y = 2x y
n
n+1 n+1
vi mi n = 0, 1, 2, ...
Chng minh rng ba dy s trn hi t v tm gii hn ca chng.

Ti liu tham kho


[1] thi Olympic ca cc nc
[2] Ti liu Gp g ton hc 2014 ti TPHCM

64

p dng mnh trong cc


bi ton logic
Phm ng Long
Trng HKH T nhin, HQG H Ni
Mnh ton hc l loi mnh ch c th cho gi tr ng hoc Sai. Khc vi cc loi mnh
vn hc, chng hn: i T quc giang sn hng v! (cu cm thn), Thy Mu i! (cu
gi), Gi g y? (cu hi),. . .
Trong bi bo ny ta gi mnh ton hc n gin l mnh v m ha gi tr ng l 1
v Sai l 0.

Mt s khi nim v cng thc trong ton mnh


Cc php ton mnh c bn sau:

nh ngha(1 (Php hi ). Php hi ca hai mnh : A v B, A and B, A B, k hiu l A.B


a,
hay AB hay
B.
AB = 1 (A = 1 v B = 1) (ng thi),
A.B = 0 (A = 0 hoc B = 0).
nh ngha 2 (Php
tuyn). Php tuyn ca hai mnh : A hoc B, A or B, k hiu l A B

A,
hay A + B hay
B.
A+B = 1 (A = 1 hoc B = 1),
A+B = 0 (A = 0 v B = 0) (ng thi).
nh ngha 3 (Php hoc loi tr ). Php hoc loi tr ca hai mnh : A hoc loi tr vi B,
A xor B, k hiu l A B.
A B = 1 (A =1 v B = 0) hoc (A = 0 v B = 1), (A, B khng cng gi tr),
A B = 0 (A =1 v B = 1) hoc (A = 0 v B = 0), (A, B cng gi tr).
nh ngha 4 (Php ko theo). Php ko theo ca hai mnh : Nu A th B, if A then B, k
hiu l A B hay A B.
A B = 0 A = 1 v B = 0,
A B = 1 (A = 1 v B = 1) hoc (A = 0 v B = 0) hoc (A = 0 v B = 1).
nh ngha 5 (Php tng ng). Php tng ng ca hai mnh : A B, ch ng khi
A v B cng mt gi tr: A B = 0 (A = 1 v B = 1) hoc (A = 0 v B = 0).
nh ngha 6 (Php ph nh ). Php ph nh ca mt mnh : Ph nh ca A, not(A), k
hiu l A.
Gi tr ca A khc vi A (i khng vi nhau).
Ch 1. Hai mnh c gi tr ngc nhau gi l i khng. Chng hn: A = Ci bng ny
mu en, B = Ci bng ny khng en th A v B l i khng.

65

Nhiu mnh khng cng gi tr gi l bt ng. V d: A = Ci bng ny mu en, B =


Ci bng ny mu xanh th A v B l i khng.
Nh vy, ta c Bng gi tr cc php ton mnh nh sau:
A
0
0
1
1

B
0
1
0
1

A+B
0
1
1
1

A.B
0
0
0
1

AB
0
1
1
0

AB
1
1
0
1

AB
1
0
0
1

A
1
1
0
0

Nhiu mnh lin kt vi nhau bi [cc] php ton mnh to thnh Biu thc mnh .
Mnh Y ph thuc vo mt hay nhiu bin mnh X1, X2,. . . gi l mt Hm mnh . Hai
mnh ni vi nhau bi mt bng gi l mt ng thc mnh . ng thc mnh lun
ng vi mi b gi tr cc bin gi l Hng ng thc mnh !
V d 1. A = x chia ht cho 3 l mt hm mnh mt bin.
B = x + y l s nguyn chn l mt hm mnh hai bin.
(A + B) (AB) (A B) l mt biu thc mnh .
Mt s tnh cht ca cc php ton mnh hay Cc hng ng thc mnh :
1. Tnh giao hon: a) A.B = B.A,
b) A + B = B + A.
2. Tnh kt hp: a) (A.B).C = A.(B.C),
b) (A + B) + C = A + (B + C).
3. Tnh phn phi: a) A.(B + C) = (A.B) + (A.C),
b) A + (B.C) = (A + B).(A + C).
4. Phn t trung ho: a) A.1 = 1.A = A,
b) A + 0 = 0 + A = A.
5. Lut kh: a) A.A = A.A = 0,
b) A + A = A + A = 1.
6. Lut nut: a) A + 1 = 1,
b) A.0 = 0.
7. Lut ly ng: a) A + A = A,
b) A.A = A.
8. Ph nh kp: A = A.
9. Lut De Morgan: a) A + B = A.B,
b) AB = A + B.
10. Chuyn i php xor : A B = A .B + A.B.
11. Chuyn i php ko theo: A B = A + B.
12. Chuyn i php tko theo: A B = A.B + AB.
C nhiu cch chng minh hng ng thc mnh , nhng cch lp bng l hay c dng.
Chng minh tnh cht 10:

66

A
0
0
1
1

B
0
1
0
1

AB
0
1
1
0

A B+A.B
0
1
1
0

R rng hai v lun bng nhau!


Ngi ta chng minh c rng:

i s Boole
1. Tp hp B = 0, 1, cng cc php ton hi, tuyn v ph nh
(1.1 = 1, 1+1 = 1, 1 = 0, 1.0 = 0, 1+0 = 1, 0 = 1, 0.1 = 0, 0+1 = 1, 0.0 = 0, 0+0 = 0)
xc nh trn B l mt i s. l i s Boole.
2. Quy v ba php ton c bn: Mi biu thc mnh c th vit li di dng ch gm
ba php ton hi, tuyn v ph nh (da theo tnh cht 10, 11 v 12).
3. Biu thc tng minh: Nu bit bng gi tr ca mt biu thc th s bit tng minh biu
thc . Cng c th ni rng: Bit bng gi tr ca mt hm mnh tc l tm c hm .
Ch 2. Php XOR (hoc loi tr ) rt hay dng nn cc nh in t, tin hc thit k ra
cc t bo (chp) AND, OR, XOR v NOT.
V d 2. T bng gi tr ca mt hm hai bin A v B nh sau:
A
0
0
1
1

B
0
1
0
1

f(A,B)
0
1
1
0

Ta c biu thc tng minh f(A,B) = A .B + A.B = A B.


V d 3. Thit k hai cng tc cu thang A v B khi n ang ti th bt cng tc no cng
sng v ang sng th bt cng tc no cng tt.
Gii:
Cch 1 (in k thut): Ngy trc cha c in t th ngi ta thit k nh sau:
y A v B l cc cng tc i pha d dng kim c.
Cch 2 (in t): Ngy nay c ton mnh v in t th tng t lm nh v d 1 ta c
S = A B v ch cn mt con chp XOR l !
Gi s mt con chp XOR nh hnh bn:
A
B
A XOR B
V d 4. Tm biu thc mch in t cho ba cng tc A, B v C cho mt phng ln c ba ca
ra vo sao cho khi bt mt trong ba cng tc th n S i trng thi.
Gii:
67

Bng gi tr tm biu thc ca n S qua cc bin A, B v C nh sau:


A
0
1
0
0
1
0
1
1

B
0
0
1
0
1
1
0
1

C
0
0
0
1
0
1
1
1

S
0
1
1
1
0
0
0
1

A.B. C
A .B. C
A .B .C

T bng trn ta c
S = A.B . C + A .B. C +A .B .C + A.B.C =
= (A.B +A .B). C + (A .B + A.B).C
= (A xor B)C + AxorB C
= (A xor B) xor C.
Nh vy ta ch cn dng hai con chp XOR thi.
Ta ni nh sau :
ABC
Cn nu dng in k thut th rt phc tp: Tn dng 2 cng tc A v B, b i hai on
thay vo lp thm cng tc C chuyn mch kp nh hnh v di y:
cng tc C bt mt nht th hai kim tip xa xung pha di hay ln trn (theo hnh v)
mt cch ng thi. Hai kim kt ni vi nhau v khng truyn in sang nhau!
V d 5. Tm biu thc mch in t cho bn cng tc A, B, C v D cho mt phng ln c bn
ca ra vo sao cho khi bt mt trong bn cng tc th n S i trng thi. Khi qut ha bi
ton!
Gii:
Bng cch lp bng nh trnta cng i n cng thc
S = ((A xor B) xor C) xor D.
Ta ni nh sau :
68

ABCD
Bng cch ni thm cng tc nh th, ngi ta gii quyt bi ton n cng tc (n>1).
Khi qut:
Mch in t cho n cng tc A1, A2, . . . , An cho mt phng ln c n ca ra vo sao cho khi
bt mt trong n cng tc th n S i trng thi l
Sn = ((A1 xor A2) xor . . . An-1) xor An. (1)
Chng minh (quy np)
1)- R rng (1) ng vi n = 2 (Xem v d 3).
2)- Ga s (1) ng vi n = k, ta s chng minh n cng ng vi n = k + 1.
Ta c
Sk = ((A1 xor A2) xor . . . Ak-1) xor Ak tha mn bi ton.
Phi chng minh cng thc sau cng tha mn bi ton
Sk+1 = ((A1 xor A2) xor . . . Ak) xor Ak+1 tha mn bi ton.
Ta s ch ra l bt mt cng tc i bt k, 1 i k+1 th Sk+1 i trng thi.
Tht vy:
- Nu i k th theo gi thit Sk i trng thi m Ak+1 vn cha i trng thi dn n Sk
xor Ak+1 i trng thi lin.
- Nu i = k+1 th do Sk khng i trng thi m Ak+1 i trng thi dn n Sk xor Ak+1
dn n i trng thi lin.
T 1) v 2) suy ra Bi ton c chng minh.
Cn nu dng in k thut th rt phc tp. Nhng trc y ngi ta cng c gii php
ri (Xem hnh v!).

Mt s bi tp lgic gii bng ton mnh

Bi tp 1. Trong mt cuc iu tra c 3 nhn chng A, B v C cng ngi vi nhau v nghe


kin ca nhau. Cui cng ban iu tra hi li tng ngi tm xem ai ni ng. Kt qu l: A
v B i khng nhau, B v C i lp nhau v C th bo A v B u ni sai. Vy ban iu tra tin
ai?
Gii:
Theo u bi ta c cc ng thc sau:

69

A.B + A .B = 1,
BC + B C = 1,
C. A .B + C .A + C .B = 1

(1)
(2)
(3)

Nhn (1) v (2) ta thu c:


(AB + A B).( BC + B C) = 1,
A.B .B.C + A.B.B .C + A .B.B.C + A .B.B .C = 1.
Ta c s hng u v cui bng 0 (theo tnh cht 5), B.B = B v B.B = B (tnh cht 7), nn:
(4)
A.B .C + A .B.C = 1.
Nhn (3) v (4) v rt gn c A .B.C = 1, tc l A sai, B ng v C sai. Tm li B l nhn
chng ni ng!
Bi tp 2. C 2 lng A v B 2 bn ng. Dn lng A thi lun ni tht, hi iu ng th gt
u, sai th lc u. Dn lng B lun ni di, hi iu ng th lc u, sai th gt u. Mt ngi
khch l n mt trong hai lng , nhng khng bit mnh ang lng no, gp mt ngi dn,
khng bit dn lng no, v h hay qua li gia hai lng. Ngi khch mun hi ch mt cu
ngi dn c gt u th bit mnh ang lng A, lc th bit mnh ang lng B.
Bn hy gip ngi khch ny vi!
Gii:
Gi X l cu hi: y l lng A?, Y l cu hi: Bn l ngi lng A ?. Cu hi ca ngi
khch l mt biu thc to ra t X v t Y, v c bng gi tr nh sau:
X
1
1
0
0

Y
1
0
1
0

Q
1
0
0
1

Cu hi
(1) Q=XY
(2)
(3)
(4) Q=X Y

ngha
(1): Nu ang lng A v gp dn lng A th mun ngi gt, ta phi hi cu ng.
(2): Nu ang lng A v gp ngi dn lng B th mun ngi gt, ta phi hi cu sai.
70

(3): Nu ang lng B v gp ngi dn lng A th mun ngi lc, ta phi hi cu sai.
(4): Nu ang lng B v gp ngi dn lng B th mun ngi lc, ta phi hi cu ng.
T bng gi tr trn tm c biu thc tng minh ca cu hi:
Q = X.Y + X .Y .
Cu hi tng minh l y l lng A v bn l ngi lng A ? hoc y khng l lng A
v bn khng l ngi lng A?.
Phi hp cho gn hn, ta c:
Cu hi = Bn l ngi lng ny ?.

Lt lun

C nhiu bi ton mo c th dng ton mnh tm ra li gii mt cch chnh xc.


Tuy nhin cng khng t bi ton kh c th a v ton mnh c. . .

Ti liu tham kho


1. Sch hng dn Ton ri rc Nguyn Duy Phng Hc vin Bu chnh Vin thng
H Ni.
2. By phng php gii bi ton logic ng Huy Run Khoa Ton C hc Tin hc
Trng i hc Khoa hc t nhin H Ni.
3. 80 Bi ton thng minh H Ngc c Mng Internet. . .

71

Mt s vn lin quan n
hm phn nguyn
L Th Anh oan
THPT chuyn L Qu n, Khnh Ha
S hc l mt trong nhng ch th v v p ca ton s cp. Trong hu ht cc k thi
chn hc sinh gii th bi ton s hc thng xuyn xut hin v lun l mt thch thc ln i
vi hc sinh. Ton v phn nguyn l mt dng ton hay, l mt trong nhng nn tng c bn. S
dng thnh tho ton v phn nguyn hc sinh s c thm mt k nng hay mt cng c gii
ton, nht l gii cc bi ton lin quan n s nguyn.
B cc ca bi vit nh sau:
Phn I. Khi nim v cc tnh cht c bn ca hm phn nguyn.
Phn II. Mt s dng ton v hm phn nguyn.
Phn III. Mt s ng dng ca hm phn nguyn.
Phn IV. Hm phn nguyn trong cc thi chn hc sinh gii.
Trong phm vi bi vit ny, ti khng c tham vng a ra nhng sng kin mi, ch mun
trao i cng qu thy c ng nghip v hm phn nguyn, cc tnh cht c bn, mt s ng dng
v mt s bi ton v phn nguyn trong cc thi chn hc sinh gii m ti su tm v tp
hp li trong thi gian qua. Rt mong nhn c s gp v xy dng t qu thy c. Xin chn
thnh cm n qu thy c.

Khi nim v cc tnh cht c bn ca hm phn nguyn

nh ngha 1. Vi mi s thc x, phn nguyn (floor) ca x l s nguyn ln nht khng vt


qu x v c k hiu l bxc.
S thc {x} = x bxc c gi l phn l (fractional part) ca s thc x.
Trn (ceiling) ca x l s nguyn nh nht khng nh hn x v c k hiu l dxe.
Cc hm s y = f (x) = bxc, y = g (x) = dxe v y = h (x) = {x} c gi l hm phn
nguyn, hm trn v hm phn l.
Nhn xt 1. nh ngha v phn nguyn cn c hiu theo mt trong hai cng thc sau:
x 1 < bxc x hoc
bxc x < bxc + 1
.

 
Mnh 1. a) Nu a v b l hai s nguyn vi b > 0 v a = bq + r th q = ab v r = ab .b.
b) Vi mi s thc x v mi s nguyn n, ta c bx + nc = bxc + n v dx + ne = dxe + n.
c) Nu x l s nguyn th bxc + bxc = 0. Nu x khng l s nguyn th bxc + bxc = 1.
d) Hm phn nguyn l mt hm khng gim, tc l: x y bxc byc .
e) bxc + byc bx + yc bxc + byc + 1.
f) bxc . byc bxyc ,vi mi s thc khng m x, y.
j k  
g) Vi mi s thc x v mi s nguyn n, ta c bxc
= nx .
n
h) Vi mi s thc dngx
  v mi s nguyn dngn th s cc s nguyn dng l bi ca n
v khng vt qu x l nx .
72

nh l 1 (nh l Legendre). S
m ca s nguyn t ptrong phn tch tiu chun ca n! c
Pjnk jnk jnk jnk
tnh theo cng thc: ep (n) =
= p1 + p2 + p3 +
pi
i1

nh l 2 (nh l Hermite). Vi mi s nguyn dng n v mi s thc x, ta c






bnxc = bxc + x + n1 + + x + n1
.
n
nh l 3 (nh l Beatty). , l cc s v t dng sao cho 1 + 1 = 1. Khi 2 tp (dy)
{An }
}
thnh hai phn hoch trn tp cc s
1 = {bc , b2c , ...} v {BnT
1 = {bc , b2c , ...} lpS

nguyn dng N , tc l: {An }


{B
}
=

v
{A
}
{Bn }
n 1
n 1
1
1 = N .

Mt s dng ton v hm phn nguyn


Dng 1: Chng minh mt biu thc.

Bi ton 1. Vi mi s thc x v y, ta c b2xc + b2yc bxc + byc + bx + yc.


Gii. Ta c x = bxc+{x}v y = byc+{y}. Khi b2xc+b2yc = 2 bxc+b2 {x}c+2 byc+b2 {y}c
v bx + yc = bxc + byc + b{x} + {y}c.
Ta ch cn chng minh: b2 {x}c + b2 {y}c b{x} + {y}c.
Do tnh i xng nn ta c th gi s {x} {y}.
V {x} 0nn b2 {x}c + b2 {y}c b2 {x}c b{x} + {y}c.
Vy ta c iu phi chng minh.
j k j k
j
k
(q1)p
Bi ton 2. (Gauss) Vi hai s nguyn t cng nhau p v q, ta c pq + 2p
+

+
=
q
q
(p1)(q1)
.
2

Gii. V (p, q) = 1 nn

ip
q

khng l s nguyn. Khi

j k
ip
q

(qi)p
q

= p+

j k
ip
q

ip
q

p 1, 1 i q 1. Do
   

   


  
p
2p
(q 1) p
p
(q 1) p
(q 1) p
p
2
+
+ +
=
+
+ +
+
= (p 1) (q
q
q
q
q
q
q
q
Bi ton 3. Chng minh rng vi mi s nguyn dng n, hiu

n  
P
n
k=1

n

b nc lun l mt s chn.

n  
P
n

= {s Z+ : ks n} . nn
l s cc cp (c tnh th t) (s, k) tha mn
k
k=1

1 s, k n v ks n. D thy b nc l s cc cp (k, k) tha mn 1 k n v k 2 n nn


n  
P

n
hiu

n l s cc cp (c tnh th t) (s, k)sao cho 1 s 6= k n v sk n. Hin


k

Gii. Ta c

k=1

nhin, nu cp(s, k)tha mn tnh cht ny th cp (k, s) 6= (s, k) cng tha mn. Do s cc
cp tha mn tnh cht trn phi l mt s chn, tc l ta c iu phi chng minh.
Bi ton 4. Chng minh rng vi mi s nguyn dng n, ta c
%

 $r

1
3 1
n+
=
n +
.
2
4 2

73

Gii. t k =

k

n+

1
2

jq
k

;m=
n 34 + 12 . Ta c


1
1
1
1
1
< k + 1 k n < k + k2 k + n < k2 + k + .
n+
2
2
2
4
4

V n nguyn dng nn phi c k 2 k + 1 n k 2 + k.


Tng t
r
3 1
1
3
1
m n + < m + 1 m2 m + n < m2 + m + m2 m + 1 n < m2 + m.
4 2
4
4
4
Do ta phi c k = m. Ta c iu phi chng minh.
Dng 2: Gii phng trnh.
Bi ton 5. Tm cc nghim thc ca phng trnh:
4x2 40 bxc + 51 = 0.

(1)

Gii. Ta c

suy ra

3
2

(2x 3) (2x 17) = 4x2 40x + 51 4x2 40 bxc + 51 = 0,

40bxc51
17
x 2 v 1 bxc 8. Khi x =
hay
2
$p
%
40 bxc 51
bxc =
2

(2)

Ln lt thay bxc {1, 2, 3, 4, 5, 6, 7, 8} vo phng trnh (2). Ta thy bxc = 2, 6, 7, 8 tha


phng trnh (2). Vy cc nghim ca (1) l

29 189 229 269


;
;
;
.
2
2
2
2
Bi ton 6. Gii phng trnh:
bx bxcc = 1.
Gii. Ta c 1 bx bxcc < 2.
* x 2 bxc 2 bx bxcc 4 phng trnh v nghim.
* 1 x < 2 bxc = 1 1 bx bxcc < 2 phng trnh nghim ng mi x.
* 0 x < 1 bxc = 0 bx bxcc = 0 phng trnh v nghim.
* 1 < x < 0 bxc = 1 bx bxcc = x < 1 phng trnh v nghim.
* x = 1 bxc = 1 bx bxcc = 1 phng trnh nghim ng.
* x < 1 bxc 2 bx bxcc > 2 phng trnh v nghim.
Bi ton 7. Gii phng trnh:


Gii. t

 

2x 1
4x + 1
5x 4
+
=
.
3
6
3
2x 1
3y + 1
=yx=
.
3
2
74

Thay vo phng trnh ta c





1
5y 1
5y 1
byc + y +
=
b2yc =
.
2
2
2
t


2t + 1
4t + 2
4t + 2
5y 1
= t (t Z) y =

=t0
t < 1.
2
5
5
5


Do t Z nn t {2; 1; 0; 1; 2} y 53 ; 15 ; 15 ; 35 ; 1 Vy tp nghim ca phng trnh l


2 1 4 7
S = ; ; ; ;2 .
5 5 5 5
Bi ton 8. Gii phng trnh:
x4 = 2x2 + bxc

(3)

Gii. Ta c

(3) bxc = x2 x2 2

x2 2 2 x 2 bxc 0 bxc {1; 0}


+bxc = 0 x = 0
+bxc = 1 x = 1

x2 > 2 bxc > 0 x > 2 x2 (x2 2) =

x< 3

bxc
x

1 x2 2

q
bxc = 1 x =

1+

1
x

<1x<

2<

2.

Mt s ng dng ca hm phn nguyn

Dng 1: Cc bi ton lin quan chia ht.


Gii. Tm tt c cc s nguyn dng n sao cho tp hp An = {1, 2, ..., n} c s cc s chia ht
cho 3 bng s cc s chia ht cho 5 hoc 7 hoc 35.
      n
. Do ta cn tm
Gii. Ta c s cc s chia ht cho 3, 5, 7, 35 ln lt l n3 , n5 , n7 , 35
n n n  n 
tt c cc s nguyn dng n sao cho 3 = 5 + 7 35 . t n = 35k + r vi 0 r < 35
ta c ng thc trn tng ng vi

 j
35k + r
rk j
rk j
rk
= 7k +
+ 5k +
k+
.
3
5
7
35
r
V 0 r < 35 nn 35
= 0 v ng thc trn tr thnh



 j k j k
jrk jrk
2k + r
2k + r
r
r
11k +
= 11k +
+
hay
=
+
.
3
5
7
3
5
7

Bng cch thay trc tip r = 0, 1, 2, ..., 34 v tnh v phi ta tm c cc b s (k, r)nh sau: (0, 1) ,
(0, 2) , (0, 5) , (0, 7) , (0, 8) , (0, 10) , (0, 11) , (0, 14) , (0, 15) , (0, 16) , (0, 17) , (0, 20) , (0, 21) , (0, 22) , (0, 23) , (0,
(0, 29) , (0, 30) , (0, 31) , (0, 32) , (1, 15) , (1, 21) , (1, 30) .

75

Bi ton 9. Tm s nguyn dng n ln nht sao cho 2013! chia ht cho5n


Gii. S n cn tm l s m ca 5 trong phn tch thnh tch cc tha s nguyn t. Theo nh
l Legendre ta c
X  2013   2013   2013   2013   2013 
=
+
+
+
= 402 + 80 + 16 + 3 = 501.
n=
i
2
3
4
5
5
5
5
5
i1
Vy n = 501 l s cn tm.
Bi ton 10. Gi s k v n l cc s nguyn dng. Chng minh rng

n
n1
(k!)k +k +...+k+1 | k n+1 !.
Gii. Ta c




n
2
k!|k!, k!(k!)k | k 2 !, k 2 !(k!)k | k 3 !, ..., (k n )!(k!)k | k n+1 !
Nhn v theo v ta c
k!k! k 2 !





2
n
k 3 ! ... (k n )!(k!)k+k +...+k |k! k 2 ! k 3 ! ... k n+1 ! .

Suy ra iu phi chng minh.


Bi ton 11. Gi s n l hp s. Chng minh rng cc s Cn1 , Cn2 , ..., Cnn1 u chia ht cho n.
Gii. Gi p l mt c nguyn t ca n v s l mt s nguyn sao cho ps n ps+1 . Ta cn
s
n!
chng minh n khng l c ca Cnp = (ps )!(np
s )! .
s

V p|n nn ch cn chng minh p khng l c ca Cnp . Gi s pk |Cnp . Khi iu phi chng


minh tng ng vi k = 0. Tht vy, p dng nh l Legendre ta c

s  
s  s
s 
X
X
X  n  X  ps  X  n ps  X
n
p
n ps

k=
i
i
i
i
i
p
p
p
p
p
pi
i=1
i=1
i=1
i1
i1
i1
s  
s  s
s  
s  s
X
X
X
X
p
n
p
n
=

+
= 0,
i
i
i
i
p
p
p
p
i=1
i=1
i=1
i=1
v

j sk
p
pi

l cc s nguyn vi 1 i s.

Dng 2: Cc bi ton s hc khc.


Bi ton 12. Vi n nguyn dng cho trc, phng trnh x + 2y = n c bao nhiu nghim
nguyn dng?
Gii. Ta c 2y = n x n 1. Do tng ng mi gi tr ca y ta c x = n 2ychnh l
mt nghim ca phng trnh. S nghim ca phng trnh chnh
 n1l s cc gi tr c th c ca
y, l b s ca 2 m khng vt qu n1. Vy phng trnh c 2 nghim nguyn.
Bi ton 13. Cho n l mt s nguyn dng tha mn n! c ng 2002 ch s 0 ng tn cng.
Chng minh rng n 8024.
Gii. Gi s n! = 2 5 q, trong (q, 2) = (q, 5) = 1. p dng nh l Legendre ta c
Xjnk
Xjnk
=
>

=
.
i
i
2
5
i1
i1
76

.
.
Do n! .. 10 , n! .. 10 nn s ch s 0 ng
 cng trong biu din thp phn
 n l .
P ca
P  n tn
n
= 2002. Vi mi n t p (n) =
. Ta c
Vy ta cn tm n nh nht sao cho =
5i
5i
i1

i1

p (n) p (n + 1) vi mi n 1 v

 
 
 
 

8024
8024
8024
8024
8024
p (8024) =
+
+
+
+
= 1604 + 320 + 64 + 12 + 2 = 2002.
5
52
53
54
55
Do n 8024 (pcm).
Bi ton 14. Chng minh rng vi mi s nguyn dng m v n ta c
(m, n) = 2

m1
X
k=0


kn
+ m + n mn
m

Gii. t d = (m, n) , m = dm0 , n = dn0 , m0 , n0 Z, (m0 , n0 ) = 1. Ta c


 (
  
n 1 nu kn
khng l mt s nguyn
kn
kn
m
+ n
=
kn
m
m
n
nu m l mt s nguyn
Mt khc

kn
m

kn0
m0

gi tr ca k sao cho
2

m1
X
k=0

.
l mt s nguyn khi v ch khi k ..m0 . Do trong tp hp {1, 2, ..., m 1}

 

m1
1
= d 0 =d1
m0
m

kn
m

l mt s nguyn. Ta c



m1
X  kn  m1
X  kn  
kn
kn
=2
=
+ n
=mn m n d.
m
m
m
m
k=1
k=1

Suy ra iu phi chng minh.


Dng 3: Tm s hng tng qut ca dy s. Tnh tng.
Bi ton 15. Cho (un ) l dy s th t tng dn ca cc s t nhin l khng chia ht cho 3.
Tm s hng tng qut ca dy s trn.
Gii. Xt theo s d th tt c cc s t nhin khng chia ht cho 3 u c dng 3p 1 hoc
3p + 1, y l hai s chn hoc hai s l lin tip ty theo p chn hay l. Khi p chn p = 2k, th
hai s dng 6k 1 v 6k + 1 l hai s l. Tt c cc s dng ny chnh l cc s hng ca dy p
cn tm. Xp th t tng dn ta s c: u2k = 6k 1 v u2k+1 = 6k + 1. Nh vy vi n = 2k + r,
r = {0, 1}, ta c
jnk
jnk
jnk
jnk

j n k
un = 6
+ {1, 1} = 6
+ 2 {0, 1} 1 = 6
+ 2r 1 = 6
+2 n2
1.
2
2
2
2
2
 
Vy un = 2n + 2 n2 1.
Bi ton 16. Cho (un ) l dy s th t tng dn ca cc s t nhin khng chnh phng. Tm
s hng tng qut ca dy s trn.

77

Gii. Xt dy cc s t nhin (vn ). Ta c


nh hn un (b loi t dy (vn )). Nh vy
k 2 + 1 un (k + 1)2 1. Trn mi on

un = vn + k, trong k l s cc s chnh phng


un2 phi nm 2 gia hai s chnh phng lin tip:
i + 1; (i + 1) 1 (gia hai s chnh phng lin
k1
P
tip) c 2i s t nhin. m cc s hng ca dy (un ) c gi tr nh hn k 2 ta c
2i = k 2 k
i=1

s hng. Nh vy ch s n s tha mn
k2 k + 1 n k2 + k
hay
r

1 1
n+ k
4 2

r
n

3 1
+
4 2

r
r
1 1
1 1
3 1
n+ + 1 n+ k n +
4 2
4 2
4 2
jq
k



nn k =
n + 14 + 21 hay k =
n + 21 .


Vy un = n +
n + 12 .
n j k
P
Bi ton 17. Tnh tng Sn =
k .
k=1

Gii. t m = b nc suy ra m2 n (m + 1)2 . Xt cc s hng ca Sn trn mi on


i2 k (i + 1)2 c 2i + 1 s hng, cc s hng ny u c gi tr l i. Nh vy ta c
2 +2i
n j
n j
n j
j k X
X
X iX
X
X
k m1
k m1
k
Sn =
k =
k +
k =
i (2i + 1) +
k

k=1

i=1 k=i2

k=m2

i=1

k=m2


m (m 1) (2m 1) m (m 1)
m (m 1) (2m + 5)
+
+ n + 1 m2 m = nm
3
2
6

Hm phn nguyn trong cc thi chn hc sinh gii

Bi ton 18. (AIME 1985) C bao nhiu s trong 1000 s t nhin u tin c th biu din
di dng
b2xc + b4xc + b6xc + b8xc , x R.
Gii. t hm f (x) = b2xc + b4xc + b6xc + b8xc .
Nu n l mt s nguyn dng th f (x + n) = f (x) + 20n(chng minh bng quy np).
Nh vy ta hn ch li, ch chng minh cho 20 s nguyn dng u tin vi x (0; 1]. Ta
c khi x th gi tr f (x) ch bin i khi b2xc , b4xc , b6xc , b8xc t gi tr nguyn. Trong na
, l m n, n = 2, 4, 6 hoc 8. Ta thy c 12 phn
khong x (0; 1] mi s bin i khi x = m
n
s (theo th t tng dn) l
1 1 1 1 3 1 5 2 3 5 7
; ; ; ; ; ; ; ; ; ; v 1.
8 6 4 3 8 2 8 3 4 6 8
Vy ch c 12 trong s 20 s nguyn dng u tin c biu din tha . V 1000 = 50.20nn c
50.12 = 600 s nguyn tha yu cu bi ton.
Bi ton 19. (Canada 1998) Tm s nghim thc ca phng trnh:
jak jak jak
+
+
= a.
2
3
5
78

Gii. V v tri l mt s nguyn nnacng phi l mt s nguyn. t a = 30q + r, trong


q, r l cc s nguyn, 0 r 29. Ta c phng trnh cho tng ng vi phng trnh sau
j r k j r k j r k
jrk jrk jrk
+
+
= 30q + r q = r
+
+
31q +
2
3
5
2
3
5
Nh vy, vi mi gi tr ca q sao cho a = 30q + r tha mn phng trnh cho. Do r ch c th
nhn 30 gi tr nn phng trnh cho c 30 nghim thc.
Bi ton 20. (Austrian - Polish 1998) Cho m, n l cc s nguyn dng, n m. Chng minh
rng:
n j
X
k

m
k2
km n + m 2 4 1 .
k=1
k2

Gii. Vi k > m ta c k < 2k suy ra k < 2 m tc l k m < 2k hay


th
j
k
k2
k m = 1.

k2

k m < 2. V vy nu k > m

T chng minh bt ng thc cho ta ch cn chng minh


m j
X

k2

km

m2

k=1

k2

m
X

k2

24.

k=1

m
km 2 4 .

2
m
> 2k > k, do k k m 2 4 . T ta c iu phi chng minh.
k2

Vi k = 1, 2, 3 th 2 4 k
Vi k 4, 2 4

m
4

k2

Bi ton 21. (Russia 1999) Chng minh rng vi mi s nguyn dng n > 2,
n2 n
X
o n2 1
.
k
2
k=1

Gii. (Chng minh bng phng php quy np). Vin = 1, hin nhin ng. Gi s ng vi
n. Ta i chng minh khng nh ng vi n + 1. Ta c

n < n2 + 1 < n2 + 2 < .... < n2 + 2n < n + 1


nn vi mi i = 1, 2, ..., 2n ta c
r
n
o
i2
i
n2 + i = n2 + i n < n2 + i + 2 n =
.
4n
2n
Do
(n+1)2

n2 n
2n
X n o X
X n o n2 1 1 X
o (n+1)
n2 1 2n + 1
(n + 1)2 1
k =
k +
k <
+
i+0 =
+
=
2
2n i=1
2
2
2
2
k=1
k=1
k=n +1

Vy khng nh ng vi, theo nguyn l quy np ta c iu phi chng minh.


Bi ton 22. (Australia 1999) Gii h phng trnh sau:

x + byc + {z} = 200


{x} + y + bzc = 190, 1

bxc + {y} + z = 178, 8


79

Gii. Cng ba phng trnh v theo v ta c 2x + 2y + 2z = 568, 9 x + y + z = 284, 45.


Tr tng cp phng trnh ta c

{y} + bzc = 84, 45


bxc + {z} = 94, 35

{x} + byc = 105, 65


Ta c 84 = b84, 45c = bbzc + {y}c = bzc bzc = 84, {y} = 0, 45. Tng t ta c byc = 105,
nn y = 105, 45. Do x = 94, 65 v x = 84, 35.
Bi ton 23. (APMO 1999) Cho a1 , a2 , ... l dy cc s thc tha mn
ai+j ai + aj , i, j = 1, 2, ...
Chng minh rng
a1 +

a2 a3
an
+
+ +
an , n N .
2
3
n

Gii. (S dng phng php quy np mnh) Vi n = 1, n = 2 khng nh hin nhin ng.
By gi ta gi s khng nh ng vi n k,vi mi s nguyn k 2, tc l
a1 a1 ,
a2
a2 ,
a1 +
2
vdots
a2
ak
a1 +
+ +
ak
2
k
Cng v theo v cc bt ng thc trn ta c
ka1 + (k 1)

a2
ak
+ +
a1 + a2 + ... + ak .
2
k

Cng a1 + a2 + + ak vo hai v ca (5) ta c



a2
ak 
(k + 1) a1 +
+ +
(a1 + ak ) + (a2 + ak1 ) + ... + (ak + a1 ) kak+1 .
2
k
Chia 2 v ca (??) cho k + 1 ta c
a1 +

a2
ak
kak+1
a2
ak
ak+1
+ +

a1 +
+ +
+
ak+1 .
2
k
k+1
2
k
k+1

Vy theo nguyn l quy np ta c iu phi chng minh.


Bi ton 24. (Korea 2000) Cho p l s nguyn t, p 1 (mod4) . Tnh gi tr ca biu thc

 2 !
p1 
X
k
2k 2
2
.
S=
p
p
k=1
Gii. Ta c

2k 2
p

2k 2
=

2k 2
p

  2
 2
k
k2
k
,
=

p
p
p

80

(4)

(5)

nn

  2
 2  2
2k 2
k
k
2k

=2

.
p
p
p
p

Ta thy: * Nu {x} < 12 th 2 {x} {2x} = 2 {x} 2 {x} = 0.


* Nu {x} 12 th 2 {x} {2x} = 2 {x} (2 {x} 1) = 1.
n 2o
Do , S bng cc s k [1, p 1] sao cho kp 21 hay s cc lp thng d k khc 0 sao


;
p

1
modulo p.
cho k 2 ng d vi mt s nm trong on p+1
2
2
Do p 1 (mod4) nn tn ti s nguyn d sao cho d 1 (modp). Chia p 1 lp thng d
khc 0 thnh p1
cp c dng (a, da) . Do (da)2 a2 (modp) nn trong mi cp c ng mt
2


;
p

1
. T suy ra c tt c p1
lp thng d
s thng d bnh phng nm trong on p+1
2
2

 p+1
2
.
khc 0 sao cho k ng d vi mt s nm trong on 2 ; p 1 modulo p, tc l S = p1
2
Bi ton 25. (CRMO 2001) Tm s cc s nguyn dng x tha
jxk j x k
=
.
99
101
x  x 
Gii. Ta c 99
= 101 = 0 x {1, 2, ..., 98} 98 s tha (6)
x  x 
= 101 = 1 x {101, 102, ..., 197} c 97 s tha (6)
99
Tng qut, nu
jxk j x k
=
= k, k 1
99
101
th
x {101k, 10k + 1, ..., 99 (k + 1) 1}

(6)

(7)

c 99 2k s tha (6).
Tp (7) khc rng khi
99 (k + 1) 1 101k k 49.
Vy s cc s nguyn dng x tha l
Bi ton 26. (CRMO 2003) Nu x l s nguyn dng ln hn 7 th
jnk .
..7.
Cn7
7

(8)

Gii. Ta c
x  x 
= 101 = 0 x {1, 2, ..., 98} c 98 s tha (8).

 x 
 99
x
=
= 1 x {101, 102, ..., 197} c 97 s tha (8).
99
101
Tng qut, nu
jxk j x k
=
= k, k 1
99
101
th
x {101k, 10k + 1, ..., 99 (k + 1) 1}
c 99 2k s tha (8).
Tp (9) khc rng khi
99 (k + 1) 1 101k k 49.
Vy s cc s nguyn dng x tha l
98 +

49
X

(99 2k) = 2499.

k=1

81

(9)

Bi ton 27. (INMO 2009) Tm tt c cc s thc x sao cho bx2 + 2xc = bx2 c + 2 bxc .


Gii. Cng 1 vo hai v ca phng trnh ta c (x + 1)2 = (bx + 1c)2 . Gi s x + 1 0.
Khi bx + 1c x + 1 0. Do


(bx + 1c)2 (x + 1)2 (x + 1)2 = (bx + 1c)2 .
V x + 1 0 nn x {..., 3, 2, 1} .
Gi s x + 1 > 0. Ta c


(x + 1)2 (x + 1)2 = (bx + 1c)2 .
Hn na,


(x + 1)2 1 + (x + 1)2 = 1 + (bx + 1c)2 .
Do
bxc + 1 = bx + 1c (x + 1) <


 q
1 + (x + 1)2 = 1 + (bxc + 1)2 .

Suy ra
 q

2
x n; 1 + (n + 1) 1 , n 1.
Vy tp nghim ca phng trnh l
{x Z|x 1}

[ 

!
q
n; 1 + (n + 1)2 1 .

n1

ss

82

Mt s dng ton
v bt phng trnh hm
Nguyn Vn Giang
Trng THPT Chuyn Bo Lc, Lm ng
Chuyn phng trnh hm v bt phng trnh hm l mt chuyn kh quan trng trong
chng trnh luyn thi hc sinh gii THPT cc cp. N xut hin nhiu trong cc thi hc sinh
gii nhng nm gn y. Trong bt phng trnh hm l phn gy kh khn hn cho cc em
hc sinh v gio vin ph trch ging dy. Qua tham kho cc ngun ti liu v thc t ging dy,
ti chn lc v h thng mt s dng bi tp bt phng trnh hm c trnh by theo cu
trc:
1) Mt s bi ton bt phng trnh hm c bn.
2) Mt s bi ton bt phng trnh hm gii bng phng php chuyn qua gii hn.
3) Bt phng trnh hm lin quan n tam gic.
4) Bi tp.
Do thi gian chun b khng nhiu v kin thc cn hn ch nn c nhng cha c gii
quyt trit mong c s gp chn thnh ca mi ngi chuyn c tt hn. Xin chn
thnh cm n!

Mt s bi ton bt phng trnh hm c bn

Bi ton 1. Xc nh cc hm s f (x) lin tc trn R tho ng thi cc iu kin sau:


(
f (x) 0; x R (1.1)
f (x + y) f (x) + f (y); x, y R (1.2)
(
f (0) 0
Li gii. Thay x = y = 0, ta c
hay f (0) = 0.
f (0) 2f (0)
Vy nn f (0) = f (x + (x)) f (x) + f (x) 0. Suy ra f (x) = 0. Th li ta thy tho mn
iu kin bi ton mn iu kin bi ton.
Bi ton 2. Cho trc hm s h(x) = ax; a R. Xc nh cc hm s f (x) lin tc trn R tho
ng
( thi cc iu kin sau:
f (x) ax; x R (2.1)
f (x + y) f (x) + f (y); x, y R (2.2)
Li gii. T gi thit, ta c: h(x + y) = h(x) + h(y). t f (x) = h(x) + g(x). Khi , ta c
g(x) 0; x R v g(x + y) g(x) + g(y); x, y R.
Theo bi ton 1, ta c: g(x) = 0; x R. Vy f (x) = h(x) = ax; a R. Th li ta thy tho
mn iu kin bi ton.
Bi ton 3. Cho a > 0. Xc nh cc hm s f (x) lin tc trn R tho ng thi cc iu kin
sau:(
f (x) ax ; x R (3.1)
f (x + y) f (x)f (y); x, y R (3.2)
Li gii. Ta c f (x) > 0; x R. Khi logarit ho hai v (3.1), (3.2), ta c

83

(
ln f (x) (ln a)x; x R
ln f (x + y) ln f (x) + ln f (y); x, y R
(
(x) (ln a)x; x R
Ta c:
(x + y) (x) + (y); x, y R

t ln f (x) = (x),

t (x) = g(x)+(ln a)x, ta c: hm g(x) tho mn iu kin bi ton 2 nn g(x) = 0; x R


v (x) = (ln a)x.
Vy f (x) = ax ; x R Th li ta thy tho mn iu kin bi ton mn iu kin bi ton.
Bi ton 4. Xc nh cc hm s f (x) lin tc trn R tho ng thi cc iu kin sau:

f (x) 0; x R (4.1)
f ( x + y ) f (x) + f (y) ; x, y R (4.2)
2
2
Li gii.
t f (0) = a, f (x) a = g(x). Khi ta c

g(x) 0; x R (4.10 )
vi g(0) = 0 Thay y = 0 vo (4.1) v (4.2)
g( x + y ) g(x) + g(y) ; x, y R (4.20 )
2
2

g(x)
x
; x R
g( )
.
2
2
g(0) = 0
x+y
g(x) g(y)
Suy ra g(
)
+
; x, y R hay
2
2
2
(
g(0) = 0, g(x) 0; x R
theo bi ton 1 th g(x) = 0; x Rvaf (x)laconst.
g(x + y) g(x) + g(y); x, y R
Th li f (x) = c tho mn iu kin bi ton.
Bi ton 5. Xc nh cc hm s f : R R tho mn iu kin sau:
1
1
1
f (xy) + f (xz) f (x).f (yz) , x, y, z R (5)
2
2
4
Li gii. T (5) thay x = y = z = 0 ta c:

2
1
1
1
1
1
f (0) + f (0) f (0).f (0)
f (0)
0 f (0) =
2
2
4
2
2
1
Tng t, thay x = y = z = 1 ta c: f (1) = .
2
1
1
Ta s chng minh f (x) , x R v f (x) , x R.
2
2
1
1
Thay y = z = 0 v s dng f (0) = ta c f (x) , x R
2
2
1
1
Thay y = z = 1 v s dng f (1) = ta c f (x) , x R.
2
2
1
Vy ta c f (x) , x R. Th li thy tha mn bi ra.
2
Bi ton 6. Xc nh cc hm s f : R R tho mn iu kin sau:
f (x + y) + f (y + z) + f (z + x) 3f (x + 2y + 3z), x, y, z R (6)
Li gii.
Thay y = z = 0 vo (6) ta c: f (x) + f (0) + f (x) 3f (x) f (x) f (0), x R
x
x
Thay x, y bi v z bi ta c: f (x) + 2f (0) 3f (0) f (x) f (0), x R.
2
2
84

T hai kt qu trn ta suy ra f (x) f (x), x R hay f (x) c (c l hng s). Th li thy
tha mn bi ra.
Bi ton 7. Xc nh cc hm s f : R+ R tho mn iu kin sau:

f (x) + f (y) f (x + y) , x, y R+ (7.1)

2
f (x) + f (y)
f (x) f (y)

, x, y R+ (7.2)
x
y
x+y
f (x)
Li gii. t g(x) =
th t (7.2) ta c: g(x) + g(y) g(x + y), x, y R+ (7.3). Ta chng
x
minh cc iu sau:
a) g(nx) ng(x), n N, x R+ . iu ny d dng chng minh bng quy np da vo tnh
cht (7.3) ca hm g. T y ta c:
f (nx) n2 f (x), n N, x R+ . (7.4)
b) f (2n x) = 4n f (x). Tht vy, trong (7.1) cho y = x ta c: 4f (x) f (2x).
Mt khc, theo a) th f (2x) 4f (x). Do f (2x) = 4f (x), x R+ .
T y ta thu c mt c im ca hm g l g(2n x) = 2n g(x), n N, x R+ .
c) g(nx) = ng(x), n N, x R+ . Tht vy, gi s tn ti
n0 N, x0 R+ sao cho g(n0 x0 ) < n0 g(x0 ). Khi , chn r N sao cho 2r > n0 th
2r g(x0 ) = g(2r x0 ) = g(2r x0 + n0 x0 n0 x0 )
g(n0 x0 ) + g((2r n0 )x0 ) (Tnh cht (7.3) ca hm g )
< n0 g(x0 ) + g((2r n0 )x0 )
< n0 g(x0 ) + (2r n0 )g(x0 ) (S dng a) )
= 2r g(x0 ) (Mu thun)
d) f (x) 0, x R+ . Tht vy,
10f (x) = 2(f (x) + f (2x)) (Theo phn b) )
f (3x) ( Theo (7.1) )
9f (x) ( Theo (7.4) )
Suy ra f (x) 0, x R+ . Ko theo g(x) 0, x R+ .
e) g l hm n iu gim, v g(x + y) g(x) + g(y) g(x), x, y R+ .
f) g(x) = ax, x R+ . Tht vy, t c) ta c g(q) = g(1).q, q R+ v g l hm n iu gim
nn ta c g(x) = ax, x R+ .
T ta suy ra f (x) = ax2 , x R+ . Th li thy tha mn bi ra.
Bi ton 8. Xc nh cc hm s f : R+ R tho mn iu kin sau: f (x + y) f (x).f (y)
2002x+y , x, y R.
Li gii.
- Ta thay x = y = 0 c: f (0) (f (0))2 1 f (0) = 1
- Thay y bi x v f(0) = 1 ta c: 1 f (x).f (x) 1 f (x).f (x) = 1, x R.
- Thay y = 0 ta c: f (x) 2002x , x R.. Nhng khi th f (x).f (x) 2002x .2002x =
1, x R.
Kt hp vi f (x).f (x) = 1, x R, ta c f (x) = 2002x , x R.
Th li thy ng.
Bi ton 9. Cho cc s dng M, a. Tm cc hm s f (x), g(x) : R R tho mn iu kin
|f (y) f (x) g(x)(x y)| M |x y|2+a ; x, y R (8)
Li gii. Gi s c cc hm s f (x), g(x) : R R tho mn iu kin. Thay i vai tr ca
x, y ta c:
85

|f (x) f (y) g(y)(y x)| M |y x|2+a ; x, y R (80 ). Cng v (8) v (8), ta c


|[g(x) g(y)](x y)| |f (y) f (x) g(x)(x y)| + |f (x) f (y) g(y)(y x)|
|[g(x) g(y)](x y)| 2M |x y|2+a ; x, y R (800 )


g(x) g(y)
a


x y 2M |x y| ; x, y R, x 6= y. C nh x cho y x, ta c
g 0 (x) = 0; x R suy ra g(x) = c(const); x R



f (x) f (y)
c 2M |x y|a ; x, y
Thay g(x) = c vo (8) v lm tng t nh trn, ta c:
xy
R, x 6= y v f 0 (x) = c f (x) = cx + d.
Th li g(x) = c ; f (x) = cx + d thy ng.

Mt s bi ton bt phng trnh hm gii bng phng


php chuyn qua gii hn

Bi ton 10. Gi s f l hm s tho mn cc iu kin sau:


f (2x) x + f (f (x)), x R+ . (1.1)
Chng minh rng: f (x) x, x R+ .
x
x
x
Li gii. T (1.1) ta c f (x) + f (f ( )) > , x R+ (1.2)
2
2
2
Do ta c th gi s rng: f (x) an x, x R+ (1.3) trong an l hng s.
x
x
x
x
1 + a2n
Kt hp (1.1), (1.2) v (1.3) ta c: f (x) + f (f ( )) + an f ( )
x, x R+ .
2
2
2
2
2
1 + a2n
(1 + an )2
1
, n N . Ta c an+1 an =
>
Xt dy (an ) xc nh bi a1 = ; an+1 =
2
2
2
0, n N nn (an ) l mt dy s tng.
Hn na, d thy an < 1, n N .
1 + a2
Suy ra dy (an ) l hi t v k hiu a l gii hn ca n th a =
, ngha l a = 1.
2
1 + a2n
x, x R+ ta c f (x) x, x R+ .
Do , cho n + th t f (x) >
2
Bi ton 11. Tm tt c cc hm s f : R R+ tho mn cc iu kin sau:
f (x) 1 + x, x R. (2.1)
f (x + y) f (x).f (y), x, y R (2.2).
Li gii. Theo gi thit f (x) > 0, x R.
T (2.1), cho x = 0 ta c f (0) 1.
T (2.2), cho x = 0 ta c f (0) f 2 (0) f (0) 1. Suy ra f (0) = 1.
Cng t (2.2) ta suy ra rng f (x1 + x2 + + xn ) f (x1 ).f (x2 )...f (xn ), xi R, 1 i n.
x
x
x x
Do f (x) = f ( + + + ) f n ( ), x R, n N . Kt hp vi (2.1) ta c f (x)
n n
n
n
x n
n x

f ( ) (1 + ) , x R, n N . T bt ng thc ny, cho n +,


n
n
ta c f (x) ex . Hn na, ta c 1 = f (0) = f (x + (x)) f (x)f (x) ex ex = 1.
Do f (x) = ex . Th li thy tha mn bi ra.
Bi ton 12. Chng minh rng khng tn ti hm s f : R+ R+ sao cho
f (x + y) f (x) + y.f (f (y)), x, y R+ (3.1).
Li gii. Gi s tn ti hm s f tha yu cu bi ton.
86

Thay x = 1 v y bi x ta c f (1 + x) f (1) + x.f (f (1)). iu ny suy ra rng lim f (x) =


x+

+, v do lim f (f (x)) = +.
x+

Mt khc, t (3.1) cho y = 1 ta c


f (x + 1) f (x) + f (f (x)), x R+ (3.2)
Suy ra: lim (f (x+1)f (x)) = +. Do tn ti x0 R+ sao cho f (x0 +k)f (x0 +k 1) >
x+

2, k 1 (3.3)
By gi, chn mt gi tr xc nh n N sao cho n x0 + 1. Trong (3.3), cho k ln lt nhn
gi tr 1, 2, . . . , n v sau cng cc bt ng thc thu c, ta c:
f (x0 + n) f (x0 ) > 2n, n x0 + 1 (3.4)
Hn na, v f (x0 ) > 0 nn vi cch chn n > x0 + 1 ta c n > x0 + 1 f (x0 ). Kt hp vi
(3.4) ta c:
f (x0 + n) > 2n + f (x0 ) > x0 + n + 1 (3.5)
Do , ta c:
f (f (x0 + n)) f (x0 + n + 1) + f (x0 + n) (x0 + n + 1).f (f (x0 + n)) (3.6)
V f (f (x0 + n + 1)) > 0 nn bi (3.5), ta c
f (x0 + n + 1) + f (x0 + n) (x0 + n + 1).f (f (x0 + n)) > f (x0 + n + 1). (3.7)
Hn na bi (3.2), ta c:
f (x0 + n + 1) f (x0 + n) + f (f (x0 + n)) (3.8)
Mt khc, v f (x0 + n) > 0 nn ta c:
f (x0 + n) + f (f (x0 + n)) > f (f (x0 + n)). (3.9)
Kt hp (3.6), (3.7), (3.8), (3.9) ta suy ra f (f (x0 + n)) > f (f (x0 + n)), mu thun. Ta c iu
phi chng minh.
Bi ton 13. Tm tt c cc hm s f : R R tha mn iu kin
|f (x) f (q)| 5(x q)2 , x R, q Q (4.1).
Li gii. Vi mi x, x0 R, (x x0 ) ta chn s hu t q nm gia x v x0 th:
|f (x) f (x0 )| = |f (x) f (q) + f (q) f (x0 )|
|f (x) f (q)| + |f (q) f (x0 )|
5(x q)2 + 5(q x0 )2
5(x x0 )2 + 5(x x0 )2 = 10(x x0 )2

Vy nn lim |f (x) f (x0 )| = 0 hay lim f (x) = f (x0 ).


xx0

xx0

Suy ra f (x) l hm s lin tc ti mi x0 R.


Mt
khc, t nh
gi trn ta nhn c:



f (x) f (x0 )
f (x) f (x0 )


= 0 hay f 0 (x0 ) = 0, x0 R.
lim
x x0 10 |x x0 | . Suy ra: xx
x x0
0
Do , f (x) c, x R (c l hng s). Th li thy tha mn bi ra.
Bi ton 14. Gi F l tp hp tt c cc hm s f : R+ R+ tha mn iu kin
f (3x) f (f (2x)) + x, x R+ . (5.1)
Tm s thc ln nht sao cho vi mi f F th f (x) .x, x R+ .
x
x
x
1
Li gii. D thy hm s f (x) = F (tha (5.1) ) v hin nhin nn .
2
2
2
2
87

x
x
2x
)) + > , x R+ (5.2)
3
3
3
Do ta c th to dy n f (x) n x, x R+ (5.3)
trong n l hng s. Kt hp (5.1), (5.2) v (5.3) ta c:
x
2x
x
2x
x
2x
1 + 2n2
f (x) + f (f ( )) + n f ( ) + n2
=
x, x R+ .
3
3
3
3
3
3
3
1
1 + 2n2
Xt dy (n ) xc nh bi 1 = ; n+1 =
, n N . Ta d dng chng minh c
3
3
1
(n ) l mt dy s tng v b chn trn bi . V do n phi hi t. K hiu l gii hn ca
2
1 + 22
1
n th =
, ngha l a = .
3
2
1 + 2n2
x
Do , t f (x)
x, x R+ cho n + th ta c f (x) , x R+ .
3
2
1
Vy a = l s cn tm.
2
T (5.1) ta c f (x) f (f (

Bt phng trnh hm lin quan n tam gic

Trong tam gic ABC, ta c cc bt ng thc quen thuc sau:

3 3
1) sin A + sin B + sin C
2
3
2) cos A + cos B + cos C
2

A
B
C
3) tg + tg + tg 3
2
2
2

A
B
C
4) cot g + cot g + cot g 3 3
2
2
2
Nhn xt: Da vo cc bt
t c hay khng lp cc hm
ng thc trn ta c th m

fflf (A) + f (B) + f (C) 3f ( )


3

tng qut tho mn iu kin fflf (A) + f (B) + f (C) 3f ( ) vi mi tam gic ABC? Chng
3
ffl
ta cng tm hiu qua cc bi ton sau:
Bi ton 15. Cho hm s f (t); t (0; ). Chng minh cc iu kin (1.1) v (1.2) sau y l
tng ng:
x+y
f (x) + f (y) 2f (
); x, y, x + y (0; ) (1.1)
2
x+y+z
f (x) + f (y) + f (z) 3f (
); x, y, z, x + y + z (0; ] (1.2)
3
Li gii.

x+y+z
z
+
x+y+z

z
+Gi s: x y z, ta c f (z) + f (
) 2f
(1.3)
3
2
T (1.1) v (1.3), ta c:

x+y+z
z+
x+y+z
x+y

3
f (x) + f (y) + f (z) + f (
) 2 f (
)+f

3
2
2

88


4f

x+y+z
3


;

x, y, z, x + y + z (0; ] . Suy ra (1.2)

+T (1.2), gi s x z y, t z =

x+y
ta c (1.1).
2

Bi ton 16. Xc nh hm s f (t); t (0; ) tho mn iu kin: A < B f (A) < f (B)
(2).
Li gii.
+(2) tho vi mi cp gc nhn A,B tng ng vi f (t) = f0 (t) l mt hm ng bin trong

(0; ]
2

f0 (t) khi 0 < t


2
+Xt hm s g0 (t) =
.
f0 ( t) khi < t <
2
Ta chng minh g0 (t) tho mn iu kin bi ton. Tht vy:
o A, B nhn th (2) tho

o Xt 0 < A < < B < ; A + B <


2
B > A, g0 (B) = f0 ( B) > f0 (A) = g0 (A)
+Ta chng minh mi hm s f (t); t (0; ) tho mn iu kin bi ton u c dng:

g0 (t), khi 0 < t


2 .
f (t) =

g0 (t), khi < t <


2

Tht vy, t g0 (B) f (B) vi B t, ta c 0 < A <


< B < , A + B < th B >
2
A, f (B) g0 (B) = f0 ( B) > f0 (A) = g0 (A) = f (A)

sin t, khi 0 < t


2 . Chng minh vi mi tam gic
Bi ton 17. Xt hm s f (t) =

1 + cos t, khi < t <


2

3 3
ABC ta u c f (A) + f (B) + f (C)
(3)
2
Li gii.

3 3
+Tam gic ABC nhn (hay vung) th (3) c dng quen thuc sin A + sin B + sin C
2

3 3 0
+Tam gic ABC t: C >
th (3) c dng sin A + sin B + 1 + cos C
(3 ) ng do:
2
2

3 3
sin A + sin B + 1 + cos C sin A + sin B + sin C
2

Bi tp
1) Tm tt c cc hm n nh f : N N tha mn f (f (n))

n + f (n)
, n N.
2

2) Tm tt c cc hm s f : [0; 1] R tho mn iu kin:


f (x) 2xf (x2 ), x [0; 1]
3) Tm tt c cc hm s f : (0; +) (0; +) tho mn iu kin:
f 2 (x) f (x + y).f (f (x) + y), x, y > 0
4) Tm tt c cc hm s f : R R tho mn iu kin:
f (x3 + x) x f 3 (x) + f (x), x R
89

5) Tm tt c cc hm s f : [1; +) [1; +) tho mn cc iu kin:


f (x) 2(x + 1), x 1 v xf (x + 1) = f 2 (x) 1, x 1.

Ti liu tham kho


1) Nguyn Vn Mu, Phng trnh hm, NXBGD 1997.
2) Trnh o Chin. Giari bt ng thc hm bng phng php chuyn qua gii hn, TL
Trng CSP Gia Lai.
3) Ng Quang Dng, Chuyn phng trnh hm, TL Trng THPT Phm Thnh Trung.
4) Titu S, Functional equation.

90

Mt s p dng khai trin Taylor


trong bt ng thc v tnh gii hn
Hong t H
Trng THPT Trn i Ngha, k Lk
Khai trin Taylor c vai tr quan trng trong gii tch. N c mi lin h cht ch vi cc tnh
cht c trng c bn ca hm s nh tnh n iu, tnh li lm,... Tuy nhin khai trin Taylor
rt t c cp trong sch gio khoa v sch tham kho. Trong bi vit ny, ti trnh by ng
dng ca khai trin Taylor xy dng, chng minh bt ng thc v tnh gii hn ca hm s.

Khai trin Taylor

nh l 1 (nh l Taylor). Hm f : (a; b) R kh vi lin tc cp n trn khong (a; b) v c


o hm cp n + 1 quanh im x0 (trong ln cn ca im x0 ), khi vi x (a; b), tn ti im
nm gia x v x0 sao cho
f (x) =

n
X
f ( k)(x0 )
k=0

k!

(x x0 )k + Rn+1 (f ; x),

(1)

trong
1
Rn+1 (f ; x) =
n!p

x x0
x

(x )n+1 f n+1 (), p R, p > 0.

(2)

Cng thc (1) c gi l cng thc Taylor i vi hm f vi phn d Rn+1 di dng


Schlomilch-Roche.
Bng cch chn cc gi tr p > 0 hon ton xc nh, ta thu c nhng trng hp ring i vi
phn d Rn+1 (f ; x). y ta xt khi p = n + 1 th t (2), ta c phn d ca cng thc Taylor
di dng Lagrange l:
Rn+1 (f ; x) =

f (n+1) ()
(x x0 )n+1 , = x0 + (x x0 ), 0 < < 1.
(n + 1)!

Khi
f 0 (x0 )
f 00 (x0 )
f (n) (x0 )
f (n+1) ()
2
n
f (x) = f (x0 )+
(xx0 )+
(xx0 ) +...+
(xx0 ) +
(xx0 )n+1 , (3)
1!
2!
n!
(n + 1)!
f (n+1) ()
(x x0 )n+1 gi l phn d (dng Lagrange) v thng k hiu tt l
(n + 1)!
0 [(x x0 )n ] (dng Pano).
trong s hng

Nhn xt 1. Nu b phn d th v phi ca (3) l mt a thc bc n ca (x x0 ), a thc ny


l duy nht sai khc mt i lng v cng b bc n ca (x x0 ). V vy c th coi hm f (x)
trong min gn x0 nh l mt a thc bc n ca (x x0 ). Ngha l
f (x) f (x0 ) +

f 0 (x0 )
f 00 (x0 )
f (n) (x0 )
(x x0 ) +
(x x0 )2 + ... +
(x x0 )n .
1!
2!
n!

91

Vn dng khai trin Taylor xy dng v chng minh bt


ng thc
Trong cng thc (3), xt khi n = 1 ta c
1
f (x) = f (x0 ) + f 0 (x0 )(x x0 ) + f 00 ()(x x0 )2 .
2

(4)

Nu ta chn hm s f (x) kh vi bc hai trn I v f 00 (x) 0, x I th t (4), ta c


f (x) f (x0 ) + f 0 (x0 )(x x0 ).
Kt qu trn c pht biu trong nh l di y.
nh l 2. Mi hm s f (x) c o hm lin tc ti x0 , kh vi cp hai trn I v f 00 (x) 0, x I
u tha mn
f (x) f (x0 ) + (x x0 )f 0 (x0 ).
(5)
Tng t, ta cng c
nh l 3. Mi hm s f (x) c o hm lin tc ti a, kh vi cp hai trn I v f 00 (x) 0, x I
u tha mn
f (x) f (x0 ) + (x x0 )f 0 (x0 ).
(6)
T hai kt qu ca nh l 2 v nh l 3, ta c th xy dng bt ng thc.
V d 1 (Bt ng thc Bernoulli).
Vi mi s thc n, xt hm s f (x) = (1 + x)n , x 1. Ta c
f (x) = n(1 + x)n1 lin tc ti 0 v f 00 (x) = n(n 1)(1 + x)n2 0, x 1.
p dng nh l 2, ta c
0

f (x) f (0) + (x 0)f 0 (0), x 1


hay
(1 + x)n 1 + xn, x 1.
V d 2.
1
Xt hm s f (x) = ln x, x (0, +). Ta c f 0 (x) = lin tc ti 1 v
x
1
00
f (x) = 2 < 0, x (0, +). Khi , theo nh l 3, ta c
x
f (x) f (1) + f 0 (1)(x 1)
hay
1 + ln x x, x > 0.
Du "=" xy ra khi v ch khi x = 1.
V d 3.

92

ex
Xt hm s f (x) = ln(1 + ex ) trn R. Ta c f 0 (x) =
lin tc ti x = 0 v f 00 (x) =
x
1
+
e
ex
> 0, x R. Theo nh l 2, ta c
(1 + ex )2
f (x) f (0) + f 0 (0)(x 0)
hay
ln

1
1 + ex
x, x R.
2
2

V d 4.
Gi s A0 , B0 , C0 l ba gc ca tam gic nhn cho trc.
Xt hm s f (t) = sint, t (0, ).
Ta c f 0 (t) = cos t lin tc trn R, f 00 (t) = sin t < 0, t (0, ).
Theo nh l 3, ta c
sin A sin A0 + (A A0 ) cos A0 .

(7)

V A0 l gc nhn nn cos A0 > 0, do


(7)

sin A
tan A0 + A A0 .
cos A0

Tng t, ta cng c
sin B
tan B0 + B B0 ,
cos B0
sin C
tan C0 + C C0 .
cos C0
Cng cc bt ng thc trn, ta c
sin A
sin B
sin C
+
+
tan A0 + tan B0 + tan C0 + (A + B + C) (A0 + B0 + C0 )
cos A0 cos B0 cos C0
hay
sin A
sin B
sin C
+
+
tan A0 . tan B0 . tan C0 .
cos A0 cos B0 cos C0
Vy ta c bi ton:
Gi s A0 , B0 , C0 l ba gc ca tam gic nhn cho trc. Chng minh rng, vi mi tam gic
ABC bt k ta c
sin A
sin B
sin C
+
+
tan A0 . tan B0 . tan C0
(8)
cos A0 cos B0 cos C0
hoc
Gi s A0 , B0 , C0 l ba gc ca tam gic nhn cho trc. Tm gi tr ln nht ca biu thc
P =

sin A
sin B
sin C
+
+
.
cos A0 cos B0 cos C0

Nhn xt 2. Trong cng thc (8), nu chn A0 , B0 , C0 c th, ta s c c cc bt ng thc


tng ng.

Chng hn, vi A0 = B0 = C0 = , ta c
3

3 3
sin A + sin B + sin C
.
2
93

V d 5.
Gi s A0 , B0 , C0 l ba gc ca tam gic cho trc.

Xt hm s f (t) = cos t, t (0, ). Ta c


2
B
C
A
cos
cos
cos
2 +
2 +
2 cot A0 + cot B0 + cot C0
A0
B0
C0
2
2
2
sin
sin
sin
2
2
2
V d 6.
Gi s A0 , B0 , C0 l ba gc ca tam gic cho trc.

Xt hm s f (t) = tan t, t (0, ). Ta c


2
A0
B0
C0
A
B
C
A0
B0
C0
cos2
. tan + cos2
. tan + cos2
. tan 2 cos
cos
cos .
2
2
2
2
2
2
2
2
2
V d 7.
Gi s A0 , B0 , C0 l ba gc ca tam gic nhn cho trc.

Xt hm s f (t) = cot t, t (0, ). Ta c


2
sin2 A0 . cot A + sin2 B0 . cot B + sin2 C0 . cot C 2 sin A0 sin B0 sin C0 .
Nhn xt 3. Khng ch dng li vic xt du o hm cp 2, chng ta cn c th xt du ca
o hm cp 3 v tng qut cho cp n. T c th to ra rt nhiu bi ton.
nh l 4. Mi hm s f (x) kh vi n bc ba v f 000 (x) 0, x I th vi mi cp im phn
bit x, x0 I u c
f (x0 )
f 00 (x0 )
f (x)
(x x0 ).

+ f 0 (x0 ) +
x x0
x x0
2!
Tng qut, ta c cc kt qu sau
nh l 5. Gi s n N l mt s chn, f (x) kh vi n bc n
v f (n) (x) 0, x I th vi mi cp im phn bit x, x0 I, ta u c


f (n) (x0 )
f 0 (x0 )
n1
(x x0 ) + ... +
(x x0 )
.
f (x) = max f (x0 ) +
xI
1!
(n 1)!
nh l 6. Gi s n N l mt s l, f (x) kh vi n bc n
v f (n) (x) 0, x I th vi mi cp im phn bit x, x0 I, ta u c


f 0 (x0 )
f (n1) (x0 )
2
n
(x x0 )f (x) = max f (x0 )(x x0 ) +
(x x0 ) + ... +
(x x0 ) .
xI
1!
(n 1)!
nh l 7. Gi s n N l mt s chn, f (x) kh vi n bc n
v f (n) (x) 0, x I th vi mi cp im phn bit x, x0 I, ta u c


f 0 (x0 )
f (n) (x0 )
n1
(x x0 ) + ... +
(x x0 )
.
f (x) = min f (x0 ) +
xI
1!
(n 1)!
nh l 8. Gi s n N l mt s l, f (x) kh vi n bc n
v f (n) (x) 0, x I th vi mi cp im phn bit x, x0 I, ta u c


f 0 (x0 )
f (n1) (x0 )
2
n
(x x0 )f (x) = min f (x0 )(x x0 ) +
(x x0 ) + ... +
(x x0 ) .
xI
1!
(n 1)!
94

Vn dng khai trin Taylor tnh gii hn ca hm s

3.1

Cng thc Maclaurin

Cng thc Maclaurin l cng thc Taylor ti x0 = 0.


f (x) = f (0) +

f 00 (0) 2
f (n) (0) n
f 0 (0)
x+
x + ... +
x + Rn+1 (f ; x),
1!
2!
n!

trong phn d (dng Lagrange) l


f (n+1) (x) n+1
Rn+1 (f ; x) =
x , 0 < < 1.
(n + 1)!
v phn d (dng Pano) l 0(xn ).
Cng thc Maclaurin ca mt s hm c bn
1. ex = 1 +

1
1
1
x + x2 + ... + xn + 0(xn ).
1!
2!
n!

2. sin x = x

1 3 1 5
(1)n 2n+1
x + x ... +
x
+ 0(x2n+1 ).
3!
5!
(2n + 1)!

3. cos x = 1

(1)n 2n
1 2 1 4
x + x ... +
x + 0(x2n ).
2!
4!
(2n)!

1
1
1
(1)n+1 n
4. ln(1 + x) = x x2 + x3 x4 + ... +
x + 0(xn ).
2
3
4
n
5. (1 + x)m = 1 + mx +

m(m 1) 2
m(m 1)...m n + 1 n
x + ... +
x + 0(xn ).
2!
n!

6.

1
= 1 x + x2 ... + (1)n xn + 0(xn ).
1+x

7.

1
= 1 + x + x2 + ... + xn + 0(xn ).
1x

8. arctan x = x

x3 x5
(1)n1 2n1
+
+ ... +
x
+ 0(x2n ).
3
5
2n 1

V d 8. Khai trin cc hm sau theo cng thc Taylor n s hng 0(xn )


1. f (x) =

1
ax + b

2. f (x) =

x2 + 5
x2 + x 12

3. f (x) = ln(ax + b)
Hng dn
1
1
=
a
ax + b
b(1 + x)
b
n
P
1
S dng khai trin
=
(1)k xk + 0(xn ).
1 + x k=0
n
P
1
ak
Khi f (x) =
=
(1)k k+1 xk + 0(xn ).
ax + b k=0
b

1. Ta c f (x) =

95

(9)

x2 + 5
3
2
=1
+
.
2
x + x 12
x+4 x3
a
a
3. f (x) = ln(ax + b) = ln[b(1 + x)] = ln b + ln(1 + x).
bn
b
P
1
k1 1 k
n
S dng khai trin
=
(1)
x + 0(x ).
1 + x k=1
k
 k
n
P
k1 1 a
(1)
Khi ln(ax + b) = ln b +
xk + 0(xn ).
k b
k=1

V d 9. Cho hm s f (x) = 1 + x2

1. Khai trin f (x) = 1 + x2 n s hng bc 4


2. Ta c f (x) =

2. Tnh f (4) (0).


Hng dn
1
1. f (x) = 1 + x2 = (1 + x ) 2 = (1 + w)
1
1
1
f (x) = (1 + w) 2 = 1 + w w2 + 0(w2 ) (bc 4 the x bc 2 theo w).
2
8
1
1
= 1 + x2 x4 + 0(x4 ).
2
8

1
f (4) (0) 4
4!
2. x4 =
x f (4) (0) = = 3.
8
4!
8
V d 10. Khai trin theo cng thc Taylor hm s

1. f (x) =

x2
n s hng 0(x5 )
1 + x3

2. f (x) = ln(1 + sin x) n s hng 0(x3 )


3. f (x) = sin2 x n s hng 0(x4 )
Hng dn
x2
1
= x2 .
, w = x3
3
1+x
1+w
= x2 (1 w + w2 w3 + ...) = x2 (1 x3 + x6 x9 + ...) = x2 x5 + 0(x5 ).

1. f (x) =

w2 w3 w4
+

+ ...
2
3
4 
2 
3
x3
x3


x
+ ...
x
+ ...
sin x2 sin x3 sin x4
x3
6
6
= sin x
+

+... = x
+ ...
+
...
2
3
4
6
2
3
x2 x3
=x
+
+ 0(x( 3)).
2
6
1 1
3. f (x) = sin2 x = cos 2x.
2 2
w2 w4
4x2 16x4
cos 2x = cos w = 1
+
... = 1
+
+ 0(x4 ).
2!
4!
2!
4!
1 4
2
4
f (x) = 1 x + x + 0(x ).
3
2. f (x) = ln(1 + sin x) = ln(1 + w) = w

96

3.2

Tnh gii hn

i vi gii hn hm s dng v nh c cha cc hm lng gic, ta thng ngh n vic s


sin x
dng gii hn lim
= 1. Khai trin Taylor cho ta cng thc n gin v cng rt tng qut
x0 x
xc nh phn chnh ca hm s. Do tnh gii hn ca hm s, ta c th dng cng thc
Taylor ti mt cp no .
Bi ton 1. Tnh gii hn

sin(sin x) x 3 1 x2
L = lim
.
x0
x5

0
Hng dn (Dng )
0
Khai trin t thc thnh a thc Taylor vi chnh xc n 0(x5 ) khi x 0.
19
S: L =
90
Bi ton 2. Tnh gii hn

1 + 2 tan x ex + x2
L = lim
.
x0
arc sin x sin x

0
Hng dn (Dng )
0
Khai trin t thc thnh a thc Taylor vi chnh xc n 0(x3 ) khi x 0.
S: L = 2.
Bi ton 3. Tnh gii hn

L = lim

x0

1
1

2
x
sin2 x


.

Hng dn (Dng )
0
Bin i v dng
0 

 2

1
1
sin x x2
1
L = lim

= lim
. S: L = .
2
2
2
2
x0
x0
x
3
sin x
x sin x
Bi ton 4. Tnh gii hn
r
r 

1
x
x
L = lim+
aarctan
barctan
(a > 0, b > 0).
x0 x x
a
b
Hng dn (Dng .0)
0
Bin i v dng
0

r
t
t
aarctan
barctan

a
b
t x = t khi x 0+ th t 0+ v L = lim+
.
3
t0
t
Khai trin t thc thnh a thc Taylor vi chnh xc n 0(t3 ) khi t 0+ .
ba
S: L =
.
3ab

Bi ton 5. Tnh gii hn


3

L = lim (cos(xex ) ln(1 x) x)cot x .


x0

97

Hng dn (Dng 1 )
Ta c

lim cot x3 ln[cos(x.ex )ln(1x)x]

L = ex0

Tnh lim (cot x3 ln[cos(x.ex ) ln(1 x) x]) .


x0
2

S: L = e 3 .

Ti liu tham kho


1. Nguyn Vn Mu, Bt ng thc nh l v p dng, Nh xut bn Gio dc.
2. Nguyn Vn Mu, Trnh o Chin, Trn Nam Dng, Nguyn ng Pht, Chuyn chn
lc a thc v p dng, Nh xut bn Gio dc.
3. Nguyn Vn Mu, Trn Nam Dng, Nguyn V Lng, Nguyn Minh Tun, Chuyn
chn lc Lng gic v p dng, Nh xut bn Gio dc.
4. Nguyn Vn Mu (thng 4/2012), Cc chuyn chuyn ton bi dng hc sinh gii trung
hc ph thng, K yu hi ngh khoa hc.
5. Nguyn Vn Mu, ng Huy Run, Nguyn Thy Thanh, Php tnh vi phn v tch phn
hm mt bin, Nh xut bn i hc Quc Gia H Ni.

98

Phng php gii bi ton chia ht


Nguyn Th Minh Nguyt
Trng THPT Chuyn Nguyn Du, kLk
Bi ton chia ht l mt dng ton quan trng trong chng trnh s hc ca hc sinh THCS
v thng xut hin trong thi hc sinh gii. C nhiu phng php gii bi ton chia ht. Vic
vn dng phng php li phi ty thuc vo dng ton. Bi vit ny trnh by mt s phng
php gii bi ton chia ht.

nh ngha v mt s tnh cht v chia ht

nh ngha 1. Cho hai s nguyn a v b (trong b 6= 0). Khi lun c duy nht hai s nguyn
q v r sao cho a = bq + r (vi 0 r < b).
Ta ni: a l s b chia, b l s chia, q l thng v r l s d c th l 0; 1; 2; ; | b | 1. c
.
bit nu r = 0 th ta ni a chia ht cho b (hoc a l bi ca b hoc b l c ca a) . K hiu b..a
(hoc a | b).

1.1

Mt s tnh cht v chia ht thng dng

T nh ngha trc tip suy ra cc tnh cht sau: Cho a, b, c, d l cc s nguyn.


Tnh cht 1. + Nu a 6= 0 th a | a, 0 | a.
+ Nu b | a th b | ac
+ Nu b | a v c | b th c | a
+ Nu c | a v c | b th c | (ax + by) vi x, y nguyn
+ Nu b | a v a | b th a = b
+ Nu c | a v d | b th cd | ab
+ Nu b | a v c | a th BCNN (b, c) | a
+ Nu c | ab v (b, c) = 1 th c | a
+ Nu p | ab vi p l s nguyn t th p | a hoc p | b

2
2.1
2.2

Phng php gii cc bi chia ht


Phng php phn tch thnh tch v phn tch tch tng
Phn tch thnh tch

chng minh A(n) chia ht cho p ta phn tch A(n) = B(n).p; cn nu khng th a ra
cch phn tch nh vy ta c th phn tch p = k.q
Nu (k, q) = 1 th chng minh A(n) cng chia ht cho k v q
.
.
Nu (k, q) 6= 1 th phn tch A(n) = B(n).C(n) v chng minh B(n)..k, C(n)..q
V d 1. Cho n l s nguyn dng. Chng minh rng 2n | (n + 1)(n + 2) (2n)
Li gii. Ta c
(n + 1)(n + 2) .(2n) =
99

Vy
2n | (n + 1)(n + 2) (2n)
V d 2. Cho m, n l hai s chnh phng l lin tip. Chng minh rng
mn m n + 1 chia ht cho 192
Li gii. V m, n l hai s chnh phng l lin tip nn chng c dng
m = (2k 1)2 v n = (2k + 1)2
Do
mn m n + 1 = (m 1)(n 1) = [(2k 1)2 1][(2k + 1)2 1]
= (4k 2 4k)(4k 2 + 4k) = 16k 2 (k 1)(k + 1)
.
.
.
m (k 1)k(k + 1)..3 v (k 1)k.k.(k + 1)..4 v (3, 4) = 1 nn k 2 (k 1)(k + 1)..12 Vy mn
.
m n + 1 = 16k 2 (k 1)(k + 1)..16.12 = 192

2.3

Phn tch tch tng

chng minh A(n) chia ht cho p, ta bin i A(n) thnh tng nhiu hng t v ri chng
minh mi hng t chia ht cho p.
V d 3. Cho a, b l hai s nguyn dng v a + 1, b + 2007 chia ht cho 6. Chng minh 4a + a + b
chia ht cho 6. ( tuyn sinh HKHTN-HQG H Ni 2007 - 2008)
.
Li gii. Ta c: 4a + a + b = (4a + 2) + (a + 1) + (b + 2007) 2010 m (4a + 2)..2 v
.
.
4a + 2 = 4a 1 + 3 = (4 1)(4a1 + 4a2 + + 1) + 3..3 v (2; 3) = 1 nn 4a + 2..6 v
a + 1, b + 2007, 2010 cng chia ht cho 6. Vy 4a + a + b chia ht cho 6.

2.4

Bt ng thc m rng:

Vi n N ta c:
an bn = (a b)(an1 + an2 b + + abn2 + bn1 )
Vi n l s t nhin l ta c:
an + bn = (a + b)(an1 an2 b + abn2 + bn1 )
Suy ra: Hai s nguyn a, b; n l s t nhin l v a 6= b th an + bn chia ht cho (a + b)
V d 4. Cho n l s nguyn dng v k l s t nhin l. Chng minh rng
1k + 2k + 3k + + nk chia ht cho 1 + 2 + 3 + + n
Li gii.
t
S = 1k + 2k + 3k + + nk
Vit li
S = nk + (n 1)k + (n 2)k + + 1k
100

Suy ra
2S = (1k + nk ) + (2k + (n 1)k ) + (3k + (n 2)k ) + + (nk + 1k )
V k l s l do (1k + nk ); (2k + (n 1)k ); (3k + (n 2)k ) chia ht cho n + 1 nn
.
2S ..(n + 1)

(1)

Mt khc
S = 1k + 2k + 3k + + nk
S = (n 1)k + (n 2)k + (n 3)k + + 1k + nk
Suy ra
2S = [1k + (n 1)k ] + [2k + (n 2)k ] + [3k + (n 3)k ] + + [(n 1)k + 1k ] + 2nk
V k l s l do [1k + (n 1)k ]; [2k + (n 2)k ]; [3k + (n 3)k ]; chia ht cho n v 2nk chia
ht cho n nn
.
2S ..n
(2)
.
.
T (1),(2) v (n; n + 1) = 1 suy ra 2S ..n(n + 1) v v n(n + 1) l s chn nn S .. = 1 + 2 + + n.
.
Vy S ..(1 + 2 + + n)

2.5

Phng php xt php chia c d

.
chng minh biu thc A(n)..p ta xt tt c cc s d trong php chia n cho q, chia n cho q
c cc s d l 0; 1; 2; ; q 1.
V d 5. Cho x, y, z l cc s nguyn tha mn (x y)(y z)(z x) = x + y + z. Chng minh
rng x + y + z chia ht cho 27.
Li gii:
Nu 3 s x, y, z cng chia cho 3 c cc s d khc nhau th x + y + z chia ht cho 3 nhng hiu
x y, y z, z x cng khng chia ht cho 3 (v l) nn trng hp ny khng xy ra.
Nu 3 s x, y, z chia cho 3 ch c 2 s c cng s d th x + y + z khng chia ht cho 3 nhng
.
.
c 1 hiu x y, y z, z x chia ht cho 3 (x y)(y z)(z x)..3 nn x + y + z ..3 (v l) nn
trng hp ny khng xy ra.
Do 3 s x, y, z chia cho 3 c cng s d gi l r nn t x = 3p + r; y = 3q + r; x = 3k + r
vi r = 0; 1; 2 do x y = 3(p q); y z = 3(q k); z x = 3(k p) chia ht cho 9 suy ra
x + y + z = (x y)(y z)(z x) chia ht cho 27.
B 1. Mt s chnh phng chia cho 3 c s d l 0 hoc 1; chia cho 5 c s d l 0, 1 hoc
4. Do mt s c dng 3k + 2; 5k + 2 hoc 5k + 3 khng l s chnh phng
V d 6. Cho 3 s nguyn x, y, z tha mn x2 + y 2 = z 2 . Chng minh rng xyz chia ht cho 60.
Li gii. Ta c 60 = 3.4.5 V (4, 3, 5) = 1
.
+ Chng minh xy ..3. Tht vy: Nu x, y cng khng chia ht cho 3 th theo h qu x, y chia 3
d 1 nn x = 3q + 1, y = 3p + 1. Khi x2 + y 2 = 3(3q 2 + 3q + 3p2 + 3p) + 2 chia cho 3 d 2 nn
.
.
.
z 2 chia 3 d 2 (v l). Vy x..3 hoc y ..3 nn xyz ..3
.
+ Chng minh xyz ..4

101

Nu x, y l cc s l th x = 2k + 1, y = 2l + 1 nn x2 + y 2 = 2(2k 2 + 2k + 2l2 + 2l) + 2 l s


chn v khng chia ht cho 4 nn x2 + y 2 khng l s chnh phng. Do trng hp ny khng
xy ra.
.
.
Nu x, y cng l s chn th xy ..4 xyz ..4
Nu x l s chn v y l s l (Trng hp x l s l, y l s chn tng t) th x = 2k, y = 2l+1
v z l s l c dng z = 2q + 1. Do
x2 + y 2 = z 2 4k 2 + 4l2 + 4l + 1 = 4q 2 + 4q + 1
k 2 = q(q + 1) l(l + 1) chia ht cho 2
.
.
k ..2 do x..4.
.
Vy xyz ..4
.
+ Chng minh xyz ..5
.
.
.
Nu x..5 hoc y ..5 th xyz ..5
Nu x, y cng khng chia ht cho 5 th x2 , y 2 chia cho 5 d 1 hoc 4. Khi x2 + y 2 chia cho
.
5 d 0, 2 hoc d 3 nhng s chnh phng z 2 chia cho 5 ch c th l d 0, 1 hoc 4 nn z ..5 nn
.
xyz ..5
.
Vy xyz ..60

2.6

Phng php xt ng d thc

nh ngha 2. Cho a, b l cc s nguyn v n l s nguyn dng. Ta ni a ng d vi b theo


mun n, k hiu a b (mod n) nu a v b c cng s d khi chia cho n.
Tnh cht 2. Nu a b (mod n), c d (mod n), th a c b d (mod n); ac bd (mod n).
H qu 1. a b (mod n) th a c b c (mod n); ac bc (mod n)
am b m

(mod n)vim N

V d 7. Cho a1 , a2 , , an l cc s nguyn v a1 + a2 + + an chia ht cho 6. Chng minh


rng a31 + a32 + + a3n chia ht cho 6.
.
Li gii. Vi mi s nguyn n, n3 n = n(n 1)(n + 1)..6 nn n3 n(mod6)
a31

Thay n = ai ta c a3i ai (mod6) a31 + a32 + + a3n a1 + a2 + + an 0(mod6) Vy


+ a32 + + a3n chia ht cho 6.

V d 8. Vit lin tip cc s 111; 112; 113; ; 888 c s A = 111112113 888. Chng
minh rng 1998 | A.
Li gii. Ta c 1998 = 2.999. V A l s chn nn 2 | A. Mt khc, A = 111.1000777 +
112.1000776 + + 888. Do 1000 1 (mod 9)99 1000n 1 (mod 9)99; vi n N. Suy ra
A 111 + 112 + 113 + + 888 0 (mod 9)99 hay 999 | A V (999, 2) = 1 nn 1998 | A.

2.7

Phng php quy np

Gi s cn chng minh

.
A(p)..q vi p p vi p, q N.

Ta cn chng minh
102

(3)

.
+ Bc 1: (3) ng vi n = p ngha l chng minh A(1)..q.
.
+ Bc 2: Gi s (3) ng vi n = k p, tc l A(k)..q. Ta chng minh (3) ng vi n = k + 1,
..
tc A(k + 1).q.
Theo nguyn l quy np kt lun (3) ng
V d 9 (HSG 9 TQ 1987). Chng minh rng s c thnh lp bi 3n ch s ging nhau th
chia ht cho 3n vi n l s nguyn dng.
Li gii. Ta cn chng minh

.. n
.3

(4)

.
+ Vi n = 1 ta c ..3. Vy (4) ng vi n = 1
.
+ Gi s (4) ng vi n = k (vi k N), tc l ta c: ..3k . Ta cn chng minh (4) ng vi
.
n = k + 1, tc phi chng minh ..3k+1 .
Tht vy: == . + . + .

.
= .(+1)..3+1

.
( v theo gi thit quy np ..3k v +1) 1 + 1 + 1 0( mod 3))
Vy (4) ng vi n = k + 1. Do s c thnh lp bi 3n ch s ging nhau th chia ht
cho 3n vi n l s nguyn dng.

2.8

Phng php s dng nh l Fermat nh

nh l 1 (nh l Fermat). Vi mi s nguyn a v p l s nguyn t ta c: ap a( mod p).


c bit: Nu (a, p) = 1 th ap1 1( mod p)
Chng minh. Gi s p l s nguyn t v (a, p) = 1. S d khi chia a, 2a, , (p 1)a cho p l
r1 , r2 , , rp1 i mt khc nhau (V chng hn r1 = r3 thi p | (3a a) = 2a p = 2 (v l) )
v cc s d l 1, 2, 3, , p 1. Ta c:
Nhn tng ng cc ng d thc ta c 1.2 .(p1) r1 r2 rp1 ( mod p) m 1.2 (p
1) = (p 1)! V r1 r2 rp1 = (p 1)! Nn (p 1)!ap1 (p 1)!(modp) suy ra ap1 1(modp)
(v ((p 1), p) = 1. Vy ap a( mod p).
.
V d 10. Cho p, q l hai s nguyn t khc nhau. Chng minh pq1 + q p1 1..pq
Li gii. V p, q l hai s nguyn t v p 6= q nn (p, q) = 1. p dng nh l Fermat ta c:
.
.
.
.
pq1 1( mod q) v q p1 1( mod p) pq1 1..q v q p1 1..p. Mt khc pq1 ..p v q p1 ..q v
.
(p, q) = 1 nn pq1 + q p1 1..pq.
V d 11. Chng minh rng A = 130 + 230 + 330 + + 1130 + 1230 chia ht cho 11.
Li gii. Vi mi a = 1; 2; 3; ; 10; 12 th (a; 11) = 1. Do theo nh l Fermat b th
a10 1(mod11) a30 1(mod11) vi mi a = 1; 2; 3; ; 10; 12. Nh vy: A 1 + 1 + 1 + 1 +
1 + 1 + 1 + 1 + 1 + 1 + 0 + 1 = 11 0(mod11) Hay A chia ht cho 11.

2.9

Phng php tm ch s tn cng ca mt s

V d 12. Chng minh rng s 0, 3(19831983 19171917 ) l s nguyn. ( thi hc sinh gii nm
1983).
103

Li gii. 19831983 34k+3 ( mod 10) m 34k 1( mod 10) nn 34k+3 33 7( mod 10). Mt
khc: 19171917 741+1 7( mod 10) nn 19831983 19171917 chia ht cho 10. Vy s 0, 3(19831983
19171917 ) l s nguyn.
V d 13. Chng minh rng vi mi n N, +5 chia ht cho 11.
Li gii. Theo nh l Fermat ta c: 310 1(mod11) v 210 1( mod 11).
Ta tm d trong php chia 24n+1 v 34n+1 cho 10, tc tm ch s tn cng ca chng:
Ta c: 24n+1 = 2.16n 2(mod10) 24n+1 = 10k + 2 (vi k N)
34n+1 = 3.81n 3(mod10) 34n+1 = 10l + 3 (vi l N)
M 310k 1( mod 11) v 210l 1( mod 11) nn +5 310k + 2 + 210l + 3 + 3 32 + 23 + 5
0(mod11) Vy vi mi n N, +5 chia ht cho 11.

2.10

Phng php phn chng

chng minh p - A(n) ta ln lt nh sau:


+ Gi s ngc li p | A(n)
+ Chng minh iu ngc li l sai.
V d 14. Chng minh rng vi mi n N th n2 + 11n + 39 khng chia ht cho 49
.
.
Li gii. Gi s n2 + 11n + 39..49. Ta c n2 + 11n + 39 = (n + 9)(n + 2) + 21..7 (n +
.
.
.
.
.
9)(n + 2)..7 (n + 9)..7v(n + 2)..7 (v (n + 9) (n + 2) = 7..7) nn (n + 9)(n + 2)..49. Mt khc
.
.
n2 + 11n + 39 = (n + 9)(n + 2) + 21..49 21..49 (v l). Vy vi mi n N th n2 + 11n + 39 khng
chia ht cho 49.
V d 15. Gi s p = k.2t + 1 l s nguyn t l vi t l s nguyn dng v k l s t nhin l.
Gi thit x v y l cc s t nhin m p | (Chng minh rng x v y ng thi chia ht cho p.
Li gii. Gi s p - x v p - y. Do p l s nguyn t nn theo nguyn l Fermat nh ta c ; theo
gi thit th
p 1 = k.2t nn suy ra
(5)
Theo gi thit. Do
k l nn chia ht cho

(6)

T (5) , (6) suy ra mu thun. Vy gi thit phn chng sai do x, y cng chia ht cho p.

2.11

Phng php s dng nh l Dirichlet

Nguyn tc ngn ko Dirichlet: Nu em n + 1 vt xt vo n ngn ko th c t nht 1 ngn


ko cha 2 vt tr ln.
M rng: Nu em nk + 1 vt xp vo n ngn ko th c t nht 1 ngn ko cha t k + 1
vt tr ln.
V d 16. Trong mt bng vung gm c 5 5 vung, ngi ta vit vo mi vung 1 trong
3 s 1; 0; -1 sao cho mi c ng 1 s. Chng minh rng trong cc tng ca 5 s theo mi ct,
mi hng, mi ng cho phi c t nht 2 tng s bng nhau.
Li gii. Ta c 5 ct, 5 dng v 2 ng cho nn c 12 tng. Mi nhn 1 trong 3 gi tr 1;
0; -1 nn mi tng nhn cc gi tr t 5; 4; ; 1; 0; 1; ; 5. Ta c 11 s nguyn t -5 n
5. Vy theo nguyn tc Dirichlet phi c t nht 2 tng s bng nhau. Vy trong cc tng ca 5
s theo mi ct, mi hng, mi ng cho phi c t nht 2 tng s bng nhau.

104

V d 17. Chng minh rng lun tn tai s c dng chia ht cho 2012.
Li gii. Ly 2013 s c dng: 2011; 20112011; ; . Ly 2013 s ny chia cho 2012 theo nguyn
l Dirichlet th tn ti 2 s c cng s d khi chia cho 2012. Gi s hai s l v (vi k > p > 0)
khi 2012 | = . Vy tn ti s tha mn bi ton.

Bi tp luyn tp
Bi tp 1. Cho 11 | (16a + 17b)(17a + 16b) vi a, b l hai s nguyn. Chng minh rng 121 |
(16a + 17b)(17a + 16b)
Bi tp 2. Cho a, b l hai s t nhin. Chng minh rng 5a2 + 15ab b2 chia ht cho 49 khi
3a + b chia ht cho 7.
Bi tp 3. Vi mi s t nhin chn n th 20n + 16n 3n 1 chia ht cho 323
Bi tp 4. Cho 2012 s t nhin bt k a1 , a2 , a3 , , a2012 . Chng minh rng tn ti s chia ht
cho 2012 hoc tng mt s s chia ht cho 2012.
Bi tp 5. Cho 5 s nguyn phn bit ty a1 , a2 , a3 , a4 , a5 . Chng minh (a1 a2 )(a1 a3 )(a1
.
a4 )(a1 a5 )(a2 a3 )(a2 a4 )(a2 a5 )(a3 a4 )(a3 a5 )(a4 a5 )..288. (Tuyn sinh lp 10 HSPHN,
1993)
Bi tp 6. Chng minh rng t 2n+2 1 s nguyn bt k lun tm c 2n s m tng ca chng
chia ht cho 2n
Bi tp 7. a) Tm tt c cc s t nhin n 2n 1 chia ht cho 7.
b) Chng minh rng vi mi s nguyn dng n th 2n + 1 khng chia ht cho 7.
(Thi ton quc t 1964)
Bi tp 8. Trong 100 s t nhin t 1 n 100 cn chn n s (n 2) sao cho tng 2 s phn
bit bt k c chn c tng chia ht cho 6. Hi c th chn n s tha mn bi ton trn vi n
ln nht l bao nhiu?

Ti liu tham kho


* Mt s chuyn ton hc bi dng HSG - H Ni 2004
* Cc bi ton c bn s hc - Phan Huy Khi
* S hc - Nguyn V Thanh - Nh xut bn gio dc
* K yu cc hi tho Ton hc, tp ch ton hc v tui tr 2007 -2014
* Cc thi hc sinh gii, tuyn sinh THPT,. . .
* Internet: ienantoanhoc.net. . .

105

Mt s phng php gii ton t hp


Cao Trn T Hi
THPT Chuyn L Qu n, Ninh Thun
L thuyt t hp ng mt vai tr kh quan trng trong cc mn khoa hc v c bit l trong
Tin hc v Ton ng dng. C th ni, l thuyt bi ton t hp vi c s l cc bi ton m, bi
ton tn ti, bi ton lit k, bi ton ti u c nhiu ng dng v thng xut hin hu khp
cc lnh vc khoa hc nh: Xc sut Thng k, Sinh hc Di truyn, Ha hc Cu trc , . . .
Khi gii mt bi ton t hp ni chung, cc quy tc t hp ni trn thng ny sinh mt cch
t nhin nh quy tc cng, quy tc nhn, t hp, chnh hp, hon v, . . . Tuy nhin, vi cc cng
c c s trn, chng ta thng ch gii c nhng bi ton dng n gin nht. Vi cc bi
ton c yu cu phc tp hn trong cc k thi hc sinh gii, cn n cc phng php nng cao
cao hn.
C nhiu phng php gii bi ton t hp da trn cc nn tng l thuyt khc nhau. V d
phng php song nh da vo l thuyt tp hp v nh x, phng php qu o da vo mt
nh l c bn v s ng i ngn nht gia hai im ca li nguyn, phng php thit lp h
thc truy hi da vo tng quy np, phng php hm sinh s dng cc kin thc tng hp ca
i s v gii tch, . . .Trong bi vit ny, chng ti s tp trung vo gii thiu cc phng php
gii bi ton t hp bi ton t hp nh vy.

Phng php song nh

Phng php song nh da vo mt tng rt n gin nh sau: Nu tn ti mt song nh


t A vo B th |A| = |B| . Do , mun chng minh hai tp hp c cng s phn t, ch cn xy
dng mt song nh gia chng. Hn na, ta c th m c s phn t ca mt tp hp A bng
cch xy dng song nh t A vo mt tp hp B m ta bit cch m. y l mt phng
php khng mi nhng rt th v. N c th a ra nhng li gii kh bt ng cho nhng bi
ton t hp kh.
Bi ton 1. Cho X = {1, 2, . . . , n}. Mt tp con ca X c gi l tp bo nu mi phn t ca
ca tp con u khng nh hn s phn t ca n. Ch rng tp rng cng l mt tp bo.
Khi t an l s cc tp con bo ca X sao cho mt tp bo bt k khng cha hai s lin tip,
bn l s cc tp con ca X m hai phn t bt k hn km nhau t nht 3 n v. Chng minh
an = b n .
Li gii.
Cch 1. Ta i tnh ln lt an , bn .
Tnh an .
Vi mi k, gi M l h cc tp bo c k phn t tho mn bi. Gi N l h cc tp con c
k phn t ca {k, k + 1, . . . , n k + 1}
Xt f : M N cho bi quy tc nh sau: Gi s {m1 , m2 , . . . ., mk } M vi n mi k,
i = 1, 2, .., k. Khng gim tng qut ta gi s mi < mj vi i < j.
t m1 = n1 , m2 1 = n2 , m3 2 = n3 , . . . , mk k + 1 = nk .
Do {m1 , m2 , . . . ., mk } khng cha hai s lin tip nn m1 < m2 1 < . . . < mk k + 1. suy
ra k n1 < n2 < . . . < nk n k + 1.
Ta nh ngha f ({m1 , m2 , . . . ., mk }) = {n1 , n2 , . . . , nk }, khi {n1 , n2 , . . . , nk } N. Ta d
dng chng minh c f l song nh. Mt khc |N | chnh l cch chn k phn t t tp

105

k
. Vy an =
{k, k + 1, . . . , n k + 1} v bng Cn2k+2

n
P

k
vi quy c Cki = 0 nu i > k
Cn2k+2

k=0

hoc k < 0.
Tnh bn .
tng ca chng ta cng l thit lp mt song nh i t h cc tp con ca X m hai phn
t bt k hn km nhau t nht 3 n v sang mt tp khc d m hn. Lc ny da vo tng
hnh thnh ch tnh an ta cng mnh dn thit lp mt song nh tng t.
Gi Ck l h cc tp con c k phn t ca X m hai phn t bt k hn km nhau t nht 3
n v, Dk l h cc tp con c k phn t ca tp {1, 2, . . . , n 2k + 2} .
Ta thit lp mt nh x f : Ck Dk nh sau : Gi s {c1 , c2 , . . . , ck } Ck , ta c th gi thit
c1 < c2 < . . . < ck . t di = ci 2i + 2 vi i = 1, 2, .., k. V ci+1 ci 3 nn di+1 di 1. Do
1 d1 < d2 < . . . < dk n 2k + 2. Ta nh ngha f ({c1 , c2 , . . . , ck }) = {d1 , d2 , . . . , dk } suy ra
{d1 , d2 , . . . , dk } Dk . Ta cng d dng chng minh c f l song nh.
q
Mt khc |Dk | chnh l cch chn q phn t t tp {1, 2, . . . , n 2k + 2} v bng Cn2k+2
.
n
n
n
P
P
P k
Khi bn =
|Ck | =
|Dk | =
Cn2k+2 ( ta cng quy c nh trn )
k=0

k=0

k=0

n y suy ra c an = bn . Vy ta c iu phi chng minh.


Nhn xt 1. R rng hai nh x s dng trong tnh an v bn c v kh ging nhau : s xut
k
hin m m ca Cn2k+2
. iu ny gi cho ta lin tng ti mt song nh trc tip gia an v bn .
y chnh l c s hnh thnh cch gii th hai ca bi ton.
Cch 2. Gi Ak l h cc tp bo c k phn t tho mn bi, Bk l h cc tp con ca X
c tnh cht hai phn t bt k hn km nhau t nht 3 n v.
Ta thit lp mt nh x f : Ak Bk nh sau : Gi s {a1 , a2 , . . . , ak } Ak . Ta c th gi s
k a1 < a2 < a3 < . . . < ak n. t b1 + k 1 = a1 , b2 + k 2 = a2 , . . . , bk = ak .
Ta c ai+1 ai + 2 vi mi i = 1, 2, .., k 1 ai+1 ai 2 bi+1 bi 3 v b1 1, bk n.
Ta nh ngha f ({a1 , a2 , . . . , ak }) = {b1 , b2 , . . . , bn } = {a1 + 1 k, a2 + 2 k, . . . , ak }
Suy ra {b1 , b2 , . . . , bn } Bk . Do vy f l mt nh x t Ak vo Bk . Ta d dng chng minh
c f l song nh. V tn ti mt song nh gia Ak v Bk nn |Ak | = |Bk | t suy ra an = bn .
Nhn xt 2. R rng cch th hai ngn gn v sng sa hn nhiu so vi cch th nht trn.
Tuy nhin mi cch u c ci hay ring ca n. Cch gii th nht tuy di nhng ay chnh l
c s hnh thnh tri thc phng php cho ngi mi bt u lm quen vi phng php song
nh v cng l c s gip ta gii quyt nhng vn hc ba hn sau ny.
Bn cnh nhng ng dng ca php song nh th ta cng c th s dng nh x n nh, ton
nh gii cc bi ton t hp. C th nh sau:
Vi A, B l cc tp hp hu hn. Xt f l mt nh x i t A vo B. Khi , nu f l n
nh th |A| |B|, nu f l ton nh th |A| |B|. Hn na, nu f l n nh v khng ton nh
th |A| < |B|, nu f l ton nh v khng n nh th |A| > |B|.
Bi ton 2. (IMO 1989) Mt hon v (a1 , a2 , . . . , a2n ) ca {1, 2, . . . 2n} , n N c gi l c tnh
cht P nu tn ti t nht mt gi tr ca i {1, 2, . . . , 2n 1} sao cho |ai ai+1 | = n. Chng
minh rng vi mi gi tr ca n th s hon v c tnh cht P lun ln hn s hon v khng c
tnh cht y.
Li gii. Chng ta s bt u t nhng trng hp nh trc .
D dng kim chng khng nh ng vi n = 1.
Gi s n 2. Gi A, B tng ng l h cc tp khng c v c tnh cht P . Ta cn chng
minh |A| < |B|. By gi ta s chng minh da trn tng thit lp mt nh x f t A vo B
sao cho f l n nh nhng khng l ton nh.
106

t T = (a1 , a2 , . . . , a2n ) l mt phn t ca A. Do T khng c tnh cht P nn hai phn t


lin tip no cng hn km nhau mt khong khc n.
Do phn t cch a1 khong cch n l ar trong 2 < r < 2n + 1.
t T0 = f (T ) = (a2 , a3 , . . . , ar1 , a1 , ar , ar+1 , . . . , a2n ). Khi {x1 , xr } l cp lin k c
khong cch n. V vy T0 B nn f l mt nh x t A vo B.
Chng minh f l n nh:
Gi = (x1 , x2 , . . . , x2n ) v = (y1 , y2 , . . . , y2n ) l cc hon v trong h A. Gi thnh phn
cch x1 , y1 khong cch n n v ln lt trong v l xr v ys khi
3 xr 2n, 3 ys 2n.
Gi s
f () = f () (x2 , x3 , . . . , xr1 , x1 , xr , xr+1 , . . . , x2n )
= (y2 , y3 , . . . , ys1 , y1 , ys , ys+1 , . . . , y2n )
trong (x1 , xr ) v (y1 , ys ) l cc cp cch nhau n duy nht tng ng trong f () v f ().
Do ta suy ra r = s. Tht vy, nu r 6= s th ta s tm c mt cp s (xs , xs+1 ) tho mn
|xs xs+1 | = n, tri gi thit A.
T suy ra xi = yi , i = 1, 2, . . . , 2n hay = . Vy f l n nh.
Chng minh f khng l ton nh.
rng f (A) cha tt c cc hon v ca s ch gm 1 cp lin k cch nhau n v B gm
cc hon v cha t nht mt cp lin k cch nhau n nn f (A) h con thc s ca B.
Chng hn, ta thy (1, n + 1, 2, n + 2, 3, 4, . . . , n, n + 3, . . . , 2n) thuc B nhng khng thuc
f (A). Suy ra f khng l ton nh.
Vy ta c iu phi chng minh.
Nhn xt 3. Chng minh trn c th m rng ra vi khong cch k bt k ch khng nht thit
phi l n (k {1, 2, . . . , n}). Phng php s dng song nh l mt phng php gii rt hay,
rt p, th hin s tinh t , ngu hng ca n. tm ra mt li gii cho mt bi ton bng
phng php ny qu thc khng h n gin, nhng khi tm ra th c cm gic v cng tuyt
vi. Hi vng cc bn t nhiu tm ra cho mnh nhng kinh nghim gii quyt nhng bi ton
t hp hc ba hn sau ny.

Phng php thit lp h thc truy hi

Phng php thit lp h thc truy hi l phng php gii bi ton vi n i tng thng
qua vic gii bi ton tng t vi s i tng t hn bng cch xy dng cc quan h no ,
gi l quan h quy. S dng quan h ny, ta c th tnh c i lng cn tm nu ch rng
vi n nh, bi ton quy v xc nh cng thc s hng tng qut ca dy s cho bng cng thc
truy hi. Chng hn, cng thc s hng tng qut ca dy sai phn tuyn tnh (xem [2]) rt hay
s dng trong phng php ny. Ta minh ha phng php thit lp h thc truy hi ny thng
qua mt s bi ton sau y.
Bi ton 3. C bao nhiu cch lt ng i kch thc 3 2n bng cc vin gch kch thc
12 ?
Li gii. Gi cn l s cch lt ng i kch thc 3 2n. D thy c1 = 3. tnh cn , ta chia
cc cch lt ng i kch thc 3 2n thnh n lai, trong lai th k l cc cch lt m phn
ng i 3 2k u tin c ph kn hon ton, nhng khng tn ti i < k sao cho phn ng
i 3 2i u tin c ph kn hon ton.
Gi Ak l tp hp cc cch lt lai k th r rng cn = |A1 | + |A2 | + . . . + |An |. D dng nhn
thy |A1 | = 3cn1 (phn ng i 3 2 c lt kn bng 3 cch, phn cn li c lt bng cn1
cch).
107

Tip theo, c th chng minh d dng rng, ch c hai cch ph phn ng i 3 2k cho cc
cch ph thuc Ak vi k = 2, 3, . . . , n, chnh l cch ph v cch ph thu c bng cch ly i
xng.
T suy ra |Ak | = 2cnk . Nh vy, ta c cn = 3cn1 + 2cn2 + . . . + 2. y l dng cng thc
truy hi bc v hn. thu c mt cng thc truy hi bc hu hn, ta thay n bng n + 1 suy
ra cn+1 = 3cn + 2cn1 + 2cn2 + . . . + 2.
T , tr hai ng thc cui cng v theo v, ta c cn+1 cn = 3cn cn1 v cui cng l
cn+1 = 4cn cn1 .

Phng trnh c trng X 2 4X + 1 = 0 c hai nghim phn bit X = 2 3, X = 2 + 3


v c1 = 3, c2 = 11 nn ta c c cng thc tng qut

n
n
3+1
31
cn =
2+ 3 +
2 3 .
2
2
Bi ton 4. (D tuyn IMO-1996) Cho bng vung n n(n > 1). Hi c bao nhiu cch nh
du cc vung trong bng sao cho trong mi hnh vung 2 2 c ng hai vung c nh
du?
Li gii.
Gi sn l s cch nh du trong bng n n. Xt tp T gm cc vung nm trong ct cui
cng tnh t phi sang v hng cui cng tnh t trn xung di, ta gi An l cc cch nh du
m c hai vung k nhau trong T cng c nh du hoc khng cng c nh du v Bn
l cc cch nh du m cc vung trong T c nh du xen k.
Ta thy vi mi cch nh du trong bng n n, b nh du cc trn T s c mt cch
nh du trong bng (n 1) (n 1). Do mi cch nh du trong Bn s ng vi mt cch
nh du thuc Bn1 , cn mi cch nh du trong An s ng vi mt cch nh du thuc An1
v mt cch nh du thuc Bn1 .
T |Bn | = |Bn1 | , |An | = |An1 | + |Bn1 | (n > 2)
Hn na sn = |An | + |Bn | , n > 1 nn sn = 2sn1 sn2 , n > 3
Bng cch tnh trc tip ta c s2 = 6, s3 = 14 v phng trnh c trng cho dy s tuyn
tnh cp hai c nghim kp X = 1 nn sn = 2n 10, n > 1.

Phng php hm sinh

Phng php hm sinh l mt phng php hin i, s dng cc kin thc v chui, chui
hm (c bit l khai trin Newton suy rng, cng thc Taylor). y l phng php rt mnh
gii bi ton t hp.
Cho dy s a0 , a1 , a2 , . . . , an , . . . Chui hnh thc A (x) = a0 + a1 x + a2 x2 + + an xn + . . .
c gi l hm sinh ca dy {an }.
tng c bn ca phng php hm sinh nh sau: Gi s ta cn tm cng thc tng qut
ca mt dy s {an } no . T cng thc truy hi hoc nhng l lun t hp trc tip, ta tm
c hm sinh
A (x) = a0 + a1 x + a2 x2 + + an xn + . . .
Khai trin A (x) thnh chui v tm h s ca xn trong khai trin ta tm c an . Cng
thc khai trin thng s dng
Cng thc nh thc Newton suy rng
( 1) x2
( 1) . . . ( n + 1) n
+ +
x + ...
(1 + x) = 1 + x +
2
n!
Cng thc khai trin Taylor

108

f 00 (0) 2
f (n) (0) n
x + +
x + ...
2!
n!
Bi ton 5. Tm s hng tng qut ca dy s Fibonacci c xc inh bi f0 = 1, f1 = 1, fn+1 =
fn + fn1 , n N .
Li gii. Xt hm sinh
F (x) = f0 + f1 x + f2 x2 + + fn xn + . . .
= f0 + f1 x + (f0 + f1 ) x2 + + (fn1 + fn2 ) xn + . . .
= f0 + f1 x + x2 (f0 + f1 x + . . . ) + x (f1 x + . . . )
= f0 + f1 x + x2 F (x) + x (F (x) f0 )
1+x
T suy ra F (x) =
1 x x2
Tip theo, ta khai trin F (x) thnh chui.

1 1
1+ 5
1 5
2
Ta c , l nghim ca phng trnh x x 1 = 0, vi =
, =
.

2
2
A
B
Vit F (x) di dng F (x) =
+
vi A, B l hng s. Bng cch ng nht hai
1 x 1 x

1+ 5
1 5

v , ta c A = , B =
hay A = , B = .
2 5
2 5
5
5
1
T , s dng cng thc
= 1 + x + x2 + + xn + . . . ta c
1x
F (x) = A (1 + x + + n xn + . . . ) + B (1 + x + + n xn + . . . )
= A + B + (A + B)x + + (An + B n )xn + . . .
!n+1
!n+1
1
+
1

1
5
1
5
suy ra fn = An + B n do fn =

.
2
2
5
5
f (x) = f (0) + f 0 (0)x +

Nhn xt 4. Bi ton tm cng thc s hng tng qut ca dy Fibonacci c th gii bng
phng php s dng phng trnh sai phn tuyn tnh cp hai kh gn. Tuy nhin, chng ti
mun s dng bi ton ny lm v d minh ha phng php hm sinh (mt phng php kh
mi v xa l cn trang b kin thc v ton cao cp nh chui ly tha v khai trin Taylor)
to ra s gn gi ca phng php hm sinh.
Bi ton 6. C 2n im trn ng trn. Hy tm s cch ni 2n im ny bng n dy cung
khng ct nhau.
Li gii. Gi cn l p s bi ton. Ta c c1 = 1, c2 = 2 v quy c c0 = 1. Khi ta d dng
xy dng c cng thc truy hi
cn = cn1 c0 + cn2 c1 + + c0 cn1 .
Xt hm sinh A (x) = c0 + c1 x + c2 x2 + + cn xn + . . .
T cng thc truy hi trn, ta c
(A (x))2 = c0 + (c0 c1 + a1 a0 ) x + + (cn c0 + cn1 c1 + + c0 cn ) xn + . . .
= c1 + c2 x + + cn+1 xn + . . .

T suy ra


1
x(A (x)) = A (x) 1 x (A (x)) = xA (x) x xA (x)
2
1
1
1 + (1 4x) 2
1 (1 4x) 2
nn xA (x) =
hoc xA (x) =
.
2
2
2

109

2
= x +

1
1 4x
=
4
4

Do xA (x) khng c s hng khng cha x, tc l n trit tiu ti x = 0 nn


1
1 (1 4x) 2
xA (x) =
2


 

1
1
1 1
1
2 ...
n + 1 (4x)n

P
1 1
2 2
2
2
xA (x) =
4 4 n=0
n!
   

3
2n 3
1 1
...
(4x)n

P
1
2 2
2
2
xA (x) =
2 n=1
n!
1.3 . . . (2n 3)2n1 xn
P
xA (x) =
n!
n=1
1.3 . . . (2n 3)2n1 xn1
1.3 . . . (2n 1)2n xn
P
P
A (x) =
A (x) =
.
n!
(n + 1)!
n=1
n=0
1.3 . . . (2n 1)2n
(2n)!
Cn
Do cn =
=
= 2n .
(n + 1)!
n!(n + 1)!
n+1
Nhn xt 5. Ngi ta nh ngha cc s Catalan c cho bng cng thc truy hi c0 = 1,
cn = cn1 c0 + cn2 c1 + + c0 cn1 . Khi theo bi ton trn ta xc nh c cc s Catalan
Cn
cn = 2n . Ngoi ra ta c th xc nh cc s Catalan bng phng php qu o v nguyn l
n+1
i xng gng nh sau:
Gi s A(a; ), B(b; ) l cc im c to nguyn, hn na b > a 0, > > 0 v
0
A (a; ) l im i xng vi A qua trc Ox. Gi qu o t A n B l ng i t A n B
ch qua cc im nguyn v ch i theo hng ln trn v qua phi. Khi s cc qu o t A
n B ct trc Ox hoc c im chung vi Ox bng s cc qu o t A0 n B. T ta thu
c cng thc tnh tt c cc s Catalan.

Dng cng c s phc

S phc c rt nhiu ng dng trong nhiu ngnh ton hc khc nhau nh hnh hc, i s,
s hc, . . . Trong mc ny s trnh by mt s ng dng ca s phc trong gii bi ton t hp.
tin theo di chng ti trnh by mt s kt qu c bn v s phc nhng khng chng minh.
Chi tit cc chng minh xin xem [3].
Cho p l mt s nguyn t l. Khi phng trnh xp = 1 c p nghim phc phn bit
2k
2k
+ i sin
vi k = 0, 1, 2, . . . , p 1.
xk = cos
p
p
T ng thc xp 1 = (xp 1) (xp1 + xp2 + + 1) v x0 = 1 ta suy ra phng trnh
xp1 + xp2 + + 1 = 0 c p 1 nghim phc phn bit
2k
2k
xk = cos
+ i sin
vi k = 1, 2, . . . , p 1.
p
p
Gi l mt nghim bt k ca phng trnh xp1 + xp2 + + 1 = 0, trong p l mt s
nguyn t l. Khi v tri ca phng trnh l a thc bt kh quy trn trng
s thc v tp

2
p1
n
2n
(p1)n
nghim ca phng trnh l {, , . . . , }. Hn na , , . . . ,
l mt hon v ca
2
p1
{, , . . . , }, trong n nguyn t cng nhau vi p.
n
P
Gi s f (x) =
ak xk l mt a thc vi h s thc v m l mt s nguyn dng, t
k=0

2
2
z = cos
+ i sin . Khi
m
m
110

m1
P

f (z j ) =

m1
n
P P

j=0

ak z jk =

j=0 k=0

n
P
k=0

ak

m1
P

z jk .

j=0

Gi A l tp con ca {1, 2, . . . , n} gm cc s chia ht cho m. T 1 + z k + z 2k + + z (m1)k


m1
P
bng m khi k chia ht cho m v bng khng khi k khng chia ht cho m. Suy ra
f (z j ) =
j=0


m1
n
P P
P
ak z jk = m
ak .
j=0 k=0

Do

ak =

kA

kA
P
1 m1

f (z j ).

j=0

Bi ton 7. (Bay Area Math Circle 1999) Cho m, n l cc s nguyn dng. Mt hnh ch nht
c to thnh bng cch xp mt s hnh ch nht 1 m v n 1. Chng minh ta c th ch
s dng mt loi 1 m hoc n 1 to thnh hnh ch nht cho.
Li gii. Gi s hnh ch nht c dng a b, r rng a, b l cc s nguyn dng. Ta cn chng
minh a chia ht cho n hoc b chia ht cho m.
2
2
2
2
+ i sin , = cos
+ i sin .
Gi = cos
m
m
n
n
Chia hnh ch nht a b thnh a.b hnh ch nht n v. ng vi hnh ch nht n v ct
x v hng y ta t s phc x y . Tng cc s phc mi hnh ch nht n 1 bt u l vung
ct x, hng y no :
n 1
x y (1 + + + n1 ) = x y
=0
1
Tng cc s phc mi hnh ch nht 1 m bt u l vung ct x, hng y no :
m 1
x y (1 + + + m1 ) = x y
= 0.
1
Trong khi tng tt c cc s phc trn hnh ch nht l
a 1 b 1
( + 2 + + a )( + 2 + + b ) =
= 0.
1 1
V vy a = 1 hoc b = 1, do a chia ht cho n hoc b chia ht cho m. Vy ta c c iu
phi chng minh.
Bi ton 8. Cho p l mt s nguyn t l v s nguyn dng k khng vt qu p 1. Tm s
tp con X ca tp {1, 2, . . . , p} bit rng X cha ng k phn t v tng tt c cc phn t ca
X chia ht cho p.
Li gii. t A = {X {1, 2, . . . , p} : |X| = k}, Aj = {X A : S(X) j(modp)} vi j =
0, 1, . . . , p 1. Khi
|A| = Cpk . V A = A0 A1 Ap1 v Ai Aj l tp rng vi mi i 6= j nn
|A| = |A0 | + |A1 | + + |Ap1 |.
Xt a thc P (x) = xp1 + xp2 + + x + 1, a thc ny c p 1 nghim phc phn bit.
Gi s l mt nghim bt k ca P (x) th tp nghim ca P (x) l {, 2 , . . . , p1 } v p = 1.
Do phng trnh xp = 1 c p nghim phc phn bit , 2 , . . . , p P
nn ta c th vit xp 1 =
k
2
p
pk
(x )(x ) . . . (x ), so snh h s ca x
ta c (1)
S(X) = 0. Ch rng
XA

S(X) = j nu X Aj .
Do
p1
p1
P
P
|Aj | j = 0. V vy l nghim ca a thc Q(x) =
|Aj | xj
j=0

j=0

Nn ta c
111

|A0 | = |A1 | = = |Ap1 | suy ra |A0 | =

|A0 | + |A1 | + + |Ap1 |


|A|
=
.
p
p

Cpk
. Vy s tp con ca X cha ng k phn t v tng tt c cc phn t ca
p
Cpk
X chia ht cho p l
.
p
Do |A0 | =

ng dng vo mt s bi ton S hc-T hp

L thuyt t hp khng ch gii quyt cc bi ton c t ra trong chnh l thuyt ny m


cn nhiu ng dng th v trong cc ngnh ton hc khc, v d nh trong i s, s hc, hnh
hc t hp, l thuyt xc sut, . . .
Cc h s nh thc thng c ny sinh mt cch t nhin trong s hc modular, trong i
s giao hon, trong l thuyt i s Lie modular, v vy, nhng ng thc lin quan n h s nh
thc ng mt vai tr c bit quan trng.
Di y, chng ta xt mt s v d lin quan n ng dng ca t hp trong cc lnh vc
khc nhau ca ton hc.
Bi ton 9. Cho p l mt s nguyn t. ng trn c chia thnh p cung bng nhau. Hi c
bao nhiu cch t p cung bng a mu khc nhau (Hai cch t mu thu c bng mt php quay
c coi l ging nhau)?
Li gii. Mi mi cung c a cch t mu, nh vy c ap cch t mu p cung (vi quy c c
nh v tr). Trong s ny c a cch t mu bng ch mt mu. Vi mi cch t mu dng 2 mu
tr ln, ta c th dng php quay to ra p cch t mu khc c tnh trong ap cch t mu
trn nhng khng c tnh theo cch tnh bi. Nh vy s cch t mu tho mn iu kin
ap a
+ a.
bi l
p
Nhn xt 6. Ta c th chng minh nh l nh Fermat, Cho p l s nguyn t v a l s nguyn,
khi ap a chia ht cho p bng cch p dng kt qu ca Bi ton trn.
Bi ton 10. (nh l Lucas)
Cho p l s nguyn t v hai s nguyn dng m, n tha mn m n c biu din
di dng p-phn nh sau : m = aS aS1 . . . a0 |p , n = bS bS1 . . . b0 |p vi bS 6= 0. Khi Cnm
aS1
CbaSS .CbS1
. . . .Cba00 (modp).
Li gii. Do Cpk 0 (modp) vi mi k = 1, 2, . . . , p 1 nn (a + b)p ap + bp (modp). T
k

bng quy np ton hc , ta d dng chng minh c (a + b)p ap + bp (modp) vi mi a, b


nguyn.
S

S1

Hn na (1 + x)n = (1 + x)bS p (1 + x)bS1 p . . . (1 + x)b0 v ly theo modulo p ta c


S bS
S1 bS1
(1 + x)n (1 + xp ) (1 + xp )
. . . (1 + x)b0 (modp).
H s ca xm v tri l Cnm . Do biu din ca n di dng p phn l duy nht nn h s
aS1
. . . .Cba00 nn ta suy ra c iu phi chng minh.
ca xm v phi l CbaSS .CbS1
Bi ton 11. (TST VN2010) Gi Sn l tng bnh phng cc h s trong khai trin ca (1 + x)n .
Chng minh rng S2n + 1 khng chia ht cho 3.
 2n
  2n

P k k
P k k
4n
2n
2n
Li gii. Ta c (1 + x) = (1 + x) (1 + x) =
C2n x
C2n x .
k=0

So snh h s ca x2n hai v ta c


112

k=0

2n
P

k
C2n

2

2n
C4n

suy ra S2n + 1 =

k=0

2n
P

k
C2n

2

2n
+ 1.
+ = C4n

k=0

Gi s 2n c biu din di dng tam phn l 2n =

m
P

ak .3k vi ak {0, 1, 2}. Nu ak

k=0

{0, 1} , k = 1, .., m.
Khi 4n c biu din tam phn l 4n =

m
P

2ak .3k . Ta thy

k=0

m
P

ak l s chn v gi 2p =

k=0

m
P

ak .

k=0

a2
a1
am
2n
(mod3).
. . . .C2a
.C2a
C2a
Theo nh l Lucas ta c C4n
m
2
1
a2
a1
am
am
a1 a2
2p
M C2a1 .C2a2 . . . .C2am = 2 .2 . . . .2 = 2 = 4p nn S2n +1 = 4p +1 1 (mod3) hay S2n +1
khng chia ht cho 3. y l iu phi chng minh.
Nu tn ti ai = 2.
Khng mt tnh tng qut, gi s i l s nh nht sao ai = 2.Khi h s tng ng ca 4n
trong h tam phn l 1. Do C12 = 0 (v s chp ln hn s phn t) nn li p dng nh l Lucas
2n
suy ra C4n
0(mod3) hay S2n + 1 khng chia ht cho 3. Bi ton gii quyt hon ton.

Nhn xt 7. Cch tip cn bi ton ny, yu cu chng minh S2n + 1 khng chia ht cho 3 c
hai mc ch. Th nht, tnh c S2n bng cch so sch h s khi khai trin nh thc Newton.
2n
, t gi n cch khai trin theo modulo.
Th hai, a ra bn cht t hp ca C4n

Ti liu tham kho


[1] Nguyn Vn Mu (ch bin), Cc chuyn chn lc t hp v ri rc, NXB GD, 2008.
[2] Nguyn Vn Mu, Mt s bi ton chn lc v dy s, NXB GD, 2005.
[3] Nguyn Vn Mu, S phc v p dng, NXB GD 2009.
[4] Trn Nam Dng, Bi ging t hp, Tp hun GV Chuyn Ton ti Nng 2013.
[5] Titu Andreescu, Zuming Feng, Combinatorial problems, from the training the USA IMO
Team.
[6] P. S. Bravo, Problem solving method in combinatorics,On appoach to Olympiad problems,
Berkhauser. .

113

Mt s ng dng ca thng d bc hai


H Duy Ngha
Trng THPT Phan nh Phng, k Lk
Tip theo chuyn Thng d bc hai ca thy Trn Quang Vinh trnh by cun Cc
chuyn ton hc bi dng hc sinh gii do S gio dc v o to Ph Yn pht hnh, trong
chuyn ny ti trnh by mt s tnh cht ca thng d bc hai vi mun khng nguyn t v
ng dng gii cc bi ton lin quan n phng trnh nghim nguyn bc cao.

Thng d bc hai vi mun nguyn t

Trong phn ny ti ch gii thiu li cc khi nim tnh cht ca Thng d bc hai, vic chng
minh cc tnh cht c th xem ti [1].
nh ngha 1. Cho a,m l hai s nguyn dng, CLN(a,m)=1. S a c gi l thng d bc
hai theo mun m nu phng trnh ng d x2 a ( mod m)c nghim. Ngc li ta ni a khng
l thng d bc hai theo mun m.
V d 1. S 2 l thng d bc hai theo mun 7 v 3 th khng l thng d bc hai theo mun
7 v 12 , 22 , 32 , 42 , 52 , 62 ng d ln lt vi 1, 4, 2, 2, 2, 4, 1 theo mun7.
Mnh 1. Gi s p l s nguyn t l, a l s nguyn khng chia ht cho p. Khi phng
trnh ng d x2 a (mod p),( 1.3) khng c nghim hoc c ng hai nghim khng ng d
theo mun p.
Chng minh. Gi s (1.3) c nghim x = x0 . Ta cn chng minh x = x0 l nghim khng
ng d vi x0 . Tht vy, gi s (1.3) c mt nghim khc l x = x1 , khi x20 x21 (mod p).
Tc l:
x20 x21 = (x0 x1 )(x0 + x1 ) 0(mod p).
Do , p|(x0 x1 ) hoc p|(x0 + x1 ). iu ny chng t x1 x0 , hoc x1 x0 .
nh ngha 2. Cho a l mt s nguyn v p l s nguyn t l. K hiu Legendre l s c xc
nh nh sau:

 
0 nOu p | a
a
(a, p) = 1 va lthng d bEc
hai theo m  un p
=
1 nOu

(a, p) = 1 va kh  ng lthng d bEc


hai theo m  un p
-1 nOu
Theo v d trn, ta k hiu
nh l 1. Cho a, p l nhng s nh trn ta c:
p1
 
a
i)
a 2 (mod p); (Tiu chun Euler)
p
p1
 
 
1
1
ii)
= 1,
= (1) 2 ;
p
p
    
a
b
ab
iii)
=
p
p
p
   
a
b
iv) Nu a b (mod p)th
=
.
p
p
114

nh l 2. (B Gauss). Cho p l s nguyn t l v (a, p)=1.


 Nu
 S l s cc thng d
p
1
p1
aln hn
th
= (1)S , ni khc hn
dng b nht ca cc s nguyn a, 2a, . . .
2
2
p
p1


2
P
2ka
S=
, vi [x] l phn nguyn ca x.
p
k=1
 
5
V d 2. Tnh
. Ta c cc thng d dng b nht theo mun 13 ca cc s: 1.5,
13
13
2.5, 3.5, 4.5, 5.5, 6.5 l cc s 5, 10, 2, 7, 12, 4 v trong cc s ny c 3 s ln hn
nn
2
 
5
= (1)3 = 1.
13
H qu 1. Cho p l s nguyn t l, ta c :
i) 2 l thng d bc hai theo mun p nu p 1 (mod 8).
ii) -2 l thng d bc hai theo mun p nu p 1 (mod 8), p 3 (mod 8).
iii) -3 l thng d bc hai theo mun p nu p 1 (mod 6).
iv) 3 l thng d bc hai theo mun p nu p 1 (mod 12).
v) 5 l thng d bc hai theo mun p nu p 1 (mod 10).
  
p
q
nh l 3. (Lut thun nghch bc hai). Nu p, q l hai s nguyn t l th c:
=
q
p
p1q1
2 .
(1) 2
 
3
V d 3. Tnh
.
41
  
 
3
41
3
1.20
p dng lut thun nghch bc hai ta c :
= (1)
= 1. Suy ra
=
41
3
41
   
41
2
=
= 1.
3
3
nh l 4. Cho x, y l hai s nguyn t cng nhau v a, b, c l cc s nguyn ty . Nu p l
s nguyn t l c ca ax2 + bxy + cy 2 nhng khng l c ca abc th D = b2 4ac l thng
d bc hai theo mun p. c bit nu p| (x2 Dy 2 )v (x,y)=1 th D l thng d bc hai theo
mun p.
Chng minh. t N = ax2 + bxy + cy 2 , ta c 4aN = (2ax + by)2 Dy 2 . Do :
(2ax + by)2 Dy 2 (mod p), (1.11a)
Ngoi ra, y khng chia ht cho p (nu khng th (2ax+by)v x chia ht cho p)nn tn ti y1
sao cho yy1 = 1 (mod p). Khi nhn hai v ca phng trnh (1.11a) vi y12 ta c:
(2axy1 + byy1 )2 D(yy1 )2 D (mod p).
Vy D l thng d bc hai theo m un p.

Thng d bc hai vi mun khng nguyn t

Phn ln cc phng trnh nghim nguyn bc cao ta thng chuyn v phng trnh ng d
bc cao vi mun khng nguyn t. Do , vic m rng cc tnh cht ca thng d bc hai vi
mun khng nguyn t c vai tr rt quan trng. Sau y l cc tnh cht ca n.
115

nh ngha 3. Cho a l s nguyn, b l s nguyn t l, gi s b c phn tch ra cc tha s


nguyn t dng b = p1 1 .p2 2 ....pr r .K hiu Jacobi c nh ngha :
 a   a 1  a 2  a r
=
.
...
b
p1
p2
pr
nxK hiu Jacobi l s m rng ca k hiu Legendre, khi p l s nguyn t th k hiu Jacobi
trng vi k hiu Legendre. Tuy nhin khi p l hp s th k hiu Jacobi cha cho ta bit phng
trnh ng d x2 a (mod p) n c nghim hay khng. Mc d vy n c nhiu tnh cht tng
t k hiu Legendre .
nh l 5. Cho a l s nguyn, b l s nguyn dng vi b c phn tch thnh cc tha s
nguyn t b = p1 1 .p2 2 ....pr r .. Khi a, l thng d bc hai theo mun b nu ch nu a l thng
d theo m un pi i , vi mi i=1,2,..,r.
Chng minh. Nu a l thng d bc hai theo mun b th d dng suy ra a l thng d bc
hai theo mun pi i vi mi i = 1, 2, .., r.
Ngc li, gi s a l thng d bc hai theo mun pi i , tc l vi mi s pi i lun tn ti xi l
nghim ca phng trnh x2 a (mod pi i ).
Khi theo nh l thng d Trung hoa th tn ti s x sao cho x xi (mod pi i )vi mi
i = 1, 2, .., r. Do ,
x2 x2i (mod pi i ) vi mi i.
Suy ra x2 a (mod b).
nh l 6. Gi s n l s nguyn dng
l , a v b l hai s nguyn t cng nhau vi n. Khi :
a  b 
=
;
i) Nu a b (mod n)th
n
n
 a   b   ab 
=
;
ii)
n
n
n
n1
 
1
iii)
= (1) 2 ;
n
n2 1
 
2
iv)
= (1) 8 ;
n
Chng minh. D thy, tnh cht (i), (ii) suy ra trc tip t nh ngha 2.1 v nh l 1.5. Ta
cn chng minh cho tnh cht (iii) v (iv).
V pi 1 chn nn
(1 + (pi 1))i 1 + i (pi 1) (mod 4) v
(1 + i (pi 1)) (1 + j (pj 1)) 1 + i (pi 1) + j (pj 1) (mod 4)
Do ta c:
p1 1 .p2 2 ...pr r 1 + 1 (p1 1) + 2 (p2 1) + ... + r (pr 1) (mod 4)
n1
1 (p1 1) 2 (p2 1)
r (pr 1)
Hay

+
+ ... +
(mod 2)
2
2
2
2
T suy ra:

p1 1 1
p2 1 2
p r 1 r
   1  2  r
1
1
1
1
=
.
...
= (1) 2 (1) 2 ...(1) 2
n
p1
p2
pr
p2 1
pr 1
n1
p1 1
+2
+...r
2
2
2 = (1) 2 ;
= (1)
1

116

n1

1
= (1) 2 .
Vy
n
Bng cch chng minh tng t, ta cng c c :
n2 1
1 (p1 1) 2 (p2 1)
r (pr 1)

+
+ ... +
(mod 8)
8
8
8
8
n2 1
 
2
= (1) 8 ..
Hay
n


nh l 7. (Lut thun nghch bc hai i vi k hiu Jacobi) . Nu m, n, l cc s nguyn dng


n1m1
m  n 
2
l, nguyn t cng nhau th
= (1) 2
n
m
Chng minh. Gi s m, n phn tch thnh cc tha s nguyn t dng: m = p1 1 .p22 ....pr r .,
n = q11 .q22 ....qss .
Khi ta c:
pi 1 q j 1
r P
s
P
p i 1 qj 1
j
i
m  n  Q
r Q
s
j
i
i=1 j=1
2
2
2
2 = (1)
=
=(1)
n
m
i=1 j=1
Ngoi ra, theo nh l 2.3 ta c :
r
s
P
P
n1
n1
pi 1
qj 1

(mod 2) ,
i

(mod 2) nn
j
2
2
2
2
i=1
j=1
m1n1
m  n 
2 .
= (1) 2
n
m

H qu 2. Cho a, b l nhng s nguyn v c, d l nhng s l ta c:
a a
.
c
d

a + bc
c


=

a  a 
;
=
c
cd

Mt s tng lin quan n k hiu Legendre

nh l 8. Nu a, b l cc s nguyn ty , v p l s nguyn t tha (a, p)=1th




p1
P ax + b
= 0.
p
x=0
Chng minh. V (a,p)=1 nn cc s ax+b, x=0,1,2,..,p-1 lp thnh h thng d y theo
p1
p1
mun p v c
l thng d bc hai ,
khng thng d bc hai v mt s chia ht cho
2
2
p. Do theo k hiu Legendre ta c:


p1
P ax + b
p1
p1
=
.1+
.(1) + 0 = 0.
p
2
2
x=0
0

nh l 9. Cho f(x) l a thc bc k vi h s nguyn. Gi f (x)p = a0 + a1 x + ... + akp0 xkp vi


p1
p0 =
ta c:
2
 


p1

P f (x)
k
0
ap1 + a2(p1) + ... + ak0 (p1) (mod p), k =
.
p
2
x=0
Chng minh. t Sn =

p1
P

xn vi n>0, Khi ta c:

x=0

117

Nu n chia ht cho p-1 th Sn =

p1
P

xn 1(mod p).

x=0

Nu n khng chia ht cho (p-1) th Sn

p1
P

xr , vi r n(mod p 1), 0 < r < p 1 . D

x=0

chng minh c Sn 0(mod p). T suy ra;


!
 p1

0
kp0
p1

P f (x)p
P
P f (x)

=
ai Si ap1 + a2(p1) + ... + ak0 (p1) (mod p).
p
p
x=0
x=0
i=0
nh
a, b, c l cc
 l 10. Cho 
 snguyn ty v p l s nguyn t tha (a, p)=1. Khi

 tng:
p1
P ax2 + bx + c
a
a
. Bng
nu b2 4ac khng chia ht cho p v bng (p 1)
nu .
p
p
p
x=0
!
 p1
  p1  2
P (2ax + b)2 D
4a P ax + bx + c
=
vi D = b2 4ac, t
Chng minh. Ta c
p x=0
p
p
x=0
cc s ax+b, x=0,1,2,..,p-1 lp thnh h thng d y theo mun p ta c:
  p1  2
 p1  2

P x D
4a P ax + bx + c
=
= S,hay
p x=0
p
p
x=0

   p1  2

 
p1
P ax2 + bx + c
a P x D
a
=
=S
.
p
p x=0
p
p
x=0
Theo nh l 3.2, ta c S = (ap1 ) 1(mod p).
Ngoi ra, ta c |S| pnn S {1, p 1}.


p1
P x2 D
Gi s S = p 1 th c duy nht mt s hng ca tng
bng 0 v tt c cc s
p
x=0


p1
P x2 D
u bng 1. Tc l tn ti duy nht x0 {0, 1, 2, ..., p 1}
hng cn li ca tng
p
x=0
sao cho p | (x20 D) , v ta cn chng mn x0 = 0.

Tht vy, nu x0 6= 0 ta c p | (p x0 )2 D
suy ra px0 {0,1,2...,p-1}dn n c hai phn t thuc tp {0, 1,
2, ...,p1} p | (x20 D)(mu
p1
P x2
thun). Do x0 = 0 hay p |D. Ngc li, gi s p |D ta c S =
=p-1.T suy ra:
p
x=0


 
p1
P ax2 + bx + c
a
= (p 1)
, v vi (D, p)=1 th
Nu p |D th
p
p
x=0


 
p1
P ax2 + bx + c
a
=
.
p
p
x=0

Bi tp p dng

Bi tp 1. Chng minh phng trnh y 2 = 41x + 3, (4.1) khng c nghim nguyn (x,y).
Li gii. Chng minh phng trnh (4.1) khng c nghim tc l ta phi
minh phng

 chng
3
trnh ng d y 2 3 (mod 41)khng c nghim, hay l ta phi chng t
= 1. Suy ra t
41
V d 1.10.
Bi tp 2. Chng minh rng phng trnh y 2 = x3 5khng c nghim nguyn (x, y).
Li gii. Gi s phng trnh (4.1) c nghim nguyn (x, y), khi x phi l s l . Tht vy,
nu x chn th ta c y 2 = 3(mod 8) (v l), suy ra x l s l .
118

Hn na nu x = 3(mod 4)th y 2 = 2(mod 4)(v l). Do x phi c dng x = 4k + 1, khi


ta c:y 2 + 4 = 4k(16k 2 + 2k + 3)suy ra 4 l thng
(16k 2 + 2k + 3).Tuy
 d
 bc hai theo mun

1
4
=
nhin theo k hiu Jacobi ta c:
2
2
16k + 12k + 3
16k + 12k + 3


1
2
Hn na, ta c 16k + 12k + 3 3(mod 4)nn theo nh l 2.3 suy ra:
=
16k 2 + 12k + 3
(1)1 = 1.
(mu thun vi 4 l thng d bc hai theo mun (16k 2 + 2k + 3).
T suy ra phng trnh cho khng c nghim nguyn.
Bi tp 3. (M rng ca bi 4.2). Chng minh phng trnh y 2 = x3 + k(4.3) khng c nghim
nguyn (x, y) nu k c dng: k = (4n 1)3 4m2 , trong m, n l nhng s nguyn v s nguyn
t p = 4f + 3 khng l c ca m.
Li gii. Gi s phng trnh (4.3) c nghim nguyn (x, y) , khi thay k = (4n 1)3 4m2 vo
phng trnh (4.3) ta c: y 2 x3 1 (mod 4), lp lun theo bi (4.2) ta c x phi c dng
x = 4k + 1. Tip tc t a = 4n 1, ta c (4.3) tng ng: y 2 + 4m2 = x3 + a3 y 2 + 4m2 =
(x + a)(x2 ax + a2 ).
Suy ra: y 2 + 4m2 = 1 (mod 4), ( doa 1 (mod 4). Do , lun tn tai s nguyn t p
3 (mod 4) l c ca y 2 + 4m2 , hay y 2 + 4m2 = 0 (mod p).
  

1
4m2
=
= 1. ( dop = 4f + 3).
Mt khc ta c theo k hiu Jacobi ta c:
p
p
Bi tp 4. Tm tt c cc cp s nguyn dng (x, n) l nghim ca phng trnh x3 + 2x + 1 =
2n (4.4)
Li gii. Ta c phng trnh (4.4) tng ng :x(x2 + 2) = 2n 1. D chng minh c
x(x2 +2)lun chia ht cho 3 vi mi s nguyn dng x nn 2n 1 cng chia ht cho 3. Suy ra n chn.
Ngoi ra, t phng trnh (4.4) ta thy rng, nu x chn th v tri l s l. Do , x phi l s l.
T suy ra: Nu n = 1 th phng trnh tr thnh : x3 +2x+1 = 2 x = 0 ( Khng l nghim).
Nu n = 2 th phng trnh tr thnh: x3 + 2x + 1 = 4 x = 1, nghim ca phng trnh l cp
(1, 2) . Nu n > 2 th phng trnh tng ng : x3 + 2x + 1 0 (mod 8 ) x 5 (mod 8).
Mt khc ta c, x3 + 2x + 1 = 2n (x + 1)(x2 x + 3) = 2n + 2 . Gi p l c nguyn t ca
(x2 x + 3) khi p l v -2 l thng d bc hai theo mun p . Do , theo H qu 1.8 p phi
c dng 8k + 1, 8k + 3, do (x2 x + 3) 3n (mod 8) hoc (x2 x + 3) 1 (mod8) .Mt khc
ta c x 5 (mod 8) nn x2 x + 3 7 (mod 8) .T suy ra 7 3n (mod 8) . (iu ny khng
ng vi n=3). Vy phng trnh (4.4) C duy nht cp nghim (1,2).
Bi tp 5. Cho n l s nguyn dng, phng trnh x2 + 3y 2 = n c nghim nu ch nu tt c
cc nhn t nguyn t ca n c dng 3k 1v c s m chn.
Li gii. gii bi ton, ta chng minh b sau:
B 1. Mt s nguyn t p c th vit di dng p = x2 + 3y 2 nu v ch nu p = 3 hoc
p = 3k + 1, k Z+ .
Vi p = 3 th hin nhin l b ng.
Vi p > 3 gi s p = x2 + 3y 2 ta s chng minh p c dng p = 3k + 1, k Z+ .
V p = x2 + 3y 2 nn (p, x)=1 v (p, y)=1. Do tn ti s y 0 sao cho yy 0 1(mod p). T
phng trnh p = x2 + 3y 2 ta suy ra, (xy 0 )2 3 (mod p) suy ra -3 l thng d bc hai theo
p1  
p1
 
 
3
3
3
= (1) 2
nn
= (1) 2 . Ngoi ra theo lut
mun p. Hn na ta c
p
p
p
119

p131
  
p
p
3
2 nn
= (1) 2
= 1 hay p 1( mod 3).( C th
thun nghch bc hai ta c
p
3
3
suy ra trc tip t H qu 1.8).
Ngc li, gi s p = 3k + 1 , khi tn ti s nguyn a sao cho a2 3 ( mod p).Ta s chng
 

minh tn ti cc s nguyn x, y sao cho 0 < x, y < p v p |a2 x2 y 2 . Tht vy t b =


p ,
2
2
xt (b + 1) cc s au + v, vi (u, v) {0, 1, 2, ..., b} {0, 1, 2, ..., b} , V (b + 1) > pnn tn ti
cp (u1 , v1 ) 6= (u2 , v2 )sao cho au1 + v1 au2 + v2 (mod p), t x = |u1 u2 | , y = |v1 v2 |ta c

0 < x, y < p v ax + y 0(modp). Suy ra a2 x2 y 2 0(modp). Vy ta c p |a2 x2 y 2 .


T suy ra, p |(a2 + 3)x2 (3x2 + y 2 )hay (3x2 + y 2 ) = lp,vi l Z+ . Hn na, ta c
0 < x2 < p, 0 < y 2 < pnn suy ra l = {1, 2, 3} .
Vi l = 1 ta c p = 3x2 + y 2 .
Vi l = 2 ta c 2p = 3x2 + y 2 , khi tn ti cp s (x, y) sao cho ,2p 0( mod 4)(mu thun).
Vi l = 3 ta c 3p = 3x2 + y 2 , khi y chia ht cho 3, t y = 3y1 ta c p = x2 + 3y1 2 . iu
ny suy ra nu p 3v p l s nguyn t dng 3k 1v p |x2 + 3y 2 th p|x, p|y. Tht vy, nu
0
0
2
2
0 2
(x,p)=1
 tn ti y sao cho xy 1(modp), v x 3y (modp)ta suy ra(xy ) 3(modp),
 th
3
= 1(iu ny mu thun). Vy l=3 khng tha.
hay
p
Vy B c chng minh.
m
Q
gii bi ton Bi tp 4.5 , ta xt n = a2 .b vi b khng chnh phng, suy ra b =
pi ,
i=1

trong pi = 3hoc pi 1(mod3).Khi theo B 1 ta c pi = xi 2 + 3yi 2 nn suy ra . Do


n = a2 b = (ax)2 + 3(ay)2 .
Bi tp 6. Chng minh rng khng tn ti cc s nguyn dng a, b, c sao cho

a2 + b 2 + c 2
3(ab + bc + ca)

l mt s nguyn.
Li gii. Gi s tn ti cc s nguyn dng a, b,c n sao cho
a2 + b2 + c2 = 3n(ab + bc + ca)
a2 + b2 + c2 = (3n + 2)(ab + bc + ca)

iu ny suy ra 3n + 2c mt c nguyn t dng p 2( mod 3). Ta chn mt c l ca 3n + 2sao


cho p2i1 |(3n + 2) v p2i khng l c ca 3n + 2.Khi ta c: pi |(a + b + c) p|(ab + bc + ca).
2
Th c a b(modp)ta 
c:p|4(a
+ ab + b2 ). Ngoi ra, ta c: 4(a2 + ab + b2 ) = (2a + b)2 +

3
3b2 nn theo B 1 ta c
= 1, hay p 1(mod3)(iu nu mu thun).
p
Bi ton c gii quyt.
Bi tp 7. Chng minh rng s xm + y n khng chia ht cho 4kxy 1 vi mi s nguyn dng
x, y, k, m, n.
m
n ..
Li gii.
h m i Gi sh ntn
i ti cc s nguyn dng x, y, k, m, n sao cho x + y .4kxy 1. Ta t
m1 =
, n1 =
ta c:
2
2


1
m1 2
n1 2 ..
(i) Nu m = 2m1 , n = 2n1 th (x ) + (y ) .4kxy 1, do ,
= 1(iu ny mu
4kxy 1
thun v 4kxy 1 3(mod4).


y
m1 2
n1 2 ..
(ii) Nu m = 2m1 , n = 2n1 +1th (x ) +y(y ) .4kxy1 suy ra
= 1. Ta s chng
4kxy 1
120


minh iu ny mu thun. Tht vy, nu y l s l, ta c:

y
4kxy 1


=

1
4kxy 1



y
4kxy 1

y1  
1
= 1.
(1)(1) 2
y



2
Cn trng hp y chn ta t y = 2 y1 , t 1, y1 N. Khi , ta c:
= 1 v
4kxy 1







t 

y1
y1
y
2
y1
=
= 1 Suy ra,
=
=
4kxy 1
42t kxy1 1
4kxy 1
4kxy 1
4kxy 1
1.


xy
n1 2 ..
m1 2
= 1.
(iii) Nu m = 2m1 + 1, n = 2n1 + 1thx(x ) + y(y ) .4kxy 1 suy ra
4kxy 1

 
 

xy
4xy
1
Mt khc ta c:
=
=
= 1.
4kxy 1
4kxy 1
4kxy 1
C ba trng hp u mu thun vi gi thit ban u. Vy bi ton c gii quyt.
t

Bi tp 8. (POL4, IMO Shortlisted problems). Cho a, b, c l cc s nguyn v p l s nguyn


t l. Chng minh rng nu s f (x) = ax2 + bx + c chnh phng ti 2p-1 gi tr nguyn lin tip
ca x th p|(b2 4ac).
Li gii. Bi ny nu khng s dng thng d bc hai s tht kh, c th xem li gii ti ti
liu [3].Sau y l cch gii ng dng thng d bc hai.
Gi s tn ti s nguyn x0 sao cho f (x0 ), f (x0 + 1), ..., f (x0 + 2p 2) l chnh phng. Khi
:
i) Nu p|av (p, b) = 1th f (x) bx + c(modp) nn tp {f (x) : x = x0 , ..., x0 + p 1} c p-1
phn t lp thnh h thng d y theo mun p. Ngoi ra, {x0 , ..., x0 + p 1} l h thng d
p1
l chnh phng theo mun p. (iu ny mu thun
y theo mun p, nn c tt c
2
vi c p-1 chnh phng). Do p|bhay p|(b2 4ac).
ii) Nu (p, a) = 1, th tas chng minh p|(b2
4ac).
Tht vy, gis (p, (b2 4ac))


 = 1,khi
2
2
p1
x
+p1
0
P ax + bx + c
P
a
a
ax + bx + c
theo nh l 3.3 ta c:
=
. Suy ra
=

p
p
p
p
x=0
x=x0
1.

 2
x0 +p1
P
ax + bx + c
3 .Trc ht ta s chng minh khng
Mt khc ta s chng minh
p
x=x0
tn ti 3 s u, v, w {x0 , ..., x0 + p 1}sao chof (u) f (v) f (w)(modp). Tht vy, gi s
f (u) f (v) 0(modp)suy ra p| (a(u + v) + b) (u v) do , a(u + v) b(modp), tng t, ta
cng c ca(u + w) b(modp). Suy ra a(u w) 0(modp)hay p|a(u w)(mu thun).
Vy khng tn ti 3 s u, v, w {x0 , ..., x0 + p 1}sao chof (u) f (v) f (w)(modp). T
suy ra
 2
 
 

x0 +p1
P
ax + bx + c
p+5
p+5

2 p
= 3.( iu ny mu thun).
p
2
2
x=x0
Vy p|(b2 4ac).( Bi ton c gii quyt ).

Bi tp t luyn

Bi tp 9. Tm tt c cc nghim nguyn khng m ca phng trnh 12x + y 4 = 2008z .


HD: B (0;0;0) l nghim ca phng trnh. Ngoi ra, ta c v tri c dng a2 + b2 hoc a2 + 3b2
nn mi c nguyn t ca 2008z u c dng 8k+1 hoc 8k+3. (Chng minh v l.)
121

Bi tp 10. Cho tp A = {1, 2, 3..., 2005}. Chng minh rng c th tm trong A cc phn t
(x,y) sao cho x2 + 3y 2 3 l bi ca 2011. HD. Xt tp M = {x2 3, x A} , N = {x2 3, x A}.
Ch ra trong 2 tp c hai phn t phn bit ng d theo m un2011.
Bi tp 11. Cho a, b l hai s nguyn dng tha a.b khng chnh phng. Chng minh tn ti
v hn s nguyn dng n sao cho (an 1) (bn 1)khng chnh phng.

Ti liu tham kho


[1] Nguyn Vn Mu, (2011), Cc chuyn ton hc bi dng hc sinh gii , S gio dc
v o to Ph Yn.
[2] Dusan DjuKie, (2007), Quadratic Congruences, Springer.
[3] Dusan DjuKie, Ivan Matic, A Collection of Problems Sugested for the Internatinal Mathematical Olympiads:1959-2009, Springer.
[4] Mt s ti liu trn mng Internet.

122

Mt s ng dng ca bt bin
trong dy s v s hc
H Thng
Trng THPT Chuyn Nguyn Du, k Lk
Bt bin l mt i lng, tnh cht khng thay i trong qu trnh chng ta thc hin cc php
bin i. Khi nim c nghin cu v ng dng trong nhiu lnh vc khc nhau ca ton hc
nh trong php bin hnh ca b mn hnh hc vi c bt bin v khong cch gia hai im trong
php di hnh, t s khong cch trong php ng dng; trong b mn i s T hp, . . . Trong
ni dung bi vit ny ti ch xin cp n mt s ng dng ca bt bin trong dy s v s hc.

Cp s cng

Bi ton 1. Cho dy s (un ) , n N , bit vi mi s nguyn dng n c un = vn+1 vn (bt


bin l mi s hng ca dy s l hiu ca hai s hng lin tip ca mt dy s khc) th ta c
u1 + u2 + + un = vn+1 v1 .
Chng minh. Ta c u1 + u2 + + un = (v2 v1 ) + (v3 v2 ) + + (vn+1 vn ) = vn+1 v1 .
V d 1. Tnh tng Sn = u1 + u2 + + un , n N , bit s hng tng qut un trong cc trng
hp
a) un = n.
(2n + 1)2 1
n (n + 1)
(2n + 1)2 (2n 1)2
Sn = 1 + 2 + + n =
=
.
Gii. un = n =
8
8
2
b) un = n2 .
(2n + 1)3 (2n + 1) (2n 1)3 + (2n 1)
Sn = 12 + 22 + + n2 =
24
(2n + 1)3 (2n + 1)
n (n + 1) (2n + 1)
=
.
24
6
c) un = q n (q 6= 1) .

Gii. un = n2 =

q n+1 q n
q1
q n+1 q
q (1 q n )
Sn = q 1 + q 2 + + q n =
=
.
q1
1q

Gii. Ta c un = q n =

2
un1
V d 2 (VMO nm 2001). Cho dy s thc (un ) xc nh bi u1 = , un =
3
2 (2n 1) un1 + 1
vi mi n 2. Tnh tng ca 2001 s hng u tin ca dy s.
1
1
Gii. T gi thit c dy s cho c cc s hng lun lun dng v

= 4n 2
un un1


n
n
P
P
1
1
1
1
3
4n2 1
+
(4i 2) + 2 =
=

=
.
un i=2 ui ui1
u1 i=1
2
2
2
1
1
Suy ra un = 2
=

.
4n
1
2n 1 2n+ 1

2001
2001
P
P
1
1
1
4002
Vy
un =

=1
=
.
2n 1 2n + 1
4003
4003
n=1
n=1
123

p 2
xn1 + 4xn1 + xn1
1
V d 3 (VMO nm 2009). Cho dy s thc (xn ) xc nh bi x1 = , xn =
2
2
vi mi n 2.
n 1
P
Vi mi s n nguyn dng, tyn =
. Chng minh rng dy s (yn ) c gii hn hu hn.
2
i=1 xi
Hy tm gii hn trong trng hp .
Gii. Ta thy dy s (xn ) cho l dy s thc dng, n iu tng. Nu dy s b chn trn
th n c gii hn hu hn gi s bng a

1
Ta c a v 2a = a2 + 4a + a a = 0 (v l)
2
Vy dy s (xn ) khng b chn trn nn lim xn = +
Vi n 2, ta c
p
1
1
1
2xn xn1 = x2n1 + 4xn1 x2n xn xn1 = xn1

= 2
xn1 xn
xn
Nn


n
n 1
P
P
1
1
1
1
= 2+

=6
lim yn = 6
yn =
2
x1 i=2 xi1 xi
xn
i=1 xi
Bi ton 2 (Cp s cng). Cho dy s (un ) , n N bit u1 v un+1 = un + a, n 1 (vi a l
cc hng s cho trc).
a) Chng minh rng vi mi s n nguyn dng c un = u1 + (n 1) a
n (2u1 + (n 1) a)
b) Sn = u1 + u2 + + un =
, n N .
2
Chng minh.
a) Ta c un+1 un = a, n 1 (bt bin)
Nn c un = (un un1 ) + (un1 un2 ) + + (u2 u1 ) + u1
Do un = u1 + (n 1) a
b) T cu a v v d 1 c
1 + 2 + + (n 1)
an (n 1)
n (2u1 + (n 1) a)
Sn = nu1 +
a = nu1 +
=
, n N
2
2
2
vn+1
vn
(bt bin l mi s hng ca dy s u l t ca hai s hng lin tip ca mt dy s khc m
vn+1
cc s hng u khc 0) th ta c u1 u2 . . . un =
.
v1
v2 v3
vn+1
vn+1
Chng minh. Ta c u1 u2 . . . un = . . . .
=
v1 v2
vn
v1
Bi ton 3 (Cp s nhn). Cho dy s (un ) , n N , bit vi mi s nguyn dng c un =

Bi ton 4 (Cp s nhn). Cho dy s (un ) , n N bit u1 v un+1 = un a, n 1 (vi a l mt


hng s cho trc).
a) Chng minh rng vi mi s n nguyn ln hn 1 c un = u1 a(n1)
1 an
b) Vi a khc 1 th c Sn = u1 + u2 + + un = u1
, n N .
1a
Chng minh.
a) Khi au1 = 0 un = u1 a(n1) = 0, n > 1
un+1
un un1
u2
Khi au1 6= 0 un 6= 0, n > 1 v
= a(bt bin) un =
. . . u1 = u1 an1
un
un1 un2
u1
124

b) V a khc 1 v theo cu a v v d 1c) c Sn = u1 + u2 + + un = u1 (1 + a + + an1 ) =


1 qn
, n N .
u1
1q
V d 4 (VMO bng B nm 2004). Cho dy s thc (xn ) xc nh bi
x1 = 1, xn+1 =

(2 + cos 2) xn + cos2
(2 2 cos 2) xn + 2 cos 2

trong l mt tham s thc. Tm dy s (yn ) xc nh bi yn =

n
P

1
, n 1 c
i=1 2xi + 1

gii hn hu hn. Hy tm gii hn trong trng hp .


Gii. Vi mi s nguyn i 1, ta c
(4 + 2 cos 2) xi + 2cos2 + (2 2 cos 2) xi + 2 cos 2
2xi+1 + 1 =
(2 2 cos 2) xi + 2 cos 2
6xi + 3
>0
=
(2 2 cos 2) xi + 2 cos 2


1
1
14
1
2
2
2
Nn
=
+ 2 sin
sin + 1 + 2 sin 2xi + 1 =
2xi+1 + 1
3 xi
3 2xi + 1


1
1
1

sin2 =
sin2
2xi+1 + 1
3 2xi + 1
Theo tnh cht ca cp s 
nhn c:

1
1
1
2
2

sin = i1
sin
2xi + 1
3
2x1 + 1


1
1
1
2
= i1
sin + sin2

2xi + 1
3
3



3
1
1
2
yn =
1 n
sin + n sin2
2
3
3
Khi 6= k, k Z lim yn = +
1
Khi = k, k Z lim yn =
2

Mt s bt bin i vi dy s

Bi ton 5. Cho dy s thc (un ) , n N gm cc s hng lun khc 0, bit u0 , u1 v un+1 =


aun un1 + b, n 1 (vi a, b l cc hng s). Ta c mt s kt qu (bt bin) sau:
un+1 + un1 b
1)
= a, n 1.
(1)
un
2) un+1 un1 un 2 + bun = c (c = u2 u0 u1 2 + bu1 ) .
(2)
2
2
3) un + un1 b (un + un1 ) aun un1 + c = 0, n 1.
(3)
4) Phng trnh x2 (b + aun1 ) x + un1 2 bun1 + c = 0, n 1
(4)
c nghim x = un . c bit nu bit dy s cho l dy s nguyn v a, b l hai s nguyn cho
trc th ta lun c bit s l mt s chnh phng.
Chng minh.
1) Theo nh ngha dy s cho ta suy ra (1).
2) T (1), ta suy ra n 2 c
125

un + un2 b
un+1 + un1 b
=
= a un+1 un1 un 2 + bun = un un2 un1 2 + bun1 =
un1
un
= c
3) Thay gi thit dy s cho vo (2), n 1 ta c
(aun un1 + b) un1 un 2 + bun = c un 2 + un1 2 b (un + un1 ) aun un1 + c = 0
4) T (3) ta suy ra (4).
Nhn xt 1. Ta c th xt mt s bi ton tng t nh Bi ton 3.

V d

u0 = u1 = 1
. Chng minh rng
V d 5. Cho dy s (un ) , n N xc nh bi
u 2+2
un = n1
, n 2
un2
tt c s hng ca dy s cho u l s nguyn.
Nhn xt 2. Bi tp ny cho c dng (2), tnh c u2 = 3, b = 0, a = 4, c = 2.
Gii.
Phng php 1. Dng phng php quy np, chng minh un+1 = 4un un1 , n 1 V
u0 = u1 = 1, nn un Z, n N.(PCM)
Phng php 2. T gi tht c un > 0, n N n 2, ta c
un+1 + un1
un + un2
un un2 un1 2 = 2 un+1 un1 un 2 = un un2 un1 2
=
= =
un
un1
u2 + u0
=4
u1
Do un+1 = 4un un1 , n 1.
V u0 = u1 = 1, nn un Z, n N.(PCM)
(
u0 = 0, u1 = 1
V d 6. Cho dy s (un ) , n N xc nh bi
.
un+2 = 2014un+1 un 3, n 0
a) Tm tt c s t nhin n un l mt s nguyn t.
b) Chng minh rng c mt s nguyn M khng i, sao cho vi mi s nguyn dng n, s
M + un+1 un1 + un l mt s chnh phng.
Gii.
a) T gi tht c: u2 = 2014u1 u0 3 = 2011 l mt s nguyn t
Dy s (un ) c gi tr t nhin, v un+1 > un + 4 > 0, n 1 .
Gi s un+1 (n 1) l mt s nguyn t
Theo kt qu (2) c
un+1 un1 un 2 3un = u2 u0 u1 2 3u1 = 4 un+1 un1 = un 2 +3un 4 = (un 1) (un + 4) .
.
.
M un + 4..un+1 , nn un 1..un+1 un = 1 n = 1
Vy ch c s u2 = 2011 nguyn t.
b) Theo trn c un+1 un1 un 2 3un = 4 un+1 un1 + un + 8 = (un + 2)2
Vy M = 8 tha mn yu cu bi.
V d 7 (VMO bng A nm 1997). C bao nhiu hm s f : N N tha mn f (1) =
1, f (n) f (n + 2) = f 2 (n + 1) + 1997.

126

Gii. Gi s f : N N l mt hm s tha mn .
t un = f (n) u1 = 1, un un+2 un+1 2 = 1997
(*)
u
+
u
u
+
u
u
+
u
n1
n+1
1
3
n
n+2
=
= =
.
un un+2 un+1 2 = un1 un+1 un 2 hay
un+1
un
u2
u1 + u3
p
t
= (p, q N ; (p, q) = 1). Thay vo trn c
u2
q
.
q (un + un+2 ) = pun+1 un+1 ..q, n N .
(**)
.
T (*) v (**) c vi n > 1 c un un+2 un+1 2 = 1997..q 2 q = 1 (1997 l s nguyn t)
Vy u3 = pu2 1 = u2 2 + 1997 u2 (p u2 ) = 1998
Vy u2 l c nguyn dng ca 1998 = 2.33 .37, c 2.4.2 = 16 cch chn u2
1988
N
o li ng vi mi cch chn u2 trn, chn p = u2 +
u2
Th hm s f xc nh bi f (1) = 1, f (2) = u2 , f (n) = un = pun1 un2 , n 3 tha mn
cc yu cu.Do c 16 hm s tha mn.
V d 8 (VMO bng B nm 1997). Cho dy s nguyn (an ) , n N xc nh bi
(
a0 = 1, a1 = 45
an+2 = 45an+1 7an , n 0
a) Tm s c nguyn dng ca s an+1 2 an an+2 theo n.
b) Chng minh rng 1997an 2 + 7n+1 .4 l s chnh phng vi mi s n.
Gii.
a) T gi thit suy ra cc s hng dy s u nguyn dng v tng t bi ton 3 vi mi
s t nhin n c
an+2 + 7an
an+1 + 7an1
an+2 + 7an
= 45
=
an+1
an+1
an
2
2
an+1 an+2 an = 7 (an an+1 an1 ) = = 7n (a1 2 a2 a0 ) = 7n+1 v a1 2 a2 a0 = 7
Vy s c nguyn dng ca s an+1 2 an an+2 l n + 2.
b) Theo trn v gi thit c an+1 2 (45an+1 7an ) an = 7n+1
Nn x = an+1 l nghim nguyn dng ca phng trnh x2 45an x + 7an 2 7n+1 = 0
Nn = 1997an 2 + 4.7n+1 phi l mt s chnh phng (PCM)
V d 9. Cho dy s nguyn (an ) , n N xc nh bi
(
a0 = 1, a1 = 4
.
an+1 = 4an an1 , n 1
Chng minh rng vi mi s n nguyn dng ta c (x = an1 , y = an ) u l nghim nguyn
dng ca phng trnh x2 + y 2 = 4xy + 1.
Gii. . T gi thit cho c dy s nguyn (an ) , n N l dy tng, cc s hng u nguyn
dng . Ta c
an+1 + an1
an+1 + an1
an+2 + an
=4
=
an+1 = 4an an1
an
an
an+1
2
2
2
an+2 an+1 an+1 = an+1 an1 an = = a2 a0 a1 = 1
Nn (4an an1 ) an1 a2n = 1 a2n1 + a2n = 4an an1 + 1
Vy (x = an1 , y = an ) l nghim nguyn dng ca phng trnh x2 + y 2 = 4xy + 1.
V d 10. Tm tt c nghim nguyn dng ca phng trnh x2 + y 2 = 4xy + 1.

127

Gii. Vi x, y l hai s nguyn dng


- Nu x = y th khng tha mn phng trnh.
- Nhn thy nu (x, y) l nghim ca phng trnh th (y, x) cng l nghim ca phng trnh
.Ta xt x < y.
Nu x = 1 th y = 4 v (x = an1 , y = an ) v d 9 u l nghim ca phng trnh .
Nu 1 < x < y th ta xt dy s nguyn (un ) , n N xc nh bi u0 = y, u1 = x, un+1 =
4un un1 , n 1.
Tng t trn c (un1 , un ) l nghim nguyn ca phng trnh
Ta c x2 1 = y (y 4x) > 0 y < 4x u2 > 0
(y x) (y 3x) = x2 + y 2 4xy + 2x2 = 1 + 2x2 > 0 y > 3x 0 < u2 < u1 < u0
Nu u2 > 1 th ta tip tc qu trnh s c mt s nguyn dng k m uk = 1 uk1 = 4.
Do c s n nguyn dng m (x = an1 , y = an ).
Vy nghim nguyn dng ca phng trnh cho l (x = an1 , y = an ) v (x = an , y = an1 )
vi n l s nguyn dng ty .

Xc nh an : Xt phng trnh c trng x2 4x+1 = 0 c hai nghim x1 = 2+ 3, x2 = 2 3.


n+1
n+1 
1 
Nn an =
2+ 3
2 3
2 3
x2 + y 2
= k c nghim nguyn
V d 11. Tm tt c s k nguyn dng phng trnh
xy 1
dng. Khi hy tm tt c nghim nguyn dng ca phng trnh .
Gii. Xt x, y l hai s nguyn dng, t nht mt s ln hn 1. Theo bt ng thc AM GM
x2 + y 2
c k =
>2k3
xy 1
Phng trnh c vit li x2 kxy + y 2 + k = 0. Xt hm s f (x) = x2 kxy + y 2 + k
2
k
=1+
3 x = y = 1 (loi)
- Nu x = y th c x2 (2 k) + k = 0 x2 =
k2
k2
- Nu x, y phn bit, ta nhn thy (x, y) l nghim ca phng trnh th (y, x) cng l nghim
ca phng trnh, xt x > y. Gi (x0, y0) l nghim ca phng trnh m x0 nh nht.
t f (x) = x2 kxy0 + y0 2 + k.
Ta tm c s x1 sao cho x1 x0 > y0 v f (x0 ) = f (x1 ) = 0
k
f (y0 ) = (2 k) y0 2 + k > 0 y0 2 <
3 y0 = 1
k
2

x0 2 + 1
2
x0 = 2
k=
= x0 + 1 +
N
k=5
x0 = 3
x0 1
x0 1
Bng cch lm tng t v d 10, ta c nghim nguyn dng ca phng trnh cho khi
k =5 l (x = an1 , y = an ), (x = an , y = an1 ),(x = bn1 , y = bn ) v (x = bn1 , y = bn ) vi n l s
nguyn dng ty .
Trong a0 = 1, a1 = 2, an+1 = 5an an1 , n 1 v b0 = 1, b1 = 3, bn+1 = 5bn bn1 , n 1
V d 11 (VMO bng A 1999). Cho hai dy s (xn ) , (yn ) vi n N c xc nh nh sau
x0 = 1, x1 = 4, xn+2 = 3xn+1 xn , n 0 v y0 = 1, y1 = 2, yn+2 = 3yn+1 yn , n 0
1) Chng minh rng vi mi s t nhin n, ta c xn 2 5yn 2 + 4 = 0.
2) Gi s a, b l hai s nguyn dng m a2 5b2 + 4 = 0. Chng minh rng tn ti s t
nhin m xk = a, yk = b.
Gii.
1) Hin nhin x0 2 5y0 2 + 4 = x1 2 5y1 2 + 4 = 0 v c hai dy s c cc s hng u
128

nguyn dng. Ta gi s vi 1 k n c xk 2 5yk 2 + 4 = 0


T gi thit c
xn+2 + xn
xn+1 + xn1
=3=
xn+2 xn xn+1 2 = xn+1 xn1 xn 2 = ... = x2 x0 x1 2 = 5
xn+1
xn
Nn xn 2 (3xn xn1 ) xn1 = 5 v xn+1 2 (3xn+1 xn ) xn = 5
Suy ra xn+1 2 = 7xn 2 xn1 2 + 10
Tng t c yn+1 2 = 7yn 2 yn1 2 2
Nn xn+1 2 5yn+1 2 + 4 = 7 (xn 2 5yn 2 + 4) (xn1 2 5yn1 2 + 4) = 0
Theo phng php quy np ton hc c xn 2 5yn 2 + 4 = 0, n N
2) Bng phng php quy np chng minh c
3xn + 5yn
3yn + xn
3xn+1 5yn+1
3yn+1 xn+1
, yn+1 =
, n 0 hay xn =
, yn =
xn+1 =
2
2
2
2
Gi s a, b l hai s nguyn dng m a2 5b2 + 4 = 0, a, b cng tnh cht chn, l
- Nu b = 1 th a = 1 v vi k = 0 th xk = a, yk = b
- Nu b = 2 th a = 4 v vi k = 1 th xk = a, yk = b
- Nu b > 2 v a2 5b2 + 4 = 0 4b2 < a2 = 5b2 4 < 9b2 2b < a < 3b
Xt hai dy s (un ) , (vn ) vi n N c xc nh nh sau
3un 5vn
3vn un
u0 = a, v0 = b, un+1 =
, vn+1 =
, n 0
2
2
3b a
3a 5b
< a, v1 =
< b u nguyn dng cng tnh cht chn, l v tha
Ta c u1 =
2
2
phng trnh
u1 2 5v1 2 + 4 = 0
Nu v1 2 th ta tip tc qu trnh phi c s nguyn dng k m
vk = 1, uk 2 5vk 2 + 4 = 0 uk = 1 (uk , vk ) = (x0 , y0 ) (a, b) = (xk , yk )
V d 12 (VMO 2012). Xt cc s t nhin l a, b m a l c ca b2 + 2 v b l c ca a2 + 2.
Chng minh rng a, b l cc s hng t nhin ca dy s t nhin
(vn ) xc nh bi v0 = v1 = 1 v vn = 4vn1 vn2 vi mi n 2.
Gii. Gi d = (a, b) d/a, d/b m a/a2 v theo gi thit b/a2 + 1 d/2
Li c a, b l s t nhin l nn d = 1. Vy c (a, b) = 1
.
.
.
Xt s N = a2 + b2 + 2 N ..a, N ..b m (a, b) = 1 N ..ab N = kab
Vy (a, b) l nghim t nhin l ca phng trnh x2 + y 2 + 2 = kxy, vi k l s nguyn dng
v x, y nguyn dng l.
Theo bt ng thc AM GM ta c k 3. Hn na x, y l hai s nguyn dng l nn k l
s nguyn dng chn.
Nu (x, y) l nghim ca phng trnh th (y, x) cng vy. Nn ta ch cn xt y x.
Gi (x0, y0) l mt nghim nguyn dng l ca phng trnh x2 + y 2 + 2 = kxy m y0 nh
nht. t f (x) = x2 ky0 x + y0 2 + 2.
x2 + 2
Theo nh l Vi et: y1 = 0
y0 x0 sao cho
kx0

2
k=3
2
2
f (y0 ) = f (y1 ) = 0 f (x0 ) = (2 k) x0 + 2 0 x0

k=4
k2
2
2
M k nguyn dng chn nn k = 4 c phng trnh x + y + 2 = 4xy (1).
Theo trn x, y cng tnh cht chn, l. Nu x, y cng chn th
x2 + y 2 + 2 2 mod (4) , 4xy 0 mod (4) (v l)
129

Vy x, y cng l .
Ta chng minh cp(x = vn1 , y = vn ) vi mi s n nguyn dng u l nghim nguyn dng
l ca (1). T gi thit c dy s cho l dy s tng vi n 1, cc s hng u nguyn dng
l, ng thi c
vn + vn2
vn+1 + vn1
=
= 4 vn+1 vn1 vn 2 = vn vn2 vn1 2 = ... = v2 v0 v1 2 = 2 Theo
vn1
vn
gi thit v chng minh trn c (4vn vn1 ) vn1 vn 2 = 2 vn 2 + vn1 2 + 2 = 4vn vn1 (n 1)
Nn (x = vn1 , y = vn ) v (x = vn , y = vn1 ) l nghim nguyn dng l ca (1) o li Nu (a,
b) l mt nghim nguyn dng l ca (1) vi 1 a b. Ta chng minh c s n nguyn dng
sao cho (a = vn1 , b = vn )
Nu a = 1 th b =1hoc b = 3 nn n = 1 hoc n = 2
Nu a > 1 th a < b. Xt dy s (un ) xc nh bi u0 = b, u1 = a v un = 4un1 un2 vi
mi n 2ta c a2 + b2 + 2 = 4ab a2 + 2 = b (4a b) > 0 b < 4a
C (b a) (b 3a) = a2 + b2 4ab + 2a2 = 2a2 2 > 0. Nn 3a < b < 4a
Suy ra u0 = b > u1 = a > u2 = 4a b > 0. Nu u2 = 1 th u3 = 1 suy ra (u0 = b, u1 = a) =
(v3 , v2 )
Nu u2 > 1 th tip tc qu trnh phi c s m nguyn dng sao cho um = um+1 = 1 v
ta c (u0 = b, u1 = a) = (vm+1 , vm ) (PCM)
Bi ton 6. Cho s m nguyn dng cho trc v dy s nguyn (un ) , n N bit u0 , u1 v
un+1 = aun + bun1 , n 1 (vi u0 , u1 , a,b l cc hng s nguyn cho trc). Ta gi dy s nguyn
(rn ) , n N vi rn un mod (m) , 0 rn < m vi mi s n N th c s nguyn dng T >1 v
s n0 N cho trc sao cho rT k+n = rn , n n0 , k N.
Chng minh. Ta c (rn1 , rn ) c ti a m2 gi tr khc nhau
V un+1 = aun + bun1 rn+1 (arn + brn1 ) mod (m) , n 1 .
Nn trong dy gm m2 + 1 cp (r0 , r1 ) , (r1 , r2 ) , ..., (rm2 , rm2 +1 ) c t nht hai cp trng nhau.
Gi n0 l s t nhin nh nht sao cho c s nguyn dng T m rn0 = rn0 +T , rn0 +1 = rn0 +1+T
rn0 +2 = rn0 +2+T . . .
T suy ra rT k+n = rn , n n0 , k N
Ch 1. Trong bi ton 4 trn nuun+1 = aun + bun1 , n 1, b 1 mod (m) th s n0 b
nht bng 0, v nu n0 1 brn0 1 rn0 1 rn0 +1 + arn0 mod (m)
M brn0 +T 1 rn0 +T 1 rn0 +T +1 + arn0 +T mod (m) rn0 +1 + arn0 mod (m) rn0 1
rn0 +T 1 = rn0 1
Nn rn0 1 = rn0 +T 1 , rn0 = rn0 +T (V l vi s n0 b nht)
un1 2 + 3
, n 2. Chng
V d 13. Cho dy s (un ) , n N xc nh bi u0 = u1 = 1 v un =
un2
minh rng u2013 l s nguyn chia ht cho 19.
Gii . Tng t v d 5 ta c un+1 = 5un un1 , n 1 m u0 = u1 = 1, nn dy s cho
l dy s nguyn, xt dy s (rn ) , n N vi rn un mod (19) , 0 rn < 19
Ta c bng sau
n
rn

0
1

1
1

2
4

3
0

4
15

5
18

6
18

7
15

8
0

9
4

10
1

11
1

12
4

13
0

14
15

15
18

16
18

17
15

18
0

Suy ra r10k+n = rn , n 0, k N r2013 = r10.210+3 = r3 mod (19) = 0 mod (19).


.
Nn u2013 ..19
130

V d 14. Cho dy s nguyn (un ) , n N xc nh bi u0 = 19, u1 = 98 v un+2 = un+1 +un , n


0. Tm s d ca php chia u20 + u21 + u22 + ... + u22013 cho 8.
Gii. Xt dy s (rn ) , n N vi rn un mod (4) , 0 rn < 4 un = rn + 4t u2n =
(rn + 4t)2 rn2 mod (8) an mod (8) (0 an < 8)
Ta c bng sau
n
rn
an

0
3
1

1
2
4

2
1
1

3
3
1

4
0
0

5
3
1

6
3
1

7
2
4

8
1
1

9
3
1

10
0
0

Li c 335.6 = 2010 u20 +u21 +u22 +...+u22013

11 12 13 14 15 16 17 18
3
3
2
1
3
0
3
3
1
1
4
1
1
0
1
1
334

P 2
u6k + u26k+2 + ... + u26k+5 +u26.335 +u26.335+1 +

k=0

u26.335+2 + u26.335+3
Suy ra u20 + u21 + u22 + ... + u22013 6 mod (8)
Vy s d ca php chia u20 + u21 + u22 + ... + u22013 cho 8 l 6.
V d 15 (VMO bng A 1995). Cho dy s nguyn (an ) , n N xc nh bi a0 = 1, a1 = 3 v
an+2 = an+1 + 9an , vi n l s t nhin chn, an+2 = 9an+1 + 5an , vi n l s t nhin l.
1) Chng minh rng a21995 + a21996 + ... + a22000 chia ht cho 20.
2) a2n+1 khng l s chnh phng vi mi s t nhin n.
Gii.
1) Vi mi s t nhin n, ta gi rn an mod (4) , un an mod (5) , 0 rn 3, 0 un 3
Ta lp bng sau
n
rn
un

0
1
1

1
3
3

2
0
2

3
3
3

4
3
1

5
2
4

6
1
3

7
3
2

8
0
4

9
3
1

10
3
2

11
2
3

12
1
1

13
3
4

14
0
3

15
3
2

16
3
4

17
2
1

18
1
2

Ta c r6k+m = rm vi mi s t nhin k, m. Nn r1995 = r6.332+3 = r3 = 3


a21995 + a21996 + ... + a22000 r32 + r42 + r52 + r02 + r12 + r22 0 mod (4)
Tng t c u8k+m = um , k, m N, m 2,u1995 = u8.249+3 = u3
a21995 + a21996 + ... + a22000 u23 + u24 + u25 + u26 + u27 + u28 0 mod (5)
M (4;5) =1 nn a21995 + a21996 + ... + a22000 chia ht cho 20
Cch 2.
Vi mi s t nhin n, ta gi
yn an mod (10) , 0 yn 9 an = yn + 10t a2n yn2 mod (20)
Ta lp bng sau
n
yn

0
1

n
yn

19
3

1
3
20
6

2
2

3
3

4
1

21
9

22
3

5
4
23
2

6
3

7
7

8
4

24
9

25
1

9
1
26
2

10
7
27
3

11
8
28
1

12
1
29
4

13
9
30
3

14
8
31
7

15
7
32
4

16
9
33
1

17
6

18
7

34
7

Ta c y24k+n = yn , n 4
2
Suy ra a21995 + a21996 + ... + a22000 y27
+ y42 + y52 + y62 + y72 + y82 0 mod (20)
131

35
8

36
1

.
Vy a21995 + a21996 + ... + a22000 ..20
2) T tnh cht tun hon ca dy (rn ) , n N
ta c a2n+1 r2n+1 mod (4) 3; 2 mod (4)
Nu a2n+1 l s chnh phng th a2n+1 0; 1 mod (4) (v l) .
Vy a2n+1 khng phi l s chnh phng
V d 16. Cho dy s nguyn (an ) , n N xc nh bi a0 = 103, a1 = 23 v an+2 = 4a2n+1 an .
Chng minh rng c v s s hng ca dy chia ht cho 2013.
Gii.
Vi mi s t nhin n, ta gi rn an mod (2013) , 0 rn 2012
Chng minh tng t bi ton 4, ta c s T nguyn dng sao cho
rT k+m = rm , k, m N,a2 = 4.232 103 = 2013 r2 = 0
.
Suy ra rT k+2 = r2 = 0 aT k+2 ..2013, k N
Vy c v s s hng ca dy chia ht cho 2013.
Ch 2. Bng chng trnh tin hc, ta kim chng c s T = 938 v c tt c s hng ca
dy chia ht cho 2013 l cc s hng aT k+2 , aT k+170 , aT k+422 , aT k+590 (T = 938)
V d 17 ( thi chn i tuyn trng THPTC Nguyn Du klk nm 2013 -2014). Cho
dy s nguyn (un ) tha mn : u1 = 1; u2 = 3v un+2 = 3un+1 + un vi mi s n nguyn dng.
Chng minh rng vi mi s nguyn dng k, tn ti s n nguyn dng un chia ht cho 10k .


Gii. Xt dy s (vn ) vi vn un mod 10k , 0 vn < 10k th un vn mod10k
vn nhn ti a 10k gi tr khc nhau
(vn ; vn+1 ) nhn ti a 10k .10k = 102k gi tr khc nhau
c m> n sao cho (vn ; vn+1 ) = (vm ; vm+1 )


um un mod10k 

um+1 un+1 mod10k



um1 um+1 3um un+1 3un un1 mod10k C nh vy ta c: umn+2


u2 mod10k v umn+1 u1 mod10k

umn umn+2 3umn+1 u2 3u1 0 mod10k
.
c umn ..10k
V d 18 (VMO 2011). Cho dy s nguyn (an ) , n N xc nh bi a0 = 1, a1 = 1 v
an = 6an1 + 5an2 vi mi n 2. Chng minh rng a2012 2010 chia ht cho 2011.
Nhn xt 3. Nu ta dy s d ca php chia an cho 2011 nh v d trn, th bng chng trnh Tin
hc, chng minh c c mt dy tun hon chu k 2010 c a2012 a2 2010 mod (2011)
.
a2012 2010..2011.(Thc t khng th gii nh vy).
Gii. Xt dy s nguyn (bn ) , n N xc nh bi b0 = 1, b1 = 1 v bn = 6bn1 + 2016bn2
bn an mod (2011) vi mi n 2.
Phng trnh c trng ca dy (bn ) , n N l: x2 6x 2016 = 0 (x 48) (x + 42) = 0


= 41

=
1
n
n
90
Nn bn = .48n + .(42) . V vy c

49 90bn = 41.48 +
48 42 = 1

=
90
n
49.(42) (n 0)
132

Ta c 2011 l s nguyn t nn theo nh l Fecma c


90b2012 = 41.482010+2 + 49.(42)2010+2 90b2 mod (2011) , (90; 2011) = 1
b2012 b2 2010 mod (2011)
.
Vy a2012 2010..2011 (PCM)
V d 19 ( kim tra i tuyn Ton 12, tnh k lk, nm hc 2011 2012). Cho dy s
(un ) , n N
c xc nh nh sau:

un+3

u0 = u1 = 3
u2 = 9
= 3un+2 un , n 0

1/ Chng minh rng c ba s thc a, b, c khng i m a < b < c v


u n = an + b n + c n
vi mi s t nhin n.
2/ Tm s d ca php chia
 2011   2012 
c
+ c
cho 12. (Vi
[t]
ch phn nguyn ca s thc t)

Gii: Xt hm s f (x) = x3 3x2 + 1 f (1).f (1/2) < 0; f (1/2)f (1) < 0, f (2 2).f (3) <
0.
Nn phng trnh f(x) = 0 c ba nghim phn bit a, b, c vi 1 < a < 1/2 < b < 1 <
2 2<c<3
Xt dy s
(vn ) , n N : vn = an + bn + cn v0 = v1 = a + b + c = 3,
v2 = a2 + b2 + c2 = (a + b + c)2 2 (ab + bc + ca) = 9
a3 3a2 + 1 = 0 an+3 3an+2 + an = 0; tt : bn+3 3bn+2 + bn = 0, cn+3 3cn+2 + cn = 0 Cng
li c vn+3 = 3vn+2 vn , n N Nn dy s ny tha mn cc gi thit ca dy s(un ) xc nh
duy nht nn c PCM 2/ C 0< a +b = 3 c < 1 nn b > - a > 0 suy ra
bn > |a|n an 0 < an + bn a2 + b2 = 9 c2 < 1, n 2
un 1 < cn = un (an + bn ) < un , un Z [cn ] = un 1
Vy
 2011   2012 
c
+ c
= u2011 + u2012 2
Gi rn l s d ca php chia un cho 12 . Ta c
n
rn

0
3

1
3

2
9

3
0

4
9

5
6

6
6

7
9

8
9

9
9

10
6

11
9

12
6

13
0

14
3

15
3

16
9

17
0

18
9

Nn chng minh c rn+14 = rn vi mi s t nhin n m 2011 =143x14 +9, 2012 =143x14


+10 nn
 2011   2012 
c
+ c
= u2011 + u2012 2 r9 + r10 2 1 (mod12)
. Nn s d cn tm l 1.
133

V d 20. Cho dy s (un ) , n N lp thnh mt cp s cng, c s hng u u1 = 1, cng


sai d = 19. Tnh u2925 v tm tt c s hng ca dy s l s nguyn dng c tt c cc ch s
u l ch s 5.
Gii. u2925 = u1 + 2924.d = 1 + 2924.19 = 55555
Gi s un = 55...5 (c k ch s 5, k ngun dng) (n > 1) Nn c 1 + (n 1) .19 =

5
10k 1 171n 175 = 5.10k 171n1 35 = 10k , n = 5n1 , n1 N
9
Hay c 10k 35 mod (171) 171 = 9.19; 10m 1 mod (9) ; 1018 1 mod (19) 1018m
1 mod (171) ; 105 136 35 mod (181)
1018m+5 35 mod (171) , m N (1)
Gi r l s d ca php chia 10m cho 171. Ta c bng sau
n
r

0
1

1
10

2
100

3
145

4
82

5
136

6
163

7
91

8
55

9
37

10
28

11
109

12
64

13
127

14
73

15
46

16
118

17
154

Do 10k 35 mod (171) k = 18m + 5, m N



5 1018m+5 + 35
Vy cc s hng un vi n = 5n1 =
, m N c ng 18m+5 ch s 5 v ch c
171
cc s hng m thi.

Ti liu tham kho


1) Nguyn Vn Mu (ch bin). Cc chuyn ton hc bi dng HSG cp THPT, K yu
HI NGH KHOA HC nm 2011) .
2) Nguyn Vn Mu (ch bin). Cc chuyn ton hc bi dng HSG cp THPT, K yu
HI NGH KHOA HC nm 2012) .
3) on Qunh (Ch bin), Ti liu chuyn ton, I S 10. NXB Gio dc nm 2009.
4) Mt s thi chn hc sinh gii cc cp.

134

18
1

a thc Fibonacci v ng dng


L Th Kim Uyn
Trng THPT Ng Gia T, tnh klk
Cc a thc Fibonacci c nghin cu bi E.C. Catalan (1883) l cc a thc c xc nh
bi cc h thc truy hi ging nh trng hp cc s Fibonacci. Mt cch tng qut lp cc a
thc ny c th c xc nh bi cc h thc truy hi tuyn tnh cp hai. Vic nghin cu vn
ny s rt c ch trong vic tm hiu su hn ton hc ph thng, trong cng tc ging dy v bi
dng hc sinh gii.
Trong bi ny, chng ti cp ti mt s tnh cht ca cc a thc dng Fibonacci, a thc
xc nh bi h thc truy hi tuyn tnh cp hai v mt s bi ton p dng trong trng THPT.

Cc nh ngha

nh ngha 1. [4] Dy a thc {an (x)} c gi l mt dy a thc xc nh bi h thc truy


hi tuyn tnh cp hai nu n tha mn h thc truy hi tuyn tnh
an (x) = p(x)an1 (x) + q(x)an2 (x)

(1)

trong p(x) v q(x) l a thc nhn gi tr thc hoc phc vi bin x.


Khi p(x) = x, q(x) = 1, a1 (x) = 1 v a2 (x) = x ta uc a thc Fibonacci. K hiu fn (x).
Dy a thc {fn (x)} tha mn h thc truy hi
fn+2 (x) = xfn+1 (x) + fn (x) vi mi n Z+
trong f1 (x) = 1, f2 (x) = x c gi l mt dy a thc Fibonacci.
Mnh 1. Gi s {an (x)} v {bn (x)} vi n Z+ , l hai dy a thc xc nh bi h thc truy
hi tuyn tnh cp hai (1). Nu
a0 (x) = b0 (x) v a1 (x) = b1 (x)
th hai dy {an (x)} v {bn (x)} l trng nhau, ngha l an (x) = bn (x) vi mi n Z+ .
Chng minh. Ta chng minh mnh trn bng phng php qui np. Theo gi thit mnh
ng vi n = 0, 1. Gi s mnh ng vi mi n = 2, . . . , j 1 vi mi j 1. Khi
aj (x) = p(x)aj1 (x) + q(x)aj2 (x) = p(x)bj1 (x) + q(x)bj2 (x) = bj (x).
Cho nn mnh ng vi n = j. Vy mnh ng vi mi n Z+ .

Mt s cng thc v a thc xc nh bi h thc truy


hi tuyn tnh cp hai

nh l 1. [4] (Cng thc Binet tng qut) Cho dy a thc {an (x)} xc nh bi h thc truy
hi tuyn tnh cp hai (1). Khi
(
a1 (x)(x)a0 (x) n
0 (x) n
(x) a1 (x)(x)a
(x) nu (x) 6= (x)
(x)(x)
(x)(x)
an (x) =
na1 (x)n1 (x) (n 1)a0 (x)n (x)
nu (x) = (x)
trong (x) v (x) l cc nghim ca phng trnh c trng
t2 p(x)t q(x) = 0.
135

Chng minh.
V (x), (x) l nghim ca phng trnh c trng
t2 p(x)t q(x) = 0
nn theo nh l Viet ta c (x) + (x) = p(x), (x)(x) = q(x). H thc (1) c th vit li
an (x) = ((x) + (x)) an1 (x) (x)(x)an2 (x)
hay
an (x) (x)an1 (x) = (x) [an1 (x) (x)an2 (x)]
=(x) [(x) (an2 (x) (x)an3 (x))] = = n1 (x) [a1 (x) (x)a0 (x)] .
Do an (x) = (x)an1 (x) + n1 (x) [a1 (x) (x)a0 (x)] . V vy


an (x)
(x) an1 (x)
a1 (x) (x)a0 (x)
=
+
.
n
n1
(x)
(x) (x)
(x)

(2)

t bn (x) = an (x)/ n (x). T (2) ta thu c


bn (x) =

(x)
a1 (x) (x)a0 (x)
bn1 (x) +
.
(x)
(x)

Hn na, nu an = can1 + d, n 1 th

n
a cn + d c 1 nu c 6= 1
0
c1
an =

a0 + nd
nu c = 1.
Do nu (x) 6= (x) th

n
(x)
a1 (x) (x)a0 (x) ((x)/(x))n 1
bn (x) = b0 (x)
.
+
(x)
(x)
(x)/(x) 1


1
a1 (x) (x)a0 (x) n
n
n
= n
(x)b0 (x) +
( (x) (x)) .
(x)
(x) (x)
hay


an (x)
1
a1 (x) (x)a0 (x) n
n
n
= n
(x)a0 (x) +
( (x) (x)) .
n (x)
(x)
(x) (x)
V vy
an (x) =

a1 (x) (x)a0 (x) n


a1 (x) (x)a0 (x) n
(x)
(x).
(x) (x)
(x) (x)

Nu (x) = (x) th bn (x) = b0 (x) + n

a1 (x) (x)a0 (x)


hay
(x)

an (x)
a1 (x) (x)a0 (x)
= a0 (x) + n
n
(x)
(x)
nn
an (x) = a0 (x) n (x) + n n1 (x) [a1 (x) (x)a0 (x)]
= na1 (x)n1 (x) (n 1)a0 (x)n (x).

136

Bi ton 1. [3]( thi chn i tuyn Olympic Ton THPT Vit Nam nm 1999, bng A) Cho
hai dy s (xn ), n = 0, 1, 2, . . . v (yn ), n = 0, 1, 2, . . .c xc nh nh sau:
x0 = 1, x1 = 4 v xn+2 = 3xn+1 xn vi mi n = 0, 1, 2, . . .
x0 = 1, y1 = 2 v yn+2 = 3yn+1 yn vi mi n = 0, 1, 2, . . .
Chng minh rng x2n 5yn2 + 4 = 0 vi mi n = 0, 1, 2, . . .
Li gii. Gi , l nghim ca phng trnh c trng t2 3t + 1 = 0. Khi

3+ 5
3 5
=
, =
2
2
p dng nh l 1 ta c

!n
!n

5+ 5 3+ 5
5 5 3 5
xn = .
.
2
2
2 5
2 5
!2n
!2n

51
51
1+ 5 1+ 5
.
.

=
2
2
2
2
!
!
2n+1

2n+1
51
1+ 5
=

2
2
v

!2n+1
1 1+ 5
+
yn =
2
5

51
2

!2n+1
.

Do
!2n+1 2

!2n+1
51
1+ 5

x2n 5yn2 + 4 =
2
2

!2n+1 2
!2n+1

1+ 5
51
+4

+
2
2
!2n+1
!2n+1

1+ 5
51
= 4.
.
+ 4 = 0.
2
2

Vy x2n 5yn2 + 4 = 0 vi mi n = 0, 1, 2, . . .
Ch 1. thun tin trong vic trnh by trong nh l 2 ta t
A1 (x) =

a1 (x) (x)a0 (x)


a1 (x) (x)a0 (x)
= A1 , A2 (x) =
= A2 ,
(x) (x)
(x) (x)
am (x) = am vi m Z+ , (x) = v (x) = .

nh l 2. Cho dy a thc {an (x)} xc nh bi h thc truy hi tuyn tnh cp hai (1). Khi
vi n, m, r l cc s nguyn dng th
am an amr an+r =

a1 [a1 ar a0 ar+1 ] anm+r


(q(x))rm [a21 a1 a0 p(x) q(x)a20 ]
a0 [a1 ar a0 ar+1 ] anm+r+1

(q(x))rm [a21 a1 a0 p(x) q(x)a20 ]


137

Chng minh. Theo nh l 1 ta c


am an amr an+r =
=[A1 m A2 m ][A1 n A2 n ] [A1 mr A2 mr ][A1 n+r A2 n+r ]
=A1 A2 mr n [ r r ] + A1 A2 n mr [r r ]


=A1 A2 [r r ] n mr mr n
=A1 A2 [r r ]mr mr [nm+r nm+r ]
r r nm+r nm+r
=A1 A2 [ ]2 (q(x))mr

2
A1 A2 [ ] a1 ar a0 ar+1 a1 anm+r a0 anm+r+1
=
A1 A2 [ ]2
(q(x))rm A1 A2 [ ]2
[a1 ar a0 ar+1 ] [a1 anm+r a0 anm+r+1 ]
=
.
(q(x))rm [a21 a1 a0 p(x) q(x)a20 ]

H qu 1. Cho dy a thc {an (x)} xc nh bi h thc truy hi tuyn tnh cp hai (1). Khi :
an+1 an1 a2n = (q(x))n1 [a2 a0 a21 ] ,
Chng minh. H qu trn c suy ra t nh l 2 trong trng hp m = n v r = 1.

Bi ton 2. [3]( thi chn i tuyn Olympic Ton THPT Vit Nam nm 1994, bng B) Cho
dy s Fibonacci (un ), n = 1, 2, . . . c xc nh bi:
u1 = u2 = 1, un+2 = un+1 + un vi mi n = 1, 2, 3, . . .
Hy tm s nguyn dng m sao cho
um
2k

um
2k+1

m1
X

s1
s.us1
2k .u2k+1

s=1

vi mi k = 1, 2, 3, . . .
Li gii. T gi thit th s m cn tm phi tha mn vi k = 1, lc
m
um
2 + u3 =

m1
X

s1
s.us1
2 .u3

s=1

hay
m

1+2 =

m1
X

s.2s1 .

(1)

s=1

t S =

m1
P

s.2s1 , khi 2S =

m1
P

s.2s .

s=1

s=1

T c
m1
X

S =2S S =

s.2

s=1
m1

=(m 1).2

m1

=(m 1).2

m1
X

s.2s1

s=1

m1
X

s1

s=1
m1

(2

m1

(s (s 1)) = (m 1).2

m1
X
s=1

m1

1) = (m 2).2
138

+ 1.

2s1

(2)

T (1) v (2) ta c 1 + 2m = (m 2).2m1 + 1. Do m = 4.


p dng H qu 1 ta c
u22k+1 = u2k+2 u2k (1)2k [u2 u0 u21 ] = u2k+2 u2k + 1

(3)

u22k = u2k+1 u2k1 (1)2k1 [u2 u0 u21 ] = u2k+1 u2k1 1

(4)

Ly (3) tr (4) v theo v ta c


u22k+1 u22k =u2k+2 u2k u2k+1 u2k1 + 2
=(u2k+1 + u2k )u2k u2k+1 (u2k+1 u2k ) + 2
=2u2k+1 u2k + u22k u22k+1 + 2.
Do u22k+1 u22k = u2k+1 u2k + 1 vi mi k = 1, 2, 3, . . .
Vi m = 4 ta c u42k+1 + u42k 2u22k+1 .u22k = u22k+1 .u22k + 2u2k+1 u2k + 1 hay u42k+1 + u42k =
1 + 2u2k+1 u2k + 3u22k+1 .u22k vi mi k = 1, 2, 3, . . .
Vy m = 4 l s duy nht tha mn yu cu bi.
Bi ton 3. [3]( thi chn i tuyn Olympic Ton THPT Vit Nam nm 1997, bng B) Cho
dy s nguyn (an ), n = 0, 1, 2, . . . c xc nh nh sau:
a0 = 1, a1 = 45
an+2 = 45an+1 7an vi mi n = 0, 1, 2, . . .
a) Tnh s cc c dng ca a2n+1 an an+2 theo n.
b) Chng minh rng 1997a2n + 7n+1 .4 l s chnh phng vi mi n.
Li gii.
a) Xt dy s nguyn
a0 = 1, a1 = 45 v an+2 = 45an+1 7an vi n = 0, 1, 2, . . .
p dng H qu 1 ta c
a2n+1 an an+2 = 7n [a2 a0 a21 ] = 7n+1

(1)

T (1) suy ra s cc c s dng ca a2n+1 an an+2 bng n + 2.


b) T (1) ta c:
a2n+1 an (45an+1 7an ) 7n+1 = 0
hay a2n+1 45an an+1 + 7a2n 7n+1 = 0 vi mi n N.
iu ny chng t phng trnh x2 45an x + 7a2n 7n+1 = 0 c nghim nguyn nn =
(45an )2 4(7a2n 7n+1 ) = 1997a2n + 7n+1 .4 phi l s chnh phng.

Dng lng gic ca a thc xc nh bi h thc truy


hi tuyn tnh cp hai

nh l 3. Cho dy a thc {an (x)} xc nh bi h thc truy hi tuyn tnh cp hai (1). Khi
p
(i) Vi nhng gi tr ca x lm cho q(x) < 0 v |p(x)| 2 q(x) ta c

p
n1 
p
sin n
sin(n 1)
an (x) =
q(x)
a1 (x)
a0 (x) q(x).
sin
sin
139

p
trong c xc nh bi h thc p(x) = 2 q(x) cos .
p
(ii) Vi nhng gi tr ca x lm cho q(x) < 0 v p(x) > 2 q(x) ta c

p
n1 
p
sinh n
sinh(n 1)
a0 (x) q(x).
an (x) =
q(x)
a1 (x)
sinh
sinh
p
trong c xc nh bi h thc p(x) = 2 q(x) cosh .
Chng minh.
p
(i) Gi s l gc tha mn h thc p(x) = 2 q(x) cos . t

p
n1 
p
sin n
sin(n 1)
bn (x) =
q(x)
a1 (x)
a0 (x) q(x).
.
sin
sin
Ta s chng minh bn (x) = an (x).
p
V |cos | 1 nn |p(x)| 2 q(x). Ta c b1 (x) = a1 (x) v
h
i
p
p
b2 (x) = q(x) a1 (x)2 cos a0 (x) q(x)
= p(x)a1 (x) + q(x)a0 (x) = a2 (x).
Mt khc
p
n2
sin(n 1)
q(x)
a1 (x)
sin
p
n2
p
p
sin(n 2)
2 q(x) cos
q(x)
a0 (x) q(x).
sin
p
n1 2 sin(n 1) cos
=a1 (x)
q(x)
sin
n 2 sin(n 2) cos
p
q(x)
a0 (x)
sin
p
n1
sin n + sin(n 2)
=
q(x)
a1 (x)
sin
p
n sin(n 1) + sin(n 3)
a0 (x)
q(x)
.
sin
p(x)bn1 (x) = 2

q(x) cos

Ta cng c
q(x)bn2 (x)

p
n3 
p
sin(n 2)
sin(n 3)
=q(x)
q(x)
a1 (x)
a0 (x) q(x).
sin
sin
p
2 p
n3
sin(n 2)
=
q(x)
q(x)
a1 (x)
sin
p
2 p
n3
p
sin(n 3)
+
q(x)
q(x)
a0 (x) q(x).
sin 
p
n1 
p
sin(n 2)
sin(n 3)
q(x)
a1 (x)
a0 (x) q(x).
.
=
sin
sin
T suy ra
p(x)bn1 (x) + q(x)bn2 (x)
140


p
n1 
p
sin n
sin(n 1)
a0 (x) q(x).
= bn (x).
=
q(x)
a1 (x)
sin
sin
Do theo Mnh 1 ta chng minh c bn (x) = an (x).
p
(ii) Gi s l gc tha mn h thc p(x) = 2 q(x) cosh . t

p
n1 
p
sinh n
sinh(n 1)
cn (x) =
q(x)
a1 (x)
a0 (x) q(x).
.
sinh
sinh
Ta chng minh cn (x) = an (x). Ta c c1 (x) = a1 (x) v
h
i
p
p
c2 (x) = q(x) a1 (x)2 cosh a0 (x) q(x)
= p(x)a1 (x) + q(x)a0 (x) = a2 (x).
Tnh ton ta c
p(x)cn1 (x)

n1 
p
p
sinh(n 1)
sinh(n 2)
q(x)
a1 (x)
a0 (x) q(x).
=2 cosh
sinh
sinh
n1
p
sinh n + sinh(n 2)
q(x)
=
a1 (x)
sinh
n1
p
p
sinh(n 1) + sinh(n 3)
q(x)
a0 (x) q(x).
.

sinh
Ta cng c
q(x)cn2 (x)

p
n3 
p
sinh(n 3)
sinh(n 2)
=q(x)
q(x)
a1 (x)
a0 (x) q(x).
sinh
sinh

p
n1 
p
sinh(n 3)
sinh(n 2)
=
q(x)
a0 (x) q(x).
.
a1 (x)
sinh
sinh
Do
p(x)cn1 (x) + q(x)cn2 (x)

p
n1 
p
sinh n
sinh(n 1)
q(x)
=
a1 (x)
a0 (x) q(x).
= cn (x).
sinh
sinh
Cho nn theo Mnh 1 ta chng minh c cn (x) = an (x).

Mnh 2. [6] (Cng thc lng gic ca cc a thc Fibonacci)


Vi x = 2i cosh z th
sinh nz
fn (x) = in1
.
sinh z
Chng minh.
V x = 2i cosh z nn

4i2 cosh2 z + 4 = 2i sinh z. Do

x + x2 + 4
(x) =
= i(cosh z + sinh z) = iez
2
x2 + 4 =

v (x) = i(cosh z sinh z) = iez . Khi


fn (x) =

n (x) n (x)
enz enz
sinh nz
= in1 z
= in1
z
(x) (x)
e e
sinh z

141

Mnh 3. [6] (Nghim ca a thc Fibonacci)


Cc nghim ca a thc Fibonacci c dng
x = 2i cos

k
, 1 k n 1.
n

Chng minh. t z = u + iv. Khi


|sinh z|2 = sinh2 u + sin2 v v |cosh z|2 = sinh2 u + cos2 v.
V u l s thc nn sinh u = 0 khi v ch khi u = 0. V vy nghim ca sinh z l nghim ca
sinh iv = i sin v v nghim ca cosh z l nghim ca cosh iv = cos v. Khi theo Mnh 2 ta c

(
sinh nz = 0
fn (x) = 0
sinh z 6= 0

sin nv = 0
z = iv

sinh z 6= 0

nv = k
z = ik/n

i sin v 6= 0.

Khi
x = 2i cosh z = 2i cosh iv = 2i cos v = 2i cos k/n
trong 1 k n 1.

Bi ton 4. Gii phng trnh:


a) g(x) = x5 + 4x3 + 3x.
b) g(x) = x6 + 5x4 + 6x2 + 1.
Li gii.
a) Ta thy rng g(x) = f6 (x), trong f6 (x) l a thc Fibonacci th 6. Theo Mnh 3
k
nghim ca f6 (x) = x5 + 4x3 + 3x c xc nh bi x = 2i cos
, 1 k 5.
6

Khi k = 1, x = 2i cos = 3i.
6

Khi k = 2, x = 2i cos = i.
3

Tng t khi k = 3, 4,
5
th
ta
c
cc
gi
tr
ca
x
tng
ng
l
0,
i,

3i. Do f6 (x)

c cc nghim l 0, i, 3i.
V vy g( x) = x(x2 + 1)(x2 + 3).
b) Ta thy rng g(x) = f7 (x), trong f7 (x) l a thc Fibonacci th 7. Theo Mnh 3
k
nghim ca f7 (x) = x6 + 5x4 + 6x2 + 1 c xc nh bi x = 2i cos
, 1 k 6.
7

Khi k = 1, x = 2i cos .
7
2
Khi k = 2, x = 2i cos .
7
3
3
Tng t khi k = 3, 4, 5, 6 th ta c cc gi tr ca x tng ng l 2i cos , 2i cos ,
7
7
2

2
3
2i cos , 2i cos . Do f7 (x) c cc nghim l 2i cos , 2i cos , 2i cos .
7
7
7
7
7
2
2
2
2 2
2
2 3
V vy g( x) = (x + 4 cos )(x + 4 cos
)(x + 4 cos
).
7
7
7

142

Ti liu
[1] Nguyn Vn Mu (2003), a thc, Nxb Gio dc H Ni.
[2] Nguyn Vn Khu - L Mu Hi (2001), Hm bin phc, Nxb Gio dc H Ni.
[3] T sch Ton hc v Tui tr (2007), Cc bi thi Olympic Ton THPT Vit Nam (1990-2006),
Nxb Gio dc.
[4] Tian-Xiao He and Peter J.-S.Shiue (2009), On sequences of numbers and polynomials defined
by linear recurrence relations of order 2, Article ID 709386, pp 6-11.
[5] T. Horzum and E. G. Kocer (2009), On some properties of Horadam polynomials, International Mathematical Forum, pp 1243-1250.
[6] T. Koshy (2001), Fibonacci and Lucas number with applications, Wiley-Interscience, NewYork,
NY, USA, pp 444-494.

143

Mt s ng dng ca tm v t
Nguyn Vn Quang
THPT Chuyn Nguyn Du, k Lk

Cc nh ngha v tnh cht

nh ngha 1. K hiu V(O,k) l php v t tm O, t s k; Q(O; ) l php quay tm O, gc


quay ). Khi ta gi V (O, k) Q(O; ) gi l php v t quay tm O. im O gi l tm v t
quay.

Nhn xt 1. Vi 4 im phn bit trong mt phng phn bit A,B,C,D sao cho ABCD khng l
hnh bnh hnh th tn ti duy nht mt php v t quay bin bin A thnh B v C thnh D
B 1. (tm v t quay) Cho A, B, C, D l 4 im phn bit trong mt phng sao cho AC
khng song song vi BD. Gi X l giao im ca AC v BD. ng trn ngoi tip tam gic ABX
v CDX ct nhau ti O (O khc X). Khi O l tm v t quay bin A thnh C v bin B thnh D.

Chng minh. Trong chng minh ta s dng gc nh hng mod gia cc ng thng. V
k hiu (a , b) gc quay t ng thng a n ng thng b.
Ta c: (OA,AC)=(OA,AX)=(OB, BX)=(OB,BD)
V (OC,CA)=(OC,CX)=(OD,DX)=(OD,DB)
Suy ra cc tam gic AOC v BOD ng dng v c cng hng. Do php v t quay tm
O bin A thnh C th cng bin B thnh D
Ch : Nu O l tm v t quay bin A thnh C v bin B thnh D, th O cng l tm v t
quay ca php bin hnh bin A thnh B v bin C thnh D.

144

[ = COD
\ v OC = OD OB = OD = k . Nn pho v t quay tm
Chng minh. V AOB
OA
OB
OA
OC
[ = COD
\ bin A thnh B v bin C thnh D.
O , t s k v gc quay AOB

Bi tp p dng

[ = CAD
\ = DAE;
\
V d 1. (IMO shortlist 2006) Cho ABCDE l ng gic li tha: BAC
[ = DCA
\ = EDA.
\ ng cho BD ct CE ti P. Chng minh rng : ng thng AP chia
CBA
i cnh CD

Gii. T gi thit ta c A l tm v t quay bin B thnh C v bin D thnh E. Theo b ta


c: ABCP v APDE l cc t gic ni tip.
\ = ABC
[ nn CD tip xc vi ng trn ngoi tip ABCP
Mt khc, ACD
Tng t CD cng tip xc vi ng trn ngoi tip APDE.
Gi M l giao im AP vi CD. Ta c: M C 2 = M P.M A = M D2 M C = M D (pcm)
V d 2. (USA MO 2006) Cho t gic ABCD, gi E v F ln lt thuc cnh AD v BC sao cho
AE
BF
=
. Bit tia FE ct tia BC v CD ln lt ti S v T. CMR: Cc ng trn ngoi tip
ED
FC
tam gic SAE, SBF, TCF v TDE cng i qua 1 im.
Gii. Gi P l tm v t quay ca php v t quay VP bin AD thnh BC, v E v F ln lt
AE
BF
thuc cnh AD v BC tha
=
nn VP : E 7 F Ta c: VP : AE BF nn PAES v
ED
FC
PBFS l cc t gic ni tip VP : DE CF nn PEDT v PFCT l cc t gic ni tip Suy ra
4 ng trn ngoi tip cc tam gic SAE, SBF, TCF v TDE cng i qua P
V d 3. (China 1992) Cho t gic ABCD ni tip ng trn tm O. ng cho AC v BD
ct nhau ti P. ng trn ngoi tip tam gic ABP v CDP ct nhau ti P v Q. Gi s rng
[ = 900 P Q.
O, P, Q l 3 im phn bit. CMR: OQOQP
Gii. Gi m v N ln lt l trung im ca AC v BD. K hiu T l php v t quay bin A thnh
B v bin C thnh D. Khi theo b ta c Q l tm v t quay ca T. V T : AC BDnn
T : M 7 N suy ra MNPQ l t gic ni tip .
\
\
Mt khc v OM
P = ON
P = 900 nn O,P,M,N cng thuc mt ng trn Do M, N, P,
Q, O cng thuc mt ng trn ng knh OP, nn
V d 4. Cho t gic ABCD, AC ct BD ti P. Gi O1, O2 ln lt l tm ng trn ngoi
tip tam gic APD v BPC. Ly M, N v O l trung im AC, BD v O1 O2 . Chng minh rng O
l tm ng trn ngoi tip tam gic MPN.
Gii.
145

Gi giao im ca ng tron ngoi tip cc tam gic APD v BPC l P v Q. K hiu T l


php v t quay bin A thnh C v bin D thnh B. Khi ta c Q l tm v t quay ca T. Khi
1
php bin hnh R = (id + T ) ( id: php ng nht) cng l mt php v t quay tha:
2
R : A 7 M
D 7 N
O1 7 O

V O1 l tm ng trn ngoi tip tam gic QAD nn O l tm ng trn ngoi tip tam
gic QMN, m QMPN ni tip nn ta c iu phi chng minh
V d 5. (im Miquel ca t gic) Gi l1 , l2 , l3 , l4 l 4 ng thng m khng c 2 ng
no trong chng song song. K hiu Cijk l ng trn ngoi tip tam gic to bi 3 ng
thng li , lj , lk ( cc ng trn ny goi l ng trn Miquel). Chng minh rng cc ng trn
C123 , C124 , C134 , C234 cng i qua 1 dim ( im ny gi l im Miquel)

Gii. Gi Pij l giao im ca li v lj . Hai ng trn C134 v C234 ct nhau ti P. Khi P l


tm v t quay ca php v t quay T : P13 7 P23 , P14 7 P24
Khi P cng l tm v t quay ca php v t quay T 0 : P13 7 P14 , P23 7 P24
p dng b ta c: C123 v C124 cng i qua P. (pcm)
V d 6. (IMO 2005) Cho t gic li ABCD c BC=AD v khng song song. Gi E v F ln
lt thuc cnh BC v AD sao cho BE v DF. Cc ng thng AC v BD ct nhau ti P, ng
thng BD v EF ct nhau ti Q, EF ct AC ti R. Chng minh rng : khi E, F thay i th cc
ng trn ngoi tip tam gic PQR cng i qua 1 im khc P.
AF
CE
=
nn T : F 7 E p dng b
Gii. Gi S l tm v t quay T : A 7 C, D 7 B V
AD
CB
ta c: SPAD, SRAF, SQFD l cc t gic ni tip, mt khc cc ng trn chnh l cc ng
146

trn Miquel ca cc t gic to bi cc ng thng AD, AP, PD, QF v S l im Miquel. Do


vy S phi thuc ng trn ngoi tip tam gic PQR. Vy ng trn ngoi tip tam gic PQR
lun i qua S khi E, F thay i

Bi tp p dng

Bi tp 1. (IMO Shorlist 2006). Gi A1, B1, C1 ln lt thuc cc cnh BC, CA v AB ca


tam gic ABC. ng trn ngoi tip ca cc tam gic AB1C1, BC1A1 v CA1B1 ct ng tr
ngoi tip tam gic ABC ti A2, B2, C2 ( vi A2 6= A, B2 6= B, C2 6= C). Cc im A3, B3, C3
ln lt i xng vi A1, B1, C1 qua trung im ca cc cnh BC, CA v AB. Chng minh rng:
tam gic A2 B2 C2 ng dng vi tam gic A3 B3 C3 .
Bi tp 2. Cho tam gic ABC ni tip ng trn tm O, gi Y thuc (O) sao cho AY l ng
i trung nh A ca tam gic ABC. Gi X l trung im AY. CMR: X l tm v t quay bin B
thnh A v bin A thnh C.
Bi tp 3. (USA 2008). Cho tam gic ABC c G l trng tm, P l im thay i trn cnh BC.
im Q v R ln lt thuc cnh AC v AB sao cho PQ//AB v PR//AC. Chng minh rng khi
P thay i trn cnh BC th ng trn ngoi tip tam gic AQR lun qua mt im c nh X
[
[ = CAX
sao cho BAG
Bi tp 4. (USA 2008). Cho tam gic ABC nhn, khng cn, v M, N, P ln lt l trung im
ca BC, CA v AB. Cc ng trug trc AB v AC ct tia AM ln lt ti D v E. Gi F l giao
im ca BD v CE vi F thuc min trong tam gic ABC. Chng minh rng A, N, F v P cng
thuc mt ng trn.
Bi tp 5. Gi A l mt giao im ca hai ng trn (C1) v (C2) c tm ln lt l O1, O2.
ng thng d tip xc vi (C1), (C2) ln lt ti ti B v C. Gi O3 l tm ng trn ngoi
\
tip tam gic ABC. Ly D i xng vi O3 qua A. M l trung im O1O2. CMR: O\
1 DM = O2 DA.

Ti liu tham kho


1. Tuyn tp thi quc t
2. Three Lemmas in Geometry - Yufei Zhao
3. Cc bi ton hnh hc su tm trn diendantoanhoc.net
147

Mt s ng dng ca php
bin i Abel
L Sng, Hunh B Lc
S GD& T Khnh Ha
Mt s bt ng thc trong cc thi IMO lin quan n hai dy s cho bi dng tng v tch
c chng minh nh php bin i tng ca Abel, thm vo vn dng BCS s d dng a n
kt qu. Bi ny c vit da trn bi ging i tuyn Khnh Ha.

Bin i Abel v v d tnh tng


Cho hai dy (ai ), (bi ). Khi
a1 b1 + a2 b2 + + an bn = (a1 a2 )b1 + (a2 a3 )(b1 + b2 ) + . . .
+(an1 an )(b1 + b2 + + bn1 ) + an (b1 + b2 + + bn )

.
V d 1. Tnh tng S1 = 1 + 2q + 3q 2 + + nq n1 .
Gii. Ta c
S1 = (1 2) + (2 3)(1 + q) + + n(1 + q + + q n1 )


q n1 1
qn 1
q 1 q2 1
+
+ +
+n
=
q1
q1
q1
q1
n
q

1
nq n
qn 1
1
n1
(1 + q + + q
n) + n
=

.
=
q1
q1
q 1 (q 1)2
y ta vn dng php bin i Abel, l d nhin c th tnh tng theo cch gn hn.

Dng gi thit cho bi tng cc s hng ca dy

Bi ton 1. Cho a1 a2 an > 0 v b1 b2 bn > 0 tha iu kin


a1 b1 ; a1 + a2 b1 + b2 ; . . . ; a1 + a2 + + an b1 + b2 + + bn .
Chng minh rng vi mi k N, ak1 + ak2 + + akn bk1 + bk2 + + bkn .

Chng minh. Xt aki bki = (ai bi ) ak1
+ aik2 bi + + bk1
= ci di vi ci = ai bi .
i
i
T gi thit ta c ci 0, c1 + c2 + + cj 0 j v di > di+1 > 0
Suy ra
ak1 bk1 + ak2 bk2 + + akn bkn = c1 d1 + c2 d2 + + cn dn
= (d1 d2 )c1 + (d2 d3 )(c1 + c2 ) + + dn (c1 + c2 + + cn ) 0, do bin i Abel, ta c
iu cn chng minh.
Bi ton 2. (Romani 1977) Cho ai > 0, 0 b1 b2 bn v
a1 + a2 + + a
k

b1 + b2 + + bk vi mi k n. Chng minh rng a1 + a2 + + an b1 + b2 + + bn .

148

Chng minh. Dng bin i Abel






a2
an
1
1
1
1
a1
+ + + = a1
+ (a1 + a2 )
+ ...
b1
b2
bn
b1
b
b
b
2
2
3
!
1
1
1
+ (a1 + a2 + + an ) .
+(a1 + a2 + + an1 ) p

bn
bn
bn1





1
1
1
1
M b1 + b2 + + bn = b1
+ (b1 + b2 )
+ ...
b1
b2
b2
b3
1
+(b1 + b2 + + bn )
bn
Nh BCS



 a1

a2
an

2
+ + +
a1 + a2 + + an
b1 + b2 + + bn
.
b1
b2
bn
T gi thit v (1), (2), (3) ta c iu cn chng minh.

(1)

(2)

(3)

Bi ton 3.
c, d khng
m, a 1, a + b 5, a + b + c 14, a + b + c + d 30. Chng

Choa, b,
minh rng a + b + c + d 10.
Chng minh. y l bi ton c th t bi 2, vi n = 4 v bk = k 2 .

Bi ton 4. (USAMO 95) Cho a1 + a2 + + an n n v ai > 0. Chng minh rng




1
1
1
2
2
2
a1 + a2 + + an
1 + + +
.
4
2
n
Chng minh. Ta bt u vi bi ton
ai > 0, b1 b2 bn 0 v a1 + a2 + + ak b1 + b2 + + bk k n th
2
a1 + a22 + + a2n b21 + b22 + + b2n .
Bi ton ny tng t bi 1, nhng vi gi thit yu hn. Dng bin i Abel
a1 b1 + a2 b2 + + an bn = a1 (b1 b2 ) + (a1 + a2 )(b2 b3 ) + + (a1 + a2 + + an )bn (3)
V
(4)
b1 (b1 b2 ) + (b1 + b2 )(b2 b3 ) + + (b1 + b2 + + bn )bn = b21 + b22 + + b2n .
Kt hp (3), (4) vi bt ng thc BCS
2
(a21 + a22 + + a2n ) (b21 + b22 + + b2n ) (a1 b1 + a2 b2 + + an bn )2 (b21 + b22 + + b2n ) .
Ta c iu cn chng minh.

1
Tr li bi ton, ch cn chn bn = n n 1 > .
2 n
Bi ton 5. (IMO 78 Php ngh) Cho f : N N n nh. Chng minh rng
n f (k)
n 1
P
P
n

.
2
k=1 k
k=1 k


n 1
P
f (k)
Chng minh. Bt ng thc vit li
1 0
k
k=1 k
f (k)
Do f n nh, t mk =
tha m1 .m2 . . . mk 1, t do AM GM m1 +m2 + +mk k
k
p dng bin i Abel, ta c
n 1
P
(mk 1)
k=1 k


n
P
1
1
1
=

(m1 + m2 + + mk k) + (m1 + m2 + + mn n) 0.
k k+1
n
k=1
149

Dng gi thit cho bi tch cc s hng ca dy

Bi ton 6. Cho x1 , x2 , . . . , xn v y1 , y2 , . . . , yn l hai dy s dng tha mn


x1 y1 , x1 x2 y1 y2 , x1 x2 x3 y1 y2 y3 , . . . , x1 x2 x3 . . . xn y1 y2 y3 . . . yn .
Chng minh rng x1 + x2 + + xn y1 + y2 + + yn .
Chng minh. Bin i Abel
x1
x2
xn
x1 + x2 + + xn = y 1 + y 2 + + y n
yn 
 y1  y2

x1
x1 x2
x1 x2
xn
= (y1 y2 ) +
+
(y2 y3 ) + +
+
+ +
yn
y1
y1
y2
y1
y2
yn
Ngoi ra, theo BCS
r
x1 x2
x 1 x 2 . . . xk
xk
+
+ +
kk
=k
y1
y2
yk
y1 y2 . . . yk
Nn x1 + x2 + + xn 1(y1 y2 ) + 2(y2 y3 ) + + nyn = y1 + y2 + + yn .
Bi ton 7. (RoMO 99) Cho hai dy s dng (xi ) v (yi )tha mn
i) x1 y1 < x2 y2 < < xn yn ;
ii) x1 + x2 + + xk y1 + y2 + + yk , 1 k n.
Chng minh rng
1
1
1
1
1
1
+
+ +

+
+ + .
x1 x2
xn
y1 y2
yn
Chng minh. Xt


1
1
1
1
1
1
+
+ +

+
+ +
x1 x2
xn
y1 y2
yn
y 1 x1 y 2 x2
y n xn
1
=
+
+ +
, t zk =
> 0; sk = (x1 y1 ) + + (xk yk )
x1 y 1
x2 y 2
xn y n
xk yk
= s1 z1 (s2 s1 ) z2 (sn sn1 ) zn 0 (do Abel)
Suy ra iu cn chng minh.

Sp th t trong dy s chng minh bt ng thc


Thc t trong nhiu bt dng thc, ta cn sp xp li dy bi ton n gin hn.

Bi ton 8. (Nga 86) Chng rng mi b s dng ty , ta c




2
n
1
1
1
1
+
+ +
<4
+
+ +
.
a1 a1 + a2
a1 + a2 + + an
a1 a2
an
Chng minh. Do v phi ca bt ng thc khng thay i th t ca cc
ak
nn ta ch cn chng minh v tri ln nht th bt ng thc cng tha mn.
Xt 0 < b1 b2 bn l dy
(ak )
c sp li nn b1 + b2 + + bn a1 + a2 + + an
1
2
n
+
+ +
Suy ra V tri
b1 b1 + b 2
b1 + b2 + + bn
150

Ghp tng cp ca v phi


2k 1
2k
2k 1
2k
4
+
<
+
< .
b1 + b2 + + b2k1 b1 + b2 + + b2k
kbk
(k + 1)bk
bk
Bi ton 9. (VN 98) Tn ti hay khng dy s (xn ) tha
i) |xn | 0, 666 vi mi s t nhin n;
1
1
ii) |xm + xn |
+
vi mi cp s nguyn dng m 6= n.
n(n + 1) m(m + 1)
Chng minh. Sp xp li vi mi s nguyn dng k b s x1 , x2 , . . . , xk theo t t xi1 xi2
xi k
Khi


|xi1 + xik | = |xi1 + xi2 | + + xik1 + xik
1
1
1

+
+ +
i1 (i1 + 1) i2 (i2 + 1)
ik (ik + 1)
k
P
1
1
1
2

ik (ik + 1)
p=1 ip (ip + 1) i1 (i1 + 1)


1
1
1

2 1

k+1
i1 (i1 + 1) ik (ik + 1)


1
1
4
2
1
2 1

k+1
1.2 2.3
3 k+1
4
Ch cn k ln, s mu thun vi |xi1 + xik | 2 0, 666 < .
3

Bt dng thc Abel


T bin i Abel ta c bt ng thc Abel.
Cho dy s (zj ), zk = z1 + z2 + + zk ; k = 1, 2, . . . , n. Khi
a) Vi mi dy (ak ) : a1 a2 an > 0 ta u c
|a1 z1 + a2 z2 + + an zn | a1 . max |zk | , 1 k n.
b) Vi mi dy (bk ) : bn bn1 b1 0 ta c
|b1 z1 + b2 z2 + + bn zn | 2bn . max |zk | , 1 k n.

Chng minh. T bin i Abel


a) |a1 z1 + a2 z2 + + an zn | |z1 | (a1 a2 ) + |z2 | (a2 a3 ) + + |zn1 | (an1 an ) + zn an
max |zk | (a1 a2 + a2 a3 + + an ) = a1 . max |zk | , 1 k n.
b) |b1 z1 + b2 z2 + + bn zn | |z1 (b1 b2 ) + z2 (b2 b3 ) + + zn1 (bn1 bn ) + zn bn |
max zk (b2 b1 + b3 b2 + + bn bn1 + bn )
2bn . max |zk | , 1 k n.
p dng: Cho a gic A0 A1 . . . An tha A0 A1 A2 = A1 A2 A3 = = An2 An1 An theo
chiu kim ng h; vi A0 A1 > A1 A2 > > An1 An . Chng minh rng A0 v An khng trng
nhau.
Chng minh. Chn trc Ox l A0 A1 , A0 trng gc ta , z = ei()
ak = Ak1 Ak lc An c ta a1 + a2 z + + an z n1 l gc gia hai on thng lin tip
= 0 th tng l s dng
6= 0 nhn hai v cho (1 z) ta c
151

(a1 a2 )(1 z) + (a2 a3 )(1 z 2 ) + + an (1 z n ) 6= 0. Tht vy,


|(a1 a2 )z + (a2 a3 )z 2 + + an z n | < |(a1 a2 )z| + |(a2 a3 )z 2 | + + |an z n |
< a1 a2 + a2 a3 + + an
Suy ra
a1 + a2 z + + an z n1 = (a1 a2 )z1 + (a2 a3 )z2 + + an (z1 + z2 + + zn ) 6= 0.

Bi tp ngh

Bi tp 1. Cho x1 x2 xn v y1 y2 yn . Gi (z1 , z2 , . . . , zn ) l mt hon v ca


n
n
P
P
(y1 , y2 , . . . , yn ). Chng minh rng
(xi yi )2
(xi zi )2 . (IMO 75)
i=1

i=1

Bi tp 2. Cho a1 , a2 , . . . , an > 0. Chng minh rng

a2
a2
a21 a22
+ + + n1 + n a1 +a2 + +an .
a2 a3
an
a1

(CHINA 85)
Bi tp 3. Cho x, y, z > 0 sao cho xyz = 1. Chng minh rng

x3
y3
+
+
(1 + y)(1 + z) (1 + z)(1 + x)

z3
3
(IMO 98 ngh)
(1 + x)(1 + y)
4
Bi tp 4. Cho x y z > 0, a > x, ab xy, abc xyz. Chng minh rng a+b+c x+y +z.
b c
c
a b c
Bi tp 5. Cho x y z > 0, a > b > c > 0, + + 3, + 2, 1. Chng minh
x y z
y z
z
rng
1 1 1
1 1 1
a) + + + + .
a b c
x y z
1
1
1
1
1
1
b)
+
+

+
+
.
n
n
n
n
n
n
y
a
c
x
z
b
a b c
b c
c
Bi tp 6. Cho 0 < x y z; a, b, c > 0; + + 3; + 2; 1. Chng minh rng
x y z
y z
z

a) a + b + c x + y + z.

b) n a + n b + n c n x + n y + n z.
Bi tp 7. Cho 0 < x y z; a, b, c > 0;

b c
c
a b c
+ + 3; + 2; 1. Chng minh rng
x y z
y z
z

a) a2 + b2 + c2 x2 + y 2 + z 2 .
b) an + bn + cn xn + y n + z n .

Ti liu tham kho


1) L Sng, Dy s v vn lin quan, 1993, NXB Nng.
2) Titu Andreescu, Mathematical Olympiad Challenges, 2000, Birkhauser.
3) Nguyn Vn Mu, Bt ng thc nh l v p dng, 2006, NXB Gio dc.
4) Nguyn V Lng, Cc bi ging v bt ng thc Cauchy, 2006, NXB HQG H Ni.
152

Mt s bi tp v dy s tun hon
L Trang Tr
Trng THPT Chuyn Nguyn Du, kLk
Dy s l mt lnh vc kh v rt rng, trong cc thi hc sinh gii quc gia, quc t cng
thng xut hin cc bi ton v dy s. gii c cc bi ton v dy s i hi ngi lm
ton phi c kin thc tng hp v s hc, i s, gii tch. Cc vn lin quan n dy s cng
rt a dng v cng c nhiu ti liu vit v vn ny, cc ti liu ny cng thng vit kh
rng v cc vn ca dy s. Trong bi vit ny ti ch cp n mt phn rt nh ca dy
s, l tnh tun hon.

Mt s kin thc lin quan

1.1

nh ngha dy s tun hon

Dy s (un ) c gi l mt dy tun hon nu tn ti s nguyn dng p sao cho


un+p = un , n N

(1)

S nguyn dng p nh nht dy (un ) tha mn (1) c gi l chu k c s ca dy.


V d 1. {1; 2; 1; 2; 1; 2; 1; 2 } l mt dy tun hon vi chu k 2.

1.2

Phng trnh sai phn tuyn tnh cp 2

nh ngha 1. Phng trnh sai phn tuyn tnh cp hai n (un ) l phng trnh sai phn dng:
aun+2 + bun+1 + cun = f (n)

(2)

Phng trnh sai phn tuyn tnh thun nht tng ng vi phng trnh (2) c dng:
aun+2 + bun+1 + cun = 0

(3)

Nghim tng qut ca (2) c dng un = xn + yn , trong xn l nghim tng qut ca (3), cn
yn l mt nghim ring no ca (2).
tm nghim ca (3) u tin ta lp phng trnh c trng ca (3) l:
ax2 + bx + c = 0
TH1. Nu phng trnh c trng (4) c hai nghim thc phn bit 1 , 2 th:
xn = An1 + Bn2
TH2. Nu phng trnh c trng (4) c nghim kp 1 = 2 = 0 th:
xn = (A + Bn)n0 .
TH3. Nu phng trnh c trng (4) c nghim phc = + i = r(cos + i sin ) vi
p
y
i2 = 1; r = x2 + y 2 ; = arctan .
x
Khi :
xn = rn (A cos n + iB sin n).
y A, B l cc hng s thc c xc nh da vo cc iu kin ban u.
153

(4)

Bi tp

3xn
1
,
xn + 3

1. Cho {xn }n l mt dy tha mn xn+1 =

n1

Chng minh {xn }n l mt dy tun hon.


Hng dn:
tan atan b
tan(a b) = 1t
m tan 6 = 13 .
ana.tanb
Ta vit li xn+1 nh sau:
xn+1 =

xn
1+

1
3

1 xn
3

Thay x1 bng tan t vi t R, ta c: x2 = tan(t 6 )


Theo quy np : xn = tan [t (n 1) 6 ], n 1.
Tht vy
xn+1

xn

1
3

h
h
tan [t (n 1) 6 ] tan 6
i
i
=
= tan t (n 1)
= tan t n
=
1+ tan 6 . tan [t (n 1) 6 ]
6
6
6
1 + 13 xn

Hm tan tun hon vi chu k nn


h
h
i
h
i

i
xn+6 = tan t (n + 5)
= tan t n + = tan t (n 1)
= xn .
6
6
6
6
Suy ra dy tun hon vi chu k 6.
2. Dy s {xn } c nh ngha nh sau:
x1 = 2, xn+1 =

2 + xn
, n = 1, 2, 3...
1 2xn

Chng minh:
a) xn 6= 0 n = 1, 2, 3...
b) Chng minh xn khng tun hon.
Hng dn:
Ta chng minh xn = tan n vi = arctan 2:
Vi n = 1 : x1 = tan(arctan 2) = 2
Gi s xn = tan n. Khi
xn+1 =

2 + xn
tan + tan n
=
= tan( + n) = tan(n + 1)
1 2xn
1 2 tan n

Vi m bt k
x2m = tan 2m =

2 tan m
2xm
=
2
1 tan m
1 x2m

Gi s xn = 0 v n = 2m l chn. Khi , x2m 0. Suy ra xm = 0.


Nu n = 2k (2s + 1)(k, s Z+ ) th xn = x2k (2s+1) =x2.2k-1 (2s+1) =0 x2k-1 (2s+1) =0
Sau k bc ta c x2s+1 = 0. Do
2 + x2s
x2s
= 0 x2s = 2
= 2
1 2x2s
1 x2s
C hai nghim ca phng trnh trn l s v t, nhng xs phi l s hu t ( v x1 = 2 Q) (
mu thun)
154

b) Gi s dy s tun hon, ngha l xn+m xn = 0 , n,m 1.


V xn = tan n nn ta c:
tan(n + m) tan n =

sin m
=0
cos(n + m)cosn

Do : xm = tan m = 0( mu thun vi cu a) .
3. ( 28th VMO 1990) Dy s {xn } c nh ngha nh sau:
x1 = a, xn+1

p
3 3x2n xn
=
, n = 1, 2, 3..., 1 < a < 1
2

a) Chng minh dy tun hon.


b) Tm a mi s hng ca dy u dng.
a) t x1 = sin k. Khi
p

3 3sin2 k sin k
3
x2 =
=
cos k sin k = sin cos k cos = sin( k)
2
2
2
3
3
3
Do x3 = sin( 3 3 + k) = x1 . Suy ra dy s tun hon chu k 2.

b) Theo cu a, mi s hng ca dy u dng th sin k > 0 v x2 = sin ( 3 k > 0

0<k< 0<a<
3
2
4. Cho a, b l hai s nguyn dng vi mt s l. Dy {un } c nh ngha nh sau:
(
1
u nu un chn
2 n
u0 = b, un+1 =
un + a nu un l
Chng minh {un } tun hon k t lc no . ( French Mathematical Olympiad, 1996)
Hng dn:
Ta chng minh c v s s hng ca dy s b thua 2a.
Gi s iu ny l khng ng v t um l s hng ln nht tha mn um < 2a
Nu um chn th um+1 = u2m < 2a
Nu um l th um+1 = um + a l chn. Do um+2 = um2+a < 3a
< 2a( mu thun)
2
Theo nguyn l Dirichlet suy ra dy s tun hon.

5. Chng minh rng dy s (un ) tha mn: un+1 + un1 = 2un n N tun hon.
Hng dn:

Phng trnh c trng: 2 2+1=0 c nghim l:

2
2

1 =
+
i = cos + i sin ,
4
4
2
2
2
2

i = cos i sin
2 =
2
2
4
4

(|1 | = |2 | = 1)

1 c chu k n nu 1 n = 1 hay 1 = cos 2


+isin 2
n
n
Vy 1,2 c chu k 8 m cng thc ca s hng tng qut un = an1 + bn2 nn suy ra (un ) tun
hon vi chu k 8.
155

6. Cho k Q\Z. Chng minh rng dy s {un } xc nh theo cng thc


u0 = 1, u1 = 1, un+1 = kun un1, n N
khng l mt dy tun hon.
Hng dn:
Khi |k| > 2 th |un+1 | |k| |un | |un1 | > 2 |un | |un1 |.
Nu|un | < |un1 | vi mi n N th ta c ngay iu phi chng minh.
Nu xy ra |un | |un1 | > 0 th |un+1 | > 2 |un ||un1 | = |un ||un1 |+|un | m |un ||un1 | 0
suy ra |un | < |un+1 | < ... v do dy un khng phi l dy s tun hon.
Xt |k| 2 vi k = pq , (p, q) = 1, 2 q Z , p Z. Bng quy np theo n ta thu c
uj =

pj
q j1

pj Z, (pj , q) = 1, j {1, n}

T suy ra
p
p pn
pn1
p.pn q 2 .pn1
pn+1
un+1 = un un1 =

=
= n ,
n1
n2
n
q
qq
q
q
q
vi pn+1 = ppn q 2 pn1 Z v (pn+1 , q) = 1. Do q 2 nn un 6= um khi n 6= m v dy {un }
khng l dy s tun hon.
7. Xc nh gi tr ca k Q dy s {un } xc nh theo cng thc
u0 = 1, u1 = 1, un+1 = kun un1, n N
l mt dy s tun hon.
Khi |k| > 2 th dy {un } khng l dy s tun hon (theo bi 4).
Khi |k| 2 vi k = pq , (p, q) = 1, 2 q Z , p Z th dy {un } khng l dy s tun hon
(theo bi 4).
Xt |k| 2 v k Z.
Vi k = 2 th
u0 = 1, u1 = 1, un+1 + un1 = 2un , n N
nn {un } l mt cp s cng vi cng sai bng -2 nn hin nhin dy {un } khng l dy s tun
hon.
Vi k = 1 th:
u0 = 1, u1 = 1, u2 = 2, u3 = 1, u4 = 1, u5 = 2, u6 = 1, u7 = 1, ...
Suy ra {un } l dy tun hon vi chu k 6.
Vi k = 0 th
u0 = 1, u1 = 1, u2 = 1, u3 = 1, u4 = 1, u5 = 1, ...
Suy ra {un } l dy tun hon vi chu k 4 Vi k = 1 th
u0 = 1, u1 = 1, u2 = 0, u3 = 1, u4 = 1, ...
Suy ra {un } l dy tun hon vi chu k 3:
Vi k = 2 th u0 = 1, u1 = 1, u2 = 1, u3 = 1, ... Suy ra {un } l dy tun hon vi chu
k 2:
156

8. Dy {un }c nh ngha nh sau: u4n+1 = 1, u4n+3 = 0 vi n 0vu2n = un vi n 1.


Chng minh dy khng tun hon.
Gi s dy cho tun hon, t T = 2r q (q l) l chu k ca dy.
Nu q = 4m + 3 v k r + 2 th
1 = u2k = u2k +T = u2k +2r (4m+3) = u2kr +4m+3 = u4p+3 = 0
(mu thun)
Nu q = 4m + 1 th
1 = u2k = u2k +3T = u2k +3.2r (4m+1) = u2kr +3(4m+1) = u4p+3 = 0
(mu thun)
Vy dy khng tun hon.
9. Dy s xn xc nh bi x0 = 2, x1 = 1 v xn+1 = xn xn1
a) Chng minh rng dy s tun hon;
b) Tm cng thc tng qut cho xn .
a) Ta tnh cc s hng u tin ca dy s th c dy
2, 1, 1, 2, 1, 1, 2, 1, . . .
Ta c x0 = x6 . Gi s xn+6 = xn vi mi n, ta c :
xn+7 = xn+6 xn+5 = xn xn+4 + xn+3 = xn xn+3 + xn+2 + xn+3 = xn + xn+1 xn = xn+1
T ta nhn thy xn+6 = xn vi mi n. Hay dy s tun hon vi chu k 6.
b) Tnh cht tun hon ca dy s gi cho chng ta n cc dy s cos(n) v sin(n).
Nu (= 2/k th cc dy ny tun hon vi chu k k. rng phng trnh
xn+1 = xn xn1 c th vit li di dng
xn+1 + xn1 = 2 cos(/3)xn

(5)

p dng cc cng thc


cos x + cos y = 2 cos

xy
x+y
xy
x+y
cos
, sin x + sin y = 2 cos
sin
2
2
2
2

ta thy cc dy s cos(n/3), sin(n/3) u tho mn phng trnh (5) t dy s xn =


c1 cos(n/3) + c2 sin(n/3) vi c1 , c2 l hng s cng tho mn (5). By gi, ta ch cn chn c1 , c2
thch hp x0 = 2, x1 = 1 l xong. Gii h
c1 cos(0) + c2 sin(0) = 2
c1 cos(/3) + c2 sin(/3) = 1
ta c c1 = 2, c2 = 0. Vy cng thc tng qut ca dy s cho l xn = 2 cos(n/3).
10. Gi s 2005 + 2005 c th c biu din nh mt a thc trong + v . Tm tng
cc h s ca a thc.
Trong khai trin ca k + k , t + = 1v = 1. Ta c, tng cc h s l:
Sk = k + k
= ( + )

k-1 + k1

= (k + k ) + k-2 + k2

Suy ra, Sk = Sk1 +Sk2 V Sk = Sk+6 nn (Sk ) l mt dy tun hon vi chu k 6 v S2005 = S1 = 1
157

Phng php s dng tnh tun hon ca dy s d

nh l 1. Cho dy s nguyn (an ) tho mnan = c1 an+1 +c2 an+2 +...+ck an+k trong c1 , c2 , ..., ck
l cc s nguyn v m l s nguyn dng ln hn 1. Gi rn l s d trong php chia an cho m.
Khi dy (rn ) tun hon.
Bi ton 1. Chng minh rng tn ti v hn s hng ca dy Fibonacci chia ht cho 2012.
Li gii.
Ta s chng minh bi ton tng qut: vi mi s t nhin n, tn ti v hn s hng ca dy
Fibonacci chia ht cho N.
Gi
ri
l s d trong php chia Fi cho n. 0 rn N 1
Xt cc cp s d khi chia hai s hng lin tip trong dy Fibonacci theo modulo N.
(r0 , r1 ); (r1 , r2 ); (r2 , r3 )....
V dy Fibonacci l v hn m ch c n2 kh nng cho mi cp s d theo modulo N nn tn ti
(ri , ri+1 ) tho mn ri ri+m v ri+1 ri+m+1 (mod N) vi m Z + .
Xt i > 1, ta c:
ri1 = ri+1 ri ri+m+1 ri+m = ri+m1 (modN )
Qu trnh c tip tc dn n rj rj+m (modN ) j 0 Suy ra 0 r0 rm r2m
... (modN ) , tc l c v hn cc s rkm tho mn yu cu bi ton. Vy bi ton c chng
minh.
Bi ton 2. Cho dy s (an ) xc nh bi:
(
a0 = 29; a1 = 105; a2 = 381
an+3 = 3an+2 + an+1 + an (1) n 0
.. Chng minh rng vi mi s nguyn dng m lun tn ti s t nhin n sao cho cc s
an , an+1 1, an+2 2 u chia ht cho m.
Li gii.
Ta b sung thm bn s hng ca dy l a1 = 8, a2 = 2, a3 = 1, a4 = 0 .
Gi s an rn (modm); 0 rn m 1 .
Xt cc b ba (rn , rn+1 , rn+2 ) . Khi tn ti hai s nguyn p < q sao cho:

r
=
r
p
q

ap aq (modm)
rp+1 = rq+1
ap+1 aq+1 (modm)

rp+2 = rq+2
ap+2 aq+2 (modm)
Kt hp vi (1) ta c:
aq+k ap+k (modm) k.
Do
ak aqp+k (modm) k.
t t = q p N th ak ak+t (modm) k

158

Suy ra ak ak+ht (modm) k N ; h N . Ni ring ta c

aht4 a4 0 (modm)
aht3 a3 1 (modm)

aht2 a2 2 (modm)
Vi h ln th ht 4 N . Khi t n = ht 4 ta c:
an 0(modm), an+1 1(modm), an+2 2(modm)
Do cc s an , an+1 1, an+2 2 u chia ht cho m( pcm).
Bi ton 3. Cho dy s an xc nh bi

a1 = 33; a2 = 49, a3 = 177
an+3 = 8an+2 8an+1 + an
Chng minh rng vi mi gi tr ca ca n, an khng chia ht cho 2013.
Ch l 2013=61.33. iu gi ta ngh ti xt theo modulo 61 (61 nguyn t). Gi rn l
s d trong php chia an cho 61. a1 33(mod61); a2 49(mod61); a3 55(mod61); .... Sau 1 s
tnh ton ta thy (rn ) l dy tun hon chu k 15 k t r1 .
Bng tnh ton 15 s hng u ca dy ta thy khng c s hng an no chia ht cho 61. Do
an khng chia ht cho 61 vi mi n, t ta c pcm.
Bi ton 4. Cho dy s (an ) xc nh bi

a1 = 111; a2 = 23
an+2 = 4a2n+1 an , n 1
Chng minh rng tn ti v s s hng ca dy chia ht cho 2015.
Ta c a3 = 2005chia ht cho 2005. Gi rn l s d trong php chia an cho 2005. T cng thc
2
2
rn+2 (mod2005)
truy hi ca dy (an ) ta c rn+2 = 4rn+1
rn (mod2005). Suy ra rn 4rn+1
ng thi dy rn tun hon k t lc no , ngha l tn ti n0 , T 1sao cho rn+T = rn , n n0
Chn n = n0 1 + T ta c rn0 1+T rn0 1 . Tng t ta cng c rn0 2+T rn0 2 , ..., r1+T =
r1 , rT = r0 Do r3+nT = r3 = 0, n N hay a3+nT chia ht cho 2005

Ti Liu tham kho


1. Dy s v cc bi ton v dy s - Trn Nam Dng.
2. Mathematical olympiad challenges - Titu Andreescu and Razvan Gelca.
3. Problema-Solving strategies - Athur Engel

159

Mt s dng ton v bt ng thc


trong t din tng t nh tam gic
on Vn Khim
Trng THPT 1 Tuy Phc, Bnh nh
Hnh tam gic v hnh t din c nhiu tnh cht ging nhau nh: tam gic l a gic c s
cnh t nht; hnh t din l hnh a din c s mt t nht; mi tam gic u c ng trn ngoi
tip v ni tip; mi t din u c mt cu ngoi tip v ni tip,. . . i vi hc sinh kh, gii
lp 11, cc em quen vi cc h thc tng t gia tam gic v t din nh:

G l trng tm ABC GA + GB + GC = 0 .

G l trng tm t din ABCD GA + GB + GC + GD = 0 .


Hoc i vi tam gic vung v t din c 3 cnh xut pht t mt nh vung gc nhau i
1
1
1
mt cng c nhng h thc tng t: 2 + 2 = 2 (a, b: l di 2 cnh gc vung; h: di
a
b
h
ng cao v t nh gc vung ca tam gic vung).
1
1
1
1
+ 2 + 2 = 2 (a, b, c: di 3 cnh vung gc i mt nu; h: di ng cao t
2
a
b
c
h
din k t nh chung ).
Vn c t ra l khi no v lm th no nhn bit c mt h thc (ng thc, bt
ng thc) c trong tam gic, liu c tn ti h thc tng t trong t din khng? Gii quyt
vn ny khng nhng cng c kin thc cho cc em, c bit l hc sinh kh gii, mt cch h
thng logic m cn to ra s hng th trong hc tp v bc u gip hc sinh bit tm ti, pht
hin kin thc.
Trong chuyn ny, chng ti a ra mt s bt ng thc trong tam gic c th m rng
sang t din nh vo tnh cht tng t nh nu, cng nh tnh tng t trong chng minh.
Cui cng, chng ti chn thnh cm n nhng ng gp qu bu ca qu ng nghip v
nhng thiu st trong chuyn ny.

Cc kin thc b tr

Ngoi cc kin thc quy nh trong chng trnh ton (SGK hin hnh), trong chuyn cn
s dng mt s kin thc nh:
1. Bt ng thc Bunhiacopsky:
n
2  n   n 
P
P 2
P 2
ai b i
ai
bi ai , bi R.
i=1

i=1

i=1

Du = xy ra khi v ch khi k R: ai = kbi ( i = 1, 2,. . . , n).


2. T s din tch.
OA.OB
S (OAB)
0
0
=
K hiu S (OAB): l din tch
Vi A, A Ox; B, B Oy ta c
0
0
S (OA B )
OA0 .OB 0
OAB
3. T s th tch.
0
0
0
Cho Ox, Oy, Oz khng ng phng v A, A Ox; B, B Oy; C, C Oz. Ta c
V (OABC)
OA.OB.OC
=
0
0
0
V (OA B C )
OA0 .OB 0 .OC 0
K hiu: V (OABC): l th tch khi t din OABC.
160

4. Cng thc Leibniz.


a) i vi tam gic ABC, G l trng tm. Ta c M A2 + M B 2 + M C 2 = 3M G2 + GA2 + GB 2 +
GC 2 , M .
c bit: khi M O: tm ng trn ngoi tip ABC, th GA2 +GB 2 +GC 2 = 3 (R2 OG2 )(
R: bn knh ng trn ngoi tip ABC).
b) i vi t din ABCD, G l trng tm. Ta c M A2 + M B 2 + M C 2 + M D2 = 4M G2 +
GA2 + GB 2 + GC 2 + GD2 , M .
c bit: khi M O: tm mt cu ngoi tip t din, th GA2 + GB 2 + GC 2 + GD2 =
4 (R2 OG2 )(R: bn knh mt cu ngoi tip t din).
5. Phng tch ca mt im i vi mt cu (O;R).


T M k cc ng thng ct mt cu (O; R) ti A, B, C, D th M A.M B = M C.M D =
OM 2 R2 ()
161

+ Nu M ngoi hnh cu th t (*) ta c M A.M B = M C.M D = OM 2 R2


+ Nu M trong hnh cu th t (*) ta c M A.M B = M C.M D = R2 OM 2

Cc kin thc trn c chng minh trong nhiu ti liu, v vy y chng ti ch ra v


khng chng minh li.

MT S BI TON M RNG T TAM GIC SANG


T DIN

Bi ton 1. a) Trong tam gic.


Bi ton 1.a: Cho A1, B1, C1 ln lt l cc im trn cnh BC, CA, AB ca ABC. K hiu
SA, SB, SC ln lt l din tch AB1 C1 , BC1 A1 , CA1 B1 .
1
Chng minh rng min {SA , SB , SC } S, vi S l din tch ABC. (K hiu min {SA , SB , SC }:
4
gi tr nh nht trong cc gi tr SA, SB, SC ).
Gii. Chng minh bng phn chng (hnh 1).

S
A > S()

1
Gi s ngc li, tc l c
SB > S()

SC > 1 S( )
4
SA
1
AC1 .AB1
1
Khi , t (*) ta c
>
> (1)
S
4
AB.AC
4
Tng t, t (**), (***) ta c
BC1 .BA1
1
> (2)
BC.BA
4
CA1 .CB1
1
> (3)
CA.CB
4
Nhn 3 bt ng thc (1), (2), (3) theo v ta c:

162

AB1 .AC1 .BA1 .BC1 .CB1 .CA1


1
>
2
2
2
a .b .c
64
(4)
(vi a = BC, b = CA, c = AB).
Mt khc, ta c

BA1 + CA1
2 BA1 .CA1
1=

(bt ng thc Cauchy)


a
a
BA1 .CA1
1

2
a
4
Tng t, ta cng c
CB1 .AB1
1 AC1 .BC1
1
;

2
2
b
4
c
4
AC1 .AB1 .BA1 .BC1 .CB1 .CA1
1

Suy ra
(5) bt ng thc (5) mu thun vi bt ng
2
2
2
abc
64
thc (4).
T bi ton c chng minh.
b) Nhn xt.
+ Cc im A1, B1, C1 thuc cc cnh tam gic cho php ta s dng t s din tch gia din
tch tam gic to thnh v din tch ABC.
1
BA1 BC1
+ T s xut hin do tch
.
c xem nh so snh A1 vi trung im ca BC;
4
BC BA
C1 vi trung im ca BA.
1
+ Vi cc im trn cnh t din, dng t s th tch, ta d on lm xut hin s . T ,
8
ta c bi ton m rng sau:
c) M rng sang t din.
163

Bi ton 1.b. Cho t din ABCD. Gi B1 , C1 , D1 , B , C , D ln lt l cc im trn cnh AB,


AC, AD, CD, BD, BC.
0
0
0
0
0
0
K hiu VA , VB , VC , VD , V ln lt l th tch t din AB1 C1 D1 , BB1 C D , CC1 B D , DD1 B C , ABCD.
1
Khi min {VA , VB , VC , VD } V
8
Gii.

Phn chng nh bi ton 1.a ta cng c cc bt ng thc sau:


AB1 .AC1 .AD1
1
VA
=
>
V
AB.AC.AD
8
0
0
BB1 .BC .BD
1
VB
=
>
V
BA.BC.BD
8
0
0
CC1 .CB .CD
1
VC
=
>
V
CA.CD.CB
8
0
0
VD
DD1 .DB .DC
1
=
>
V
DA.DB.DC
8
Nhn cc bt ng thc trn vi k hiu b, c, d, b, c, d l di cnh AB,AC,
4 AD, CD,
0
0
0
0
0
0
AB1 .BB1 .AC1 .CC1 .AD1 .DD1 .BC .DC .BD .CD .CB .DB
1
BD, BC, ta c
>
2 02 02
2
2
2
0
8
bcdb c d
Hay
 12
0
0
0
0
0
0
AB1 .BB1 .AC1 .CC1 .AD1 .DD1 .BC .DC .BD .CD .CB .DB
1
>
(6)
2 02 02
2
2
2
0
2
bcdb c d
Cng lm tng t nh bi ton 1.a, ta c 6 bt ng thc c dng
AB1 .BB1
1

b2
4
T suy ra VT(6) (v tri ca (6)):
 6  12
1
1
V T (6)
=
(7)
4
2
bt ng thc (7) mu thun vi bt ng thc (6). T ta c iu phi chng minh.
Bi ton 2. a) Trong tam gic. Bi ton 2.a. Cho ABCni tip ng trn (O;R), G l trng
tm tam gic. GA, GB, GC tng ng ct ng trn (O) ti A1, B1, C1. Chng minh rng
164

1) GA1 + GB1 + GC1 GA + GB + GC


2) S1 S (vi S1, S th t l din tch A1 B1 C1 , ABC).
Gii. (hnh 3)



GA1 .GA GB1 .GB GC1 .GC
1
1
1
2
2
1) Ta c GA1 +GB1 +GC1 =
+
+
= (R OG )
+
+
GA
GB
GC
GA GB GC



Do PG /(O) = GA1 .GA = GB1 .GB = GC1 .GC = OG2 R2


1
1
1
1
2
2
2
+
+
(1)
Hay GA1 + GB1 + GC1= (GA + GB + GC )
3
GA GB GC
(h thc Leibniz vi M O)
Theo bt ng thc Bunhiacopsky th (GA + GB + GC)2 3 (GA2 + GB 2 + GC 2 ) (2) T
(1), (2) suy ra


1
1
1
1
2
GA1 + GB1 + GC1 (GA + GB + GC)
+
+
(3)
9
GA GB GC


1
1
1
Theo bt ng thc Cauchy, (GA + GB + GC)
+
+
9 (4)
GA GB GC
T (3), (4) ta c bt ng thc cn chng minh.
Du = xy ra GA = GB = GC G O ABC u.
2) V G l trng tm ABC nn
1
S (GBC) = S (GCA) = S (GAB) = S (*)
3
S1
S (GB1 C1 ) + S (GC1 A1 ) + S (GA1 B1 )
Ta c
=
S
S
S1
S (GB1 C1 ) S (GC1 A1 ) S (GA1 B1 )

=
+
+
(do (*))
S
3S (GBC)
3S (GCA)
3S (GAB)
r


2
2
2
S1
1 GB1 .GC1 GC1 .GA1 GA1 .GB1
3 GA1 .GB1 .GC1

=
+
+

(5)
S
3 GB.GC
GC.GA
GA.GB
GA2 .GB 2 .GC 2
(bt ng thc Cauchy)
Mt khc,
165

GA1 .GA = GB1 .GB = GC1 .GC = R2 OG2 (suy ra t PG /(O) ) v GA2 + GB 2 + GC 2 =
3 (R2 OG2 ) ( h thc Leibniz)
GB 2
GC 2
GA2 + GB 2 + GC 2
GA2
+
+
Nn 3 =
=
R2 OG2
GA.GA1 GB.GB1 GC.GC1
r
GA.GB.GC
GA
GB
GC
3=
+
+
33
(bt ng thc Cauchy)
GA1 GB1 GC1
GA1 .GB1 .GC1
GA1 .GB1 .GC1

1 (6)
GA.GB.GC
S1
T (5) v (6)
1 bt ng thc cn chng minh.
S
Du = xy ra khi v ch khi xy ra du = (5) v (6)
GB
GC
GA
=
=
= 1 G O, tc l ABC u.

GA1
GB1
GC1
b) Nhn xt.
Vi li gii trn, nu thay ABC bi t din ABCD, ta ch cn thay s lng cc yu t hnh
hc tham gia vo bi gii. C th:
Trong (1) thay GA1 + GB1 + GC1 bi GA1 + GB1 + GC1 + GD1 .
Trong (2) dng bt ng thc Bunhiacopsky cho 4 cp s.
Trong (4) dng bt ng thc Cauchy cho 2 tng, mi tng gm 4 s.
ng thc (*) c thay bi
1
V (GBCD) = V (GCDA) = V (GDAB) = V (GABC) = V (ABCD).
4
Do , ta c bi ton m rng sau:
c) M rng sang t din.
Bi ton 2.b. Cho t din ABCD ni tip mt cu (O;R). G l trng tm t din. GA, GB,
GC, GD ct mt cu (O;R) th t ti A1, B1, C1, D1 . Chng minh rng
1) GA1 + GB1 + GC1 + GD1 GA + GB + GC + GD
2) V (A1 B1 C1 D1 ) V (ABCD)
Li gii: Bi ton 2.b c tin hnh tng t nh bi ton 2.a. y trnh by tm tt:
GA1 .GA GB1 .GB GC1 .GC GD1 .GD
+
+
+
1) GA1 + GB1 + GC1 + GD1 =
GA
GC
GD
 GB

1
1
1
1
2
2
GA1 + GB1 + GC1 + GD1 = (R OG )
+
+
+
GA GB GC GD


1
1
1
1
1
2
2
2
2
= (GA + GB + GC + GD )
+
+
+
4
GA GB GC GD


1
1
1
1
1
2
GA1 + GB1 + GC1 + GD1 (GA + GB + GC + GD)
+
+
+
16
GA GB GC GD
GA1 + GB1 + GC1 + GD1 GA + GB + GC + GD (bt ng thc c chng minh)
2) V G l trng tm t din nn
V (GBCD) = V (GCDA) = V (GDAB) = V (GABC) = V .
K hiu: V, V1 l th tch t din ABCD, A1B1C1D1 th t.
S dng: phprphn tch nh bi ton 2.a, t s th tch v bt ng thc Cauchy, ta chng
3
3
3
3
V1
4 GA1 .GB1 .GC1 .GD1
minh c

1
V
GA3 .GB 3 .GC 3 .GD3
Suy ra V1 V
Du = xy ra G O ABCDl t din gn u (t din c cc cp cnh i bng
nhau).
166

Bi ton 3. a) Trong tam gic.


Bi ton 3.a. Cho tam gic ni tip ng trn (O;R), sao cho O trong tam gic. OA, OB,
9R
0
0
0
OC ct BC, CA, AB th t ti A, B, C. Chng minh rng AA + BB + CC
.
2
Gii. (hnh 4)
K hiu: S, Sa, Sb, Sc ln lt l din tch ABC, OBC, OCA, OAB. Ta c Sa +Sb +Sc =
Sa Sb Sc
S
+
+
= 1 (*)
S
S
S
0
0
0
Sa
OA
OA OB
Chng minh c
=
. Tng t i vi 2 t s cn li, nn t (*) suy ra
+
+
S
AA0
AA0 BB 0
0
OC
=1
CC 0
0
0
0
AA OA BB OB CC OC

+
+
=1
AA0
BB 0
CC 0
R
R
R

= 2 (**)
0 +
0 +
AA
BB
CC 0
1
1
1
2
(1)

0 +
0 +
0 =
AA
BB
CC
R
Mt khc, theo bt ng
 thc Cauchy th 

1
1
1
0
0
0
AA + BB + CC
9 (2)
0 +
0 +
AA
BB
CC 0
0
0
0 2
T (1) v (2) suy ra AA + BB + CC . 9.
R
9R
0
0
0
Vy AA + BB + CC
.
2
Du = xy ra ABC u.
b) Nhn xt.
0
0
+ Vi gi thuyt O trong tam gic, gip ta phn tch c S = Sa +Sb +Sc v OA = AA R
nn c c h thc (1).
Ngoi ra, hai tam gic OBC v ABC c chung y BC nn t s din tch bng t s hai
di ng cao, t a v t s OA:AA.
9
+ S trong bt ng thc trn c c t (**) v (2).
2
+ i vi t din ABCD do th tch V ca t din ABCD c phn tch thnh tng th tch
ca 4 t din nh O nn s 2 trong ng thc tng t nh (**) c thay bi s 3 v s 9 trong
bt ng thc tng t (2) c thay bi s 16 (trong bt ng thc Cauchy).

167

T ta c bi ton sau:
c) M rng sang t din.
Bi ton 3.b. Cho t din ABCD ni tip mt cu (O;R) sao cho O trong t din. OA, OB,
OC, OD ct cc mt BCD, CDA, DAB, ABC th t ti A, B, C, D.
Chng minh rng
16R
0
0
0
0
AA + BB + CC + DD
3
Gii tm tt: (hnh 5)
Gi V, VA, VB, VC, VD ln lt l th tch t din ABCD, OBCD, OACD, OABC.
Ta c V = VA + VB + VC + VD
VA VB VC VD

+
+
+
= 1 (a)
V
V
V
V
Gi H, K ln lt l hnh chiu ca A, O trn (BCD).
Do OK // AH nn H, K, A thng hng. T
0
VA
OK
OA
R
=
=
0 = 1
V
AH
AA
AA0

1
1
1
1
Do , (a) R
+
+
=3
AA0 BB 0 CC 0 DD0
1
1
1
1
3

(b)
0 +
0 +
0 +
0 =
AA
BB
CC
DD
R
168

Theo bt ng thc Cauchy,ta c



1
1
1
1
0
0
0
0
+
+
+
16 (c)
AA + BB + CC + DD
AA0 BB 0 CC 0 DD0
T (b) v (c) suy ra bt ng thc cn chng minh.
0
0
0
0
0
0
0
0
Du = xy ra AA = BB = CC = DD OA = OB = OC = OD
VA
VB
VC
VD
1

=
=
=
=
V
V
V
V
4
O l trng tm t din ABCD l t din gn u.
kt thc bi vit ny, c l cng cn c mt bi ton v t din c 3 cnh xut pht t
mt nh ca t din vung gc i mt.
i vi tam gic vung ABC vung ti A, ta c nh l Pythagore: BC 2 = AB 2 + AC 2 . Mt
h thc tng t cng c i vi t din ABCD c 3 cnh AB, AC, AD vung gc i mt.
K hiu SA, SB, SC, SD l din tch BCD, ACD, ABD, ABC th t th:
2
SA2 = SB2 + SC2 + SD
(A)
Tht vy, t b = AB, c = AC, d = AD(hnh 6).
1
2
Ta c SB2 + SC2 + SD
= (c2 d2 + d2 b2 + b2 c2 ) (1)
4
Mt khc, BC 2 = b2 + c2 , CD2 = c2 + d2 , BD2 = b2 + d2 .


1
\ = 1 BC 2 .BD2 . 1 cos2 CBD
\
SA2 = BC 2 .BD2 .sin2 CBD
4
4
"
#
2
2
2 2
1
(BC
+
BD

CD
)
SA2 = BC 2 .BD2 . 1
4
4.BC 2 .BD2
1
SA2 = [(b2 + c2 ) (b2 + d2 ) b4 ]
4
1
2
SA = (b2 d2 + b2 c2 + c2 d2 ) (2).
4
T (1) v (2) suy ra (A) c chng minh.
Bi ton 4 (Bi ton 4.a.). Cho ABC vung ti A, c BC = a, AB = c, AC = b; ng cao
AH = h. Chng minh rng
169


2+1 .

2) b + c 2h 2.
K hiu P: l chu vi ABC
Gii.
p
1) Ta c P = a + b + c a + 2 (b2 + c2 ) (bt ng thc Bunhiacopsky)


P a + a 2 hay P a 1 + 2 .
Du = xy ra b = c ABC vung cn ti A.

2) Ta c b + c 2 bc (*)

1
1
1
2
bc 2h2 (**) T (*) v (**) suy ra b + c 2h 2
Mt khc, 2 = 2 + 2
h
b
c
bc
Du = xy ra b = c ABCvung cn ti A.
1) P a

Nhn xt 1. Trong chng minh trn, ta s dng 2 h thc trong tam gic vung: a2 = b2 + c2
1
1
1
v 2 = 2 + 2 .
h
b
c
S dng h thc tng t i vi t din c 3 cnh xut pht t mt nh vung gc i mt,
vi s tng ng gia chu vi tam gic l din tch ton phn Stp ca t din, ta c bi ton sau:
c) M rng sang t din.
Bi ton 5 (Bi ton 4.b.). Cho t din ABCD c AB, AC, AD i mt vung gc; h l di
ng cao t din k t A; SA, SB, SC, SD l din tch cc mt i din nh A, B, C, D tng
ng. Chng minh rng


1) Stp SA 3 + 1
9
2) SB + SC + SD h2
2
Gii tm tt:
p
2
1) Stp = SA + SB + SC + SD SA + 3 (SB2 + SC2 + SD
)
S dng h thc (A) ta c bt ng thc cn chng minh.
Du = xy ra SB = SC = SD AB = AC = AD
1
2) SB + SC + SD = (cd + bd + bc) (vi AB = b, AC = c, AD = d).
2
3
3 2 2 2
SB + SC + SD
b c d (bt ng thc Cauchy) (a)
2
r

1
1
1
1
1
3
Li c: 2 = 2 + 2 + 2 3 3 2 2 2 b2 c2 d2 3h2 (b)
h
b
c
d
bcd
9
T (a) v (b) ta c SB + SC + SD h2
2
Du = xy ra b = c = d.

Kt lun

Trn y chng ti nu ln cc bi ton trong tam gic c th m rng sang t din nh


vo php tng t trong chng minh. S lng cc bi ton nu ra khng nhiu v cng cha a
dng, tuy nhin qua cng nh cho cc em hc sinh mt hng tm ti mt kt qu tng
t t bi ton tam gic sang t din. Cc bi ton loi ny chc chn l cn nhiu, chng ti s
tip tc nghin cu trong cc nm sau.

170

Mt s dng ton v dy s
trong t hp
Mai c Thanh
Trng THPT Chuyn Nguyn Du, k Lk

Mt s dng ton tnh ton trong t hp

Bi ton 1. Mt tp hp c n phn t th c bao nhiu tp con?


Gii. Xt cc tp An = {a1 ; a2 ; . . . ; an }. t xn = |An |.
D thy A0 = ; x0 = 1.
Vi mi tp con ca A ca An+1 = {a1 ; a2 ; . . . ; an ; an+1 }, ta c:
Nu A khng cha an+1 th A l tp con ca An = {a1 ; a2 ; . . . ; an }, s cc tp A nh vy l
xn .
Nu A cha an+1 th B = A\ {an+1 } l tp con ca An , s cc tp A nh vy cng l xn .
(
x0 = 1
Do ta c cng thc dy (xn ) :
xn = 2n .
xn+1 = 2xn
Vy mt tp hp c n phn t th c 2n tp con.
Bi ton 2. C bao nhiu cch xp n nam v n n thnh 2 hng ngang quay mt vo nhau sao
cho i din vi mi nam l n?
Gii. Gi un l kt qu ca bi ton.
D thy u1 = 2.
Khi xp n + 1 nam v n + 1 n ln s sau theo yu cu ra :

(1a) (2a) (3a) . . . (na) ((n + 1) a)


(1b) (2b) (3b) . . . (nb) ((n + 1) b)

V tr ((n + 1) a) c 2 (n + 1) cch chn ngi.


V tr ((n + 1) b) c n + 1 cch chn ngi (khc gii vi ngi trn)
Cc v tr cn li c un cch xp.
Do ta c dy:

(
u1 = 2
u1 = 2
un+1
(un ) :
(un ) :
= un

un+1 = 2(n + 1)2 un


2(n + 1)2
u1

=1
2.1! u
un

n+1

2 = n
n+1
2 ((n + 1)!)
2 ((n)!)2
un
2
n
Dy (vn ) :vn =
2 l dy hng bng 1 nn (un ) :un = 2 ((n)!)
n
2 ((n)!)

171

Bi ton 3. Vi n ng thng chia mt phng nhiu nht l bao nhiu min?


Gii. Vi mt s ng thng c th t bt k trong mt phng.
Gi un l s min m n ng thng u to ra.
D thy u1 = 2.
Vi n + 1 ng thng u ta c:
Vi n ng thng u chia mt phng c un min.
ng thng th n + 1 ct n ng thng nhiu nht ti n giao im phn bit. Ngha
l trn ng thng ny c chia nhiu nht n + 1 khong. M mi khong ny chia mt min
c thnh hai min mi, tc l mi khong lm tng thm mt min so vi s lng c.
Do ta c cng thc:
(
u1 = 2
(un ) :
un+1 un n + 1
Ta c:
un = u1 +

n1
X
k=1

(uk+1 uk ) 2 +

n1
X

(k + 1) = 2 +

k=1

n2 + n + 2
n(n 1)
+n1=
2
2

Vy vi n ng thng chia mt phng nhiu nht l


n2 + n + 2
2
min khi cc ng thng i mt ct nhau v khng c ba ng no ng quy.
Bi ton 4. Cho hai s t nhin m, n tha mn 1 m n. Phng trnh x1 + x2 + + xm = n
c bao nhiu nghim nguyn dng?
Gii. Gi u (m; n) l kt qu ca bi ton.
D thy u (1; n) = u(n; n) = 1.
Vi mi nghim (x1 ; x2 ; . . . ; xm ) ca phng trnh x1 + x2 + + xm = n khi 2 m n, ta
c:
Nu xm = 1 th (x1 ; x2 ; . . . ; xm1 ) l nghim phng trnh
x1 + x2 + + xm1 = n 1.
Do , ta c u (m 1; n 1) nghim (x1 ; x2 ; . . . ; xm ) nh vy.
Nu xm > 1 th (x1 ; x2 ; . . . ; xm1 ; xm 1) l nghim phng trnh x1 + x2 + + xm = n 1.
Do , ta c u (m; n 1) nghim (x1 ; x2 ; . . . ; xm ) nh vy.
T , ta c cng thc:
(
u (1; n) = u(n; n) = 1
u(m; n) = u(m 1; n 1) + u(m; n 1); 2 m n
t v (m 1; n 1) = u (m; n), c cng thc quen thuc:
(
v (0; n) = v(n; n) = 1
m1
v (m; n) = Cnm u (m; n) = Cn1
v(m; n) = v(m 1; n 1) + v(m; n 1)
m1
Vy phng trnh x1 + x2 + + xm = n c Cn1
nghim nguyn dng.

172

Bi ton 5. Cho A = {1; 2; 3; . . . ; n}. C bao nhiu song nh f : A A tha mn f (f (x)) =


x; x A?
Gii. Bi ton khng thay i nu thay A l tp c n phn t. t li An = {1; 2; 3; . . . ; n}.
Gi un l kt qu ca bi ton.
D thy u1 = 1; u2 = 2.
Vi mi nh x f : An+2 An+2 tha f (f (x)) = x; x An+2 , ta c:
Nu f (n + 2) = n + 2 th f |An+1 : An+1 An+1 tho
f (f (x)) = x; x An+1 nn c un+1 nh x f nh vy.
Nu f (n + 2) 6= n + 2 th:
f (n + 2) = x 6= n + 2 f (x) = n + 2. x c (n+1) cch chn.
t B = An+2 \ {n + 2; x}. f |B : B B tho
f (f (x)) = x; x B nn c un cch chn f |B .
Vy c (n + 1) un nh x f nh vy.
T
( ta c cng thc:
u1 = 1; u2 = 2
un+2 = un+1 + (n + 1) .un

Vy p s bi ton l un c xc nh theo cng thc trn.

Bi tp tng t

Bi tp 1. Tm s nghim nguyn dng ca cc phng trnh:


a) x + 2y + 3z = n
b) 2x + 3y + 5z = n
Bi tp 2. Cho n im trn mt ng trn. C bao nhu cch t mu n im bi 4 mu
xanh, , vng, tm sao cho 2 im k nhau c t khc mu.
Bi tp 3. Cho n im trn mt ng trn. C bao nhiu cch ly ra m im sao cho khng c
hai im no c chn ng cnh nhau.
Bi tp 4. C bao nhiu hon v ca tp An = {1; 2; 3; . . . ; n} sao cho hai s lin tip trong hon
v c hiu ln hn 1.

173

Mt s bi ton gii bng phng


php m theo hai cch
Lng Ngc Tin
THPT Nguyn Tri, TX. Ninh Ha, Khnh Ha
Bi ton t hp xut hin trong nhiu lnh vc ca ton hc thun ty, c bit l trong i
s, l thuyt xc sut, t p, hnh hc, l thuyt ti u, khoa hc my tnh,. . . . Trong ton s cp,
cc bi ton v t hp ng mt vai tr quan trng, n c mt hu ht cc k thi: Hc sinh gii
cc cp, v ch Ton cc nc, Olympic Ton Quc T,. . . .. Bi vit ny chng ti xin gii thiu
mt cch tip cn ca mt phng php thng c s dng trong ton t hp l phng php
m theo hai cch.

M u

m theo hai cch l g? m theo hai cch l mt phng php chng minh ph bin trong
T hp, n chng minh hai biu thc bng nhau bng cch ch ra rng hai biu thc l kt
qu ca hai cch m khc nhau ca cng mt tp hp.
m theo hai cch l mt trong cc k thut chng minh ph bin c s dng rt nhiu
trong l thuyt t hp, th, s hc, i s,. . . .Ni mt cch nm na, m theo hai cch l vic
xem xt mt ma trn thch hp sau ta i tnh tng s cc phn t ca ma trn theo hai
cch khc nhau. u tin n bng tng theo dng ca tng cc phn t trn cng mt dng v
th hai n bng tng theo ct ca tng cc phn t trn cng mt ct.
Phng php m theo hai cch khng ch c s dng chng minh cc ng thc t hp
m cn c th c s dng trong cc bi ton v bt ng thc t hp, cc tr t hp, tn ti
hai khng tn ti,. . . .. Rt nhiu kt qu nn tng ca ton hc l h qu ca m theo 2 cch
nh: B bt tay (Handshaking Lemma), Tnh nhn ca hm Euler,. . . . . .
iu quan trng nht ca phng php m theo hai cch l chng ta phi xc nh ng ta
cn phi m ci g ?
Trong bi vit ny, thng qua mt s bi ton c th, chng ti gii thiu mt s cc k thut
s dng phng php m theo 2 cch gii quyt mt s kiu bi ton sau:
1. Chng minh mt ng thc t hp
2. Chng minh bt ng thc t hp
3. Bi ton cc tr t hp
4. Bi ton tn ti hay khng tn ti
c bit trong bi vit, chng ti s dng k thut m theo hai cch thng qua ma trn lin
thuc vo rt nhiu bi ton. y l mt k thut hay gip gii quyt nhiu bi ton t hp kh,
cho chng ta nhiu li gii p.

Cc kt qu b tr cho phng php m theo hai cch

 
n
Mnh 1. S tp con gm k phn t ca tp hp c n phn t l:
.
k
Mnh 2. Qui tc cng: Nu A1 , A2 , . . . An vi Ai Aj = , i, j {1, 2, . . . , n}, i 6= j, th
|A1 A2 An | = |A1 | + |A2 | + + |An |.
174

Mnh 3. Nu A1 , A2 , . . . , An l cc tp hp hu hn bt k v A1 A2 An l tch
cc ca n tp hp th |A1 A2 An | = |A1 | |A2 | |An | .
Mnh 4. Cho A = (aij )l ma trn c m n. Gi Ri l tng cc phn t dng th i, Cj l
m
n
P
P
tng cc phn t ct th j. Khi :
Ri =
Cj .
i=1

j=1

Mnh 5. ChoA l ma trn c m n cc phn t 0 v 1 ty . Cj l tng cc phn t ct


th j. Gi s rng 2 dng bt k c ng t ct cha s 1 c hai dng. Khi :
  X

n 
m
Cj
t
=
.
2
2
j=1
Chng minh. Gi T l tp tt c cc cp s 1 nm trn cng mt ct. Ta s m s phn t
ca T theo 2 cch:

m theo dng: 2 dng bt k cho ta t cp s 1, t suy ra c tt c t m2 cp s 1.
m theo ct: Cj l tng cc phn t ct th j nn suy ra c Cj s 1 trn ct th j ngha l
n


P
Cj
s c C2j cp s 1. V c tt c n ct nn c
cp s 1.
2
j=1

Mnh 6. Cho A = (aij ) l ma trn c m n. Gi Ri l tng cc phn t dng th i, Cj l


m P
n a
P
ij
= m.
tng cc phn t ct th j. Khi nu Ri > 0 vi mi 1 i mth:
i=1 j=1 Ri
Tng t, nu Cj > 0 vi mi 1 j n th:
m P
n a
m
n P
m a
P
P
P
ij
ij
= n. Chng minh. Ta c
=
i=1 j=1 Ri
i=1
j=1 i=1 Cj

n
1 P
aij
Ri j=1

!
=

m
P
i=1

1
Ri
Ri


=

m
P

1 = m.

i=1

Mnh 7. Cho A = (aij )l ma trn c m n cc phn t 0, 1.Ri l tng cc phn t dng


th i, Cj l tng cc phn t ct th j sao cho Ri > 0 vi mi 1 i m, 1 j n. Nu
Cj Ri khi aij = 1, th m n.
Chng minh. Khi aij = 1,
1
1
aij
aij
Cj Ri

vi mi 1 i m, 1 j n. T kt qu mc 1.6, ta c:
Ri
Cj
Ri
Cj
m P
n a
n P
m a
P
P
ij
ij
m=

= n.
R
C
i
i=1 j=1
j=1 i=1 j

3
3.1

Mt s bi ton s dng phng php m theo hai


cch
Chng minh ng thc t hp

 
n
Bi ton 1. (Hng ng thc Pascal) Chng minh rng, vi n 2, 1 k n. Ta c
=
k

 

n1
n1
+
. Nhn xt. V tri ca ng thc chnh l s tp con gm k phn t ca tp
k
k1
c n phn t, iu ny gi cho chng ta i m s tp con gm k phn t theo 2 cch khc
nhau. Xt tp X = {x1 , x2 , . . . ., xn }, mt tp con gm k phn t ca X s gm 2 loi: khng cha
phn t xi no v c cha phn t xi . Ta s gii bi ton c th nh sau:

175

Gii. Xt tp hp X = {x1 , x2 , . . . ., xn }. Ta sm


 s tp con gm k phn t ca X bng 2
n
cch Cch 1: S tp con gm k phn t ca X l
(1) Cch 2: Tp con gm k phn t ca X
k
s gm 2 loi: - Loi 1: Khng cha phn t x1 Tp con gm
 k phn
 t ca X khng chax1 phi
n1
c ly trong tp X\ {x1 }. Do s tp con cn tm l
. - Loi 2: C cha phn t x1
k
chn 1 tp con gm k phn t ca X c cha
 phn
 t x1 , ta ch vic chn
 thm
 k 1 phn
 t
n1
n1
n1
t tp X\ {x1 }, do s tp con cn tm l
. Vy X c tt c
+
tp
k
k1
 k 1
 

n
n1
n1
con c k phn t. (2) T (1) v (2) ta c:
=
+
k
k
k1
Bi ton 2.(ng thc Vandermode)
Cho
 
 m, n, r l cc s t nhin tha r min {m, n} Chng
r
P
m
n
m+n
. Nhn xt. V tri ca ng thc chnh l s tp con
minh rng:
=
k
rk
r
k=0
gm r phn t ca tp c m + n phn t, iu ny gi cho chng ta i m s tp con gm r
phn t theo 2 cch khc nhau. Nhn vo v phi ta li thy c s xut hin ca tp con gm k
phn t ca tp m phn t v tp con gm r - k phn t ca tp n phn t. iu ny gi cho
chng ta xy dng 1 tp con gm r phn t m n c k phn t t tp m phn t v r - k phn
t t tp c n phn t.
Gii. V tri ca ng thc chnh l s tp con gm r phn t ca tp A c m + n phn t. Ta
s m stp conc r phn t ca A bng 2 cch nh sau: Cch 1: V tp A c m + n phn t
m+n
nn A c
tp con gm r phn t. Cch 2: Ta chia tp A thnh 2 tp ri nhau A1 v A2
r
tng ng c s phn t l m v n. T A1 
ta chn
phn t, t A2 ta chn ra r - k phn t.
  ra k 
m
n
Khi s cch chn r phn t t tp A l:
. Cho k chy t 0 n r ta c tng s
k
r k


r
P
n
m
cch chn tp con gm r phn t t tp A l:
.
k
rk
k=0
Tng qut ca bi ton trn: Chon, x, y, ki (i= 1, y) l cc s t nhin. Khi ta c ng
  
 


n
x
P
(y + 1)n
n
n
n
y
P

thc
....
kj =
x
k2
ky
x
k1 ,k2 ,..,ky =0 k1
j=1

Bi ton 3. Chng minh rng

n
P
k=0

k2 =

n(n + 1)(2n + 1)
.
6

Nhn xt 1. Tng trn c th chng minh bng cc phng php khc n gin hn, sau y
chng ti xin nu ra mt cch chng minh s dng phng php m theo 2 cch.
Gii. Ta xt bi ton sau
C n + 1 ngi (khc nhau v tui) mun xem mt bui biu din. C 3 tm v vo nh ht,
gm 1 v loi 1 v 2 v loi 2 (phn bit). Mun phn phi v cho 3 ngi theo qui tc sau:
(1) V loi 1 phi cho ngi ln tui nht (trong s cc ngi c phn phi v).
(2) Ngi nhn c v loi 1 khng c nhn mt trong hai v loi 2, nhng 2 v loi 2 c
th phn phi cho cng 1 ngi.
Ta s m s cch phn phi v cho 3 ngi theo 2 cch:
Cch 1. Phn 3 v cho 3 ngi hoc phn 3 v cho 2 ngi

176

- Phn 3 v cho 3 ngi: V ngi ln nht s nhn v loi 1, 2 v cn li cho 2 ngi cn


 li (c

n+1
2 cch phn 2 v loi 2 cho 2 ngi), nn s cch phn phi trong trng hp ny l: 2
.
3
- Phn 3 v cho 2 ngi (v loi 
1 cho ngi
ln tui, 2 v loi 2 cho ngi cn li): S cch

n+1
phn phi trong trng hp ny l:
.
2

 

n(n + 1)(2n + 1)
n+1
n+1
Vy tng s cch phn phi v tha yu cu bi ton l: 2
+
=
.
3
2
6
(1)
Cch 2: Ta sp xp n + 1 ngi theo th t tng dn v tui: a1 , a2 , . . . . . . , an+1 . Ta s phn
phi v theo cch sau:
- Phn v loi 1 cho a1 , d thy khng th lm c. Vy c 0 0 cch.
- Phn v loi 1 cho a2 , khi c 1 cch phn phi 2 v loi 2 cho a1 . Vy c 1 1 cch.
- Phn v loi 1 cho a3 , khi c A22 + 2 = 2 2 cch phn phi 2 v loi 2 cho a1 , a2 . Vy c
2 2 cch
- Phn v loi 1 cho a4 , khi c A23 + 3 = 3 3 cch phn phi 2 v loi 2 cho a1 , a2 . Vy c
3 3 cch
- .....................................
- Phn v loi 1 cho an+1 , khi c A2n +n = nn cch phn phi 2 v loi 2 cho a1 , . . . , an Vy
c n n cch.
n
P
Tng s cch trong trng hp ny l: 12 + 22 + 32 + + n2 =
k 2 (2).
k=0

T (1) v (2) ta c:
 


n
n+1
Bi ton 4. Chng minh rng:(n + 1)
= (k + 1)
Nhn xt. Khc vi cc bi ton
k  
k+1
n
trc, v tri ca ng thc l tch (n + 1)v
, n chnh l s cch thc thc hin 1 cng vic
k
 
n
no gm 2 cng on: cng on 1 gm (n + 1)cch thc hin, cng on 2 gm
cch
k
thc hin. T y gi n suy ngh xt cp (x, A), vi x thuc tp gm (n + 1)phn t v A l
tp con gm k phn t ca tp c n phn t.
Gii. Ta xt tp X = {1, 2, 3 . . . ., n + 1}, x l mt phn t no ca X. Ta s m cc cp
(x, A)(khng phn bit th
 t),
 trong A l mt tp con c k phn tca
 X\ {x}. Cch 1: Ta
n
n
c n + 1 cch chn x v
cch chn tp A, do c tt c (n + 1)
cp (x, A). Cch 2:
k
k
Ta ly tp con A ca X c k + 1 
phn t,
 sau t A ta chn ra mt phn t x. Khi ta c
n+1
cp (x, A) vi x X. Ta thy c
cch ly tp A v k + 1 cch ly x. Do s cch chn
k+1


n+1
cp (x, A) l (k + 1)
.
k+1
Bi ton 5. (IMO - 1987 ) Gi pn (k) l s cc hon v ca tp A = {1, 2, . . . , n} , n 1, c ng
n
P
k im bt ng (tc l pn (i) = i). Chng minh rng:
k.pn (k) = n!.
k=0

Nhn xt 2. pn (k) l s cc hon v ca tp A c ng k im bt ng, do k.pn (k)gi cho


chng ta i m s cp (x, p) vi p l mt hon v ca A v x l mt phn t bt ng ca p.

177

Gii. Ta s m s cc cp(x, p) vi p l mt hon v ca A v x l mt phn t bt ng ca p


Cch 1: Vi mi phn t x ta c (n 1)! hon v ca A nhn x lm im bt ng, do c
tt c n(n 1)! = n! cp (x, p).
Cch 2: Mi hon v p ca A c ng k im c nh th c k cp (x, p), do tng s cp
n
P
k.pn (k).
(x, p) l:
k=0

Trong cc bi ton t 1 n 5, ta thy rng chng minh mt ng thc t hp, ta thng


da vo cc thng tin trong ng thc cn chng minh suy lun ngc, t xc nh c
mc tiu l chng ta phi m ci g.
Sau y l mt s bi ton t hp khng phi di dng ng thc tng minh.
Bi ton 6. Trong mt cu lc b, mi thnh vin thuc vo ng 3 t chuyn mn. Mi t
chuyn mn c ng 3 thnh vin. Chng minh rng s cc thnh vin bng s cc t chuyn
mn.
Nhn xt 3. Bi ton trn l mt v d in hnh cho k thut dng ma trn lin thuc nh
chng ti ni trong phn u ca bi vit. Ta gi s cu lc b c m thnh vin v n t chuyn
mn, nu ta sp xp cc thnh vin theo hng dc t trn xung, sp xp cc t chuyn mn theo
hng ngang t tri qua, ta s thu c 1 bng hnh vung v ta gi l 1 ma trn c m dng
v n ct.
Vn na ca k thut ny l khi xy dng mt ma trn th cc phn t ca n s cho ttheo
qui lut no. Thng th nhng bi ton dng ny ta s dng s 0, 1 m t cc mi quan h
ca bi ton, v khi gii quyt bi ton ta ch da trn tnh cht phn b ca 0 v 1.
Trong bi ton 6, ta gi Cj l cc t chuyn mn, Ri l cc thnh vin. Ta xy dng ma trn
nh sau: nu thnh vin i thuc vo t chuyn mn j th ta nh ngha aij = 1 v 0 cho trng
hp ngc li.
C1 C2 C3

R1 1 1 0
R2
0 1 1
R3
1 1 1
. . . .
Rm 1 0 1

. . Cn

. . 1
. . 1

. . 0

. . 1
. 1 0

Gii. Gi m, n ln lt l s thnh vin v s t chuyn mn ca cu lc b. Ta xt bng (ma


trn) m n, mi dng biu din mt thnh vin v mi ct biu din mt t chuyn mn. Nu
ngi th i thuc vo t chuyn mn j , ta t s 1, nu khng t s 0. V mi thnh vin thuc
ng v 3 t chuyn mn nn mi dng c ng 3 s 1, mi t chuyn mn c ng 3 thnh vin
nn mi ct c ng 3 s 1. T suy ra Ri = 3 vi mi i v Cj = 3 vi mi j.
Bi ton 7. Gi F(S) l h cc tp con ca tp hu
P hn S, vi
P mi x S ta k hiu d(x) l s
cc tp con thuc F(S)cha x. Chng minh rng:
d(x) =
|A|.
xS

AF(S)

Gii. Ta xt ma trn gm |S| dng v |F(S)| ct, trong cc phn t ca ma trn c cho
nh sau. Ta nh ngha phn t ca ma trn nh sau:
aij = 1 nu phn t xi ca S thuc vo tp Aj ca F(S).
aij = 0 nu phn t xi ca S khng thuc vo tp Aj ca F(S).

178

A1 A2

x1 1 0
x2
0 1
.
1 1
. . .
xm 1 0

. . . An

1 . . 1
0 . . 1

0 . . 0

. . . .
1 . . 1

Ta thy d(x) chnh l s cc s 1 dng th i, |A| l s cc s 1 ct th j.


Bi ton 8. 15 hc sinh tham gia kha hc ma h. Mi ngy, 3 hc sinh phi trc nht sau gi
hc. Sau kha hc, mi cp hc sinh u trc nht ng 1 ln. Hi kha hc ko di bao nhiu
ngy ?
Nhn xt. Rt t nhin, ta s gi s ngy
 ca kha hc l s k no . V mi ngy c 3 hc
3
sinh trc nn ton b kha hc s c k
cp hc sinh tham gia trc nht. Mt khc c tng
2


15
cng 15 hc sinh nn s c
cp hc sinh. T y ta dn n kt qu bi ton.
2
Gii.
Gi k l s ngy ca kha hc. Ta s m 
s cphc sinh theo 2 cch.
15
Cch 1: Lp hc c 15 hc sinh nn s c
cp phi tham gia trc nht. Cch 2: Mi
2
ngyc3 hc sinh tham gia trc nht nn
 tng
 s
 cp
 hc sinh tham gia trc nht ca c kha
3
3
15
l k
. T 2 cch m trn ta c: k
=
3k = 105 k = 35
2
2
2
Bi ton trn c th tng qut nh sau: Mt kha hc c m hc sinh. Mi ngy c n hc sinh
tham gia trc nht n m. Sau kha hc ngi ta thy mi cp hc sinh u trc nht ng k
ln. Hi kha hc ko di bo nhiu ngy.

Chng minh bt ng thc t hp, tm cc tr t hp

bt u, chng ta xt v d n gin sau:


Cho A, B l 2 tp hp hu hn bt k c s phn t tng ng l m, n, gi p l s phn t
ca A B.
1. Nu chng ri nhau (hnh 1), ta c ngay m + n = p. ng thc thu c chng qua l h
qu ca vic chng ta m s phn t ca A B theo hai cch.
2. Ta t ra cu hi kh t nhin l: nu A v B khng ri nhau (hnh 2) th ng thc trn
c cn ng khng? Ta s tr li cu hi trn bng phng php m.
Trc ht ta thy m + n l tng cc phn t ca A v B. Mt khc v A B 6= nn chng
c t nht 1 phn t chung. iu ny c ngha l A Bphi c s phn t t hn s phn t ca
A v B cng li, do ta c m + n > p.
Qua v d trn ta thy phng php m theo hai cch c th c s dng vo cc bi ton
v bt ng thc t hp, cc tr t hp. Rt nhiu bi ton v bt ng thc t hp, cc tr t
hp c gii quyt nh vo phng php m theo hai cch. Sau y l mt s bi tp minh
ha.
179

Bi ton 9. Cho cc s nguyn dng m,n m n > 1. Cho F1 , F2 , . . . , Fk l cc tp con gm n


phn t ca {1, 2, 3, . . . ., m} sao cho Fi Fj c nhiu nht 1 phn t, 1 i < j k. Chng minh
m(m 1)
rng: k
.
n(n 1)
 
m
Gii. Gi S l tp tt c cc tp con c 2 phn t ca {1, 2, 3, . . . ., m}. Khi |S| =
. Mi
2
 
m
tp Fi gm c n phn t nn c ng
phn t ca S l tp con ca Fi . V |Fi Fj | 1 nn
2
khng c phn t no ca S l tp con ca c 2 tp Fi v Fj . iu ny c ngha l
   
n
m
k

2
2
Bi ton 10. (IMO - 1989) Cho n v k l cc s nguyn dng, S l tp n im trn mt phng
tha mn iu kin:
(1) Khng c 3 im no ca S thng hng.
(2) Vi mi im P ca S, c t nht k im ca S m khong cch n P bng nhau.
1
Chng minh rng: k < + 2n (1).
2
k(k 1) 1
+ . V n, k l nhng s
2
8
 
k(k 1)
k
n1
nguyn dng nn bt ng thc tng ng vi: n 1
(2).
2
2
 
n
t S = {A1 , A2 , . . . , An }. C
on thng c to thnh t tp S. (3)
2
Mt khc, c mi im ca S, c t nht k im cch u n. Chng hn t A1 ta v 1 ng
trn c tm l A1 v bn knh bng khong cch t A1 n mt trong k im cch u n. Khi
c t nht
 
k
2
Gii. Bt ng thc cn chng minh c vit li vi n >

dy cung (mi dy cung l 1 on thng c to tnh t tp S)


Tip tc thc hin cho cc im cn li ta s c t nht
 
k
n
2
dy cung.
  Nhng
  v 2 ng trn c nhiu nht 1 dy cung chung nn s dy cung t nht c th
k
n
l: n

(4) T (3) v (4) ta suy ra:


2
2
     
 
k
n
n
k
n

n1
2
2
2
2
Bi ton 11. Trong mt k thi c 11 hc sinh tham gia gii 9 bi ton. Hai th sinh bt k, c
khng qu 1 bi m c hai ngi cng lm c. Gi s rng k l s nguyn dng tha mn mi
bi ton u c t nht k th sinh gii c. Chng minh rng k 3.
Gii. Ta k hiu S l b tt c cc cp th sinh m gii c cng mt bi ton no . Theo
11
gi thit 2 hc sinh bt k lm c khng qu 1 bi ton. V th |S|
.
2

180

By gi ta i m theo bi ton. C nh i {1,


 2, . . . , 9}. Ta k hiu xi l s ngi gii c
xi
bi ton th i. i vi bi ton th i c ng
cp hc sinh gii c bi ny. Do :
2
 
9
P
xi
|S| =
.
2
i=1
   
9
9
P
P
xi
11
Do ta c nh gi:

(x2i xi ) 110.
2
2
i=1
i=1
Gi s rng k l s tha mn yu cu bi ton. Khi xi k. Do ta c 9(k 2 k) 110v
ta suy ra c: k 4. Ta s ch ra k = 4 khng tha mn. Gi s k = 4 , nu tn ti i m xi 5
th:
9
P
(x2i xi ) 52 5 + 8 (42 4) > 110, mu thun.
i=1
 
11
Vy xi = 4 vi mi i. Khi |S| = 54 = n 1
1.
2
Do tn ti ng 1 th sinh khng gii c bi no. Vy nn k 3.
Bi ton 12. Mt cuc thi Olympic Ton c tng cng m bi ton, c 8 hc sinh tham gia cuc
thi. Mi bi ton c lm bi 4 hc sinh v mi cp hc sinh s lm chung s bi ging nhau.
Xc nh gi tr nh nht ca m.
 
4
Gii. Mi bi ton c lm bi 4 hc sinh do mi bi ton s c
cp hc sinh tham
2
gia
bi ton.
V tng cng c 8 hc sinh nn c tt c
 gii. gi s mi cp hc sinh lm chungk 
 
8
4
8
cp hc sinh. Vy ta c ng thc: m
=k
28k = 6m
2
2
2
T y ta suy ra 14 l c ca m hay m 14. Ta k hiu cc hc sinh theo th t t 1 n
8. Khi vi m = 14, ta c m t nh sau:

{1, 2, 3, 4} , {3, 4, 5, 6} , {5, 6, 7, 8} , {1, 2, 7, 8} , {1, 2, 5, 6} , {2, 3, 6, 7} , {3, 4, 7, 8}


{1, 4, 5, 8} , {1, 3, 5, 7} , {2, 4, 6, 8} , {1, 3, 6, 8} , {2, 3, 5, 8} , {2, 4, 5, 7} , {1, 4, 6, 7}
Vy gi tr nh nht ca m l 14.

Chng minh tn ti, khng tn ti

Ngoi vic c s dng trong cc bi ton chng minh ng thc t hp, bt ng thc t
hp, cc tr t hp, . . . Phng php m theo hai cch cn c s dng trong cc bi ton v
chng minh tn ti, khng tn ti. Sau y l mt s bi tp minh ha.
Bi ton 13. C s d liu tp ch ca th vin Quc Gia c ng 2016 loi khc nhau. Th
vin ny cho php 2013 th vin i phng kt ni khai thc c s d liu tp ch ca n. Bit
rng mi th vin a phng c php khai thc 1008 loi tp ch khc nhau v 2 th vin a
phng bt k th ch c c ti a 504 loi tp ch m c 2 th vin a phng cng c
php khai thc. Chng minh rng khng c qu mt loi tp ch trong c s d liu ca th vin
Quc Gia m c 2013 th vin a phng u khng th khai thc c.
Gii. Ta k hiu X1 , X2 , . . . ., X2013 ln lt S
l cc u tp ch m th vin con th nht, th hai,
. . . , th 2013 c th truy cp vo. t X = i = 12013 Xi = {x1 , x2 , . . . ., xn }. khi |Xi | 1008
181

v |Xi Xj | 504 vi mi i 6= j. Vi mi i = 1, 2, . . . , n ta k hiu di l s cc th vin cng mua


u tp ch xi . m s b (tp ch, th vin a phng mua tp ch ) theo hai cch ta c:
2013
P
t = d1 + d2 + + dn =
|Xi | 2013 1008. Mt khc ta k hiu S l h cc b mt tp ch xi
i=1
2

t
 
t
n
P
di
v hai th vin con cng mua tp ch . m theo tp ch cho ta: |S| =
n
. m
2
2
i=1


P
2013
theo cp th vin con cho ta: |S| =
|Xi Xj | 504
. Thnh ra ta nhn c:
2
1i,j2013


2013 1008
2013 1008
2013 1008
2013 2012

1 n
504
2
2
n
1007
Vy n > 2014 hay n 2015.
Nhn xt 4. Bi ton trn c ngun gc t bi ton v tp hp sau
n
K hiu A1 , A2 , , , , Ak l cc tp con ca tp hp {1, 2, . . . , n}, mi tp c t nht phn t
2
S
n
kn
n
v vi 2 tp bt k c chung nhau khng qu phn t. Chng minh rng: | i = 1 Ai |
.
4
k+1
Bi ton 14. Cho bng hnh vung gm 3 dng v 9 ct. Mi vung c t mu hoc mu
xanh. Chng minh rng tn ti 2 dng (khng nht thin lin tip) v 2 ct (khng nht thit
lin tip) sao cho chng giao nhau cho ra 4 hnh vung cng mu.
Gii. Ta s chng minh phn chng bi ton trn. Gi s rng khng tn ti 2 dng v 2 ct
m cc hnh vung giao nhau khng cng mu. K hiu I = {ri , rk }l cp 2 dng bt k, j l ct
ca bng hnh vung. Ta k hiuX (I, j) l tp cc hnh vung to thnh t ct j v 2 dng ri , rk .
Ta s m s cc hnh vung c mu ging nhau ca X (I, j). m theo ct: V mi ct c 3 hnh
vung v c 2 mu c t nn mi ct c t nht 2 hnh vung pha cng mu. V c 9 ct nn ta
suy ra: |X (I, j)| 9 (1). m theo dng: Theo gi thit phn chng, ta suy ra mi cp 2 dng c
nhiu nht 1 cp hnh vung (trn cng 1 ct) cng mu v nhiu nht 1 cp hnh vung (trn
cng 1 ct ) c mu xanh. Hay ta c khng qu 2 cp hnh vung cng mu i vi mi cp 2
dng. T y suy ra: |X (I, j)| 2 3 = 6 (2). T (1) v (2) ta c: |X (I, j)| 6 < 9 |X (I, j)|,
mu thun.
Bi ton 15. 17 hc sinh tham gia cuc thi gii ton gm c 9 bi ton. Bit rng mi bi ton
c ng 11 hc sinh gii ng. Chng minh rng c 2 hc sinh cng gii ng c 9 bi ton.
Gii. Gi s rng khng tn ti 2 hc sinh cng gii c 9 bi ton. Ta xt ma trn gm 17
dng v 9 ct, cc ct ca ma trn l P1 , P2 , . . . , P9 tng ng vi 9 bi ton, cc dng ca ma
trn l h1 , h2 , . . . ., h17 tng ng vi 17 hc sinh. Gi aij l phn t ca ct i v dng j. Ta nh
ngha aij nh sau: aij = 1 nu hc sinh hi gii c bi ton Pj ; aij = 0 nu hc sinh hi khng
gii c bi ton Pj . Ta s m s cp s 0 trn mi ct ca ma trn. Cch
  1: V mi hc sinh
6
gii ng c 9 bi ton nn mi ct c ng 6 s 0, do c tt c 9
= 135cp s 0 trn
2
mi ct.
Bi ton 16. (IMC - 2002) 200 hc sinh cng tham d mt cuc thi Olympic Ton. Cuc thi c
6 bi ton. Sau khi kt thc cuc thi, ngi ta thy rng mi bi ton u c t nht 120 hc sinh
lm ng. Chng minh rng c t nht 2 hc sinh sao cho mi bi ton th t nht mt trong hai
ngi lm ng.

182

Gii. Ta xt ma trn gm 6 dng (mi dng xem nh 1 bi ton) v 200 ct (mi ct xem nh
1 hc sinh). Ta nh ngha cc phn t ca ma trn nh sau: Vi bi ton th j, nu hc sinh th
i lm ng th phn t tng ng vi dng i v ct j ta ghi l 1, v ghi 0 cho trng hp ngc
li. Gi S l tp cc cp s 0 trn mi dng. Ta s m s phn t ca S theo 2 cch. m theo
ct: Gi s khng tn ti 2 hc sinh m mi bi ton th t nht mt trong 2 hc sinh lm
ng. Ngha l vi mi cp 2 hc sinh u c 1 bi ton m c 2 khng
 lm ng. Do vi mi
200
cp 2 ct, c t nht 1 cp s 0 (mi s trn mt ct), do |S|
= 19900. m theo
2
dng: Mi bi ton c gii bi t nht 120 hc sinh,
 iu ny c ngha l c nhiu nht 80 s
80
0 trn mi dng. Suy ra mi dng c nhiu nht
cp s 0. V ma trn c 6 dng nn ta c
2
 
80
|S| 6
= 18960. T 2 cch m trn ta suy ra 19900 |S| 18960. iu ny v l. Vy
2
phi c t nht 2 hc sinh sao cho mi bi ton th t nht mt trong hai hc sinh lm ng.

Kt lun

Ton hc mun mu mun v, mi bi ton u cha ng bn trong n nhng ci hay, ci


kh ring, ngi hc ton phi t tm cho mnh mt phng php tip cn theo ci cch ca h.
Bi vit ny ch th hin mt gc nhn no i vi mt lp bi ton nht nh. Chng ti rt
mong nhn c nhng gp ca nhng ai quan tm n lnh vc ny. Phn cui ca bi vit,
chng ti xin a ra mt s bi tp dnh cho nhng ai mun tm hiu thm v vn ny.

6.1

Mt s bi tp ngh

Bi tp 1. Chng minh cc ng thc sau vi m, n, k l cc s nguyn dng (bng phng


php m theo 2 cch)

 


   m

 n


P nk m + k
k1
k
n1
n P mk n + k
+
+. .. .+
=
2
+
2
= 2m+n+1
k1
k1
k1
k k=0
k
k
k=0
 


 n  2
 
k+1
P
P
P
n
k
n+k1
n
2n
i
= n
=
(d) = n ((d)l hm Phi Euler)
i
i

1
k
k
n d/n
i=1
k=0


n
P
n+k1
= F2n (Fk l s Fibonaci th k).
2k 1
k=1
Bi tp 2. Gi F(S) l h cc tp con ca tp hu
mi xP
S ta k hiu d(x) l s
P hn S,2 vi P
cc tp con thuc F(S)cha x. Chng minh rng:
(d(x)) =
|A B|.
xS

AF(S) BF(S)

n Ck
m Ck
P
P
m+k
n+k
+
Bi tp 3. (VMO - 2004) Cho trc cc s nguyn dng m, n. Tnh T =
n+k
m+k
k=0 2
k=0 2

Bi tp 4. Chng minh rng:

n
P

[(nk)/2]

2k Cnk Cnk

n
= C2n+1
, n Z +

k=0

Bi tp 5. Ti mt trng hc c 2007 hc sinh nam v 2007 hc sinh n, mi hc sinh khng


tham gia nhiu hn 100 cu lc b trng. Bit rng 2 hc sinh khc nhau v gii tnh tham
gia chung t nht 1 cu lc b. Chng minh rng c 1 cu lc b c t nht 11 hc sinh nam v
11 hc sinh na l thnh vin.
Bi tp 6. (Bulgari - MO 2006) Mt quc gia c 16 thnh ph v 36 tuyn bay ni gia chng.
Chng minh rng c th t chc mt chuyn bay vng quanh gia cc thnh ph.
183

Bi tp 7. (IMO - 1998) Trong mt cuc thi ton quc t c m th sinh v n v gim kho,
n l s l v n 3. Mt gim kho nh gi mt th sinh thng qua 2 mc: t v Cha t.
Gi s rng k l s nguym dng tha mn vi 2 gim kho bt k th nh gi ca h l trng
n1
k

.
nhau vi nhiu nht k th sinh. Chng minh rng:
m
2n
Bi tp 8. (IMO Shortlist 1986) Cho 5 s c 100 ch s c to thnh t ch s 1, 2. Ta xp
cc ch s thng nhau theo hng n v, chc, trm,. . . Bit rng 2 s bt k trong 5 s u
c t nht r hng ging nhau v mi hng sau khi xp u c 2 ch s 1, 2. Chng minh rng:
40 r 60.
Bi tp 9. c bao nhiu con xe c th t trn bn c c kch thc n n sao cho mi con xe
b n bi ti a 2 con xe khc.
Bi tp 10. Gi s rng A = [aij ]nn ma trn c n n gm cc phn t l cc s trong tp
{1, 2, 3, . . . , n}. Chng minh rng bng cch i cc ct ca A lp ra ma trn B = [bij ]nn sao
cho K(B) n , trong K(B)l s phn t ca tp hp {(i, j) : bij = j}.

Ti liu tham kho


[1] Cc thi v ch ton, olympic cc nc, olympic quc t.
[2] Yufei Zhao, Counting in two ways, MOP 2007 Black Group.
[3] Stasys Jukna, Extremal Combinatorics, Springer.
[4] Mt s ti liu t cc din n ton hc trn internet.

184

Mt s dng ton suy lun logic


Li Th nh Vn
Trng THPT Chuyn Nguyn Du, k Lk

Mt s dng ton t mu

Bi ton 1. Mi s nguyn t 1 n n c t bi mt trong hai mu: xanh hoc . Tm s


nguyn dng n nh nht sao cho vi mi cch t mu u tn ti ba s cng mu v lp thnh
mt cp s cng.
Gii. Vi n 8 ta xt cch t mu sau
+ Cc s 1, 2, 5, 6 t cng mu xanh,
+ Cc s 3, 4, 7, 8 t cng mu .
Khi khng c ba s no cng mu lp thnh cp s cng.
Vi n = 9, gi s khng tn ti cch t mu no tha mn iu kin c ba s cng mu lp
thnh cp s cng. Ta xt cch t mu s 5. Nu 5 c t mu xanh
+ Nu 4 v 6 cng mu xanh th 4, 5, 6 cng mu v lp thnh cp s cng (mu thun).
+ Nu 4 xanh v 6 th do 3, 4, 5 khng cng mu nn 3 ; do 3, 6, 9 khng cng mu
nn 9 xanh; do 1, 5, 9 khng cng mu nn 1 ; do 1, 2, 3 khng cng mu nn 2 xanh; do 2, 5,
8 khng cng mu nn 8 ; do 6, 7, 8 khng cng mu nn 7 xanh. Nh vy 5, 7, 9 cng mu
xanh (mu thun).
+ Nu 4 v 6 xanh th do 5, 6, 7 khng cng mu nn 7 ; do 1, 4, 7 khng cng mu
nn 1 xanh; do 1, 3, 5 khng cng mu nn 3 ; do 2, 3, 4 khng cng mu nn 2 xanh; do 2, 5,
8 khng cng mu nn 8 ; do 7, 8, 9 khng cng mu nn 9 xanh. Vy 1, 5, 9 cng mu xanh
(mu thun).
+ Nu 4 v 6 cng mu th do 2, 4, 6 v 4, 6, 8 khng cng mu nn 2, 8 cng xanh. Vy
2, 5, 8 cng mu xanh (mu thun).
Vy vi n = 9 tn ti cch t mu tha mn iu kin c ba s cng mu lp thnh cp s
cng.
Hay n = 9 l s nguyn dng nh nht tha mn iu kin.
Bi ton 2. Mi im ca mt phng c t bng mt trong
hai mu: xanh hoc . Chng

minh rng tn ti mt tam gic u, vi cnh bng 1 hoc 3, c 3 nh cng mu.


Chng minh. Ly tam gic ABC c ba cnh bng 1. Nu ba im A, B, C cng mu th bi
ton c chng minh.
Ta xt trng hp tam gic ABC c hai nh khc mu, khng mt tnh tng qut gi s A
v B khc mu.
Dng tam gic cn PAB c P A = P B = 2, khi P khc mu vi A hoc B. Gi s P khc
mu vi A. Gi M l trung im AP. Khi M cng mu vi A hoc P. Do tnh i xng ta gi
s M cng mu vi A. Dng cc tam gic u AMQ v AMN.
+ Nu Q (hoc N) cng mu vi A th tam gic AMQ (hoc tam gic AMN) c cc cnh bng
1 v ba nh cng mu.

+ Nu Q v N khc mu vi A th Q v N cng
mu vi P. Mt khc ta c P QN = 60 , QN =
3. Do tam gic PQN u c cnh bng 3 c ba nh cng mu.

185

Bi ton 3. Xt lc gic A1 A2 A3 A4 A5 A6 c tt c cc cnh v cc ng cho c t mu xanh


hoc , trong khng c tam gic Aj Ak Am (1 j < k < m 6) no c ba cnh u l mu
xanh. Gi Rk (k = 1, 2, . . . , 6) l s on Ak Aj (j 6= k) mu . Chng minh rng
6
P
(2Rk 7)2 54.
k=1

Chng minh. Nhn xt


i) Rk 5 vi k = 1, 2, . . . , 6. Do 2Rk 7 3
ii) Nu Rk 2 vi k = 1, 2, . . . , 6 th |2Rk 7| 3 (2Rk 7)2 9

6
P

(2Rk 7)2 54

k=1

Ta xt cc trng hp vi 0 Rk < 2.
Khng mt tnh tng qut ta gi s R1 Rj vi j = 2, . . . , 6. Xt hai trng hp sau
TH1: R1 = 0. Do cc tam gic A1 Ak Am (1 < k < m 6) c t nht mt cnh mu nn
Ak Am l cnh mu , do Rk = 4 (2 k 6). Vy
6
P
(2Rk 7)2 = (7)2 + 5(2.4 7)2 = 54.
k=1

TH2: R1 = 1tc l c ng mt cnhA1 Aj mu nn 1 Rj 5. Do cc tam gic


A1 Ak Am (2 k < m 6; k, m 6= j) c t nht mt cnh mu nn Ak Am l cnh mu , do
3 Rk 4 (2 k 6, k 6= j). Vy
6
P
(2Rk 7)2 (5)2 + (2.1 7)2 + 4(2.4 7)2 = 54
k=1

Vy ta lun c

6
P

(2Rk 7)2 54 vi k = 1, 2, . . . , 6.

k=1

Bi ton 4. Cho 9 im phn bit cng nm trn mt ng trn. Ta t mu cho 36 on thng


ni chng bi cc mu xanh hoc . Gi s rng mi tam gic c to thnh t 3 trong s 9
im cha t nht mt cnh mu . Chng minh rng c 4 im tha mn 6 on thng ni
chng u c mu .
Chng minh. Nu c mt im c ni vi 4 im X1 , X2 , X3 , X4 bi cc ng mu xanh
th do iu kin khng c tam gic no c ba cnh u l mu xanh nn tt c 6 ng
186

X1 X2 , X1 X3 , X1 X4 , X2 X3 , X2 X4 , X3 X4 u c mu . Nh vy nu c mt im m s ng
mu xanh ni n vi cc im khc t nht l 4 th bi ton c chng minh.
Ta xt trng hp mi im c ni vi cc im khc bi t nht 5 ng mu (tc nhiu
nht l 3 ng mu xanh). Nu mi im u c ni vi 5 im khc bi cc ng mu
95
khng l s nguyn nn khng xy ra trng hp ny. Do tn ti im B ni vi 6
v
2
im khc bi cc ng mu . Ta k hiu cc ng l BY1 , BY2 , BY3 , BY4 , BY5 , BY6 . V
trong 5 ng Y1 Y2 , Y1 Y3 , Y1 Y4 , Y1 Y5 , Y1 Y6 c t nht 3 ng c cng mu, ta gi s cc ng
Y1 Y2 , Y1 Y3 , Y1 Y4 c cng mu.
+ Nu ng Y1 Y2 , Y1 Y3 , Y1 Y4 cng mu xanh thi do iu kin mi tam gic c t nht mt
cnh mu nn Y2 Y3 , Y3 Y4 , Y2 Y4 l cc ng mu . Do cc im B, Y2 , Y3 , Y4 c ni vi
nhau bi cc on mu .
+ Nu ng Y1 Y2 , Y1 Y3 , Y1 Y4 cng mu thi do tam gic Y2 Y3 Y4 c t nht mt cnh mu
nn ta gi s Y2 Y3 l cc ng mu . Do cc im B, Y1 , Y2 , Y3 c ni vi nhau bi
cc on mu .
Bi ton 5. Gi s n l mt s t nhin khng nh hn 3. Trong mt phng cho tp hp P gm
n im, trong khng c 3 im no thng hng. Mi cp im thuc P c ni bi mt on
t mu bi mu hoc mu trng. Tm s nh nht ca cc on thng mu , sao cho bt c
tam gic no, vi 3 nh thuc P, u c t nht mt cnh mu .
Gii. Gi s cn tm l d(n).

Theo hnh v, ta c d(3) = 1 v d(4) = 2.


Xt vi n 5. S on thng ni cc im thuc P l Cn2 . Ta xt f l mt cch t mu cc
on v k hiu f (n) l s on mu . Ta c f (n) d(n).
Gi thit rng trong cch t mu f c t nht mt on trng l AB (A, B P ). Xt tp hp
Q = P \ {A, B} gm n 2 im. T cch t mu f , ta b i on AB v tt c cc on c u
mt l A hoc B, ta c cch t mu f* cc on ni cc cp im ca Q tha mn iu kin ca
bi ton. Ta gi s cc on mu trong cch t mu f* l f (n 2). Ta c f (n 2) d(n 2).
Ly im C Q v xt cc on CA, CB. Do tam gic ABC c t nht mt on mu
nn trong hai on CA, CB c t nht mt on mu . V Q c n 2 im nn suy ra
187

f (n) n 2 + f (n 2). Do , ta c:
+ Nu n l s chn (n 6) th
f (n) (n 2) + (n 4) + + 4 + f (4) 2 + 4 + + (n 2)
n(n 2)
f (n)
4

+ Nu n l s l th
f (n) (n 2) + (n 4) + + 3 + f (3) 1 + 3 + + (n 2)
(n 1)2
.
f (n)
4

Mt khc ta xt tp P gm n im trong khng c im no thng hng. Ta chia P lm


hai tp con P1 v P2 ri nhau theo cch;
n
+ Nu n chn th mi tp P1 v P2 gm im.
2
n1
n+1
+ Nu n chn th tp P1 gm
v P2 gm
im.
2
2
Ta t mu cho cc on thng theo quy tc:
+ Nu on thng c hai u mt cng thuc P1 hoc P2 th n c t mu .
+ Nu on thng c hai u mt thuc hai tp khc nhau th n c t mu xanh.
Khi s on mu l
n(n 2)
+ Nu n l s chn f (n) = 2C 2n =
4
2
+ Nu n l s l th
(n 1)2
1 (n 1) (n 3) 1 (n 1) (n + 1)
+
=
.
f (n) = C 2n 1 + C 2n + 1 =
2 2
2
2 2
2
4
2
2
n(n 2)

khi n = 2k
4 2
Vy ta c d(n) =

(n 1) khi n = 2k + 1
(k N )
4
Bi ton 6. (Hellenic Mathematical Competitions 2012). Cho mt s nguyn n c dng n =
3k (k 2). Tam gic u ABC c chia thnh n2 tam gic u nh bng cc ng thng song
song vi cc cnh ca n. Mt hnh r qut bao gm ba tam gic nh ng cnh nhau nh phn
c t mu trong hnh v (ng vi trng hp n = 6 (k = 2)). Ta t mu cho tt c cc im
bi ba mu: xanh, , trng sao cho cc im k nhau th khc mu nhau. Phn loi cc hnh r
qut l mu xanh, , hay trng da vo mu ca nh chung ca ba tam gic. Hi c bao nhiu
hnh r qut c to thnh v mi loi c bao nhiu hnh?
Gii. Ta gi im l nh chung ca ba tam gic ca mt hnh r qut l tm ca hnh r qut
.
+ Mi im nm trn cnh ca tam gic ABC (tr cc nh A, B, C) l tm ca ng mt
hnh r qut. M trn mi cnh ta c n 1 im. Vy c 3(n 1) hnh r qut.
+ Mi im nm trong tam gic ABC u l tm ca mt hnh lc gic u nn mi im
l tm ca 6 hnh r qut. Mt khc s im nm trong tam gic ABC l
188

1 + 2 + 3 + + n 2 =

(n 1)(n 2)
2

Vy s hnh r qut c tm nm trong tam gic ABC l 6

(n 1)(n 2)
= 3(n 1)(n 2)
2

Vy tng s hnh r qut c to thnh l 3(n 1) + 3(n 1)(n 2) = 3(n 1)2 .


Chng ta t mu cho nh A v gi mu ny l mu cp 1 v gi hai mu cn li l mu cp
2.
Ta xt hnh tam gic u c chia thnh 9 tam gic u nh (nh hnh v) v gi n l tam
gic c bn. T mu cp 1 cho mt nh ca tam gic c bn. Do cc im lin k khng c cng
mu nn hai nh cn li v tm ca tam gic cng c cng mu cp 1.

V n c dng n = 3k nn ta c th nhm n2 tam gic nh thnh k 2 tam gic c bn. Do


cc nh B, C ca tam gic ABC phi c cng mu vi nh A (mu cp 1). Mt khc mi tam
189

gic c bn c tm ca n c t mu cp 1 nn ta c k 2 im c mu cp 1. Ngoi ra tn ti
(k 1)(k 2)
1 + 2 + + k 2 =
im khng l tm ca k 2 tam gic c bn (l nh chung
2
ca hai tam gic c bn) c t mu cp 1.
(k 1)(k 2)
Vy ta c s im nm bn trong tam gic ABC c mu cp 1 l k 2 +
=
2
3k 2 3k + 2
(n 1)(n 2)
v s im nm bn trong tam gic ABC c mu cp 2 l

2
2
3k 2 3k + 2
(3k 1)(3k 2) 3k 2 3k + 2
=

= 3k 2 3k.
2
2
2
Trn mi cnh ca tam gic ABC (tr hai u mt) c n 1 = 3k 1 nh c t cc mu
cp 1 v cp 2, do hai im u mt ca cnh c t mu cp 1 nn trong 3k 1 im c
k 1 im c t mu cp 1 v 2k im c t mu cp 2.
Theo lp lun tnh s r qut trn ta tnh c s r qut c mu cp 1 l 6 k 2 + 6
(k 1)(k 2)
+3(k 1) = 9k 2 6k +3. v s r qut c mu cp 2 l 6(3k 2 3k)+6k = 18k 2 12k
2
Vy s r qut c mu cng mu vi nh A l 9k 2 6k + 3 = n2 2n + 3 v s r qut c
18k 2 12k
= 9k 2 6k = n2 2n.
mu l mt trong hai mu khng cng mu vi nh A l
2
Bi ton 7. (Japan Mathematical Olympiad 2012): Cho mt hnh ch nht c kch thc 2 x
100 c chia thnh 2 x 100 hnh vung nh. Chng ta t mu cho 200 hnh vung bi cc
mu xanh v . C bao nhiu cch t mu cc hnh vung tha mn iu kin sau
i. C t nht mt hnh vung mu v mt hnh vung mu xanh.
ii. Ta gi cc hnh vung cng mu c chung cnh l cc hnh c lin kt. Khi cc hnh
vung c mu lin kt vi nhau thnh mt min (nh vy cc hnh vung c mu xanh cng
lin kt vi nhau thnh mt min).
(Chng ta xem hai hnh sau khi t mu cng phn bit nu c mt php quay hay mt php
tnh tin bin hnh ny thnh hnh kia).
Gii. Xt 204 im nm trn cc cnh ca hnh ch nht, mi im hoc l mt nh ca
mt hnh vung hoc l nh chung ca ng hai hnh vung k nhau.
Nu chng ta xt ln lt cc im bt u t gc tri pha bn di i dc theo cc cnh
ca hnh ch nht th do hnh ch nht c chia lm hai min c mu khc nhau (xanh v )
nn s c chnh xc mt im sao cho ti im mu ca hai hnh vung k nhau chuyn t
sang xanh v s c chnh xc mt im sao cho ti im mu ca hai hnh vung k nhau
chuyn t xanh sang . Hai im c bit ny khng th l cc nh ca hnh ch nht ban u.
Nu ta chn ra hai im t 200 im (tr cc nh ca hnh ch nht) th chng ta c th xc
nh c duy nht mt ng zig-zag ni hai im tha mn l ng bin ca min mu
v min mu xanh.
Do s cch t mu cc hnh vung tha mn iu kin bi ton bng s cch chn ra t
200 im mt im ti mu ca vung chuyn t sang xanh v mt im ti mu ca
2
vung chuyn t xanh sang . Vy s cch t mu l 2 C200
= 39800 cch.
Bi ton 8. (Japan Mathematical Olympiad 2012): Cho n l s nguyn dng, xt mt hnh
vung c chia thnh 2n 2n li hnh vung. Chng ta t mu cho ng 2n2 hnh vung trong
li tha mn iu kin nu mt hnh vung c t mu th tt c cc hnh vung c chung
ng mt nh vi n khng c t mu. C bao nhiu cch t mu ng 2n2 hnh vung tha
mn iu kin?
(Chng ta xem hai cch t mu c hai hnh to thnh m sau mt php quay hoc php tnh
tin hnh ny bin thnh hnh kia l hai cch t mu phn bit).
190

Gii. Chng ta s xt li hnh vung c kch thc n n, trong mi hnh vung ca li c


kch thc l 2 n v. Nh vy mi hnh vung ca li mi s cha 4 hnh vung ca li ban
u, ta gi mi hnh vung ca li mi l mt block. Do iu kin ca bi ton mi block cha
nhiu nht l 2 hnh vung c t mu. Mt khc do ta phi t c ng 2n2 hnh vung ca
li ban u, do mi block phi c ng 2 hnh vung c t mu.
Vi mi block ta dn nhn cho 4 hnh vung ca n l A, B, C, D.
Do trong mi block c 2 hnh vung c t mu tha mn iu kin nn ta c th chn mt
hnh vung trong cp A v D, v mt hnh vung trong cp B v C t mu.
Ta gi cc block c chung cnh hay ch chung mt nh trong li n n l cc block ln cn.
Mi hnh vung trong mt block c th chung cnh hoc chung ng mt nh, hoc khng c g
chung vi cc hnh vung trong block ln cn. Vic chung nh nhng khng chung cnh ca hai
hnh vung cc block ln cn ch c th xy ra khi nhn ca chng l A v D hoc B v C. Do
vic quyt nh t mu cho A hay D trong mt block l c lp vi vic quyt nh t mu
cho B hay C trong block .
By gi chng ta s xem xt n vic trong mi block nn chn hnh vung A hay hnh vung
D t mu. Ta s bt u vi block nm gc trn bn tri v chn A hoc D t mu, tip
n ta xt block nm ngay bn phi block xt v chn A hoc D t mu (nh th vn tha
mn iu kin ca bi ton), v chng ta c tip tc lm nh th cho n block cui cng ca
dng u tin. Chng ta quay li block u tin bn tri ca dng th hai lp li cch lm nh
dng u tin, i t tri qua phi. C xt nh th cho n block cui cng nm gc di bn
phi ca li n n.
Theo nh cch t mu ny ta nhn thy nu block X hnh vung D c t mu th trong
tt c cc block cng hng bn phi block X, cc hnh vung dn nhn D phi c t mu. V
nu c mt hng na di hng cha block X th block nm ngay bn di X phi c hnh vung
D c t mu.
Nh th, vi mi k = 1, 2, . . . , n tn ti mt s nguyn ak tha mn iu kin: dng th k
ca li (cc block tnh t tri qua phi), n ak hnh vung nhn A c t mu v ak hnh
vung nhn D c t mu. Hn th na ta c iu kin 0 a1 a2 ak n (1)
Nh th vi dy s nguyn khng m {ak ; 1 k n} tha mn iu kin (1) ta s c th t
191

mu hnh vung A hoc D dng th k sao cho tt c n ak block u tin bn tri th hnh
vung A c t mu v ak block cn li th hnh vung D c t mu.
Vi mi k = 1, 2, . . . , n ta t bk = ak + k. Khi ta c 1 b1 < b2 < < bk 2n
Nh th s cch chn A hoc D t mu cho n2 block th bng s cch chn n s nguyn
n
.
phn bit nm gia 1 v 2n (bao gm c 1 v 2n), bng C2n
Chng ta xt tng t cho cch chn hnh vung B hoc C t mu
n 2
Vy p n ca bi ton l (C2n
)

Bi tp

Bi tp 1. ( thi Olympic Colombia 1997-1998) Cho s nguyn dng n. Xt hnh vung c


chia thnh n n li hnh vung v 3 mu. Ta s t mu mi on ca li bi mt trong 3 mu
trn sao cho mi vung n v c 2 cnh cng mu v 2 cnh cn li cng mu khc. Hi c
bao nhiu cch t mu tha mn iu kin?
2
p n: 32n .2n .
Bi tp 2. ( thi Olympic Czech v Slovak repubulick 1997-1998) Mi cnh v ng cho ca
mt n-gic u (n 3) c t mu hoc mu xanh. Ta chn mt nh v thay i mu ca
cc on thng nhn im lm u mt t mu thnh mu xanh v ngc li. Chng minh
rng, vi bt k cch t mu lc u th no, ta vn c th bin s cnh mu xanh xut pht t
mi nh l s chn.
Bi tp 3. ( thi Olympic Italy 1997-1998) Gc phn t th nht ca mt phng ta c
chia thnh cc hnh vung n v bi cc ng li. C th t mu cc hnh vung n v tha
mn cc iu kin sau hay khng?
a) Vi mi hnh vung ln c mt nh t ti gc, v cc cnh song song vi cc trc ta
th cha nhiu hnh vung n v c t mu hn cc hnh vung n v khng c t mu.
b) Mi ng song song vi ng phn gic ca gc phn t th nht ch i qua cc hnh
vung c t mu.
Bi tp 4. ( thi Olympic Nam Phi 1997-1998) C 6 im c ni vi nhau tng i mt bi
nhng on mu hoc mu xanh. Chng minh rng: c mt chu trnh 4 cnh cng mu.

192

Mt s ng dng ca nguyn l
Dirichlet trong ton hc
Nguyn Mnh Quyn
Trng THPT Chuyn Nguyn Ch Thanh, k Nng
Nguyn l Dirichlet - cn gi l nguyn l chim b cu (The Pigeonhole Principle)-hoc nguyn
nhng ci lng nht th hoc nguyn l sp xp vt vo ngn ko (The Drawer Principle) a ra mt nguyn tc v phn chia phn t cc lp.
Nguyn l ny c Dirichlet pht biu u tin nm 1834.
Nguyn l Dirichlet l mt cng c rt hiu qu dng chng minh nhiu kt qu su sc
ca ton hc. N c bit c nhiu p dng trong lnh vc khc nhau ca ton hc. Nguyn l
ny trong nhiu trng hp ngi ta d dng chng minh c s tn ti m khng a ra c
phng php tm c vt c th, nhng trong thc t nhiu bi ton ta ch cn ch ra s tn ti
l ri.
Ni dung ca nguyn l ny ht sc n gin v d hiu nhng li c tc dng rt ln, c nhiu
hiu qu bt ng trong gii ton. S dng n, chng ta c th chng minh c nhiu kt qu su
sc ca Ton hc. i khi c nhng bi ton ngi ta dng rt nhiu phng php khc nhau
gii m vn cha i n c kt qu, nhng nh nguyn l Dirichlet m bi ton tr nn d
dng gii quyt.

Nguyn l Dirichlet

Nu nht n + 1 con th vo n ci chung th bao gi cng c mt chung cha t nht hai con
th.

Nguyn l Dirichlet tng qut




N
Mnh 1. Nu c N vt c t vo trong k hp th s tn ti mt hp cha t nht
k
vt.

( y, [x] l gi tr ca hm trn ti s thc x, l s nguyn nh nht c gi tr ln hn hoc


bng x. Khi nim ny i ngu vi [x] gi tr ca hm sn hay hm phn nguyn ti x l s
nguyn ln nht c gi tr nh hn hoc bng x.)
Chng minh.
 
N
Gi s mi hp u cha t hn
vt. Khi tng s vt l;
k
 
 
N
N
k(
1) < k
= N.
k
k
iu ny mu thun vi gi thit l c N vt cn xp.

Nguyn l Dirichlet i ngu


Cho tp hu hn S 6= v S1, S2, Sn l cc tp con ca S sao cho |S1| + |S2| + + |Sn| >
k.|S|. Khi , tn ti mt phn t x S sao cho x l phn t chung ca k+1 tp Si (i = 1, 2, n).
193

Nguyn l Dirichlet m rng





n+m1
con
Nu nht n con th vo m 2 ci chung th tn ti mt chung c t nht
m
th, y k hiu [] ch phn nguyn ca s .
Ta chng minh nguyn l Dirichlet m rng nh sau : Gi s tri li mi chung th khng c
n
 
 


n1
n1
n+m1
=
+1 =
+1
m
m
m


n1
con, th s th trong mi chung u nh hn hoc bng
con.
m


n1
T suy ra tng s con th khng vt qu m.
n 1con.
m
iu ny v l v c n con th. Vy gi thit phn chng l sai.
Nguyn l Dirichlet m rng c chng minh. Nguyn l Dirichlet tng chng n gin nh
vy, nhng n l mt cng c rt hiu qu dng chng minh nhiu kt qu su sc ca ton
hc. N c bit c nhiu p dng trong lnh vc khc nhau ca ton hc. Nguyn l ny trong
nhiutrng hp ngi ta d dng chng minh c s tn ti m khng a ra cphng
php tm c vt c th, nhng trong thc t nhiu bi ton ta ch cnch ra s tn ti l
ri.Nguyn l Dirichlet thc cht l mt nh l v tp hu hn. Ngi ta c thpht biu chnh
xc nguyn l ny di dng sau y.

Nguyn l Dirichlet dng tp hp

Cho A v B l hai tp hp khc rng c s phn t hu hn, m s lng phnt ca A ln


hn s lng phn t ca B. Nu vi mt quy tc no , mi phnt ca A cho tng ng vi
mt phn t ca B, th tn ti t nht hai phn t khc nhau ca A m chng tng ng vi mt
phn t ca B.
Vi cng mt cch nh vy, nguyn l Dirichlet m rng c dng sau y.

Nguyn l Dirichlet dng tp hp m rng


Gi s A, B l hai tp hp hu hn v S(A), S(B) tng ng k hiu l cc slng phn t
ca A v B. Gi s c mt s t nhin k no m S(A) > k.S(B) v ta c quy tc cho tng
ng mi phn t ca A vi mt phn t ca B. Khi tn ti t nht k + 1 phn t ca A m
chng tng ng vi cngmt phn t ca B.
Ch : Khi k = 1, ta c ngay li nguyn l Dirichlet.
V chng ny dnh trnh by phng php s dng nguyn l Dirichlet gii cc bi ton
hnh hc s cp.V l , ti xin trnh by lun mt s mnh sau (thc cht l mt s pht
biu khc ca nguyn l Dirichlet p dng cho di cc on thng, din tch cc hnh phng,
th tch cc vt th) rt hay c s dng n trong nhiu bi ton hnh hc c cp ti
trong chng ny.

Nguyn l Dirichlet cho din tch


Nu K l mt hnh phng, cn K1 , K2 , , Kn l cc hnh phng sao cho Ki K vi i = 1, n
v |K| < |K1 | + |K2 | + + |Kn |, y |K| l din tch ca hnh phng K, cn |Ki | l din tch

194

hnh phng Ki , i = 1, n, th tn ti t nht hai hnh phng Hi , Hj


c im trong chung.

(1 i < j n) sao cho Hi , Hj

( y ta ni rng P l im trong ca tp hp A trn mt phng nu nh tn ti hnh trn


tm P bn knh b sao cho hnh trn ny nm trn trong A) Tng t nh nguyn l Dirichlet
cho din tch, ta c cc nguyn l Dirichlet cho di cc on thng, th tch cc vt th . . .

Nguyn l Dirichlet v hn
Nu chia mt tp hp v hn cc qu to vo hu hn ngn ko, th phi c t nht mt ngn
ko cha v hn cc qu to.
Nguyn l Dirichlet m rng cho trng hp v hn ny ng vai tr cng ht sc quan trng
trong l thuyt tp im tr mt trn ng thng. N c vai tr quan trng trong l thuyt s
ni ring v ton hc ri rc ni chung (trong c hnh hc t hp)

Nguyn l Dirichlet i ngu v hn phn t.


* Tp phn t l mt khong trn ng thng. Trong mc ny ta k hiu d(I) l di ca
khong I R.
+ Cho A l mt khong gii ni, A1 , A2 , An l cc khong sao cho Ai A(i = 1, 2, n)
v d(A) < d(A1 ) + d(A2 ) + + d(An ). Khi t nht c hai khong trong s cc khong trn c
mt im trong chung.
Chng minh.
Tht vy, gi s khng c cp no trong nhng khong cho c im trong chung.
Khi ,
d(A1 A2 An ) = d(A1 ) + d(A2 ) + + d(An ) > d(A).
Mt khc, t Ai A(i = 1, 2, , n) suy ra d(A1 A2 An ) d(A). Cc bt ng thc
trn mu thun vi nhau. Vy t nht c hai khong trong s cc khong trn c im trong
chung.
* Tp phn t l min phng gii hn bi mt ng cong phng khp kn. Trong mc ny ta
k hiu S(A) l din tch min A trong mt mt phng.
+ Nu A l mt min gii hn bi mt ng cong phng khp kn, cn A1 , A2 , , An l cc
min sao cho Ai A(i = 1, 2, , n) v S(A) < S(A1 ) + S(A2 ) + + S(An ), th t nht c hai
min trong s cc min ni trn c im trong chung.
Chng minh. Tng t nh chng minh nh l 1.

Phng php v ng dng

Nguyn l dirichlet tng chng nh n gin nh vy, nhng n l mt cng c ht sc c


hiu qu dng chng mnh nhiu kt qu ht sc su sc ca ton hc. Nguyn l Dirichlet
cng c p dng cho cc bi ton ca hnh hc, iu c th hin qua h thng bi tp
sau:
s dng nguyn l Dirichlet ta phi lm xut hin tnh hung nht th" vo chung" v
tho mn cc iu kin :
+ S th" phi hiu hn s chung
+ Th" phi c nht ht vo cc chung", nhng khng bt buc chung no cng phi
c th.

195

Thng phng php Dirichlet c p dng km theo phng php phn chng.
Ngoi ra n cn c th p dng vi cc php bin hnh.

3.1

Nguyn l cc hn

Song song vi vic s dng cc nguyn l khc nh phn chng, Dirichlet hay quy np ton
hc tm li gii cho cc bi ton kh hc ba, nguyn l cc hn cng c xem l mt phng
php rt hay, c vn dng mt cch linh hot trong vic kho st mt tp hp hu hn hay v
hn phn t m trong n tn ti gi tr nh nht hoc gi tr ln nht .
Nguyn l cc hn c pht biu n gin nh sau:
Nguyn l 1: Trong mt tp hu hn v khc rng cc s thc lun lun c th chn c s
b nht v s ln nht.
Nguyn l 2: Trong mt tp khc rng cc s t nhin lun lun c th chn c s b nht.
S dng nguyn l cc hn l mt phng php c vn dng cho nhiu lp bi ton khc,
c bit n c ch khi gii cc bi ton t hp ni chung v hnh hc ni ring.
Trong qu trnh tm kim li gii nhiu bi ton hnh hc, s rt c li nu chng ta xem xt
cc phn t bin, phn t gii hn no , tc l phn t m ti mi i lng hnh hc c
th nhn gi tr ln nht hoc gi tr nh nht, chng hn nh cnh ln nht, cnh nh nht ca
mt tam gic, gc ln nht hoc gc nh nht ca mt a gic . . .
Nhng tnh cht ca cc phn t bin, phn t gii hn nhiu khi gip chng ta tm kim
c li gii thu gn ca bi ton.
Nguyn l cc hn thng c s dng kt hp vi cc phng php khc, c bit l phng
php phn chng, c vn dng trong trong trng hp tp cc gi tr cn kho st ch tp hp
hu hn (nguyn l 1) hoc c th c v hn nhng tn ti mt phn t ln nht hoc nh nht
(nguyn l 2)Khi vn dng nguyn l ny, ta phi tin hnh cc bc sau :
Bc 1: Chng minh rng trong tt c cc gi tr cn kho st lun tn ti gi tr ln nht
hoc gi tr nh nht.
Bc 2: Xt bi ton trong trng hp ring khi n nhn gi tr ny (nh nht hoc ln nht)
Bc 3: Ch ra mt mu thun, ch ra mt gi tr cn nh hn (hay ln hn) gi tr ta ang
kho st .
Theo nguyn l ca phng php phn chng, ta s suy ra iu phi chng minh.

Bi tp p dng

Bi ton 1. Trong hnh vung cnh bng 1, t 51 im bt k, phn bit. Chng minh rng c
1
t nht 3 trong s 51 im nm trong mt hnh trn bn knh .
7
Cch gii.
1
Chia hnh vung cho thnh 25 hnh vung con bng nhau c cnh bng . Theo nguyn l
5
Dirichlet, tn ti t nht mt hnh vung con a cha t nht ba im trong s 51 im . ng
1
1
trn ngoi tip (a) c bn knh .
7
5 2
1
Vy ba im ni trn nm trong hnh trn ng tm vi ng trn (a) c bn knh .
7
Tng qut ha bi ton:
Da vo bi gii bi ton trn ta c th tng qut ha bi ton trn vi a l kch thc ca
cnh hnh vung, m l s im t bt k, phn bit. Chng minh rng c t nht n trong s m
196

a2
im nm trong mt hnh trong bn knh s 
 trong k hiu [a] l phn nguyn ca
m
2
n1
a).
Cch gii.


a2
m
hnh vung con bng nhau c cnh bng s 
Chia hnh vung cho thnh
.
n1
m
2
n1
Theo nguyn l Dirichlet , tn ti t nht mt hnh vung con c cha t nht n im trong s m
im .
ng trn ngoi tip (c) c bn knh
a2
a2
s 
s

.

m
m
2
2
n1
n1
a2
Vy n im trn nm trong hnh trn ng tm vi ng trn (c) c bn knh s 
.
m
2
n1
Bi ton 2. Cho (xi , xi , xi ), i = 1, 2, 3, 4, 5, 6, 7, 8, 9 l mt tp hp gm 9 im khc nhau c
cc ta nguyn trong khng gian.
Chng minh rng trung im ca ng ni t nht mt trong cc cp im ny c ta
nguyn.
Cch gii.
Gi ta hai im bt k trong khng gian l A(a, b, c) v B(d, e, f )
a + d b + e c + f 
,
,
.
Vy trung im ca on AB l O
2
2
2
Cc ta ca im O nguyn nu v ch nu a v d; b v e; c v f cng chn hoc cng l.
V c 23 = 8 b ba chn l khc nhau ((c, c, c); (l, l, l); (c, c, l); (c, l, l); (c, l, c); (l, c, c); (l, c, l); (l, l, c))
nn theo nguyn l Dirichlet c t nht 2 trong 9 im c cng b ba chn l nh nhau.
Vy c t nht mt cp im m im chnh gia ca chng c ta nguyn.
Tng qut ha bi ton:
Cho tp hp gm m im khc nhau c cc ta nguyn
trong khng gian.
hmi

+1
trong cc cp im ny c
Chng minh rng trung im ca ng ni t nht 8
2
ta nguyn. Cch gii:
Gi ta hai im bt k trong khng gian l A(a, b, c) v B(d, e, f )
Vy trung im ca on AB l:
a + d b + e c + f 
O
,
,
.
2
2
2
Cc ta ca im O nguyn nu v ch nu a v d; b v e; c v f cng chn hoc cng l.
V c 23 = 8 b ba chn l khc nhau h((c,ic, c); (l, l, l); (c, c, l); (c, l, l); (c, l, c); (l, c, c); (l, c, l); (l, l, c))
m
trong m im c cng b ba chn l nh nhau.
nn theo nguyn l Dirichlet c t nht
8
197

hmi
Vy c t nht 8

+1

cp im m im chnh gia ca chng c ta nguyn.

Bi ton 3. Trong mt hnh vung c cnh l 1 cha mt s ng trn. Tng tt c chu vi ca


chng l 10. Chng minh rng tn ti mt ng thng ct t nht 4 ng trn trong nhng
ng trn ?
Gii. Ta chn mt cnh hnh vung ri chiu vung gc cc ng trn xung cnh (xem
hnh 1). Ta c, hnh chiu ca mt ng trn bn knh R xung AB l mt on thng c
di 2R. V vy trn cnh hnh vung chn c nhng on thng chiu xung vi tng di
10
10
. M
> 3. Nn theo nguyn l
l

Dirichlet i ngu suy ra c mt im M no thuc AB l im trong chung ca t nht 4


on thng chiu xung. Khi , ng thng i qua M vung gc vi AB ct t nht 4 trong
nhng ng trn .
Tng qut bi ton:
Cho hnh vung c cnh 1 cha mt s ng trn. Tng di ca cc ng trn l 10.
Chng minh rng tn ti mt ng thng m n ct t nht bn trong nhng ng trn ny
(gi s s ng trn cho ln hn hoc bng 4).
Cch gii.
Chn mt cnh hnh vung chng hn l AB ri chiu vung gc cc ng trn xung cnh
no . D thy rng hnh chiu ca mt ng trn bn knh R s l mt on thng c di
2R. Gi C1 , C2 , , Cn
l chu vi ca n ng trn cho.
Khi theo gi thit, th :
C1 + C2 + + Cn = 10
Ci
.
Mt khc, ng trn vi chu vi Ci s c bn knh : Ri =
2
Vy hnh chiu ca hnh trn vi chu vi Ci s l on thng vi di l :
2Ci
Ci
=
2

Tng di hnh chiu ca n ng trn trn cnh cho l:


C1 C2
Cn
10
+
+ +
=

10
> 3. Nn theo nguyn l Dirichlet i ngu suy ra c mt im M no thuc AB l

im trong chung ca t nht 4 on thng chiu xung. Khi , ng thng i qua M vung
gc vi AB ct t nht 4 trong nhng ng trn . pcm.
M

Bi ton 4. Cho mt hnh vung v 13 ng thng, mi ng thng u chia hnh vung


thnh hai t gic c t s din tch 2 : 3.Chng minh rng trong s 13 ng thng cho, c t
nht 4 ng thng cng i qua mt im.
Cch gii. Gi d l ng thng chia hnh vung ABCD thnh hai t gic c t s din tch l
2 : 3.
ng thng d khng th ct hai cnh k nhau ca hnh vung Gi s d ct hai cnh AB v
CD ti M v N, khi n ct ng trung bnh EF ti I. Gi s
2
2
SAM N D = SBM N C th EI = IF
3
3
198

Nh vy mi ng thng cho chia cc ng trung bnh ca hnh vung theo t s 2 : 3


C 4 im chia cc ng trung bnh ca hnh vung ABCD theo t s 2 : 3
C 13 ng thng, mi ng thng i qua mt trong 4 im
Vy theo nguyn l Dirichlet c t nht 4 ng thng cng i qua 1 im.
Bi ton 5. Chng minh rng mt ng thng ch c th nhiu lm hai cnh ca mt tam gic
phn trong ca cc cnh ny.
Cch gii.
Mt ng thng d bt k lun chia mt phng ra lm hai min, cho nn theo nguyn tc
Dirichlet, tn ti mt min cha t nht hai nh, khng mt tng qut ta gi s l hai nh
A v B. Khi cnh AB nm hon ton trong na mt phng ny v khng th ct d c.
Bi ton 6.
Trong mt ci bt hnh vung cnh 18 cm c 128 ht vng. Chng minh rng tn ti hia ht
vng c khong cch ti nhau nh hn 2 cm.
Cch gii. Ly mi ht vng lm tm dng hnh trn bn knh 1cm. Cc hnh trn ny nm
hon ton trong hnh vung c cnh 20cm thu c t hnh vung cho bng cch tnh tin
bn cnh ca n mt khong 1cm ra pha ngoi.
Tng din tch ca cc hnh trn bn knh 1cm ny l 128 > 402, 112 > 400. Do tng din
tch cc hnh trn ny ln hn din tch hnh vung cnh 20cm.
Bi ton 6. Trong hnh ch nht 3 4 t 6 im. Chng minh rng trong s lun tm c
hai im c khong cch gia chng khng ln hn .
Cch gii. Chia hnh ch nht cho thnh nm hnh ABCD, DCKEF, KF N M, N F EQR, QEDAS.
V c 6 im nn theo nguyn l Dirichlet tn ti mt trong nm hnh trn, m hnh ny cha
t nht hai trong 6 im cho.
Ta a vo khi nim sau:
Gi s P l mt hnh.
t d(P ) = maxM,N P {M N }, v i
lng d(P ) gi l ng knh ca hnh
P. D thy c
nm hnh trn u c ng
knh bng 5. (Th d: d(ABCD) = AC = 5, d(DCKF E) =
CE = KE = CF = DK = 5)

T suy ra lun tm c 2 im trong s 6 im cho c khong cch khng ln hn 5.


l iu phi chng minh.
T ta c cc bi ton tng t nh sau:
Bi ton 7. Trn mt phng cho 25 im. Bit rng trong ba im bt k trong s lun lun
tn ti hia im cch nhau nh hn 1. Chng minh rn g tn ti hnh trn bn knh 1 cha khng
t hn 13 im cho.
Cch gii. Ly A l mt trong s 25 im cho. Xt hnh trn O1 (A, 1) tm A bn knh 1.
Ch c hai kh nng sau c th xy ra:
1) Nu tt c cc im cho nm trong O1 (A, 1) th kt lun ca bi ton hin nhin ng.
2) Tn ti im B 6= A (B thuc trong s 25 im cho), sao cho B 6 O1 (A, 1),v B 6=
O1 (A, 1), nn AB > 1.
Xt hnh trn O2 (B, 1) tm B, bn knh 1. Ly C l im bt k trong s 25 im cho sao
cho C 6= A, C 6= B. Theo gi thit (v da vo AB > 1), ta c min{CA, CB} < 1.
V th C O1 (A, 1) hoc C O2 (B, 1).
iu ny chng t rng cc hnh trn O1 (A, 1), O2 (B, 1) cha tt c 25 im cho. V th
theo nguyn l Dirichlet, t nht 1 trong hai hnh trn trn cha 13 im cho. l pcm.
Tng qut bi tan :
199

Cho 2n + 1 im trn mt phng (vi n 3). Bit rng trong ba im bt k trong s lun
lun tn ti hai im cch nhau nh hn 1. Khi tn ti hnh trn bn knh 1 cha khng t
hn n + 1 im cho.
Bi ton 8. Tm hnh vung c kch thc b nht, trong hnh vung c th sp xp nm
hnh trn bn knh 1, sao cho khng c hai hnh trn no trong chng c im chung.
Cch gii.
Gi s hnh vung ABCD c tm O v cnh a, cha nm hnh trn khng ct nhau v u
c bn knh bng 1. V c nm hnh trn ny u nm trn trong hnh vung, nn cc tm ca
chng nm trong hnh vung A0 B 0 C 0 D0 c tm O v cnh a 2, y A0 B 0 //AB.
Cc ng thng ni cc trung im ca cc cnh i din ca hnh vung A0 B 0 C 0 D0 cha
A0 B 0 C 0 D0 thnh 4 hnh vung nh. Theo nguyn l Dirichlet tn ti mt trong 4 hnh vung nh
m trong hnh vung ny cha t nht hai trong s 5 tm hnh trn ni trn (khng mt tnh tng
qut ta gi s l O0 v O0 ). rng v khng c hai hnh trn no (trong s nm hnh trn) ct
nhau, nn
O0 O 2.

(1)

Mt khc do O0 , O cng nm trong mt hnh vung nh (cnh ca hnh vung nh bng


a2
) nn ta li c
2
a2
O0 O
. 2.
(2)
2
T (1) v (2) suy ra:

a2
. 2 2 a 2 2 + 2.
2

(3)

Vy mi hnhvung cnh a tha mn yu cu bi, ta u c (3). By gi xt hnh vung


ABCD c a = 2 2 + 2. Xt nm hnh trn c tm l O, A0 , B 0 , C 0 , D0 (xem hnh 9) , th mi yu
cu ca bi tha mn. Tm li, hnh vung c kch thc b nht cn tm l hnh vung vi
cnh 2 2 + 2.
Bi ton 9. Chng minh rng trong mt hnh trn bn knh 1, khng th chn c qu 5 im
m khong cch gia hai im ty trong chng u ln hn 1.
Gii Chia hnh trn thnh 6 hnh qut bng nhau (tm cc hnh qut u ti tm O cho).
Ta bit rng khong cch gia hai im bt k trong mt hnh qut nh hn hoc bng 1, v
th t gi thit suy ra ti mi hnh qut c khng qu 1 im ri vo. Gi thit phn chng chn
c qu nm im tha mn yu cu bi. V l do trn nn s im khng th qu 7 (v nu
s im chn c m ln h n hoc bng 7 th theo nguyn l Dirichlet c t nht hai im c
chn nm trong mt cung hnh qut. m iu ny mu thun vi nhn xt trn.).
Vy t gi thit phn chng suy ra tn ti su im A1 , A2 , A3 , A4 , A5 , A6 v mi im nm
trong mt hnh qut sao cho khong cch gia hai im ty trong chng u ln hn 1.
Do
2 + A2 OA
3 + A3 OA
4 + A4 OA
5 + A5 OA
6 + A1 OA
6 = 360
A1 OA
Khi suy ra:

360

= 60 ( y t A7 A1 ).
min Ai OAi+1
6
i=1,6

i+1 = Ak OA
k+1
Xt tam gic Ak OAk+1 (vi {1, 2, 3, 4, 5, 6}, A7 A1 v mini=1,6 Ai OA
k+1 60 .
khi : Ak OA
200

k+1 60 nn t suy ra:


V OAk 1, OAk+1 1, Ak OA
k+1 max{Ak OA
k+1 O, OAk OA
k+1 }
Ak OA
T thao mi lin h gia cnh v gc trong tam gic Ak OAk+1 , th
Ak Ak+1 max{OAk , OAk+1 }
iu ny mu thun vi Ak Ak+1 > 1 (v h su im A1 , A2 , A3 , A4 , A5 , A6 tha mn yu cu
bi). T ta thy gi thit phn chng l sai. iu c ngha l khng th chn qu 5 im
tha mn yu cu bi. pcm.

Bi tp hc sinh t luyn:
Bi tp 1. Cho hnh trn c bn knh n, y n l s nguyn dng. Trong hnh trn c 4n
on thng u c di bng 1. Cho trc mt ng thng d. Chng minh rng tn ti ng
thng d0 hoc song song vi d, hoc l vung gc vi d sao cho d0 ct t nht hai on thng
cho.
Bi tp 2. Cho 1000 im M1 , M2 , M1000 trn mt phng. V mt ng trn bn knh 1 ty
. Chng minh rng tn ti im S trn ng trn sao cho
SM1 + SM2 + + SM1000 1000.
Bi tp 3. Cho chn ng trn cng c tnh cht l mi ng thng chia hnh vung thnh
2
hai t gic c t s din tch bng . Chng minh rng c t nht ba ng thng trong s
3
cng i qua mt im.
Bi tp 4. Cho mt bng c kch thc 2n 2n vung. Ngi ta nh du vo 3n bt k ca
bng. Chng minh rng c th chn ra n hng v n ct ca bng sao cho cc c nh du
u nm trn n hng v n ct ny.
Bi tp 5. Trong mt phng cho tp hn A c n im (n 2). Mt s cp im c ni vi
nhau bng on thng. Chng minh rng tp hp A cho, c t nht hai im c ni vi cng
s lng cc im khc thuc A.
Bi tp 6. Chng minh rng trong mi a gic li vi s cnh chn , tn ti ng cho khng
song song vi mt cnh no ca a gic.
Bi tp 7. Mt hnh lp phng c cnh bng 15 cha 11 000 im. Chng minh rng c mt
hnh cu bn knh 1 cha t nht 6 im trong s 11 000 im cho.
Bi tp 8. Gi s mi im trong mt phng c t bng mt trong 2 mu en v trng. Chng
minh tn ti mt hnh ch nht c cc nh cng mu.
Bi tp 9. Trong mt phng cho 6 im, trong khng c ba im no thng hng. Mi on
thng ni tng cp im c bi mu hoc xanh. Chng minh rng tn ti ba im trong s
su im cho, sao cho chng l cc nh ca mt tam gic m cc cnh ca n c bi cng
mt mu.
Bi tp 10. Cho hnh chp y l a gic chn cnh. Tt c 9 cnh bn v 27 ng cho ca
a gic y c t bng mt trong hai mu xanh hoc . Chng minh rng:
Tn ti ba nh ca hnh chp sao cho chng l nhng nh ca hnh tam gic vi cc cnh
c bi cng mu.
201

Bi tp 11. Cho P1 , P2 , , P7 l by im trong khng gian, trong khng c bn im no


ng phng . T mu mi on vi mt trong hai mu hoc en. Chng minh rng: c hai
tam gic n sc khng c chung cnh.
Bi tp 12. Cho hnh a gic u 9 cnh . Mi nh ca n c t mu bng mt trong hai
mu trng hoc en. Chng minh rng tn ti hai tam gic phn bit c din tch bng nhau, m
cch nh ca mi tam gic c t cng mt mu.
Bi tp 13. Cho mi im trn mt phng c t bng mt trong hai mu xanh, . Chng
minh rng tn ti mt tam gic m ba nh v trng tm cng mu.
Bi tp 14. Gi s 1 bn c hnh ch nht c 3 7 vung c sn en hoc trng. Chng
minh rng vi cch sn mu bt k, trong bn c lun tn ti hnh ch nht gm cc 4 gc l
cc cng mu.
Bi tp 15. Dng n mu t tt c cc cnh ca mt hnh lp phng sao cho mi nh u
c ba nh u c ba mu lin thuc, l ba mu ca ba cnh cha nh . Tm s n nh nht
hai iu kin sau ay ng thi c tha mn :
a) Khng c mt no c hai cnh cng mu .
b) Khng c hai nh no c cng ba mu lin thuc .
Bi tp 16. Trong mt t giy hnh vung bng giy c cnh bng 12cm c 31 l kim chm.
Chng minh rng ta vn c th ct t t giy ny ra mt hnh trn c bn knh 1cm m khng
cha mt l kim chm no.
Bi tp 17. Cho hnh trn (C) c din tch bng 8 , t 17 im phn bit , bt k Chng minh
rng bao gi cng tm c t nht ba im to thnh mt tam gic c din tch b hn 1
Bi tp 18. Trong hnh vung cnh 15 t 20 hnh vung nh cnh 1 tng i mt khng ct
nhau.Chng minh rng trong hnh vung ln c th t mt hnh trn bn knh 1 sao cho n
khng ct hnh vung no.

Bi tp p dng nguyn l cc hn:

Bi ton 10. Chng minh rng: Bn hnh trn ng knh l bn cnh ca mt t gic li th
ph kn min t gic ABCD.
Cch gii. Ly M l mt im ty ca t gic li ABCD. C hai kh nng xy ra:
1) Nu M nm trn bin ca a gic (tc M nm trn mt cnh ca t gic ABCD). Khi
M nm trong hnh trn c ng knh l cnh y. Trong trng hp ny kt lun ca bi ton
hin nhin ng.
2) Nu M nm bn trong t gic li ABCD. Khi ta c
B + BM
C + CM
D + DM
A = 360
AM
B, BM
C, CM
D, DM
A} = BM
C.
Theo nguyn l cc hn, tn ti max{AM
Khi y :
C 90
BM

(4)

T (4) suy ra M nm trong (hoc cng lm l nm trn) ng trn ng knh BC. Vy d


nhin M b ph bi ng trn ny.
Nh th do M l im ty ca t gic ABCD, ta suy ra bn hnh trn ni trn ph kn t
gic li cho.
iu phi chng minh.
202

Bi ton 11. Cho 2011 ng thng phn bit , trong ba ng thng bt k trong s chng
th ng quy.
Chng minh rng c 2011 ng thng cho ng quy ti mt im.
Cch gii. Ta s i gii quyt bi ton bng phng php phn chng: Gi s ngc li cc
ng thng cho khng i qua mt im. Ta xt cc giao im to nn bi 2011 ng thng
cho. Xt tt c cc khong cch khc 0 h t cc giao im ny ti cc ng thng cho.
Gi s A l mt giao im trong s v gi AQ l khong cch nh nht trong s v t A
n ` ng thng ` trong s 2011 ng thng.
Qua A theo gii thit, phi c t nht l 3 ng thng, v 3 ng thng ny c ` ln lt ti
B, C v D. V AQ`, th hai trong ba im B, C, D phi nm cng mt pha ca im Q, chng
hn l C v D.
Khng mt tnh tng qut, gi s QC < QD; v CP AD, v QKAD.
Suy ra: CP < QK < AQ
V l, v tri vi gi thit gi s AQ l khong cch b nht. iu v l trn chng t rng
2009 ng thng cho ng quy ti mt im.
Bi ton 12. ( thi hc sinh gii quc gia 1992-1993 bng A)
Mt nc c 80 sn bay, m khong cch gia hai sn bay no cng khc nhau. Mi my bay
ct cnh t mt sn bay v bay n sn bay no gn nht. Chng minh rng: trn bt k sn bay
no cng khng th c qu 5 my bay n.
Cch gii.
T gi thit suy ra nu cc my bay t cc sn bay M v N n sn bay O th khong cch
> 60 .
M N l ln nht trogn cc cnh ca tam gic M ON, do M ON
Gi s rng cc my bay bay t cc sn bay M1 , M2 , M3 , M4 , , Mn n sn bay O th mt

j khng ln hn 360 (i, j, n = 1, 2, 3, 4, 5 80) v tng cc gc cho bng


trong cc gc Mi ON
n
360 .
Vy:
360
> 60 n < 6
n
Suy ra iu phi chng minh.
Bi ton 13. ( thi hc sinh gii quc gia 1992-1993 bng B)
Trong tam gic ABC c ba gc nhn. Ly mt im P bt k, chng minh khong cch ln
nht trong cc khong cch t im P n cc nh A, B, C ca tam gic khng nh hn 2 ln
khong cch b nht trong cc khong cch t im P n cc cnh ca tam gic .
Cch gii.
Dng P A1 , P B1 , P C1 tng ng vung gc vi cc cnh BC, CA, AB. V tam gic ABC c
ba gc nhn nn cc im A1 B1 C1 tng ng nm trong on BC, CA v AB.
Ni P A, P B, P C ta c:
APC1 + C1P B + BPA1 + A1P C + CPB1 + B1P A = 360
Suy ra gc ln nht trong 6 gc ny khng th nh hn 60 .
Khng mt tnh tng qut, ta gi s APC1 l ln nht, khi APC1 60 .
Xt AP C1 vung ti C1 , ta c:
P C1
1
= cos APC1 cos 60 =
AP
2
T ta c: AP P C1 .
203

Nu thay P A bng khong cch ln nht trong cc khong cch t P n cc nh v thay


P C1 bng khong cch ngn nht t P ti cc cnh th bt ng thc cng c tha mn.
Bi ton 14. ( thi chn HSG quc gia 1986 -1987. Bng A)
Cho t gic li ABCD c hai ng cho AC v BD ct nhau ti E. Chng minh rng: Nu
cc bn knh ca 4 ng trn ni tip cc tam gic EAB, ECD, EDA m bng nhau th t gic
ABCD l hnh thoi.
Cch gii.
Hon ton khng mt tnh tng qut ta c th gi s rng :
CE AE, BE DE
Gi B1 v C1 tng ng l cc im i xng ca B v C qua tm E, ta c tam gic C1 EB1 .
nm trong min tam gic AED.
Gi s on thng AD khng trng vi on thng C1 B1 .
Khi ng trn ni tip tam gic AED nm bn trong ng trn ni tip tam gic AED,
ng dng (phi cnh) vi ng trn ny vi tm ng dng E, h s ng dng ln hn 1.
Nh vy: rAED > rC1 EB1 = rCBE (rAED l bn knh ng trn ni tip tam gic AED).
V l
V tri vi gi thit l: rAED = rCEB .
iu v l chng t l A C1 v D B1 .
Khi : OA = OC, OB = OD ABDC l hnh bnh hnh. Trong hnh bnh hnh ABCD c
p1 r = SAEB = SBEC = p2 r
( p1 v p2 v tng ng l na chu vi ca cc tam gic AEB v BEC).
Suy ra:

p1 = p2

BC + CE + BE
AB + BE + EA
=
2
2

AB = BC (v AE = CE)

Hnh bnh hnh ABCD c AB = BC nn ABDC l hnh thoi.


Bi ton 15. Chng minh
rng : Nu tt c cc cnh ca mt tam gic u nh hn 1 th din
3
tch tam gic nh hn
.
4
60 .
Cch gii. Gi A l gc nh nht ca tam gic ABC, suy ra: BAC
Ta c :
1
1
SABC = BH.AC = AB.AC sin A
2
2
Do :
SABC

3
3
1
1

=
< AB.AC. sin 60 < .1.1.
2
2
2
4

Suy ra iu phi chng minh


Bi ton 16. Gi O l giao im ca t gic li ABCD. Chng minh rng: Nu cc tam gic
AOB, BOC, COD, DOA c chu vi bng nhau th t gic ABCD l hnh thoi.
Cch gii. Khng mt tnh tng qut ta gi s :
AO CO,

DO BO.

204

Gi B1 v C1 tng ng l cc im i xng ca B v C qua O OB = OB1 ,


Tam gic B1 OC1 nm trong tam gic AOD.
Ta c:

OC = OC1 .

PAOD PB1 OC1 = PBOC = PAOD (trong k hiu P l chu vi)


Du bng xy ra khi v ch khi B1 D v C1 A.
Khi t gic ABCD c : OA = OC, OB = OD ABCD l hnh bnh hnh.
Mt khc ta li c :
PAOD = AB + BO + OA
PBOC = BC + BO + OC
Suy ra AB = BC.
Vy ABCD l hnh thoi.

Bi tp hc sinh t luyn:

Bi tp 19. Trn mt phng cho 2 2000 im, trong khng c bt k 3 im no thng


hng. Ngi ta t 2011 im bng mu v t 2011 im cn li bng mu xanh. Chng minh
rng: bao gi cng tn ti mt cch ni tt c cc im mu vi tt c cc im mu xanh bi
2011 on thng khng c im no chung.
Bi tp 20. Cho t gic ABCD tha mn: bn knh cc ng trn ni tip bn tam gic
ABC, BCD, CDA v DAB bng nhau. Chng minh t gic ABCD l hnh ch nht.
Bi tp 21. Trn mt phng cho 2011 im, khong cch gia chng i mt khc nhau. Ni
mi im trong s 2011 im ny vi im gn nht. Chng minh rng: vi mi cch ni khng
th nhn c mt ng gp khc khp kn.
Bi tp 22. Trn mt phng cho 2011 im tha mn: ba im bt k trong s chng u thng
hng. Chng minh rng: 2011 im cho l thng hng.
Bi tp 23. Bn trong ng trn tm O bn knh R = 1 c 8 im phn bit. Chng minh
rng: tn ti t nht hai im trong s chng m khong cch gia hai im ny nh hn 1.
Bi tp 24. Trn cc cnh ca tam gic ABC ly im C 0 thuc cnh AB, A0 thuc cnh BC
v B 0 thuc cnh AC. Bit rng, di cc on thng AA0 , BB 0 , CC 0 khng ln hn 1. Chng
1
minh rng: SABC (n v din tch).
3
Bi tp 25. Trn mt phng cho 2011 im khng thng hng. Chng minh rng: tn ti mt
ng trn i qua ba trong s 2011 im cho m ng trn ny khng cha bt k im no
trong s 2008 im cn li.
Bi tp 26. Cho t gic ABCD ngoi tip ng trn tm O. Chng minh rng: Nu cc ng
cho AC v BD giao nhau ti O th t gic ABCD l hnh thoi.
Bi tp 27. Trong khng gian cho mt s hu hn cc im m khng c bn im no trong
chng cng nm trn mt mt phng sao cho th tch ca mi t din to ra bi nh l nhng
im cho khng ln hn 1. Chng minh rng tt c cc im c th c ph bi mt t din c
th tch bng 27.
205

Bi tp 28. Trn mt ng thng nh du n im khc nhau A1 , A2 , A3 , An theo th t


t tri qua phi (n 4). Mi im c t bng mt trong 4 mu khc nhau v c bn mu u
c dng. Chng minh rng tn ti mt on thng cha ng hai im cu hai mu v t nht
hai im ca hai mu cn li.
Bi tp 29. Chng minh rng trong mt phng ta , khng th tm c nm im nguyn
l nh ca mt ng gic u.
(Mt im M (x; y) trn mt phng ta c gi l im nguyn nu c hai ta x, y ca n
u l nhng s nguyn).
Bi tp 30. Cho cc s nguyn m, n vi m < p, n < q cho p q l s thc i mt khc nhau.
in cc s cho vo cc vung con ca bng vung kch thc p q (gm p hng, q ct
) sao cho mi s c in vo mt v mi c in vo mt s. Ta gi mt vung con
ca bng l xu nu s nm b hn t nht m s nm cng ct vi n v ng thi b hn
t nht n s nm cng hng vi n. Vi mi cch in s ni trn, gi s l s xu ca bng s
nhn c. Hy tm gi tr nh nht ca s.

Ti liu tham kho


[1]
[2]
[3]
[4]
[5]
[6]
[7]
[8]

Sch gio khoa hnh hc lp 11, NXB Gio dc, 2010


Sch gio vin hnh hc lp 11, NXB Gio dc, 2010
Phng php dy hc mn ton. NXB Gio dc, 2005
Sch chuyn nng cao hnh hc, NXB i hc Quc gia TP. H Ch Minh, 2001
Sch chuyn hnh hc t hp, NXB i hc Quc gia H Ni, 2003
Tp ch gio dc v thi i.
Tp ch ton hc tui tr.
Mt s ti liu trn internet

206

Mt s phng php xc nh s hng


tng qut ca dy s
T Ton - Tin
Trng THPT Mc nh Chi, Gia Lai
Trong chng trnh ton THPT, bi ton v dy s chim mt v tr quan trng ca ton hc.
Nu tm c cng thc tng qut (CTTQ) ca dy s th ni dung ca bi ton gn nh c
gii quyt!. Vic hc sinh i tm CTTQ ca dy s thng gp nhiu kh khn. Bi vy, ti:
Mt s phng php xc nh cng thc tng qut ca dy s nhm gip hc sinh gii quyt mt
s lp bi ton c bn m trong chng trnh sch gio khoa 11 t cp n.
Ni dung ti chia thnh ba phn:
Phn I: Xc nh CTTQ ca mt s dy s thng gp.
Phn II: ng dng.
Mt s kt qu ca ti c nhiu sch tham kho v dy s. y, ngi vit mun a
ra nhng suy ngh v kinh nghim ca mnh nhm trao i, hc hi, v rt ra nhng kinh nghim
qu bu vn dng trong dy hc nhm nng cao cht lng dy v hc trng ph thng.
Trong qu trnh thc hin ti, ti nhn c s ng vin, gp v gip nhit tnh ca
ng nghip, nhn y, ti xin by t lng bit n chn thnh. Trong gii hn cho php, ti
c gng trnh by mt cch c h thng, sp xp t n gin n mc kh hn, ph hp vi tng
i tng hc sinh, nhm gip cc em ch ng nm bt kin thc v pht trin t duy trong qu
trnh hc tp. D vy, ti cng cha khai thc ht cc kha cnh phong ph ca n. Rt mong
s gp nhit tnh ca cc ng nghip ti c tt hn.

Xc nh cng thc ca mt s dy s thng gp

V d 1. (Bi 2, trang 92, i s v gi tch 11). Cho dy s (un ), bit


(
u1 = 1
. Bng phng php quy np chng minh rng un = 3n 4
un+1 = un + 3, n 1
Li gii. Vi n = 1, ta c u1 = 3.1 4 = 1. Cng thc ng vi n = 1.
Gi s cng thc ng vi n = k, k 1, tc l uk = 3k 4. Chng minh cng thc ng vi
n=k+1
Tht vy, ta c: uk+1 = uk + 3 = 3k 4 + 3 = 3 (k + 1) 4.
Nh vy cng thc ng vi n = k + 1 v do cng thc ng vi mi n N .
Nhn xt 1. i vi bi tp ny, sau khi c hc phng php chng minh quy np hc sinh
vi sc hc trung bnh hon ton lm c! Tuy nhin, vi kin thc v cp s cng, t cng thc
truy hi ca bi ton, d dng nhn ra ngay cng thc s hng tng qut m khng cn phi
chng minh!.
Mt cch tng qut, da vo nh ngha cp s cng ta gii quyt c lp bi ton sau y:
Bi ton 1. Cho dy s (un ), bit
(
u1 = x0
. Khi cng thc s hng tng qut ca dy s l un = u1 +
un+1 = un + d, n 1
(n 1) d. Tng t, ta cng gii quyt c lp bi ton v cp s nhn.
215

Bi ton 2. Cho dy s (un ), bit


(
u1 = x0
. Khi cng thc s hng tng qut ca dy s l un = u1 q n1 .
un = un1 q, n 2
V d 2. (Bi 3, trang 92, i s v gii tch 11). Cho dy s (un ), bit
(
u1 = 3
p
. D on cng thc tng qut v chng minh bng quy np.
un+1 = u2n + 1, n 1
tng: Bi ton ny i hi hc
v tm quy lut cho s
sinh phivit vi bas hng u
ca dy s. Chng hn u1 = 3; u2 = 10; u3 = 11; u4 = 12; u5 = 13; ...Nh vy c th d
on rng s hng tng qut l mt biu thc cha trong cn bc hai!. Vn l cn bc hai ca
biu thc no?

rng cc s trong cn bc hai cch unhau mt n v nu nh ta xem u1 = 3 = 9.


n y hc sinh c th d on c un = n + 8. Vic chng minh xem nh hc sinh gii
quyt c. Vi vic d on v chng minh bng quy np, hc sinh cng c th gii quyt bi
ton sau:
V d 3. (Bi 45, trang 123, i s v gii tch 11 nng cao). Cho dy s (un ), bit

u1 = 2
2n1 + 1
. Chng minh rng un =
, n 1. tng: R rng
un + 1
un+1 =
2n1
, n 1
2
bng quy np ta chng minh c! Tuy nhin, ta th tm cch a (un )v mt cp s cng hoc
cp s nhn d dng lin h vi u1 cho.
un1 + 1
2un = un1 + 1 a v mt cp s nhn. Nhng
2
v phi vng phi s 1!. By gi, nu ta t un = vn + d v thay vo th ta c 2 (vn + d) =
vn1 + d + 1 2vn + 2d = vn1 + d + 1. n y, nu 2d = d + 1 d = 1 th (vn ) l mt cp s
1
1
nhn vi cng bi q = . M v1 = u1 d = 1 suy ra vn = v1 q n1 = n1 .
2
2
2n1 + 1
n y ta c un =
, n 1. Bi ton c gii quyt xong!
2n1
Tng t nh trn, ta xt bi ton sau y:
(
u1 = 2;
V d 4. Cho dy s (un ) xc nh bi
. Tm s hng tng qut ca
un = 3un1 1, n 2
dy s.
Li gii. Ta bin i un =

tng: Ta thy rng (un ) khng phi l mt cp s nhn!. Do , ta tm cch lm mt s


1 v tri ca biu thc truy hi v chuyn dy s v mt cp s nhn.


1
1
1
1
.
Li gii. Ta c un = 3un1 1 un = 3un1 1 un = 3 un1
2
2
2
2
1
5
t vn = un v1 = v vn = 3vn1 , n 2. Do (vn ) l mt cp s nhn cng bi
2
2
5 n1
5
1
q = 3. Suy ra vn = .3 . n y ta c un = .3n1 + .
2
2
2
1
hai v ca cng thc truy hi ri t dy ph (vn ) a v mt cp
2
s nhn c v p t!. Lm th no phn tch t nhin hn? Ta lm nh sau: t un = avn + b,

Nhn xt 2. Vic bt i

216

vi a R , b R (s chn sau). Khi avn + b = 3avn1 + 3b 1 vn = 3vn1 +

2b 1
. n
a

y, mun (vn ) l
mt cp s nhn ta chn
b= 1
2b 1
=0
2
a R
a
C th chn a = 1; b =

1
ta c vn = 3vn1 , n 2. Do (vn ) l mt cp s nhn cng
2

bi q = 3.
5
1
5
Suy ra vn = .3n1 . n y ta c un = .3n1 + .
2
2
2
T hai bi ton trn ta c th tng qut c:
Bi ton 3. Xc nh c cng thc s hng tng qut ca dy s (un ) cho bi
(
u1 = x0
.
un+1 = aun + b; a 6= 0, n 1
Bng cch chn dy s (vn ) sao cho un = vn + k. T cng thc xc nh ca dy s (un ) ta
suy ra vn + k = a (vn1 + k) + b = avn1 + ak + b. n y ta chn k sao cho k = ak + b k =
b
(a 6= 1). Khi (vn ) l mt cp s nhn cng bi q = a v v1 = u1 k. Do s hng tng
1a
qut ca (vn ) l vn = v1 an1
b
Vy un = (u1 k) an1 + k vi k =
1a
Trng hp a = 1 th (un ) l mt cp s cng c cng sai d = b
Mt cch bin i khc:
Trng hp 1: Nu a = 1 th (un ) l mt cp s cng c cng sai d = b


b
b
= a un +
Trng hp 2: Nu a 6= 1 th ta vit un+1 = aun + b un+1 +
. T
a1
a1


b
b
= u1 +
an1 .
ta c un +
a1
a1
an1 1
Vy un = u1 an1 + b
.
a1
Kt lun: Dy s (un ) cho bi
(
u1 = x0
lun xc nh c cng thc s hng tng qut nh trn.
un+1 = aun + b; a 6= 0, n 1
By gi ta xt bi ton m rng cho lp bi ton sau y:
V d 5. Cho (un ) xc nh bi
(
u1 = 2
. Tm cng thc tng qut ca dy s.
un = 2un1 + 3n 1, n 2
tng: Ta khng th p dng kt qu ca Bi ton 3 v y v phi ca cng thc truy
hi l mt hm s bc nht theo n. Mun vy, ta cn lm mt 3n 1.
Li gii. Ta vit un + 3n + 5 = 2un1 + 3n 1 + 3n + 5 un + 3n + 5 = 2 [un1 + 3 (n 1) + 5]
t vn = un + 3n + 5, ta c v1 = 10 v vn = 2vn1 . Tc l (vn ) l mt cp s nhn cng bi
q = 2 v do vn = 10.2n1 . Vy un = 5.2n 3n 5.
Nhn xt: Vi vic cng vo hai v ca cng thc truy hi cho 3n + 5 khng t nhin lm.
Vy ta phn tch nh sau: 3n 5 = an + b 2 [a (n 1) + b]. Cho n = 1; n = 2 ta c
217

a = 3; b = 5.
Trong trng hp tng qut, thay 3n + 5l mt a bc nht theo n bi mt a thc f (n) ta
cng phn tch tng t f (n) = g (n) ag (n 1) (*), trong g (n) cng l a thc theo n
Tng qut: Tm cng thc tng qut ca dy s (un ) cho bi
(
u1 = x0
, trong f (n) l mt a thc theo n. Khi ta c un g (n) =
un = aun1 + f (n)
a [un1 g (n 1)] = ... = an1 [u1 g (1)]. Vn cn li l ta xc nh g (n) na l xong!
Trng hp 1: Nu a = 1 ta chn g (n)l a thc bc k + 1 c h s t do bng 0. Khi trong
(*), ta cho nnhn k + 1 gi tr s tm c g (n).
Trng hp 2: Nu a 6= 1 ta chn g (n)l a thc bc k. Khi trong (*), ta cho nnhn k gi
tr s tm c g (n).
Nh vy ta tm c cng thc s hng tng qut ca dy s trn.
Bi ton 4. Xc nh c cng thc s hng tng qut ca dy s (un ) cho bi
(
u1 = x0
un = aun1 + f (n) ; a 6= 0, n 2

Bng cch t un = g (n) + a [un1 g (n 1)], trong g (n) cng l a thc theo n. Khi
t vn = un g (n) ta c un = [u1 g (1)] an1 + g (n)
Vn lm theo nh Bi ton 4, ta cng tm c cng thc cho bi ton sau:
(
u1 = x0
Bi ton 5. 5: Xc nh c cng thc s hng tng qut ca dy s (un ) cho bi
un = aun1 + b.n ; a
Bng cch xt hai trng hp:
Trng hp 1: Nu = a th n = nn (n 1) n1 .
Ta c un bnn = [un1 b (n 1) n1 ] = ... = (u1 b) n1 . Do un = b (n 1) n +
u1 n1
Trng hp 2: Nu 6= a th ta phn tch n = kn akn1 . Khi un bkn =

.
a (un1 bkn1 ) = ... = an1 (u1 bk). Do un = an1 (u1 bk) + bk. Ta tm c k =
a
Trn y ta xt n dy s cho bi cng thc truy hi da vo phng trnh tuyn tnh cp
mt. By gi, ta da v phng trnh tuyn tnh cp hai xt bi ton rt ni ting sau y.
V d 6. Mt i th con (gm mt th c v mt th ci) k t lc trn hai thng tui c
mi thng li ra mt i th con (gm mt th c v mt th ci). Gi s t lc u thng
ging c mt i th s sinh. Hi n u thng n c bao nhiu i th?
Bi ton 6 (Bi ton Fibonacci). tng: cho tin ta gi Fn l s i th sau n thng. Khi
ta c F1 = 1; F2 = 1. D thy rng n thng ba ta c F3 = 2 + 1 = 3 i th . n thng t
ta c F4 = 3 + 2 = 5i th. . . C tip tc suy din nh vy, n thng n ta c Fn = Fn1 + Fn2 .
Nh vy y chnh l bi ton tm s hng tng qut ca dy s (Fn ) xc nh bi
(
F1 = 1; F2 = 1
.
Fn = Fn1 + Fn2 , n 3
Li gii. Ta tm cch th dy s (Fn ) bng mt dy s khc l mt cp s nhn nh sau:
T Fn = Fn1 + Fn2 Fn x1 Fn1 = x2 (Fn1 x1 Fn2 ).
Do ta phi chn x1 ; x2 sao cho
(

x1 + x2 = 1
1 5
2
. Hay x1 ; x2 l nghim ca phng trnh x + x 1 = 0 x =
.
2
x1 x2 = 1
218

1 5
1+ 5
Chn x1 =
; x2 =
, ta c Fn =
2
2

!
!n1
1 5
1+ 5
Fn1 +
2
2
"
!
n
!n #
1
1+ 5
1 5
p dng kt qu ca Bi ton 5 ta c Fn =
.

2
2
5
Bi ton trn do Fibonacci (1170 - 1250) nh Ton hc ngi tm ra sau ny nh Ton hc
ngi Php E. Lucas (1842 - 1891) t tn cho bi ton ny mang tn l dy s Fibonacci. Cn
cng thc tng qut l do nh Ton hc ngi Php Binet (1786 - 1856) tm ra v gi l cng
thc Binet.
T bi ton ny, ta rt ra kt qu tng qut sau y:
Bi ton 7. 6: Tm cng thc tng qut ca dy s (un ) xc nh bi cng thc
(
u1 = ; u2 =
(trong a, b R : a2 4b 0)
un + aun1 + bun2 = 0, n 3
Bng cch xt hai nghim x1 ; x2 ca phng trnh x2 + ax + b = 0: gi l phng trnh c
trng ca dy s
Nu x1 6= x2 th un = kxn1 + lxn2 , trong k; l l nghim ca h phng trnh
(
k+l =
.
x1 k + x2 l =
n1
( Nu x1 = x2 = x0 th un = (kn + l) x0 trong k; l l nghim ca h phng trnh
l = x0
.
k+l =

V d 7. Tm cng thc s hng tng qut ca dy s (un ) trong cc trng hp sau:


(
u1 = 1; u2 = 2
a. (un ) :
un+1 4un un1 = 0, n 2
(
u1 = 1; u2 = 3
b. (un ) :
un 4un1 + 4un2 = 0 , n 3
Li gii.

a. Phng trnh c trng x2 4x 1 = 0 x = 2 5.

1
Chn x1 = 2 + 5; x1 = 2 5 suy ra k = l = .
2
n
n i
1h
2+ 5 + 2 5 .
Vy un =
2
b. Phng trnh c trng x2 4x + 4 = 0 x = 2 suy ra k = 1; l = 2.
Vy un = (n + 2) 2n1 .
Bng cch xy dng tng t ta cng xc nh cng thc tng qut ca dy s cho bi cng
thc truy hi da vo phng trnh tuiyeens tnh cp ba.
Bi ton 8. Tm cng thc tng qut ca dy s (un ) xc nh bi cng thc
(
u1 ; u2 ; u3
un+1 + aun + bun1 + cun2 = 0, n 3
Bng cch xt phng trnh c trng x3 + ax2 + bx + c = 0()
Nu () c ba nghim phn bit x1 ; x2 ; x3 th un = xn1 + xn2 + xn3 . T ba s hng u ca
dy s suy ra cng thc tng qut.
219

Nu () c mt nghim n v mt nghim kp x1 = x2 6= x3 th un = ( + n) xn1 + xn3


T ba s hng u ca dy s suy ra cng thc tng qut.
Nu () c ba nghim bi x1 = x2 = x3 th un = ( + n + n2 ) xn1 . T ba s hng u ca
dy s suy ra cng thc tng qut.
(
u1 = 0; u2 = 1; u3 = 3
V d 8. Cho (un ), bit
. Tm cng thc tng qut
un+1 = 7un 11un1 5un2 = 0, n 3
ca dy s.
Li gii. Phng trnh c trng x3 7x2 + 11x 5 = 0 c mt nghim n v mt nghim
bi l x1 = x2 = 1; x3 = 5. Suy ra un = + n + 5n .
3
1
1
Cho n = 1; n = 2; n = 3 ta c = ; = ; = .
16
4
16
1
3
1 n
Vy un = + n + 5
16 4
16
V by gi ta xt mt lp bi ton cho bi cng thc truy hi lin tng n cc cng thc
lng gic quen thuc.

u =1
1
V d 9. Cho (un ), bit
. Tm cng thc tng qut ca dy s.
2
u
= 2u2 1, n 1
n+1

tng: T cng thc truy hi ta lin tng n cng thc nhn i ca cosin: 2cos2 x 1 =
cos2x.

Do nu ta t u1 = cos th ta c mt dy truy hi theo cos .


3
3

4
8
Li gii. t u1 = cos u2 = cos ; u3 = cos ; u4 = cos ; ...n y, hc sinh c th
3
3
3
3
2n1
v chng minh bng quy np.
d on s hng tng qut un = cos
3
Mt cch tng qut, ta c bi ton sau y:
Bi ton 9. Tm cng thc tng qut ca dy s (un ) xc nh bi cng thc
(
u1 = x0
un+1 = 2u2n 1, n 1
Bng cch xt hai trng hp: Tng hp 1: Nu |x0 | 1 th ta t cos = x0 . Khi bng
phng php quy np ta tm c un = cos2n1 

1
1
a+
, trong a l nghim ca phng
Trng hp 2: Nu |x0 | > 1 th ta t u1 =
2
a
trnh a2 2u1 a + 1 = 0chn acng du vi u1 . Khi bng phng php quy np ta tm c
1
1
un =
a2n1 + 2n1 .
2
a
rng phng trnh a2 2u1 a + 1 = 0 c hai nghim c tch bng 1 nn ta c un =

2n1 
2n1
p
p
1
u1 u21 1
+ u1 + u21 1
.
2
Mt cch tng t, da vo cng thc nhn ba ca cosin ta c bi ton nh sau:
Bi ton 10. Tm cng thc tng qut ca dy s (un ) xc nh bi cng thc
(
u1 = x0
un+1 = 4u3n 3un1 , n 1
220

Bng cch cch xt hai trng hp


Trng hp 1: Nu |x0 | 1 th ta t cos = x0 . Khi bng phng php quy np ta tm
c un = cos3n1


1
1
Trng hp 2: Nu |x0 | > 1 th ta t u1 =
a+
, trong a l nghim ca phng
2
a
trnh a2 2u1 a + 1 = 0chn acng du vi u1 . Khi bng phng php quy np ta tm c
1
1
un =
a3n1 + 3n1 .
2
a
rng phng trnh a2 2u1 a + 1 = 0 c hai nghim c tch bng 1 nn ta c un =

3n1 
3n1
p
p
1
.
u1 u21 1
+ u1 + u21 1
2

u1 =
V d 10. Cho (un ), bit
. Tm cng thc tng qut ca dy s.
2

3
un+1 = 4un 3un1 , n 1
tng: T cng thc truy hi ta lin tng n cng thc nhn ba ca cosin: 4cos3 x3cosx =

cos3x. Do nu ta t u1 = cos th ta c mt dy truy hi theo cos .


6
6
3
9
27

; ...
Li gii. t u1 = cos u2 = cos ; u3 = cos ; u4 = cos
6
6
6
6
3n1
n y, hc sinh c th d on s hng tng qut un = cos
v chng minh bng quy
6
np.

ng dng

Trong ti ny chng ti nu mt s phng php tm s hng tng qut ca dy s cho bi


cng thc truy hi. Vn l n p dng c g trong gii ton. Sau y l mt s ng dng
ca lp bi ton ny.
Bi tp 1. Trong mt phng cho n ng thng i mt khng ct nhau v khng c dng quy.
Hi rng trong n ng thng ni trn chia m phng thnh bao nhiu min?
tng: y l bi ton t hp, a bit rng c th s dng quy tc m l tnh c s min
to ra bi n ng thng ni trn. Tuy nhin, c mt ng thng chia mt phng thnh hai
min, ta gi an l s min to ra bi n ng thng ta c ngay a1 = 2. Bi ton tr thnh tm s
hng
( tng qut ca dy s xc nh bi
a1 = 2
an+1 = an + n + 1, n 1

Li gii. Gi an l s min to ra bi n ng thng ta c ngay a1 = 2. Xt ng thng


th n + 1 ct n ng thng cho ti n im v b n ng thng ny chia thnh n + 1 phn
v mi phn thuc mt min ca an . Mt khc, mi on nm trong ca min an chia min
thnh
hai min nn s min c thm l n + 1. Vy ta c dy s (an ) xc nh bi cng thc
(
a1 = 2
an+1 = an + n + 1, n 1
n (n + 1)
p dng kt qu ca Bi ton 4 ta c an = 1 +
2
Bi tp 2. Trong khng gian cho n mt phng trong khng ba mt no ct nhau v khng
c bn mt no cng i qua mt im. Hi n mt phng ny chia khng gian thnh bao nhiu
221

min? tng: Vi tng nh Bi tp 1, ta c li gii sau y:


Li gii. Gi an l s min to ra bi n ng thng ta c ngay a1 = 2. Xt mt phng th
n + 1 ct n mt phng cho ti n giao tuyn v n giao tuyn ny chia mt phng th n + 1
n (n + 1)
phn, mi min ny thuc mt min ca an v chia min thnh hai min .
thnh 1 +
2
Vy ta c dy s (an ) xc nh bi cng thc

a1 = 2
2
an+1 = an + n + n + 2 , n 1
2
(n + 1) (n2 n + 6)
p dng kt qu ca Bi ton 4 ta c an =
6
Bi tp 3. Cho dy s (xn ) xc nh bi cng thc

x1 = 2
2x + 1
xn+1 = n
, n 1
xn + 2
a. Tnh x2000
2000
P
b. Tm phn nguyn ca A =
xi Olympic 30 -4 2000
i=1

tng: Ta thy cng thc truy hi l mt phn thc cha ti gin. Bng cch bin i v
rt gn cho mu thc v dng mt dy ph th ta c kt qu.
Li gii. Ta c xn+1 =

xn 1
1
3
2xn + 1
xn+1 1 =

=1+
xn + 2
xn + 2
xn+1 1
xn 1

1
suy ra y1 = 1; yn+1 = 3yn + 1.
xn 1
3n+1 1
.
p dng Bi ton 3 ta c yn =
2
2
32001 + 1
Vy xn = 1 + n+1
v do x2000 = 2001
3
1
3 1



2000
2000
2000
P
P
P
2
1
b. Ta c A =
xi =
1 + n+1
= 2000 + 2
. Suy ra 2000 < A <
3
1
3n+1 1
i=1
i=1
i=1
P 1
2 2000
< 2001. Vy [A] = 2000.
2000 +
3 i=1 3i
t yn =

Bi tp

a.

b.

4. Tnh gii hn cc dy s cho bi cng thc truy hi:

c.
(
d.

u1 = 2
un+1 =

un + 1
,n 1
2

u1 = 3
un+1 =

2un + 4
,n 1
3

u1 = 9
un = un1 + 3n 3, n 2
u1 = 11
un+1 = 10un + 1 9n, n 1

Li gii.
222

a. p dng Bi ton 3, ta c cng thc tng qut ca dy s l un = u1 (a)n1 + b




2
2
+ 1. Do lim un = lim n + 1 = 1
2n
2
n1

(a)n1 1
=
a1

(a)n1 1
=
+b
a1

b. p dng Bi ton 3, ta c cng thc tng qut ca dy s l un = u1 (a)


!
 n1
 n1
2
2

+ 4. Do lim un = lim
+4 =4
3
3


3
c. p dng Bi ton 4, ta vit 3n 3 = n2 + n 1 (n 1)2 + (n 1) = .. = n2
2
3
3
3
3 2 3
3
3
2
2
n (n 1) + (n 1). Suy ra un n + n = un1 (n 1) + (n 1).
2
2
2
2
2
2
2
3 2 3
3 2 3
t vn = un n + n vn = vn1 = .. = v1 = u1 = 9 un = n n + 9
2
2
2
2
Do lim un = +
d. Theo Bi ton 4, ta vit 1 9n = n + 10 [ (n 1) + ] = .. = n + 1 10 [(n 1) + 1].
Suy ra un n 1 = un1 (n 1) 1.
t vn = un n vn+1 = 10vn vn = 10n un = 10n + n
Do lim un = +
Bi tp 5. Cho dy s (un ) c xc nh bi:

u1 = 0
n
P
1
u1 = 0 v
.
un1 + 2009 vi n = 2, 3, . . . t vn =
un =
k=1 uk 2009
un1 + 2011
a. Tm s hng tng qut ca dy s (un ).
vn
.
b. Tnh lim
n + 2011
Li gii.
un1 + 2009
2un1 2
un1 + 2009
1 =
un 2009 =
2009 =
un1 + 2011
u
un1 + 2011
 n1 + 2011 
2010(un1 2009)
un 1
1
un1 1
Suy ra
=
.
un1 + 2011
un 2009
1005 un1 2009
1
1
un 1
yn =
yn1 v y1 =
t yn =
un 2009
1005
2009

n1
1
1
p dng Bi ton 2 dy (yn) c s hng tng qut l yn =
.
2009 1005

n1
1

2009
2009
2009 1 (1005)n1
1005
=
Suy ra un = 
n1
1 2009(1005)n1
1
2009
1005

k1
2009.2008(1005)k1
1
1
1
1
b. Ta c uk 2009 =

k1
uk 2009
2009.2008 1005
2008
1 2009(1005)
a. Ta c un 1 =

n
P

1
1
Do
=
2009.2008
k=1 uk 2009

1
1005

0


+

n
2008
223

1
1005

1


+ +

1
1005

n1

n
=
2008



1005 1

1
2009.2008

n 


1
1005 1
vn
n
1005
Suy ra
=

.
n + 2011
2009.2008.1004(n + 2011) 2008(n + 2011)
n 



1
1005 1 1005

vn
n
= 1
Vy lim
= lim

n+ n + 2011
n+
2009.2008.1004(n + 2011) 2008(n + 2011)
2008
(
Bi tp 6. Cho dy s (xn ) c xc nh bi:

x1 = 1
, n = 1, 2, 3, .. , t
xn+1 = xn 2 + 3xn + 1

n
P

1
, n = 1, 2, 3, ...
i=1 xi + 2
Tm lim yn .

yn =

Li gii. T h thc xn+1 = xn 2 + 3xn + 1, n = 1, 2, 3, ... , (1)


Suy ra xk+1 = xk 2 + 3xk + 1 > 3xk 3.3k1 = 3k , k 1.
Chng minh bng quy np ta c xn > 3n1 . (2)
Cng t (1) suy ra xn+1 + 1 = xn 2 + 3xn + 2 = (xn + 1) (xn + 2) .
1
1
1
1
1
1
1
Do
=
=

.
=
+
xn+1 + 1
(xn + 1) (xn + 2)
xn + 1 xn + 2
xn + 2
xn + 1 xn+1 + 1


n
n
P 1
P
1
1
1
1
1
1
Vy yn =
=

=
,
=
xi + 1 xi+1 + 1
x1 + 1 xn+1 + 1
2 xn+1 + 1
i=1 xi + 2
i=1
1
Tm li lim yn =
2

Li kt

Tri qua thc tin s phm, ni dung ca ti vi s gp ca ng nghip vn dng vo


ging dy thu c nhng kt qu sau:
1. i vi nhng hc sinh tr ln c th gii quyt c lp bi ton c bit.
2. i vi hc sinh gii c th vn dng cc kt qu ny trong cc k thi hc sinh gii.
3.T cc kt qu ny, gio vin c th xy dng nn cc bi ton v dy s.
Vi mc ch nhm nng cao cht lng dy v hc, khi dy tnh sng to ca hc sinh, bn
thn ngi vit nu ra mt chuyn nh. Tuy nhin cng cha khai thc ht cc kha cnh ca
n v cng khng th trnh khi nhng thiu st mc phi. Rt mong c gi gp su sc
ti c hon thin hn. Xin chn thnh cm n.

Ti liu tham kho


1. Sch gio khoa, i s v gii tch 11, NXB GD
2. Sch gio khoa, i s v gii tch 11 Nng cao, NXB GD
3. V Giang Giai Nguyn Ngc Thu, Mt s bi ton v dy s, NXB HQG H Ni
4. Tp ch Ton hc v tui tr, Tuyn tp 30 nm tp ch Ton hc v tui tr, NXB GD
224

Mt s ng dng nguyn l Dirichlet


trong t hp v bt ng thc
Ng Minh Ngc Richard 10CT, Nguyn Vn Quang
THPT Chuyn Nguyn Du, k Lk
Trong cc k thi chn i tuyn, chn hc sinh gii cc cp hin nay, cc bi ton i s s
cp ang xut hin t dn trong cc thi, thay vo l s xut hin ngy cng nhiu ca cc
bi ton S hc, T hp. Cc bi ton ny chim t l kh cao trong thang im. c bit, cu T
hp thng c hc sinh mc nh ngm l cu kh nht trong thi. Tuy nhin, nu nm vng
mt s phng php, cu T hp hon ton c th c gii quyt. Mt trong nhng phng php
c bn nht l s dng nguyn l Dirichlet, hay cn gi l nguyn l "chung v th", nguyn l
ngn ko, v.v. . . Nguyn l Dirichlet c ni dung kh n gin, song li l cng c quan trng v
c nhiu ng dng su sc trong ton hc.
Trong gii hn khun kh ti, cc tc gi xin c trnh by cc khi nim v nguyn l
Dirichlet v ng dng ca nguyn l trong dng ton: T hp v Bt ng thc.Do c nhiu ti
liu vit v cc bi ton s dng Dirichlet nn cc bi ton trong c tc gi a ra trong phn
ng dng l nhng bi ton hay v mang tnh chn lc. Hy vng ti s gip ch cho nhng ai
cha quen vi vic s dng Dirichlet gii ton.
Mc d rt c gng nhng chc chn ti vn cn nhng thiu st, mong ngi c thng
cm v gp ti c hon thin hn.

Vi nt v nguyn l Dirichlet

Nguyn l Dirichletc t theo tn ca nh ton hc ngi c Johann Dirichlet (18051859). ng l ngi u tin xut v pht biu nguyn l ny vo nm 1834. Nguyn l ny
c rt nhiu ng dng quan trng trong hu ht cc lnh vc ton hc. i vi cc bn hc sinh,
y l cng c khng th thiu khi gp nhng bi ton m cc phng php thng thng khng
mang li hiu qu.

1.1

Nguyn l Dirichlet:

* Nguyn l Dirichlet c nhiu cch pht biu khc nhau, sau y l cch pht biu di dng
"ngn ko": Nu xp n vt vo m (m 2) ngn ko th lun tn ti t nht 1 ngn c cha t
n
e vt.(K hiu dae ch s nguyn nh nht khng nh hn a).
nht d m
Hoc: Nu xp nm + k (n > k) vt vo m ngn ko th lun tn ti t nht 1 ngn c cha t
nht n + 1 vt.
Chng minh: Gi s tt c cc ngn u c nhiu nht l m vt, khi tng s vt s khng
vt qu mn, iu ny v l. Do nguyn l c chng minh.
* Li th ca nguyn l Dirichlet l ta c th ch ra s tn ti ca mt i tng m khng
cn quan tm n tnh cht ca i tng . Chng hn khi ta phn hoch mt tp hp gm 10
phn t thnh 3 tp con, th d ta khng bit nhng phn t l g, ta vn c th khng nh
rng tn ti mt tp con c cha t nht 4 phn t.
* Khi gii ton, mun p dng nguyn l Dirichlet, cn phi nhn ra hai yu t, l "vt"
v "ngn". C kh nhiu bi ton, hai yu t ny xut hin kh mp m trong bi, i hi
chng ta phi c k nng nhn ra chng. Cc bi ton trong phn II s cho chng ta thy r
hn iu ny.
225

T hp:

* Do c nhiu chuyn ni v cc bi ton T hp nn tc gi ch xin a ra nhng bi


ton hay v mang tnh tiu biu.
V d 1. Cho bng vung kch thc 2000 2001 (hng ct). Hy tm s nguyn dng k
ln nht sao cho ta c th t mu k vung con ca bng tha mn iu kin: hai vung con
no c t mu cng ko c nh chung.
VMO 2001 Bng B
Gii.
nh s cc hng t tri qua phi, cc ct t trn xung di.
Quy c: c ta (i, j), (i, j N , i 2000, j 2001), l nm hng i, ct j.
* Chia bng vung nh hnh 1:
D thy rng bng vung c chia thnh 1000.1001 min phn bit.
* Gi s c nhiu hn 1000.1001 vung c t mu. Theo nguyn l Dirichlet, c t nht
hai vung c t mu nm trong cng mt min,tc l tn ti hai vung c t mu c
chung nh, iu ny tri vi gi thit.
Suy ra, k 1000.1001
aaaaaaaaaaaaaaaaaaaaaaaaaaaaaaaaaaaa
aaaaaaaaaaaaaaaaaaaaaaaaaaaaaaaaaaaaaaa
aaaaaaaaaaaaaaaaaaaaaaaaaaaaaaaaaaaaaaaa
aaaaaaaaaaaaaaaaaaaaaaaaaaaaaa
Hnh 1
* Ta s ch ra cch t tha k = 1000.1001 :
* T tt c cc c ta (2x 1, 2y 1), x, y N , x 1000, y 1001.
D thy cch t trn tha v s c t l 1000.1001
Vy, gi tr ln nht ca k l 1000.1001
Nhn xt 1. y l bi ton kh d, tng chia bng vung thnh cc min phn bit l rt
r rng.
V d 2. Vi mi s nguyn dng n, (n 2), ta xt mt bng vung n n. Mi vung
con c t bi mu hoc mu xanh. Tm s n nh nht sao cho vi mi cch t ta lun chn
c mt hnh ch nht kch thc m k (2 m, k n) m bn vung con 4 gc ca hnh
ch nht ny c cng mu.
thi chn i tuyn HSG lp 10 KHTN H Ni (2014 -2015).
Gii.
Gi mt hnh ch nht (HCN) tho mn bi l mt HCN tt.
* Vi n {2, 3, 4} th tn ti cch t sao cho khng tn ti HCN tt. Ta chng minh:
n N , n 5, hnh vung n n lun tn ti mt HCN tt.
* Vi n = 5, xt hnh vung 5 5.
Theo nguyn l Dirichlet, mi ct lun tn ti t nht 3 cng mu.
* Nu tn ti mt ct c 5 (xanh), d thy lun tn ti mt HCN tt vi 4 nh mu
xanh ().
* Nu tn ti mt ct c 4 hoc xanh. Gi s ct c 4 : Ta thy rng trong 4 ct
cn li nu c nhiu hn mt ct c 2 th s tn ti mt HCN tt vi 4 nh mu . Do
trong 4 ct cn li ch c nhiu nht mt ct c 2 . Tc l ta s c 3 ct c t nht 4 xanh,
do tn ti mt HCN tt vi 4 nh mu xanh.
226

* Xt trng hp tt c cc ct c t 3 , 2 xanh hoc 3 xanh, 2 : Theo nguyn


l Dirichlet, c t nht 3 ct c 3 c t cng mu, khng mt tng qut gi s cc ny c
t cng mu .
Xt 4 hng bt k trong 5 hng. Do hng cn li c ti a 3 nn tng s ca 3 ct
4 hng ny khng nh hn:
3.3 3 = 6 = 4 + 2
Do theo nguyn l Dirichlet, tn ti 2 ct c cng 2 trong 4 hng ny nn tn ti
mt HCN tt vi 4 nh mu .
Vy, lun tn ti mt HCN tt trong hnh vung 5 5. Vi n > 5, hnh vung n n cha hnh
vung 5 5 nn lun tn ti mt HCN tt.
Vy, n = 5 l gi tr nh nht tha mn bi.
Nhn xt 2. y l cu t hp ca thi chn i tuynca trng KHTN c ng trn Din
n VMF. tng n kh t nhin thng qua vic c gng tm mt cch v tha trng hp
n = 5. Tc gi on rng khng th v c nh vy, v tng s dng nguyn l Dirichlet c
ny sinh. Cng vic cn li ch l chia trng hp v gii quyt bi ton.
V d 3. Cho A v B l hai tp con ca tp {1, 2, 3, 100} tha |A| = |B| v |A B| = .
Xc nh s phn t ln nht ca tp A B sao cho vi n A, ta lun c 2n + 2 B.
thi ngh Olympic truyn thng 30/4 (2011-2012)
Gii.
V 2n + 2 B m B {1, 2, 3, , 100} nn:
2n + 3 100 n 49

Do : A {1, 2, 3, , 49}
Ta chia tp {1, 2, 3, , 49} thnh 33 tp con nh sau:
Nhm 1: Gm 16 tp con cha ng 2 phn t: {1, 4}, {2, 6}, {3, 8}, {5, 12}, {7, 16}, {9, 20}, {10, 22}, {10,
Cc tp ny u c dng {x, 2x + 2}.
Nhm 2: Gm 17 tp con cha ng 1 phn t: {25}, {26}, {27}, {29}, {31}, {33}, {34}, {35}, {37}, {39},
Nu |A| 34, khi theo nguyn l Dirichlet tn ti t nht mt trong 16 tp con nhm
mt c 2 phn t u thuc tp A, tc l tn ti 2 s x v 2x + 2 cng thuc tp A, iu ny
mu thun vi gi thit.
Suy ra,
|A| 33 |A B| = |A| + |B| |A B| = 2|A| 66
Ta chn hai tp A, B nh sau:
Chn tp A :
A = {1, 2, 3, 5, 7, 9, 10, 11, 13, 14, 15, 17, 18, 19, 21, 23, 25, 27, 29, 31, 33, 35, 37, 39, 41, 42, 43, 45, 46, 47, 49}
Chn tp B :
B = {2n + 2|n A}
R rng 2 tp A, B tha v |A B| = 66
Vy, s phn t ln nht ca tp |A B| l 66.
Nhn xt 3. Bn cht bi ton kh n gin, tuy nhin gii ng n th cn phi c s kin
nhn trong vic phn hoch tp {1, 2, 3, , 49}. y l mt trong nhng bi rt d sai p s.

227

V d 4 (IMO Shortlist 1994). Cho tp X = {1, 2, 3, , 15}. Gi M l mt tp con ca X tha


iu kin tch 3 phn t bt k ca M u khng phi l s chnh phng. Tm s phn t ln
nht ca M.
Gii.
Gi b ba phn t bt k ca X c tch l s chnh phng l mt b xu.
Chia tp X thnh 5 tp con nh sau:
A1 = {1, 4, 9}, A2 = {2, 6, 12}, A3 = {3, 5, 15}, A4 = {7, 8, 14}, B = {10, 11, 13},
Ta thy rng cc b ba phn t ca Ai (i = 1, 2, 3, 4) u l cc b xu.
Nu |M | 12 th theo nguyn l Dirichlet, c t nht 2 trong 5 tp trn l tp con ca M, suy
ra c t nht mt b xu c cha trong M. Suy ra |M | 11.
Gi s |M | = 11. p dng nguyn l Dirichlet, c 1 trong 5 tp trn l tp con ca M. M
khng cha b xu th tp B phi l tp con ca M, cc tp Ai (i = 1, 2, 3, 4) mi tp c 2 phn
t thuc M.
V B M nn ta c 10 M. Ta c hai b xu i vi 10 l (2, 5, 10), (6, 10, 15).
D thy rng thy rng nu c 3 v 12 khng thuc M th s c t nht mt trong hai b xu
trn c cha trong M.
Suy ra 3 M v 12 M. Tuy nhin ta li c hai b xu i vi 3 v 12 l hai b
(3, 12, 1), (3, 12, 9). Mt khc 1 v 9 u thuc tp A1 , nn chc chn c t nht mt trong hai
phn t ny thuc M. iu ny ng ngha vi vic mt trong hai b xu trn s c cha trong
M, gy mu thun vi gi thit.
Suy ra, |M | 10.
Ta s ch ra tp M c ng 10 phn t tha yu cu bi:
M = {1, 4, 5, 7, 10, 11, 12, 13, 14}
Vy, s phn t ln nht ca M l 10.
Nhn xt 4. Bi ton ny cng thuc dng phn hoch tp hp nh bi trn, nhng li l mt
bi ton kh kh, i hi tnh t duy t hp cao. Vic chia tp hp ch ra |M | 11 l iu d
nhn ra, nhng ch ra |M | 10 th li l c mt vn . y l mt bi ton rt hay v th v.
V d 5. Cho n + 1 s nguyn dng khc nhau v nh hn 2n. Chng minh tn ti ba s trong
n + 1 s m mt s bng tng hai s kia.
Gii.
Gi n + 1 s nguyn dng cho l a1 , a2 , an+1
Khng mt tng qut gi s:
1 a1 < a2 < an < an+1 2n 1
t
b i = ai a1

(i = 2, 3, n + 1).

Suy ra:
1 b2 < b3 < < bn < bn+1 2n 1
Xt dy 2n s a2 , a3 , , an+1 ; b2 , b3 , , bn+1 . Cc s ny nhn 2n 1 gi tr khc nhau nn
theo nguyn l Dirichlet, c t nht s trong dy trn bng nhau.
Mt khc ta c: ai 6= aj , bi 6= bj , 2 i, j < n + 1
Ngoi ra ai 6= bi , i = 2, 3, , n + 1 (do a1 6= 0)
228

Suy ra tn ti ax = by
Hay

(x 6= y, 2 x, y n + 1)
ax = ay a1 a1 + ax = ay

Vy, ta c iu phi chng minh.


Nhn xt 5. y l mt bi ton thuc dng xy dng dy s. tng l to ra mt dy c 2n
s v nhn 2n 1 gi tr khc nhau, t suy ra trong dyc hai s bng nhau. tuyn sinh
ca trng chuyn Nguyn Du k Lk (2014 2015)
* C mt cu tng t vi n = 2013, nhng li cho gi thit l cc s t nhin nn khng th
chng minh c.
V d 6. Cho 2014 s t nhin bt k. Chng minh rng lun tn ti 729 s c tng chia ht cho
729.
thi chn i tuyn tnh chuyn Nguyn Du k Lk (2014-2015)
Gii.
* Ta chng minh b n gin:
Trong 5 s t nhin bt k lun tn ti 3 s c tng chia ht cho 3.

(1)

Chng minh: Gi 5 s t nhin cho l a1 , a2 , a3 , a4 , a5 . Gi 5 s d ca 5 s ny khi chia


cho 3 tng ng l b1 , b2 , b3 , b4 , b5 .
* Nu cc s bi (i = 1, 2, 3, 4, 5) ch nhn cng mt gi tr 0, 1, 2 th d thy 3 s bt k trong
ai u c tng chia ht cho 3.
* Nu cc s bi nhn 2 gi tr trong 3 gi tr 0, 1, 2 th theo nguyn l Dirichlet, tn ti 3 s c
.
gi tr bng nhau. Gi s b1 = b2 = b3 th a1 + a2 + a3 ..3.
.
* Nu cc s bi nhn c 3 gi tr 0, 1, 2. Gi s b1 = 0, b2 = 1, b3 = 2 th d thy a1 + a2 + a3 ..3.
Vy, b (1) c chng minh.
* Ta chng minh b tip theo:
Trong 53 s t nhin bt k lun tn ti 53 s c tng chia ht cho 27.

(2)

Chng minh:
Gi tp 53 s t nhin cho l A. Ta c: |A| = 53 = 17.3 + 2.
p dng b (1), tn ti 3 phn t c tng chia ht cho 3, gi tng ny l B1
B i 3 phn t trn, p dng b (1), tn ti 3 phn t c tng chia ht cho 3, gi tng
ny l B2 C tip tc lm nh vy. . .
M ta c: 53 = 3.17 + 2 nn suy ra tn ti 17 s Bi c tnh cht tng t.
Xt 17 s Bi (i {1, 2, , 17}), chng minh tng t nh trn, ta c 17 = 5.3 + 2 nn tn
ti 5 s Cj (j {1, 2, 3, 4, 5}) sao cho mi s l tng ca 3 s Bi v chia ht cho 3. Mt khc cc
s Bi ny chia ht cho 3 nn suy ra cc s Cj chia ht cho 9.
.
Xt 5 s Cj trn, p dng b (1), tn ti s D sao cho D l tng ca 3 s Cj v D..3. Mt
khc cc s Cj ny chia ht cho 9 nn suy ra D 27. Ngoi ra, D cn l tng ca 3.3.3 = 27
phn t ca A.
T y suy ra b (2) c chng minh.
* Tr li bi ton: Ta chng minh bi ton vn cn ng vi 1457 s t nhin.
Gi tp 1457 s t nhin cho l X. p dng b (2), chng minh tng t nh trn, tn
ti tng S
vdots27.27 = 739 v S tng ca 27.27 = 729 phn t ca X.
229

Vy, bi ton c chng minh.


* Nhn xt: Mu cht ca bi ton l vic pht hin ra 729 = 36 a bi ton v cc b
n gin hn. Bi ton ny cn c th tng qut ha nh sau:
Trong 2n 1 s lun tn ti n s c tng chia ht cho n.
Li gii ca bi ton tng qut ny tng i di v c ng trn bo Ton hc v Tui tr
s 383, tc gi xin php khng nu ln y.
* Ch : Li gii trn i theo con ng: 3 33 36 . Ta c th chng minh c bi ton
bng cch s dng b (1) m khng cn chng minh b (2). Tuy nhin theo tc gi cch
lm trn s ngn gn hn.
*
Nhn xt 6. tng ca cc bi ton trn l to ra hai yu t "vt" v "ngn ko" p dng
nguyn l Dirichlet. c th l "im" v "min", "phn t" v "tp hp", v.v. . . Nm c
hai yu t ny th bi ton tr nn n gin. Tuy nhin cng c mt s bi ton m vic p dng
nguyn l Dirichlet ch l bc khi u cho mt chui suy lun, nh gi logic nh bi ton
VD4. Hy vng nhng bi ton trn s gip cc bn gp nht nhng kinh nghim trong vic gii
ton T hp.

Bt ng thc (BT):

* ng dng ca nguyn l Dirichlet trong vic gii cc bi ton i s s cp (phng trnh,


h phng trnh,. . . ) l khng ng k. Tuy nhin, vn c mt s bi ton i s c gii bng
phng php s dng nguyn l Dirichlet, in hnh l mt s bi ton v BT ba bin i xng.
* Trong phng php ny, thng th ta d on im ri ca BT ri nh gi mt s i
lng f (a) = f (b) = f (c) = 0 sao cho hp l. Mc ch l nh gi cc i lng khng thun
nht.
* Ta s xt ba VD m u thy c s hiu qu ca phng php trn:
V d 7. Cho a, b, c l cc s thc dng. Chng minh rng:
a2 + b2 + c2 + 2abc + 1 2(ab + bc + ca)
* BT trn c th chng minh bng BT Schur bc 3.Tuy nhin, chng ta ths gii quyt
n bng cch s dng nguyn l Dirichlet:
Gii.
Xt 3 s a 1, b 1, c 1. p dng nguyn l Dirichlet, c t nht 2 trong 3 s trn cng du.
Khng mt tng qut gi s a 1 v b 1 cng du, ta c:
(a 1)(b 1) 0 ab + 1 a + b 2abc 2ac + 2bc 2c
Ta cn chng minh:
a2 + b2 + c2 + 2ac + 2bc 2c + 1 2(ab + bc + ca)
(a b)2 + (c 1)2 0
iu ny hin nhin ng.
ng thc xy ra khi v ch khi a = b = c = 1.
Nhn xt 7. y l v d c bn nht v phng php s dng nguyn l Dirichlet. Trong v d
trn, ta d on im ri ti a = b = c = 1, ri s dng nguyn l Dirichlet nh gi cc i
230

lng a 1, b 1, c 1, sau s dng php nhm bnh phng hon tt chng minh. C th
thy rng phng php ny lm cho BT n gin i rt nhiu. Trong cc VD s 4, 5, 6, ta
s thy c s hiu qu ca phng php ny khi kt hp vi phng php dn bin.
V d 8. Cho a, b, c > 0. Chng minh rng:
p
abc + 3 (1 + a)3 (1 + b)3 (1 + c)3 ab + bc + ca
Gii.
p dng nguyn l Dirichlet, trong 3 s a 1, b 1, c 1 c t nht 2 s cng du. Khng mt
tng qut gi s a 1 v b 1 cng du, ta c:
(a 1)(b 1) 0 ab + 1 a + b abc + c ac + bc
p dng BT Holder ta c:
(13 + a3 )(13 + b3 )(c3 + 1) (c + ab)3
Suy ra:
abc +

p
3
(1 + a3 )(1 + b3 )(1 + c3 ) abc + c + b ac + bc + ab

ng thc xy ra khi v ch khi a = b = c = 1.


Nhn xt 8. Cm gic ban u v bi ton l s phc tp vi lp cn bc ba v cc biu thc
khng thun nht vi nhau. Tuy nhin li gii bng nguyn l Dirichlet trn li cho thy iu
ngc li.
V d 9. Cho a, b, c 0, chng minh rng:
1
abc + 2 + [(a 1)2 + (b 1)2 + (c 1)2 ] a + b + c
2
Gii.
p dng nguyn l Dirichlet, trong 3 s a 1, b 1, c 1 c t nht 2 s cng du. Khng mt
tnh tng qut gi s a 1 v b 1 cng du, ta c:
(a 1)(b 1) 0 ab a + b 1
Nh vy ta cn chng minh:
1
c(a + b 1) + 2 + [(a 1)2 ) + (b 1)2 + (c 1)2 ] a + b + c
2

(a 1)2 + (b 1)2 + (c 1)2 2(a + b 2)(1 c)


p dng BT Cauchy-Schwarz (C-S) v BT AM-GM, ta c:
(a + b 2)2
(a 1)2 + (b 1)2 + (c 1)2
+ (c 1)2
2

2|(a + b 2)(c 1)| 2(a + b 2)(c 2)


ng thc xy ra khi v ch khi a = b = c = 1.

231

Nhn xt 9. y l thi tuyn sinh lp 10 KHTN H Ni. Kh c th kim c li gii no


ph hp hn cho BT ny, c bit l trong p lc phng thi.
* Nh vy qua ba VD trn, bc u ta thy c s hiu qu ca phng php s dng
nguyn l Dirichlet trong mt s BT i xng khng thun nht.
* By gi ta s xt mt s BT s dng phng php dn bin bng cch s dng cc BT
kinh in kt hp vi nguyn l Dirichlet:
V d 10. Chng minh BT sau ng vi mi s thc dng a, b, c :
(b + c a)2
(c + a b)2
(a + b c)2
3(a2 + b2 + c2 )
+
+

2a2 + (b + c)2 2b2 + (c + a)2 2c2 + (a + b)2


2(a + b + c)2
Nhn xt 10. Vi mt BT thun nht phc tp nh trn th iu u tin ta ngh ti l s
dng phng php chun ha lm n gin ha bi ton.
Gii.
BT cho thun nht vi 3 bin a, b, c nn ta chun ha a + b + c = 1.
BT c vit li thnh:
(1 2b)2
(1 2c)2
3
(1 2a)2
+
+
(a2 + b2 + c)
2
2
2
2a 2a + 1 2b 2b + 1 2c 2c + 1
2
Xt 3 s a 31 , b 13 , c 13 . p dng nguyn l Dirichlet th t nht 2 c trong 3 s trn cng
du. Gi s 2 s l b 13 v c 13 , ta c:

2
2 1
1 
1
2
2
0 b
c
b +c
a +
3
3
3
9
By gi p dng BT Cauchy-Schwarz ta c:
(1 2c)2
(2 2b 2c)2
(1 2b)2
+

3b2 2b + 1 3c2 2c + 1
3(b2 + c2 ) 2(b + c) + 2
4a2
12a2
= 
= 2
2
9a 6a + 5
3 32 a + 13 2(1 a) + 2
Mt khc, ta c:
a2 + b 2 + c 2 a2 +

2
3

2

4
5
1
= 2a2 a +
9
3
9

Vy ta ch cn phi chng minh BT mt bin a sau:


(1 2a)2
12a2
5
+
3a2 2a +
2
2
3a 2a + 1 9a 6a + 5
6
(1 2a)2
12a2
1
1
2
+ 2
3a2 2a +
3a 2a + 1 9a 6a + 5 2
3
(3a 1)2 (13a2 6a + 5)
1

2(3a2 2a + 1)(9a2 6a + 5)
3(3a 1)2
(3a 1)2 (54a4 72a3 + 33a2 14a 5)

0
6(3a2 2a + 1)(9a2 6a + 5)
Ta cn chng minh: 54a4 72a3 + 33a2 14a 5 0
(a 1)[a(54a2 18a2 + 15) + 1] 4 0
232

(3)

V a, b, c > 0, a + b + c = 1 0 < a < 1 (3) ng.


Vy, BT c chng minh.
ng thc xy ra khi v ch khi a = b = c.
* Ch 1: Ta khng nn chun ha a + b + c = 3 v s lm cc h s trong BT ln hn, v
tnh gy nn kh khn cho cc bc bin i tip theo.
* Ch 2: bc phn tch nhn t chng minh BT mt bin sau cng, nu ta d
on c ng thc xy ra khi a = b = c = k th s lun xut hin nhn t a k hoc (a k)2 .
V d 11. Cho a, b, c l cc s thc tha mn a + b + c = 3. Chng minh:
1
1
1
1
+
+

5a2 4a + 11 5b2 4b + 11 5c2 4c + 11


4
Gii.
Xt 3 s a 1, b 1, c 1. p dng nguyn l Dirichlet th t nht 2 c trong 3 s trn cng
du. Gi s 2 s l b 1 v c 1, ta c:
0 (b 1)(c 1) b2 + c2 (2 a)2 + 1
Ta thy rng khng th p dng ngay BT C-S v BT s b i chiu. Ta s bin i BT
nh sau:
1
 1

1
1
1
1

5a2 4a + 11 20
10 5b2 4b + 11
10 5c2 4c + 11
2[b2 + (2b 1)2 ] 2[c2 + (2c 1)2 ]
9 + 4a 5a2

(4)
5b2 4b + 11
5c2 4c + 11
5a2 4a + 11
p dng BT Cauchy-Schwarz, ta c:
(3c 1)2
[3(b + c) 2]2
(3b 1)2
+

5b2 4b + 11 5c2 4c + 11
5(b2 + c2 ) 4(b + c) + 22
(7 3a)2
9a2 42a + 49

=
5(2 a)2 + 5 4(3 a) + 22
5a2 16a + 35

V T(4)

Vy ta cn chng minh:
9a2 42a + 49
9 + 4a 5a2
2(a 1)2 (35a2 103a + 112)

0
5a2 16a + 35
5a2 4a + 11
5a2 16a + 25)(5a2 4a + 11)
BT ny hin nhin ng.
ng thc xy ra khi v ch khi a = b = c = 1.
Ch : Nu ta bin i BT thnh:
1
 1

1
1
1

5a2 4a + 11
8 5b2 4b + 11
8 5c2 4c + 11
8
(2b 1)2 + b2 + 12 + 12 (2c 1)2 + c2 + 12 + 12

5b2 4b + 11
5c2 4c + 11
5a2 4a + 11
th xut hin bnh phng t ca hai phn thc, ta cn phi p dng BT C-S cho 4
s. R rng ta lm cho v tri ca BT yu i nhiu hn so vi cch gii "chun". Kt qu l
BT mtbin cui cngs khng lun ng a R :
(a 1)2 (45a2 276a + 211)
0
(5a2 16a + 35)(5a2 4a + 11)
233

(5)

V d 12. Cho a, b, c l cc s thc dng. Chng minh:


r
a b c
8abc
+ + 2+
2
b c a
(a + b)(b + c)(c + a)
Gii.
y l mt BT hon v nn ta s a n v dng i xng bng cch t:
a
b
c
x = , y = , z = x, y, z > 0, xyz = 1
b
c
a
BT cn chng minh tng ng:
p
x+y+z2+

8
2
(x + 1)(y + 1)(z + 1)

(6)

p dng nguyn l Dirichlet th t nht 2 c trong 3 s x 1, y 1, z 1 cng du. Gi s


x 1 v y 1 cng du, ta c:
(x 1)(y 1) 0 (x + 1)(y + 1) 2(xy + 1)
q
4

V T(6) 2 xy + z 2 +
(xy + 1)(x + 1)
t

xy = t (t > 0), ta cn chng minh:


r
1
4
2
2t + 2 2 + 2
1
t
(t + 1)( t2 +1)

 2t3 2t2 + 1
  4t2

1 +
1 0
t
(t2 + 1)2
(t2 1)2
2t3 3t2 + 1

0

t( 2t3 2t2 + 1 + t) (t2 + 1)2


2t + 1
(t + 1)2
2

(t 1)

0
t( 2t3 2t2 + 1 + t) (t2 + 1)2

Ti y th ta ch cn chng minh nhn t trong ngoc vung lun dng. p dng BT


AM-GM, ta c:
2t + 1
(t + 1)2

2
t( 2t3 2t2 + 1 + t) (t + 1)2
=

2t + 1
t2 +(2t3 2t2 +1)
2

+ t2

(t + 1)2
(t2 + 1)2

2t3 (t 1)2 + t4 + 2t3 + 2t2 + 2t + 1


> 0 (do ) t > 0
(2t3 + t2 + 1)(t2 + 1)2

Vy, BT c chng minh.


ng thc xy ra khi v ch khi a = b = c.
Nhn xt 11. Qua cc v d trn ta thy rng nguyn l Dirichlet khng ch c ch trong nhng
bi ton mang tnh T hp m cn gip chng ta rt nhiu trong vic gii ton BT. tng
ch o l s dng nguyn l lm n gin BT ban u, sau kt hp vi cc phng php
khc nh nhm bnh phng, dn bin, s dng cc BT kinh in,. . . Hy vng ngi c s
thy thch th vi phng php ny v thng xuyn p dng n.
234

Ti liu tham kho


* Mt s ti liu t Internet, c bit l t diendantoanhoc.net
* Tp ch Ton hc v Tui tr.
* S dng phng php Cauchy-Schwarz chng minh bt ng thc ca V Quc B Cn
Trn Quc Anh.

235

Phng php phn chng


trong hnh hc
Hong Huy Thng - Bi B Anh 10CT, Nguyn Vn Quang
Trng THPT Chuyn Nguyn Du, k Lk
Trong ton hc, hnh hc l mt phn rt quan trng, c bit n t rt sm, pht trin
khng ngng ngh t xa ti nay. Trong h thng THCS, THPT, hnh hc c xem nh l phn
quan trng nht, thu ht c rt nhiu cc bn hc sinh tm ti, hc tp. Ngay c cc k thi tuyn
sinh, Hc sinh gii, Olympic, cc bi ton hnh hc lun c gp mt v l nhng bi ton kh,
thng c dng phn loi. Do tnh pht trin lin tc, nhng bi ton hnh hc trong cc k
thi ngy cng phc tp, i hi chng ta phi tm thm nhng cng c mi, phng php mi
gii quyt chng, m cng l lm phong ph, tch ly thm kinh nghim cho mnh. Bi vit
ny xin c gii thiu mt trong nhng phng php nh th trong chng minh hnh hc: Phng
php phn chng.
Phng php phn chng trong hnh hc vn khng phi l phng php qu xa l, ngay t khi
lp 7, ta bit dng kin thc ny chng minh nhng tnh cht c bn, chng hn nh chng
minh hai gc so le trong - ng v bng nhau, ng dng ca Tin Euclid. Hay cc lp trn,
phng php ny c s dng quan trng trong vic chng minh cc nh l o, in hnh l 2
nh l Ceva, Menelaus (c th c cch chng minh khc khng s dng phng php ny), chng
minh tnh duy nht. . . v.v.
Tuy nhin, chng ti vn cho rng y l phng php mi, v v c bn, c rt t nhng bi
ton dnh ring cho n, v phng php ny vn kh mi m, hu nh t c s dng chng
minh.

C s khoa hc v mc tiu nghin cu

1.1

C s ca phng php chng minh phn chng

Phng php chng minh phn chng m t qu trnh lp lun nh sau:


Cn chng minh mnh A B.
chng minh A B ng, ta xy dng gi thit: A ng, nhng A B sai.
Bi v A B sai, m A ng nn B phi c gi tr sai, ngha l ng.
T ng, thong qua mt s php bin i tng ng dn n ng.
T gi thit v qu trnh lp lun ta c A v ng thi ng, dn n mu thun.
iu chng t gi thit ng l sai. Vy B ng hay A B ng (iu phi chng minh).

1.2

Cc bc suy lun phn chng

Bc 1: Gi s iu phn chng l sai (ph nh mnh cn chng minh).


Bc 2: T iu gi s suy ra mt s tnh cht, quan h mi, m cc tnh cht, quan h ny
mu thun vi iu cho hoc tri vi tnh cht m ta bit.
Bc 3: Kt lun iu gi s ban u l sai. T bi ton c chng minh.
Ch 1. Trong cc bc trn, bc 1 rt quan trng v cn to ra mnh ph nh iu cn
chng minh phi chnh xc.
Xin c bt u vi 2 bi ton sau:
236

Bi ton 1. Chng minh rng mt ng thng d ct c ba cnh ca mt tam gic khi v ch


khi n i qua mt nh ca tam gic.
Bi ton trn c th coi l mt tnh cht hin nhin trong chng minh hnh hc m nhiu
khi trong qu trnh lm bi, ta tha nhn kt qu ny. Tuy nhin, tnh cht ny hon ton c
th chng minh c. Vi nhng bn thng xuyn tip xc, khng ch trong hnh hc, th
kinh nghim l: Nhng bi ton mang tnh tha nhn, i hi chng minh, th cch chng minh
thng gp v hiu qu nht l phn chng!
Li gii. Xt tam gic ABC. Gi s ngc li, ng thng d khng i qua nh no ca tam
gic. Khi d chia mt phng ra lm hai min. Do 3 nh ca tam gic khng c nh no thuc
d, theo nguyn tc Dirichlet, tn ti mt min cha t nht hai nh, khng mt tnh tng qut,
l nh A v nh B. Khi cnh AB nm hon ton trong na mt phng ny v khng th
ct d c, mu thun vi gi thit l d ct tt c ba cnh ca tam gic ABC. Vy d phi i qua
mt nh ca tam gic.
Bi ton 2. Cho hnh vung ABCD c cnh bng a, M l trung im ca cnh AD, im E nm
trn BC tha mn iu kin 0 < CE < . Qua M k ng thng song song vi AE, ct cnh CD
ti F. Chng minh rng hnh thang AMFE khng th l hnh thang cn.

Li gii. Gi s AMFE l hnh thang cn.


Ta c AM = EF. (1)
Mt khc, ta li c: = = EA l tia phn gic gc ngoi ti nh E ca tam gic EFC. (2)
V t gic ABCD l hnh ch nht nn CA l tia phn gic trong ti nh C ca tam gic
EFC. (3)
T (2) v (3) suy ra A l tm ng trn bng tip gc ca tam gic EFC, ng trn bng
tip ny tip xc vi EC ti B, FC ti D.
Do : EF = BE + DF
Theo gi thit, 0 < CE < BE > EF = BE + DF > BE > = AM.
Mu thun vi (1)!
237

Vy iu gi s sai v ta c pcm
Nhn xt 1. Trn y l 2 bi ton c bn ca phng php phn chng hnh hc. tng
phn chng l r rng v khng kh chng minh n. Nhng bi ton c gii thiu sau y
l nhng bi ton ni ting t cc k thi HSG, mang v p thc s ca phng php ny, khi
chng khng d hnh thnh tng v cch chng minh:
Bi ton 3. Cho ABC c = 105o , ng trung tuyn BM v ng phn gic CD ct nhau ti
K sao cho KB = KC. Tnh cc gc ABC.
Li gii.

K AH vung gc vi BC.
Xt tam gic AHC vung ti H, ta c AM = CM = HM (v M l trung im AC).
Suy ra = = 2 = 2 (V KB = KC).
M = + nn = . Do tam gic HBM cn ti H.
Nh vy HM = HB.
Nu HA > HB. Suy ra < 45o m = 105o nn > 600 .
Do = 1800 2 < 600 . HA < HM HA < HB. Mu thun.
Nu HA < HB. Hon ton tng t suy ra v l.
Vy HA = HB.
T ta c = 45o , = 105o , = 300 .
Bi ton 4. Chng minh rng tam gic c hai ng phn gic bng nhau th tam gic l
tam gic cn. (nh l Steiner Lenmus)
Li gii. Cch 1
Gi s hai ng phn gic BD, CE ca tha mn BD = CE. Ta chng minh .
Gi s (Nu chng minh tng t) th .
Dng hnh bnh hnh BEID.
238

Ta c EIC cn ti E nn .
M nn . (1)
Xt BCD v CBE, ta c BC chung BD = CE
Nn CD > BE CD > DI.
Suy ra (2)
T (1) v (2) ta c
Mu thun.
Do iu gi s sai. Vy ABC l tam gic cn.
Cch 2

Gi F l giao im ca BD v CE
Gi s th , ta ly im I trn on thng FE sao cho . Khi t gic BICD l t gic ni
239

tip.
Mt khc nn . Suy ra .
Ta c < 900 (v ) nn t suy ra .
Do CI > BD, dn n CE > BD, tri vi gi thit.
Vy , do ABC cn ti A.
Nhn xt 2. Ngoi hai cch dng phn chng nh trn, cn c mt cch dng cng thc tnh
di ng phn gic v bin i i s nh sau:
Ta c
V nn ta c
hay
Vy b = c hay tam gic ABC cn ti A.
Bi ton 5. Cho tam gic nhn ABC c cc ng cao k t B v C ct nhau ti O. Chng minh
rng nu ng trn ni tip tam gic OAB v ng trn ni tip tam gic OAC c bn knh
bng nhau th tam gic ABC l tam gic cn. ( thi hc sinh gii lp 9 tnh Qung Ngi-2009)

Nhn xt 3. So vi bi ton v nh l Steiner - Lenmus, vic hnh thnh tng cho bi ton
ny l phong ph hn, bi bi ton c cho kh nhiu d kin, rt ph hp vi cc phng
php chng minh thng thng nh k ng ph,xt quan h cc di, tam gic. . . v.v, c
bit l gi thit c xut hin ng trn ni tip tam gic th t tng li cng c khng
nh.
Th nhng, bn c hy th chng minh xem, liu c th d dng thc hin cc cch lm .
Ti xin khng nh rng bc u, mi cch chng minh thng thng u tht bi, v trong qu
trnh tm ti, ta c th d dng nhn ra mi cch lm u thiu 1 d kin no , mi th
cha chng minh xui v m ch phc v khi tam gic cn, v cc gi thit c tnh
i xng nhau. ng trc tnh hung , t tng phn chng l t tng cui cng m cng
l ph hp nht.
240

Li gii. Gi X,Y ln lt l tm ng trn ni tip cc tam gic OAB v OAC. H,K th t


l tip im ca hai ng trn ny vi AB,AC.
T gi thit suy ra XH = YK.
Do O l trc tm tam gic ABC nn (cng ph )
Xt BXH v CYK ln lt vung ti H, K ta c: = XH = YK
Suy ra BXH = CYH (cnh huyn gc nhn)
T y ta c BH = CK.
Nu AB > AC th AH > AK (do BH = CK)
< (1)
(do OA vung gc BC)
AB < AC (tri vi iu gi s)
Nu AB < AC, hon ton tng t ta suy ra AB > AC, mu thun.
Tm li, AB = AC, tc l ABC cn ti A
Ch 2. Trong bi ton ny, ta tha nhn mt tnh cht kh hin nhin (1), nhng iu ny
cng hon ton c th chng minh c nh sau: Do AH > AK nn trn on AH, ly I sao cho
IH = AK, d chng minh IHX = AKY (c-g-c), t p dng tnh cht gc ngoi v ch
rng I nm gia A v H, ta c pcm.
Bi ton 6. Cho hnh bnh hnh ABCD, AB < AD, gc A t, E l mt im trn ng trn
ngoi tip tam gic ABD. ng thng a i qua nh C ct AB v AD ln lt ti H v K. Chng
minh rng nu E l tm ng trn ngoi tip tam gic AHK th ng thng a cha ng phn
gic ca gc BCD.

Nhn xt 4. Cng ging nh bi ton trn, tng u tin cho bi ton ny l cc phng
php hnh hc thng thng, v chc chn t ai dm ngh rng s s dng phng php phn
chng chng minh, nht l trong bi ton c rt nhiu gi thit phc tp, cc gi thit c phn
khng i xng. iu duy nht gi cho ta ngh ti phng php ny li chnh l t kt lun ca
bi ton. Sau y chng ti xin gii thiu mt cch gii s dng phng php phn chng:
241

Nu AH = AK th = . Suy ra = = = (do ABCD l hnh bnh hnh). Do ng thng a


cha tia phn gic ca
Nu AH > AK, h EI , EF th t vung gc AB v AD ( I thuc AB, F thuc AD)
Suy ra EF > EI ( khong cch t tm ti dy b hn th ln hn)
Mt khc, do t gic ABDE ni tip nn = , ta c BEI v DEF l 2 tam gic vung ng
dng
= EI.DF = EF.BI, m EF > EI nn DF > BI
Ta li c FK = AK < AH = AI DK = DF FK > BI AI = AB = DC
Mt khc, d thy DCK ng dng vi AHK nn:
1 > = AH < AK Mu thun vi iu gi s!
Nu AH < AK, hon ton tng t ta suy ra iu v l.
Tm li, ch c th xy ra AH = AK v do ta c pcm
T bi ton ny mt ln na khng nh v p ca phng php phn chng trong hnh hc,
m cng l s khng nh li tnh khng t nhin ca n. Nhng bi ton s dng phng php
phn chng u rt hay, nhng rt kh ngh ti v tm ra li gii cho n bng phng php
ny. V vy bi ton trn, tt hn ht bn c hy th to ra 1 cch gii khc mang tnh t
nhin hn, bi t rt nhiu gi thit phc tp, hon ton c th c nhiu cch gii khc cho n.
Bi ton 7. Cho hnh vung ABCD, im M thuc min trong ca hnh vung tha mn iu
kin = = 15o. Chng minh rng tam gic MBC u.
Nhn xt 5. y l mt bi ton ny kh quen thuc s dng phng php tnh ton cc gc
v k ng ph, n khng kh bng 2 bi ton trn, tuy nhin t bn bit n cch chng minh
khc cho bi ton ny. mi chnh l im mi m ca bi ton: Phn chng minh!

Theo gi thit ta suy ra: = = 75o (1) = 1500 (2)


Ta c MAD cn ti M nn MA=MD. (3)
T (1) v (3) BMA = CMD (c g c ) (4) MB = MC hay MBC cn ti M =
Gi s MBC khng u, suy ra MB > BC hoc MB < BC
Nu MB > BC, tc l MB > AB
242

> , t (1) suy ra: < 75o (5)


T (2), (4), (5) ta c: + + < 3000
0
0
> 600 < 180 260 = 600 . Do > MB < BC (v l)
Nu BC > MB, hon ton tng t ta suy ra iu v l.
Vy phi c MB = BC, kt hp vi MB = MC, ta suy ra MBC u.
Bi ton 8. Chng minh rng nu mt ng thng ng thi chia chu vi v din tch tam gic
thnh hai phn bng nhau th ng thng i qua giao im ca cc ng phn gic ca tam
gic .
Nhn xt 6. Bi ton ny thc s l mt bi ton rt kh - c v tng ln cch chng minh,
thm ch phng php quen thuc v mnh nht l k ng ph cng khng x l c. Nguyn
nhn chnh nm gi thit kh t v ri rc. V vy nu kho lo khi khai trin gi thit t nhng
suy ngh n gin th t tng phn chng minh bi ton ny li kh r.

Li gii. Xt tam gic ABC c tm ng trn ni tip I. Gi s ng thng MN chia chu vi


v din tch tam gic thnh 2 phn bng nhau, ta s chng minh MN i qua I T gi thit ta c
c:
AM + AN = BM + BC + CN (1)
SAMN = SBMNC (2)
Gi r l khong cch t I ti cc cnh ca tam gic, t (1) ta c:
r(AM + AN) = r(MB + BC + CN) SIAM + SIAN = SIMB + SIBC + SINC (3)
Ti y th tng r, ta thy rng nu I thuc MN th ng thc (3) cng chnh l (2),
c vy, ta s phn chng
Nu I nm trong tam gic AMN th tr v theo v ca (2) cho (3) ta c
SIMN = SIMN (v l)
Nu I nm ngoi tam gic AMN th tr v theo v ca (2) cho (3) ta c
SIMN = SIMN (v l)
Nh vy, I phi nm trn mt cnh ca tam gic AMN, hin nhin I khng nm trn AM,
AN, vy I thuc MN.
243

Bi ton 9. Gi P l im trong tam gic ABC, cc ng thng AP, BP, CP ct ng trn


ngoi tip tam gic tam gic ABC ln lt li K, N, M. Tip tuyn ti C ct cnh AB ko di ti
S. Bit MK = MN. Chng minh rng SC = SP.

Li gii.
Gi E, F th t l giao im ca SP vi ng trn ngoi tip tam gic ABC.
Gi s ME > MF, suy ra = < = (do MN = MK)
Do EK < FN( gc ni tip chn cc cung EK v FN)
=+<+=
= 1800 > 1800 =
Tn ti im D trn on SA sao cho =
SPD ng dng SBP SP2 = SB.SD < SB.SA = SC2 (do SC l tip tuyn ca ng trn)
SP < SC (1)
Mt khc, do ME > MF = + > + = (gc to bi tip tuyn v dy v gc c nh bn trong
ng trn).
SP > SC (cnh i din gc ln hn th ln hn). iu ny mu thun vi (1).
Tng t trng hp cn li, ME < MF, ta cng thu c iu mu thun.
Vy th ch xy ra kh nng ME = MF, khi = hay SP = SC.
Nhn xt 7. T bi ton ny mt ln na khng nh v p ca phng php phn chng
trong hnh hc, m cng l s khng nh li tnh khng t nhin ca n. Nhng bi ton s
dng phng php phn chng u rt hay, nhng rt kh ngh ti v tm ra li gii cho n
bng phng php ny. V vy cc bi ton trn, tt hn ht bn c hy th to ra 1 cch gii
khc mang tnh t nhin hn, bi t rt nhiu gi thit phc tp, hon ton c th c nhiu
cch gii khc cho n.

244

Bi tp ngh

Bi tp 1. C tn ti hay khng mt tam gic c hai ng trung tuyn nh hn na cnh i


din?
Bi tp 2. Cho ng trn (O) v I, K ln lt l trung im ca cc dy cung AB, CD. Bit
rng AB > CD v tia AB ct tia CD ti P, chng minh rng PI > PK.
Bi tp 3. Cho ng trn tm O c hai ng knh AB v CD vung gc vi nhau. Gi I v K
ln lt l trung im ca OA, OB. Tia CK ct (O) ti F. Chng minh rng khng phi l gc
vung.
Bi tp 4. Cho tam gic ABC c trng tm G. Chng minh rng nu AB + GC = AC + GB
th ABC l tam gic cn.

Kt lun

ti ny c vit theo chng trnh THPT, dng cho hc sinh, gio vin trong vic nghin
cu tham kho. Chng minh bng phng php phn chng l mt dng ton hay gip ta gii
quyt nhng bn ton vi tnh ng n c th thy ngay m nhng cch chng minh khc khng
th lm c.
Trong ti, chng ti a ra c s l thuyt, mt s bi tp, nhn xt c rt ra trong qu
trnh nghin cu. Tuy nhin nhng nhn xt trn mang tnh c nhn nn c nhiu hn ch. Mong
nhn c nhng gp b ch t Thy, C v cc bn c gi.
Xin trn trng cm n!
Bun Ma Thut, ngy 28 thng 12 nm 2014

Ti liu tham kho


1. Nguyn B ang (Tuyn chn): 279 Bi ton hnh hc phng Olympic cc nc, NXBGD
2. L Quc Hn: n sau nh l Ptoleme, NXB Gio dc, 2007
3. on Qunh (Ch bin) : Ti liu Chuyn ton Hnh hc, NXB Gio dc
4. Tp ch Ton hc v Tui tr
5. Din n ton hc VMF: http:/
diendantoanhoc.net/forum
6. Din n Mathscope: http://website.informer.com/forum.mathscope.or
7. Din n AOSP: http://www. Artofproblemsolving.com/forum
8. Mathlink.ro.

245

You might also like